Torts

Pataasin ang iyong marka sa homework at exams ngayon gamit ang Quizwiz!

ex of collateral sources: P gets 10k judgment against the D. D says my insurance has already paid you 5k. that 5k should be deducted from the 10k, owe you only 5k. That's legit. But let's say the gov, or a rich aunt, or a hospital, decides to collect only half of a bill - that half that they didn't collect is a collateral source, a benefit to P.

P can still collect the entire bill from D. by refusing to let D have that benefit, making them pay the entire bill, deter behavior in the future. Normally the P's own insurance or employer will be the one that gives them the money. But D still has to pay the bills.

what kind of notices can a D have when evaluating evidence of negligence:

1. actual notice 2. Constructive notice

In Wagon Mound 2, court was talking about foreseeability of the oil catching on fire as part of the proximate cause and foreseeability analysis.

Said that a reasonable engineer should have foreseen it, it was possible. If a reasonable person knows its foreseeabile, then likely he will remove it - that's breach, move to a different analysis

under contributory negligence, P's negligence is:

an intervening, superseding cause which makes the D's negligence no longer proximate.

a social guest, though invited, is not in law:

an invitee but is treated as merely a licensee.

a. Sudden emergency, b. you must not have caused emergency c. you must not have anticipated the emergency.

factors that must be in place to use the emergency defense:

A maj of courts have discarded willful and wanton as a limitation and have held that when the presence of the trespasser is discovered,

there's a duty to use ordinary care to avoid injuring him by active operations.

To state a claim for negligence, must show D had a duty, breached that duty. Judge determines whether a duty exists, jury determines when breach's occurred. When violation of statute applies, court's asked to adopt a criminal statute as the duty for that particular case. In deciding whether to adopt that statute, court must evaluate:

1) Person suing is a member of the class the statute was meant to protect 2) Damage suffered was the type of damage that the statute meant to protect 3) (OTHER POSSIBLE ONE taken from Perry) is it appropriate and wise to impose a tort liability for the violation

Ways you can be found vicariously liable:

1. respondeat superior 2. joint enterprise 3. Bailment 4. automobile guest statutes 5. Sometimes js have imposed vicarious liability on owners of private aircraft via statute, parents for certain intentional torts committed by teenage kids

implied assumption of risk ex: Rush v. Commercial Realty Co.

- Ps were tenants of D. P fell through floor. - Assumption of risk, in the sense of consent, requires actual knowledge of the particular risk, appreciation of its magnitude, and voluntary encountering of the risk. It is an affirmative defense for which the D bears the burden of pleading, production, and proof.

indemnity is also discussed in Slocum v. Donahue

Donahue was trying to say that Ford was 100% liable, and he was 0%, but court says that he can only get indemnity if there's a vicarious liability relationship between you and Ford. But think: if Ford is liable, will be liable on their own right, not bc of you; that's not possible, you're not Ford's manufacturer or employer; can't say that you're passively negligent bc of them. •Didn't meet requirements for contribution OR indemnity. Indemnity doesn't exist, contribution is cut off in this j. Note: we allow settling D to go after non-settling D for contribution, if they find out that they paid more than they had to. Doesn't go both ways.

Some js have done away with implied assumption totally, it's all considered to be comparative negligence since it all reduces P's recovery but does not destroy it.

Some js only will use assumption fo the risk if it's in a written or oral exculpatory k. but if it involves any other kind of conduct by the P, it's going to be deemed to be comparative neg, will be used to reduce her recovery. Just gotta look at the j - some still keep assumption fo the risk even if have gone to comparative negligence.

In most states, there are two statutes that regulate the recovery for the death of another due to tortious conduct: survival statute and wrongful death statute

Survival statute: says that a cause of action for pj survives the death of the P or D. Under wrongful death statute, a new cause of action is created by the death of someone due to tortious conduct of another. Aka, is created on bahlf of decedents' beneficiaries.

Kline v. 1500 Massachusetts Ave. Apartment Corp.

Tenant assaulted, robbed in common hallway of office building. Entrances were unguarded, leassor had notice of an increasing amount of assaults and robberies happening in hallways Does landlord have duty to protect tenants from foreseeable criminal acts committed by third parties? Yes. Bc landlord has control of hallways, only one with power to make the necessary precaution. Inn keepers liable for criminal acts of guests Landlord not an insurer, but does have to take precautions

In Anderson v. Sears, what the court did was say that for each element of damages, the amount the jury awarded did not exceed the max recovery. Say for each element, jury could award x, and what they did award was not over that.

The jury awards amounts based on the amount attorneys present in opening and closing statements, how much P asks for.

Traditionally, courts held that a right of action accrues immediately upon the infliction or occurrence of injury and that mere ignorance of failure of P to discover his cause of action or the subsequent resulting damage does not toll the statute. This is harsh. To counter this, courts have fashioned the discovery doctrine:

Under this, the statute doesn't begin to run til the negligent injury is, or should have been, discovered. This concept has been adopted in at least a majority of states. Some of these js limit the application of the doctrine to "foreign objects;" the majority apply it to all med malpractice cases. In Teeters case, court holds that the cause of action accrued when P found out she was pregnant.

Enright v. Groves: P was convicted of having dog not on a leash, but she was arrested for not showing her license. Police had no legal justification for demanding my license when she wasn't driving a car - so she was falsely arrested.

What's important: if they HAD arrested her for her dog not being on a leash, wouldn't have a claim for false imprisonment.

What if somebody tries to rob you with an unloaded gun, you don't know it's not loaded: assault, bc he had the apparent ability (if it'd appear to a reasonable prudent person that he has the ability to complete the activity he's threatening, that's assault).

Words alone probably aren't sufficient - there must be some act, movement that puts one in reasonable apprehension of an imminent battery. Even though action plus words can cause an imminent threat of battery, depends on the words: if they say you have til x time, if you don't do it, I'm going to hurt you." whatever they say has to be IMMINENT, can't happen in the future.

Neither private/public necessity or justification will allow for:

a protestor to trespass onto private property, or even public property, to protest. Can't destroy public or private property as part of your protest to stop some gov action like, say, the Vietnam War.

In contributory neg, P's neg will bar recovery on if it's

a substantial factor/cause in fact of bringing about the result

what are privileges:

basically defenses

Normally, the fact that a car leaves a road in a one-car crash is, in the absence of explanation, enough to make out a res ipsa case against the driver. Ways to prove breach:

circumstantial evidence (bananana peel cases), res ipsa.

second restatement of torts section 343A(1)

current duty to licensees regarding passive conditions: if a condition is obvious, if you can anticipate harm despite its obviousness, you owe them a duty or reasonable care.

Foundational rule: landowner could not be liable to a trespasser. Exceptions:

trespasser whose presence has been discovered; D liable for injury to a trespasser for conduct that's "willful and wanton;" cases involving passive conditions on the land - D must warn trespasser of latent danger; D is required to anticipate trespassers on a v limited area of land; tolerated intruders - this is sometimes explained or justified on the theory that the D's continued toleration of the trespasses amounts to permission to use th eland, so that the P becomes a licensee.

GENERAL STATEMENT: for a negligence case, P lawyer needs to say:

what D should've done to not be negligent/breach their duty.

gravity of harm (element for breach):

what kind of injury occurred or would occur

Maloney v. Rath:

• D's car hit P bc brakes failed. Three months before, mechanic had overhauled brakes. • Can the D delegate the responsibility of making a brake repair to an independent contractor • One who employs an independent contractor to maintain instrumentalities that threaten a serious risk of bodily harm is subject to the same liability for physical harm caused by the negligence of the contractor in maintaining such instrumentalities as though the employer had himself done the work of maintenance. • Aka, the fact that the brake failure was the result of the contractor's negligence is no defense for the D.

a. Can happen at any time after the judge sets the date - parties can get together themselves, set a date for mediation, set their own mediators. b. Judge can pick a mediator or serve as a mediator herself

mediation

types of economic losses:

medical expenses, lost wages, loss or impairment of future earning capacity

regarding wrongful death statutes: Some states say if beneficiary is living when files claim, and then dies, then her estate can collect. Others say

no, beneficiary has to be alive to collect.

(situation where retaining an independent contractor won't insulate employer from vicarious liability) Illegal activities:

one who contracts for performance of an illegal act is vicariously liable for any damage even if the agent is an independent contractor.

the approach to j and s liability in Coney is better known as just Several liability:

only liable for your percentage fault. On exam, won't know if j and s applies or Several liability - give both.

1. Settlement negotiations 2. Mediation 3. Arbitration

processes that take place throughout the trial:

There's three diff ways to look at comparative negligence:

pure, modified 50%, modified 49%

primary v. secondary assumption of hte risk:

primary - you assume the risk of turbulence on a plane. Primary exists mostly where there would be no liability anyway bc D didn't do anything negligent. Secondary: reasonable and unreasonable categories. Reasonable: if a mom ran into a building to save their kid. Unreasonable; you rean in to save your laptop instead of a kid.

If a co is negligent in selecting the contractor or in giving improper directions:

the co will be held liable for its own negligence, which has combined with that of the contractor to cause harm.

a. P attorney can call up insurance co. if suit hasn't been filed. If has been filed, P can call up the D attorney at any time for settlement negotiations. b. If settlement's reached, a legal k called a settlement agreement is written. If it's not adhered to, non-breaking party can sue, and there's a whole new case for a break of k. c. After settlement, another doc has to be filed for judge to dismiss the suit. Can enter into settlement negotiation anytime in the case

settlement negotiations:

duty under violation of a statute:

Duty: what the statute says you should do now determines what a reasonable prudent person would do. Duty isn't what a reasonable prudent person would do, duty is to not do x.

what to use when analyzing TTC:

Bottom of p90 (Restatement 217): use this to analyze TTC AND the paragraph on p91. Go through each one of the a, b, c, d.

Any chattel/personal property (NOT REAL PROPERTY) can be converted.

Can't convert a tree unless you've cut it down. You can convert stocks and bonds.

Regarding comparative negligence: buren of proof is on D to show that

P was negligent and that their negligence was a proximate cause of P's injuries

Fisher v. Carrousel Motor Hotel, Inc.: P was racially profiled, but not actually touched, just embarrassed. Here, intentional grabbing of his plate constituted battery.

"Damages for mental suffering are recoverable without the necessity for showing actual physical injury in a case of willful battery bc the basis of that action is the unpermitted and intentional invasion of the P's person and not the actual harm done to P's body.

compensatory damages:

Are intended to represent the closest possible financial equivalent of the loss or harm suffered by the P, to make the P whole again, to restore the P to the position P was in before the tort occurred.

sentimental ds and conversion:

For sentimental costs: give you fmv, perhaps a little bit more to take care of sentimental value (but not much)

immunities:

Immunities are conferred not bc of the particular facts, but bc of the status or position or rel of the favored D or activity. Does not deny the tort, but rather the resulting liability.

Let's suppose P's got 48%, D1 is only 2%, D3 is 50%. P would bypass both 49 and 50 percent rule - compare her negligence to both Ds.

Since didn't bypass these, can collect from the D - can collect 2% from one and 50% from the other.

two types of causation:

cause in fact and proximate cause

Loss of enjoyment of life: hedonic damages.

Same thing as loss of function or appearance. Same type of damages. Diff: some js just split them, include as a separate category of damages - get pain and suffering in two different ways.

when looking at cause in fact, cases dealing with informed consent can cause problems. Ex, patient says that had they know certain risks, they wouldn't have had a procedure

Some courts go by an objective test, aka what would a prudent person in the patient's position have decided if adequately informed, or subjective (argue that it must be subjective to protect bodily autonomy).

there are three basic ways you can be liable under joint and several liability:

1. If A is acting in concert with B 2. If an and B are negligent at the same time and their negligence creates and indivisible injury 3. When both of the Ds owe a common duty to the P - master and servant; when theire's some type or relationship, like employer and employee. When you fall in one of these three categories, both Ds are j and s liable to pay the whole. If they want a fair allocation f the money, have to sue each other either in the same case or afterwards.

There are different tests to define proximate cause (just like but for, substantial factor tests for cause in fact): (need to know all of the tests; diff js use diff tests for proximate cause)

1. Immediate or remote test 2. Eggshell rule test 3. Direct cause test 4. Public policy test 5. Foreseeability test

Willful, reckless, and wanton negligence

1. Usually used for punitive damages 2. To proceed in an action when there's a deliberate and conscious disregard for a known likelihood of harm to another 3. Ex: driving 100mph with a kid in the street, you see them, but fail to slow down. You know there's a high probability that you're going to cause great harm to him if you don't act.

privileges: affirmative defenses P can raisee. Types of privileges we've covered:

1. consent 2. self-defense 3. defense of property 4. retaking property

Self-defense: can use reasonable force to defend yourself against an aggressor. Can use force designed to cause serious harm only if the attacker is attaching you with force designed to cause serious bodily harm.

Ex, can only shoot someone if they are posing a threat of serious bodily harm or death to you. some js require you to run away before you submit them to serious harm, only after you've tried to get away can you shoot him. in other js, have stand your ground laws, which mean you don't have to run (though they still must be posing serious harm before you can use serious harm).

professional malpractice has two subgroups when committed in a medical situation:

1. medical malpractice 2. lack of informed consent

There's three ways a jury can be instructed on how to treat a violation of an adopted statute:

1. negligence per se 2. presumption of negligence 3. Proof of negligence (circumstantial evidence)

degrees of care:

1. ordinary care 2. extraordinary care

D1 gets a credit for the amount which they've settled for with D2.

Ex, if 2 pays 100k, 1 og owed 500k, will now owe 400k. sometimes the credit is percentage fault; ex, not dollar for dollar; first subtract dollars, then add his percentage fault. 500k, subtract 100k, have 400k, then end up with 320k. (80%) of it

Note 6, p225: normally, doc has a duty to disclose only to their patient bc that's with whom they have a physician-patient relationship.

i. Does doc have duty to disclose a patient's disease to a spouse? Likely, bc there's a reasonable change they'll be infected. Many jurisdictions say you have a duty to inform the patient, and as long as you inform them that they're contagious and should inform wife, that satisfies the duty. ii. Aka, to 3rd parties, depending on whether contagiousness is foreseeable, doc has duty to inform at least the patient that they can spread to someone else. If they don't disclose it to patient, patient hurts someone else, person can sue doc from not disclosing to the patient the info that would've kept them from being hurt

a. JUDGE determines the duty. Jury decides everything else. Does this via evidence, tells jury via jury instructions b. Definition of the (ordinary) duty: D had a duty to use the care of a reasonable, sensible person in a similar circumstance to guard against unreasonable harm to others Consider: If you didn't bump him into the pool, you don't have to save him. No duty if you didn't cause his circumstance

the first part of negligence, duty:

In order for rescue doctrine to apply, must meet the elements. If P acted negligently:

can't use it, look toward implicit assumption of the risk.

in wrongful death statutes, sometimes a state will cap:

non-economic ds.

The reason why we have this joint and several liability doctrine:

quid pro quo for contributory negligence. Aka, if P is any part negligent, can't collect at all from the Ds. In return, gave Ps the ability to collect all damages from one D if more than one D is acting in concert.

Cruz v. DaimlerChrysler Motors Corp.

§ FACTS: P husband and wife sued D auto dealership. Husband was cleaning a minivan he bought from D when both front airbags unexpectedly deployed, injuring him. Superior court granted D's motion for summary judgment as to all claims. Ps appealed. Ps argued that in dismissing their claim, the trial court erred in failing to apply the doctrine of res ipsa loquitur. The high court disagreed. § ISSUE: Did the trial court err in granting summary judgment? No § CONCLUSION: Accident occurred 3 years after D sold Ps the van. While the spontaneous deployment of air bags ordinarily did not occur in the absence of negligence, other responsible causes had not been sufficiently eliminated by the evidence to justify applying res ipsa loquitur. Ps' claim for negligent misrepresentation also failed because none of the evidence suggested that the statements of D's employee that the van was a safe vehicle and had no accident history were false when they were made. As the trial court properly granted D summary judgment on Ps' claims of negligence and misrepresentation, the wife was barred from recovering damages for loss of consortium.

1. Objective standard of care 2. Question of common practice 3. Emergency Standard of Care 4. Standard of care for the handicapped 5. Child's standard of care 6. Standard of care for mental illness

Considerations that fall under duty:

punitive damages:

Are an additional sum, over and above the compensation of P, awarded to punish D, to make an ex of D, and to deter D and others from engaging in similar conduct

Astensible agency:

P must rely upon the holding out of the D that they are an employee

regarding wrongful death statutes: Whether it's an adult or a kid, if beneficiary can't prove that there is no love and affection, maybe there is no ds for loss of love or companionship:

If no real relationship, you don't get those kinds of damages.

Two Ds can be a proximate case.

If you can est direct cause and proximate cause for both Ds.

Exs of instances where P has been held to be an invitee:

those attending free public meetings; spectators at public amusements; entering a bank to get change; use of state land open to te public, visitors to national parks

If an employee does something reprehensible or intentional, will employer have to pay punitive damages?

Usually not.

Comparative negligence does not prevent:

a D getting contribution from another D if the paying D has paid more than her fair share based upon comparative share - based upon pro-rata or percentage fault.

Note that the 'factors to consider when deciding if adoption of a statute is far, workable, and wise' are none of them decisive of themselves. You'll have to evaluate a bunch of them.

a. The factors go to the assessment if it's fair workable and wise to adopt a statute. This is done with the understanding that you've already gone through the first two questions - within the class and type of injury. § These two may be held as true, but court may still not adopt statute bc it's not fair workable and wise. Ie, in bar case statute wasn't adopted, but the regulation was. On exam, need to know the two primary factors, how to apply the third if need be.

damages are broken into two categories:

damages for the person's body property damages

Restatement of Torts section 221:

defines dispossession, assert that not every interference with the personal property of another is actionable and that physical dispossession or substantial interference with the chattel is required.

loss of earning capacity/loss of wages:

expert will come in, say that bar other evidence, will be an average person with an average wage. This is the average salary. (default rule for young people: average high school graduate). Experts will look at wages, inflation, average raises she'd get for next x years, add all that up, say this is the amount of money she'd make each year. Expert predicts this

Lack of informed consent ex - Scott v. Bradford

a. Facts: P operated on by D. D caused an issue, P had to have more surgeries to fix it. P said D hadn't told her the risks; said she would've refused surgery if she'd known them b. Rule: doc has a duty to inform a patient of his options and risks. If this isn't done, patient's consent is defective. If treatment's unauthorized, it's battery. c. Analysis: scope of a doc's communications must be measured by patient's need to know enough to enable him to make an intelligent choice. Aka, full disclosure of all material risks incident to treatment must be made. A risk is material if it would be likely to affect patient's decision. Here, in a patient-oriented standard jurisdiction

Pokora v. Wabash Ry. Co.

a. Facts: P, driving a car, approached a rr crossing. Couldn't see'all the tracks. Looked as well as he could, drove ahead, struck by a train of the D. b. Procedure: trial court found for D bc of contributory negligence. Affirmed by appeallate court; SC granted certiori Issue: was P guilty of contributory for not getting out of the car and look if they couldn't otherwise see if a train was coming. Court says no.

discussing negligence of evidence ex: Ortega v. Kmart Corp.

a. Facts: shopper slipped on a puddle of milk on the floor of D store. b. Issue: Can the store be held liable for damages? Yes c. Conclusion: SC held that: (1) evidence of the store's failure to inspect the premises within a reasonable period of time was sufficient to allow an inference that the condition was on the floor long enough to give the store owner the opportunity to discover, remedy it; (2) P showed D had constructive notice of the dangerous condition if he could show that the site hadn't been inspected within a reasonable period of time so that a person exercising due care would have discovered the hazard.

notes after Bundt:

a. P may bring a series of separate actions against multiple tortfeasors liable for the same damage, and take each to judgment, as long as he only collects on one b. P's entitled to only one compensation; fll satisfaction of his claim, by full payment, would prevent its further enforcement. c. Any atrial satisfaction of a claim must be credited to the other parties who are also liable

In Bartlett and Coney, negligent conduct of more than one actor combined to produce an injury to P. The negligence of each is found to be a proximate cause of the injury. The injury has therefore been treated as indivisible, and this has commonly been regarded as the third situation in which the principle of joint and several liability should be applied. This doctrine of joint and several liability og was based on two maj tort principles that have been changing:

i. contributory negligence ii. comparative negligence. Involves contribution among joint tortfeasors. A joint tortfeasor paying more than his share of P's damages may recover from other tortfeasors on either a per capita basis or a comparative fault basis.

complaint. Here, will reduce client's facts to writing, state facts, state theories of liability. Ex, Count 1 Negligence. Might have Count 2, Assault § Each count involves a different theory Complaint must be drafted, filed in courthouse, served against D. D gives to insurance co. co gives it to their lawyer.

if case's elements are est, facst are true, you'll file a legal doc called a:

a. must admit, deny, admit and deny. If doesn't know facts, can say 'neither admit nor deny.' b. Must address every count listed. c. Puts forth Affirmative Defenses i. If D can establish one of these, D basically wins

in an answer, D must:

Continuing tort:

in some professional malpractice cases, courts have found that the statute of limitations didn't begin to run until the course of treatment was complete

Types of economic damages:

past and future med expenses, lost wages, loss or impairment of future earning capacity

who can file a claim for conversion:

person who owns the property, person in possession of the property

1. disabled person's held to the standard of a reasonable, prudent disabled person in a similar circumstance. a. What if he was walking outside? Diff surroundings, riskier. Will have to go to the jury. When he's walking outside, look at likelihood, gravity, burden of precaution - same elements you'd look at in any other case. 2. What if the person is stronger, has some kind of ability through genetics that they're stronger than average person - should that be used to hold that person to a higher standard? Yes, if you have enhanced ability to hear, see, etc., you should be held to the standard of a person who has those abilities

what's the rule from Roberts v. the State of Louisiana:

How can the D that pays all the money get contribution:

1. Can bring absent D into the same suit (through third party practice; everything resolved in one lawsuit); 2. Can sue absent D in a later, second lawsuit (here, absent D gets to re-litigate the percentage of fault, his liability; much more work than just doing it all at once).

Parvi v. City of Kingston: False imprisonment - police took drunk guy to an abandoned golf course to dry out. Gives us the elements of false imprisonment:

1. D intended to confine him. 2. P was conscious of the confinement. 3. P didn't consent to the confinement. 4. The confinement wasn't otherwise privileged.

discussing evidence of negligence ex: Goodard v. Boston and Maine R.R. Co.

1. Facts: P fells on a banana peel on rr station's platform Conclusion: peel could've been left by another passenger just getting off the train.

Two different tests to est cause in fact:

i. Substantial factor ii. But for test.

Simple negligence:

1. Failure to use the care of a reasonable prudent person - ex, us driving a car Ex: driving 60 in 55mph zone

big takeaway from Reynolds v. Texas and Pac. Ry. Co.

"Where D's negligence greatly multiplies the chances of accident to the P, and is of a character naturally leading to its occurrence, the mere possibility that it might have happened without the negligence is not sufficient to break the chain of cause and effect between the negligence and the injury."

Review of Proximate Cause

1. We understand that proximate cause is a hard subject. Most of the tests are a value judgment, don't know whether they've been satisfied until the jury tells you. 2. In addition to public policy, can use a different doctrine to cut off liability, even with everything else established: shift in liability

Briefly explain the difference between the duty owed to a licensee v. an invitee

A social guest is a licensee. The property owner owes a duty to warn of passive conditions that are not open and obvious that the P doesn't know about AND that property owner does know about. If owner doesn't know, there's no duty. Regarding active conditions: possessor's duty is to not willfully and wantonly injure the P, just use simple reasonable care to guard against an unreasonable risk of harm.

intro to subject of joint tortfeasors:

A. The problem in cases just covered is if D's liability to P should be cut off even though the D's conduct was both negligent and a factual cause of the P's injury. Is there a system by which this problem can be handled and a test by which to make the determination: can be suggested that the problem can be treated in connection with any one of the four elements of a negligence case - duty, breach, causation, damages. Ex, duty approach comes in Palsgraf. But most courts have treated the question in the causation element.

In Watson, P should sue both the person who left the gas and the person who threw the match.

Look at cause in fact of both, proximate cause analysis of both separate, then assign a percentage fault to them.

Courts have held that landowners have a duty to maintain natural and artificial conditions on private land such that they don't' dangerously obscure highway visibility. In Salevan:

P contends that the D had notice of the passage of baseballs outside of its park into the street, had notice of the danger of persons using that street, and their failure to take reasonable precautions to safeguard the public was nuisance. D took insufficient precautions to protect people along the street from a knowable risk. It's generally held that an abutting owner is under no duty to remove snow and ice that falls naturally upon the road. However, fi the owner has altered the condition of the premises so that surface water or snow is discharged upon the road, he may be directly held liable for negligence or nuisance. Notably, some js have extended owners' responsibility for abutting sidewalks. Courts have held that landowners have a duty to maintain natural and artificial conditions on private land such that they don't dangerously obscure highway visibility.

public necessities (a defense):

Public necessities cover situations when the actor (could be private city or public official) a public necessity exists when an actor does an act to prevent a danger to the public. In this case, the private actor burnt someone's house down to prevent the spreading of the fire. It was a public necessity bc if it hadn't been burnt down, it would've fueled the fire to spread further throughout the city. This not only applies to fires, applies to anything that submits the public to danger. Another ex: covid. A reasonable mistake will be a defense here - if the actor reasonably believd that the destruction of the property was necessary to avoid a spreading of the calamity, he will have a defense that he made a reasonable mistake. People destroying other people's property that'll fall in public necessity defense usually doesn't happen today. Usually what'd happen is when police break down your door - happen a lot with parents whose kids are hiding drugs in their house - can tear down doors, rip mattresses

justification (a privilege/defense):

Says that in these circumstances, what you did is the only thing that you could do. In ex case from book, bus driver had a bus full of rowdy kids. Bus driver took them to the police station, kept them confined during the drive. Parents say false imprisonment, court says no, bc driver was justified in taking them to the station bc he had a duty to protect city property, and when someone destroys city property he has a further duty to help the police investigate. Best way to help them investigate was take perpetrators to them. Some states have statutes that provide a justification defense in certain situations

with respect to the shopkeepers' privilege:

Shopkeeper can detail the shoplifter to do a reasonable investigation for a reasonable period of time. This keeps keepers from false imprisonment claims, IIED claims, but the privilege is NOT a defense and won't protect the keeper from defamation claims. So if in the process of detaining or investigating a particular shoplifter, the store employees say 'this person is a shoplifter;' if another customer hears that, turns out to be false, that's the grounds for the defamation claim of slander.

(Gore factor) "The difference between the punitive damages awarded by the jury and the civil penalties authorized or imposed in comparable cases:"

The civil penalties in State Farm case = the civil fine that the state could've assessed was only like 10mill. Policy decision by legislature was the 10mill. If disparity is great between civil liberty that could've been awarded and what jury awarded, that's a strike in favor of them being excessive. So in this case, were deemed excessive; lower court later awarded just 9:1.

ordinary care:

Use care of reasonable prudent person in same or similar circumstances. Use that if you're driving your car. Taxi driver, bus driver has a higher duty in some js than you and I do.

• With trespass to chattle, nominal damages won't do: you have to show real damages. Whereas in land trespass, just show entry to the land, not damage to the land. Chattel, you need damage OR dispossession (which equals damages) before can have a claim for trespass to chattel.

With respect to trespass to land: if can show a trespass that is intentionally done, can also get punitive damages.

Rescue doctrine says that the duty owed to the car driver will be extended to anybody who tries to rescue the driver. The duty is owed to the person driving, that person's rescuer. So if you breach a duty to the driver, that IS a breach to the one who's going to rescue later. Cause in fact relates to both the car crash person and the rescuer. Proximate cause - can rely on the duty owed to the victim, a breach of that duty. But he HAS to est his own proximate cause.

a. Aka, if Suzuki was cause in fact of victim being harm's way, then that's also a breach of duty to the rescuer and a cause in fact of any injury the rescuer suffers. BUT rescuer must est his own proximate cause. If under foreseeability test, question will be if injury to this rescuer was foreseeable (to victim, yes. For rescuer, will be a jury question).

something that's extraordinary is not probable.

a. An injury is foreseeable if it could possibly occur. How do you decide if something could possibly occur - leav it to the jury.

violation of a statute ex - Perry v. S.N. and S.N.

a. Facts: suit arising out of abuse of kids at a day care center. Ds saw abuse, failed to report b. Issue: can Ps maintain a cause of action for negligence based on the Family Code, which requires any person having cause to believe a kid's being abused to report the abuse c. Rule: statute requires any person who has cause to believe there's child abuse to file a report with police. Is this statute a basis for tort liability? d. Conclusion: bc a decision to impose negligence per se could not be limited to cases charging serious misconduct, but would impose immense potential liability under an ill-defined standard on a broad class of people whose rel to abuse is extremely indirect, hold that it's not appropriate to have the statute est a duty and standard of conduct in tort.

how the arrangement of three different breach factors can affect the case's outcome:

a. If likelihood and gravity's low, burden is high, jury usually will say no breach occurred. b. Gravity of harm has a special place. Even if likelihood and burden's low, gravity can easily outweigh those c. when likelihood and gravity is great and burden is low, is going to go to jury (jury's probably going to find breach) d. If the likelihood is low but burden is great, there may be a breach. e. If life and death is involved, sometimes doesn't matter if burden's high. Even if likelihood is great, if gravity is great enough, then a jury's probably going to find breach.

Hodges v. Carter takeaways:

a. Not legal malpractice to fail to anticipate a substantial change in law requiring an overruling of current precedent. Nor is it malpractice to fail to anticipate correctly the resolution of an unsettled legal principle. b. Professionals aren't supposed to know everything, just what the ordinary member of the profession does c. In attorney negligence cases the P-client must show that but for the lawyer's negligence they would've been successful in prosecuting or defending the claim

Bc of medical malpractice concerns, res ipsa has been given special treatment in the med field. Docs, hospitals have special rules in some states.

a. Some of those rules allow the use of res ipsa only when foreign bodies are left inside of an incision or some kind of burn injury. Something like nerve damage needs an expert.

discuss negligence per se under "the three ways in which a jury can be instructed on how to treat a violation of an adopted statute:

a. Two types of negligence per se: nps with an excuse, nps with an excuse that they acted as a reasonable person. Some js don't allow for an excuse, you're just negligent. Doesn't matter if reasonable person would've violated it. b. Once a statute is adopted as the duty, a violation will be treated as negligence. If excuse is permitted, negligence will be forgotten. Jury must then look at the totality of the situation in deciding if the D acted as a reasonable prudent person. c. Only thing that matters is if, given the totality of circumstances, D did/didn't act as a reasonable person. d. In Zeni, if you were in a negligence per se jurisdiction, and she didn't have an excuse, she'd be automatically be negligent. But if you were in a jurisdiction that does allow for an excuse, her walking on the wrong side of the road is no longer going to be automatic negligence. Now jury will look at the totality of circumstances.

private necessity (contrast to public necessity):

danger is such that it doesn't expose the public to a destruction, but the problem exposes only the private person to certain destruction. In Vincent case, p140, the owner of a boat that was docked to keep it from being bounced around in a storm tied off on someone's dock and stayed there all night. During the storm, smashed against dock, hurt dock. Dock owner sued boat owner, owner's defense was private necessity - if hadn't used dock, boat would've been destroyed in the waterway. The person who tied the boat will not be prosecuted criminally, nor with they be liable for a trespass to chattel or conversion. However, whatever damages is done to the dock, the boat owner must compensate the dock owner for. Ex, if you go steal bread bc about to die, not liable for conversion or trespass of chattel, you will have to pay the price of the bread. •Private necessity, you won't be technically liable for a tort. What you will be required to pay, though, is the fmv of the thing that you used and/or any damage you caused to it.

Restatement 217 comment e:

defines physical intermeddling with chattel as follows: intentionally bringing about a physical contact with the chattel.

damages, namely compensatory and nominal damages, can be put under these categories:

economic non-economic

Murrell v. Goertz Intentional liability for intentional torts: normally, the employer is not vicariously liable for the intentional torts of the employee, even when they're operating in the scope of employment. There are just exceptions - ex if the employee is furthering the employer's business through the commission of the intentional tort

ex: at gas station, drive off without paying, you resist, he punches you - that's intentional battery. He was furthering service station's business by making people pay. Also liable to if he tells employee to hit people who run off, affirm me for doing so) Note 4: what about punitive damages? The employer is liable for compensatory damages, but what about punitive damages? There may be a different rule. Depends on whether owner ratified the act, was negligent in hiring

income tax and damages:

fed gov tax code does not count compensatory damages as income. Not taxed. But once you invest it, get interest on it, will have to pay taxes on that. punitive damages ARE considered to be taxable, though.

1. Duty 2. Breach 3. Causation 4. Damages

for a case to be well-grounded in negligence, there must be four elements:

factors that should be considered in likelihood of harm (breach element)

foreseeability, forewarning, instrumentality of item in question (is the item very dangerous)

Full satisfaction v. partial satisfaction:

get the full amount of your damages. Means you can't collect any more money from any Ds; automatically releases any other Ds. Collecting the total amount of judgment from one D releases all the other Ds A settlement is often a partial satisfaction. Aka, I settled the case for only part of the money, I can get the other part from the other D, provided that the j doesn't say a settlement is a release of all other Ds. Bottom line: don't wanna release people you don't wanna release. do want to settle. Gotta know what the rule is in your j. if intend to settle with only one, call it a partial satisfaction, say I should be able to pursue the other D for the money. But if your j says even a partial settlement releases all Ds, you don't need to settle unless you can get both to settle at the same time.

What's the duty that's owed to a licensee:

go back to passive and active conditions. Has a duty to warn him of any latent passive conditions that are known to the owner and unknown to the licensee. Active conditions, like shooting a gun or riding around the house wildly: you have to not wantonly and recklessly injure them (wantonly and recklessly: morphed from intentional behavior to negligence - will be liable for simple negligence, failure to not use reasonable care. That's what the notes say. Use reasonable care in your behavior to licensee).

Tolling the statute of limitations:

if P is a minor, temporarily incompetent bc of memory loss, come, etc, temporary insanity; fraud (if doc commits malpractice on day 1, day 10 you say there's pain, he says I treated you right, that pain is normal, fraudulently hid it from you - accrues on day that you discover the fraud;

statute of limitations and continuing torts:

if a doc continues to treat you, and he had committed malpractice on day 1, but continues to treat you for two years, when does the cause of action accrue? Injury, or day he stops treating you? if he's treating you for the same condition, continues to commit malpractice, it's a cont tort, cause of action accrues on day he stops treating you.

Joint tortfeasors - two or more persons whose negligence at some point created an indivisible injury to the p. acts combined to create an indivisible injury in the P.

indivisible means you can't tell which D caused which aspects of the injury. If there's no way to see which D caused the harm, can't allocate which caused which part of the injury. If, in contrast, one car runs over an arm, the other the leg, then it is possible to divide up between Ds. But if you can't divide it, both Ds will be liable for the injury

in causation, jury really doesn't have to be told what foreseeability is

just be told that that is the test, then lawyers can make the argument that it does or does not exist

3 factors to consider when deciding breach:

likelihood of harm, gravity of harm, and burden of precaution

what is slight negligence:

many js just don't want to put a percentage contribution and assign that to P

normally in a wrongful death claim,

one must be the rep of the other beneficiaries. Ex, one can file on behalf of themselves and the decedent's kids. If a sibling files, files on behalf of other siblings, you'd better give them notice you're representing them and what, if anything, you win.

In some js, the licensee category is broken down into two subgroups:

ordinary licensee; "bare" licensee. Bar licensee include salespersons, canvassers, social visitors who drop in without express invitation. The duty owed to bare licensees is less than that owed to ordinary licensees

Non-economic damages:

physical pain and suffering, mental anguish, loss of function or appearance, emotional distress from legal malpractice, litigation-induced stress, loss of enjoyment of life

The D has the burden of prof for both contributory and comparative negligence

show that P had a duty owed to herself to protect herself from injury. She breached that duty by not doing something a reasonable prudent person would not do, and that action was a cause in fact and proximate cause of her own injury. D must prove those things, duty, breach, causation, by a preponderance of the evidence.

Single v. Dual intent:

single - all a person has to intend is to touch. If that exists, has purpose to touch or with the substantial certainty that touching occurs, if touching occurs, battery occurs if the touching is harmful or offensive. Dual intent: the person has to not only intent to touch but 2. Has to intend that the touch is harmful or offensive. Depends on the j which type of intent is used to constitute battery. Wagner is a single intent j.

Maj of states have modified the common law collateral source rule

some apply only to particular causes of action, like med malpractice, others say that info about the collateral source is admissible in evidence.

Diff js have diff standards on what you have to show to get IIED:

some require physical damage, others require experts to say that you suffer emotional damage, others just listen to what you have to say

public policy test (for proximate cause): ex is Kelly case

superceding and intervening causes aren't really another test, are something that under direct cause test, foreseeability test, or immediate and remote teset may cut of proximate cause. If superceding, cut it off; if just intervening/foreseeable, don't cut it off). Next test is public policy

To have an intentional tort be actionable, have to show

the actor did the act for the purpose of causing the tort OR with substantial certainty that the tort would occur.

the benefit of having a public necessity defense:

the actor does not have to pay compensation to the person's whose property is destroyed. Some states have statues or ordinanaces that will sometimes make the state responsible for paying for certain destruction that's caused by a public necessity. So maybe the actor doesn't have to pay - maybe there's a city ordinance that'll allow you to make a claim with the state or city, have them pay for the destruction of the property.

a. Types of causation i. Cause in fact § Ex: I'm in my back yard, barbecuing, some fire gets out and burns down neighbor's house. If not for your actions, houses wouldn't have burnt down ii. Proximate cause Based upon foreseeability. Ex, a house burned down because a tornado blew your CONTAINED fire towards it (beyond a certain line, liability must stop).

the third element of negligence, causation:

There's going to be some subjectivity in play, even in an objective standard of informed consent:

what kind of discussion would a reasonable prudent doc give to someone who is an immigrant, mentally handicapped, etc. If reasonable doc would have gotten an interpreter, and this doc didn't, might've been breach. Still an objective standard

Apparent authority: if you're in the hospital and a doc comes up, you don't know who he's employed by. If the hospital holds out the doc as being its employee, and if the patient relies on that holding out, the doc will be deemed to be an employee, hospital will be vicariously liable for doc's negligence. If hospital, doc put themselves out as hospital's employee, that's enough to est a employer-employee rel, even though tey may be an indep contractor in reality. Sensible v. apparent agency:

with apparent agency, there needs to be some kind of affirmative statement by the hostpial or doc that doc is an employee of hospital "our docs are the best in town," etc.

Restatement section 222A: what facts influence courts in determining if conversion has taken place where there's been interference with a chattel:

"1. Conversion is an intentional exercise over a chattel which so seriously interferes with the right of another to control it that the actor may justly be required to pay the other the full value of the chattel. 2. In determining the seriousness of the interference and the justice of requiring the actor to pay the full value the following factors are important: A. The extent and duration of the actor's exercise of dominion or control B. The actor's intent to assert a right in fact inconsistent with the other's right of control C. The actor's good faith D. The extent and duration of the resulting interference with the other's right of control E. The harm done to the chattel F. The inconvenience and expense caused to the other

Conversion - Pearson v. Dodd: P loses chattel which he rightfully possessed, and D finds it, converted it to his own use. The most distinctive feature of conversion is its measure of damages, which is the value of the goods converted. Not every wrongful interference with the personal property of another is a conversion. Where the intermeddling falls short of the complete or very substantial deprivation of possessory rights in the property, the tort committed is not conversion, but the lesser wrong of trespass to chattels. Restatement has marked the distinction by defining conversation as:

"An intentional exercise of dominion or control over a chattel which so seriously interferes with the right of another to control it that the actor may justly be required to pay the other the full value of the chattel. Less serious interferences fall under the def of trespass to chattels. This diff is more than just semantics. The measure of ds in trespass is not the whole value of the property interfered with, but rather the actual diminution in its value caused by the interference. A judgment for conversion can be obtained with only nominal damages, whereas liability for trespass to chattels exists only on a showing of actual damage to the property interfered with. The general rule has been that ideas or info aren't subject to legal protection, but the law has developed exceptions to this rule. Where info is gathered and arranged at some cost and sold as a commodity on the market, it's properly protected as property. where ideas are formulated with labor and inventive genius, like with books or research, they're protected. Conversion may occur when one who's authorized to use chattel uses it in a manner exceeding the authorization

Restatement 434:

"One who by statute or by administrative regulation is under a duty to provide specified safeguards or precautions for the safety of others is subject to liability to the others for whose protection the duty is imposed for harm caused by the failure of a contractor employed by him to provide such safeguards or precautions."

Elements of trespass to chattel:

"One who commits a trespass of chattel is subject to liability to the possessor of the chattel if, but only if: 1. He dispossesses the other of the chattel, or 2. The chattel is impaired as to its condition, quality, or value, or 3. The possessor is deprived of the use of the chattel for a substantial time or 4. Bodily harm is caused to the possessor, or harm is caused to some person or thing in which the possessor has a legally protected interest.

Regarding the duty of landowners to people outside the land:

"We think that, except for extreme situations, the question of the landowner's or possessor's attention to the condition of his roadside trees under a general standard of "reasonable care to prevent an unreasonable risk of harm" is to be decided as a question of fact upon the circumstances of the individual case." With regard to most conditions on land that arise in the state of nature, most courts have held that there is no duty upon the landholder to protect persons outside the premises.

regarding intentional infliction of emotional distress:

"Whether an actor's conduct is extreme and outrageous depends on the facts of each case, including the relationship of the parties, whether the actor abused a position of authority over the other person, whether the other person was especially vulnerable and the actor knew of the vulnerability, the motivation of the actor, and whether the conduct was repeated or prolonged." Aka: P's sensitivities may be a factor in deeming D's conduct extreme and outrageous. Ex, Police act cruel to someone who just was raped

Restatement 423:

"one who carries on an activity which threatens a grave risk of serious bodily harm or death unless the instrumentalities used are carefully maintained, and who employs an indep contractor to maintain such instrumentalities, si subject to the same liability for physical harm caused by the negligence of the contractor in maintaining such isntrumentalities as though the employer had himself don the work of maintenance"

Intentional infliction of emotional distress: State Rubbish Collectors Ass'n v. Siliznoff: "when one intentionally subjects another to the mental suffering incident to serious threats to his physical well-being, whether or not the threats are made under such circumstances as to constitute a technical assault."

"one who, without privilege to do so, intentionally causes severe emotional distress to another is liable for such emotional distress and for bodily harm resulting from it." Common carriers and innkeepers have been held to a higher standard of conduct and sometimes held liable for using insulting language to their passengers and patrons

a restatement of common law doctrine of joint and several liability:

"the common law doctrine of joint and several liability holds joint tortfeasors responsible for the P's entire injury, allowing P to pursue all, some, or one of the tortfeasors responsible for his injury for the full amount of the damages."

Cole v. Turner: Provides a def of battery. This is the old definition. Wallace v. Rosen provides another def. Restatement gives another def: touch just has to be offensive or harmful. Another thing the restatement does: has some element of transfer of intent in it - aka if you intend assault but commit battery, will be liable for the battery. P38. Doesn't require anger or insolence. In Wallace case, there is dispute about whether she intended to push the woman down the stairs. Most judges will let this go to the jury about whether the touching was harmful or offensive. Most js use the Restatement rule, not the old rule. The old rule:

"the least touching of another in anger is battery. If two or more meet in a narrow passage, and without any violence or design of harm, the one touches the other gently it won't be battery. If any of them use violence against the other, to force his way in a rude inordinate matter, it is a battery; or any struggle about the passage, to that degree as may do hurt, is a battery." ALT: "battery is the knowing or intentional touching of one person by another in a rude, insolent, or angry manner. Any touching, however slight, may constitute an assault and battery." Restatement section that provides NEW def of battery: section 18 - just requires harmful or offensive contact

Cole v. Turner:

"the least touching of another in anger is battery; if two or more meet in a narrow passage and without any violence or design of harm if one touches the other gently it's not battery; if any of them use violence against the other to force his way in a rude way, perhaps, it's battery."

In Osborne, what was the risk the statute meant to prevent - people being injured by the poison by:

(IMPORTANT) not knowing it's poison. Meant to guard against poison-caused injury. a. What about if woman gets startled by the poison, falls and hits her head - does that injury fall in the realm of injuries that the statute was meant to prevent? Logical argument is that this poison's labeling was specifically meant to prevent against ingesting it. b. WHAT INJURY IS MOST LIKELY TO OCCUR if statute isn't followed. Look at legislative history, title of statute to get a hint about this. Look at the intent of what kind of injury is mean to be guarded against. On exam, will just give you a statute, will have to argue both ways the person was/was not in the class meant to be protected Say that it would depend on legislative history on exam - say that P has asked judge to adopt x statute, judge may adopt this as the duty. Will require on these two factors. Whether these factors apply will apply on title, legislative history.

McCoy v. American Suzuki Motor Corp. - The Rescue Doctrine

1 "Danger invites rescue." The initial tortfeasor is Suzuki. They produced a defective car that, while one was driving, went off the road. P helped them out, but somebody comes and hits him. he wants to sue Suzuki - wouldn't have been there if their care wasn't defective. 2 Suzuki, in making the car, owed a duty to the driver of the car to produce a car that wasn't defective. Breached the duty when they put the car on the market. When the car ran off the road, the negligence caused in fact the accident.

Fuller v. Preis - intervening cause ex

1 Here, P was his own intervening cause - he blew his brains out. D drove into his car, then sometime later, bc of head injury, he has an irresistible impulse to commit suicide. a. D says P was his own superseding, intervening cause. It was intentionally done, should cut off D's liability from the accident. In this j, the court says no. if you negligently cause and accident, dead person from that accident develops an impulse to kill himself, you're going to be responsible for his death. b. In this j, court says we have a new rule to deal with suicides - their acts won't cut off og tortfeasor's negligence IF the og negligent act creates a condition in the P that gives the P an irresistible impulse to kill themselves. That's a minority rule. c. The majority rule is that irresistible impulses WILL cut off D's negligence. If dead person KNOWS what he is doing, if he plans it out, the fact that he can't control himself is irrelevant, and D's og negligence is cut off. In most js, if he knows what's going on, it'll cut off og negligence. Diff js have diff rules.

Derdiarian v. Felix Contracting Corp.

1 Hole in the middle of the road. People in this hole, fixing gas main. Negligent act: not putting barricades on the road to not keep cars from driving into the hole, not having flagman to lead cars around the hole. What's the risk when you don't have those things: a car may drive into the hold negligently and injure someone in it. a. D says this was a freakish accident, unforeseeable. Court says no, the freakish nature of the accident doesn't cut off liability if the injury that occurred is in the risk of harm created by the negligent behavior. What's in the risk of harm here: that a car will go into the hole, hurt somebody in it. doesn't matter the injury happened in a freakish way instead of guy just getting smashed. What does matter is if the person harmed was within the risk of negligent behavior. If so, it's foreseeable, and proximate cause is est

Kelly v. Gwinnell is an ex of the public policy test.

1 Public policy is a separate proximate cause test: says proximate cause doesn't exist bc public policy dictates that D not be liable for this type of injury.

In Sindell, D was negligent in putting a defective drug on the market. P was innocent. Burden is going to be kinda shifted to them to exonerate themselves. Depending on the j, the Ds each can come forward with evidence to prove that it wasn't their pill. If can prove that, then they don't have to pay anything

1 This is a cause in fact case. a. Manufacturers of the drug had a duty, they breached that duty in manufacturing the drug; did it in a negligent way. Question becomes whether the P can show that a particular manufacturer was the cause in fact of the injury. Court came up with market share liability idea.

Daubert v. Merrell Dow Pharmaceuticals, Inc.

1 Two minors sue, claim they have birth defects bc their moms took a drug made by the D. question: whether certain expert scientific testimony is admissible to prove that the drug actually caused the birth defects. Causation can be proved even when we don't know precisely how the damage occurred, if there's sufficiently compelling roof that the agent must have caused the damage SOMEHOW 2 One method of proving causation in these circumstances is to use statistical evidence. If enough people who eat a food get sick, the food probably caused it.

Kelly relates that for proximate causes reasons, won't hold social host liable (note that majority opinion says yes, they're liable, though. Dissenting opinion says proximate cause shouldn't be est - use some policy reasons. Say that unlike a bar, social host might not know when a guest is drunk. And what can they do to control him? He might threaten them. What do you expect host to do to stop a guest from leaving?

1 Universal rule: if you serve liquor to a minor, you will be the proximate cause of any accidents he causes. But for adults, we say that adult negligence in drinking too much and driving IS the superceding, intervening cause that cuts off any liability the social host has. 2 Employers can be liable for serving liquor at employee functions, events that happen on the job. Depends - some js treat them just like social hosts. What about bars: different rules. Dram laws. Statute gives the person injured a claim against a bar bc bar is a commercial establishment. Even when there's not a statute, some js will say that common law says it's ok to hold them liable.

Summers v. Tice gives us an alternative way to prove a cause in fact:

1 when two parties are negligent, one of them causes an injury to P, we don't know who did it. ordinarily, P'd have to prove it. but bc she's innocent, Ds are negligent, going to switch burden of proof of causation to you two. If impossible to prove who it was, each going to be jointly and severally liable to all the damages. When two parties are negligent to the P, but only one injured to P, burden will shift to the parties for them to prove by a preponderance that they were not the cause in fact. If can't prove, either one or both of them will be liable. They're both liable 100%, but each will probably end up paying half. But if one doesn't have money, P can collect all from the other.

In Smith v. Providence Health and Services, court holds that trial court erred in dismissing P's claim. CAN recover from loss of opportunity. This case describes three ways that states have responded to a situation where a D's medical malpractice has reduced patient's chance of recovery or of a better outcome:

1) A few js relax the trad requirement and permit the patient to recover as long as the lsot chance was a substantial factor in producing the harm. 2) Some js view the patient's injury as the loss of the opportunity for a better outcome, rather than as the death or worse outcome. In those js, patient's permitted to recover damages but only a proportion of the full amount. 3) Js continue to require the trad causation requirement. "in an action alleging med malpractice, the P can't recover for a loss of an opportunity unless opportunity was greater than 50%"

When you're able to est the res ipsa elements, case will go to jury. But judge will give further instruction, will go 'if you find D was in possession, and that this is the type of accident that ordinarily doesn't occur without D being negligent, this is what you must do with that finding:" aka, that finding raises one of three effects:

1) An inference of negligence 2) A presumption of negligence which requires the jury to find negligence of D doesn't produce evidence sufficient to rebut the presumption. i. If judge tells jury that shall presume that D was negligent unless they have an excuse - if excuse is given, then presumption evaporates, must look at totality of circumstances, see if D more likely than not was negligent 3) It not only raises a presumption but also shifts the ultimate burden of proof to D and requires him to prove by a preponderance of all the evidence that the injury wasn't cause by his negligence.

McCoy v. American Suzuki Motor Corp.: Brief

1) FACTS: claimant was driving when the car which preceded him swerved off the roadway and rolled. The claimant stopped to render assistance, and at the direction of a police officer placed flares to warn approaching vehicles. Concerned that the flares were insufficient, the claimant continued further up the road, and, using flares, manually directed traffic to the inside lane. While walking back to his vehicle the claimant was struck by a hit-and-run vehicle. The claimant brought the products liability claim against the corporations, which manufactured the vehicle that rolled. The claimant based his claim upon the rescue doctrine alleging that he was injured while a rescuer. The corporation moved for summary judgment, stating that 1) the rescue doctrine does not apply to product liability actions; and 2) even if it does, the claimant must still prove proximate cause. The trial court granted the motion and dismissed the action. 2) ISSUE: Is the rescue doctrine applicable to product liability actions, and if so, is the claimant still required to prove proximate cause? Yes 3) CONCLUSION: court concluded that the rescue doctrine did apply to product liability actions. However, the rescue doctrine did not relieve him of the burden of proving proximate cause. A rescuer shows the defendant proximately caused his injuries is in keeping with general principles of liability. Finally, the issue of whether the third vehicle striking the claimant was an intervening cause was one for the jury.

In Daubert, SC lists several factors judges can consider in determining whether to admit expert scientific testimony:

1) If the theory or technique employed by the expert is generally accepted in the scientific community 2) whether it's been subjected to peer review and publication 3) whether it can be and has been testsed; whether the known or potential rate of error is acceptable

In Smith v. Providence Health and Services, we're given three elements/practice concerns to detail a theory of recovery:

1) P must est that they lost a substantial chance of a better medical outcome due to the D's medical negligence. 2) P must plead with specificity the lost chance of a better medical outcome - aka, P must plead the percentage and quality of their loss of chance. 3) P has suffered the physical harm that he might well have avoided had he received proper medical care. P who demonstrates that a doc's negligence reduced his chance of a good outcomes from 33 percent to zero percent could recover damages, but only to the possible extent of the 33 percent of damages resulting from adverse med outcome.

Elements of rescuer doctrine used by this j:

1) The D was negligent to the person rescued and such negligence caused the peril or appearance of peril to the person rescued 2) The peril or appearance of peril was imminent 3) A reasonably prudent person would have concluded such peril or appearance of peril existed; 4) The rescuer acted with reasonable care in effectuating the rescue.

Note Yabara case. If P can est elements of res ipsa, jury's going to be told:

1. 'if you find that these elements are in play, you shall find D was negligent unless D can show by a preponderance of evidence he wasn't negligent.' In a fifty-fifty case, it matters who has the burden.

Three ways that states have responded to the situation where a D's med malpractice has reduced patient's chance of recovery or of a better outcome:

1. A few js relax the trad requirement and permit the patient to recover as long as the lost chance was a substantial factor in producing the harm. 2. About half the states have viewed the patient's injury as the loss of opportunity for a better outcome, rather than as the death or worse outcome. In those js, patient is permitted to recover damages, but only a proportion of the full amount. 3. Many js continue to require the trad causation requirement.

222A also gives the ways in which an actor may convert a chattel, ie intentionally exercise dominion and control over it so that seriously interferes with the owner's right to control it that it's just to require the actor to pay its full value:

1. Acquiring possession of it, eg stealing the chattel 2. Damaging or altering it, eg intentionally running over an animal and killing it 3. Using it, eg a bailee seriously violates the terms of the bailment 4. Receivint it, eg obtaining possession after a purchase from a thief 5. Disposing of it - eg, a bailee wrongfully sells the chattel 6. Misdelivering it, eg delivery to wrong person by mistake so that the chattel is lost 7. Refusing to surrender it, eg bailee refuses to return the chattel

a. Usually filed once discovery finishes, but CAN be done beforehand b. Motion says: "we've done discovery, but are no facts that show P had damages, so case should be dismissed for an inability to show damages." c. Filer says: "there's no genuine issue of material fact, thus we're entitled to judgment as a matter of law." i. In this motion, no witnesses testify; judge only looks at written docs attached as exhibits to the motion. This is why it's done after discovery - so there's evidence to present.

1. After discovery, one partly, likely D, will file a Rule 56 motion for summary judgment

The only tool at the jury's disposal for "making the P whole again" is money.

1. All losses, even non-economic ones, must somehow be translated into money damages. 2. ALSO: all damages must be included in one lump sum award, on the particular day the verdict is returned, in that single lawsuit that the P's permitted to bring. a. Most js allow new trials only if the damages awarded are so high or low that they shock the conscience.

Notes following Elbaor:

1. Alt names for Mary Carter agreements: Gallagher covenants, Loan Receipt Agreements, High/Low agreement 2. As the dissent here pointed out, most js permit the agreements as longa s they are disclosed to the court and the other parties A few, like TX, have found mary Carter agreements void as against public policy even when disclosed.

Four elements necessary to prove existence of a joint venture/enterprise:

1. An agreement, express or implied, among the members of the group 2. A common purpose to be carried out by the group 3. Community of pecuniary interest in that purpose, among the members; 4. An equal right to the voice in the direction of the enterprise, which gives an qual right of control

Elements of a joint enterprise implies a pecuniary enterprise in the venture (Not so for joint venture). Elements for joint enterprise:

1. An agreement, express or implied, among the members of the group 2. A common purpose to be carried out by the group 3. Community of pecuniary interest in that purpose, among the members; (who shares in the profits - in Popejoy, only the daughter) 4. An equal right to a voice in the direction of the enterprise, which gives an equal right to control. Element three: have to have pecuniary interest - this is why hunting trips, beach trips, trips for pleasure don't count as joint enterprises. HOWeVER: may be jointly and severally liable. In Popejoy, this court says the most important element is common pecuniary interest.

gross negligence:

1. An extreme departure from the ordinary duty to use reasonable care 2. Ex: driving 80 in a 55mph zone 3. willful, reckless, wanton care - conscious disregard for high probability of harm to another.

Children have been given special protection. More is required of a possessor of land regarding kids (Possessor of land: the owner of the property; the person who's in possession of the property. Two categories. P can sue both). Three approaches to this:

1. Attractive nuisance doctrine: the old rule regarding kids. This was changed by restatement, that many states have adopted. The old rule is still being used in some states, though. Attractive nuisance - if something on premises attracts kids, lures them onto the property, if it is dangerous to the kids, the owner had a duty to use reasonable care to guard against harm to the kids. Under this old rule, the thing had to be attractive - if kid just inadvertently there, the doctrine didn't apply, no duty owed to kids. 2. Second Restatement section 339 thought that was harsh, say it doesn't matter if the kid is lured there or not, they're there, they're injured, so came up with "Artificial Conditions Highly Dangerous to Trespassing Children" - "a possessor of land is liable for physical harm to kids trespassing caused by an artificial condition if the place where the condition exists is one upon which the possessor knows or has reason o know that kids are likely to trespass, and the condition is one of which the possessor knows or has reason o knows will involve an unreasonable risk of death or serious harm, and the kids do not discover the risk in meddling with it, and the utility to the possessor of maintaining the condition and burden of eliminating the danger are slight as compared with the risk to the kids involved, and the possessor fails to exercise reasonable care to eliminate the danger or otherwise to protect the kids. • Duty to kids: use reasonable care to eliminate the danger or otherwise to protect the kids. • Diff between this and the railroad case we had, where got his foot caught (no duty til he's discovered): here, a duty is owed to kids even before they're discovered 3. P559: section 51 of third restatement has replaced the earlier section on kids. Third gets rid of all these diff condition - duty is to use reasonable care to guard against an unreasonable risk of harm to trespassing kids. That means you gotta inspect, make things safe through either warning or correct. Aka, same duty that's owed to an invitee. !!! the factors under restatement second are still relevant to restatement third bc though they're not preconditions, they are just relevant to determine if the duty of reasonable care was breached. Aka, relevant to whether a breach occurred, NOT whether a duty was owed. • Some states still use the second restatement - gotta see what your state uses. • How define 'kid?' Look in your j, see the ages to which the restatement has been applied. Is there in some cases that this rule only applies when a kid is around 4 or 5. There may be some norm in the case law; alternatively, can argue that a 13-year old knows and appreciates the risks involved • Third restatement says that it doesn't matter if it's an artificial or natural condition that hurt the kid - same duty to use reasonable care applies. Restatement second is limited to artificial conditions, though - third doesn't.

a. Judge will decide which issues will be presented to jury, which witnesses will testify, which exhibits can be presented, etc. b. Out of this conference comes the Final Pretrial Order - final controlling doc for the trial, states what the sides can and can't do

1. Between sj motion and trial, will be a final pretrial conference

Retaking property: ex is that you go in a store, buy something on credit, lie about credit score, they discover before you leave that you lied, they jump in the car and chase you down the road, try to wrestle the good away from you - can use reasonable force to take property away from you if they're in hot pursuit. If you go home, lock your door, they can't. Ex: Bonkowski v. Arlan's dept store: store has a privilege that if they suspect you inside or outside the store of having stolen something, can come to you and do a reapsoanble investigation, if necessary detain you wihle they do it. both the investigation and detention must be reasonable, and they must have probable cause to believe you have shoplifted from them. Shopkeeper's Privilege. Elements:

1. Can only use reasonable force if you've demanded they return the property, and they refuse. 2. Have to be in hot pursuit. BUT if during pursuit they fight you, you can use reasonable force to defend yourself

Harris v. Jones: Employer ridiculed employee bc of speech impediment. Gives us the elements of emotional distress:

1. Conduct must be intentional or reckless. 2. Conduct must be extreme and outrageous. 3. There must be a causal connection between the wrongful conduct and the emotional distress. 4. The emotional distress must be severe

in loss of chance cases, we change the damages and WHAT you have to prove more likely than not so the docs don't get away scott free:

1. Damages- we give you ds not for the value of your life but your loss of chance. How do we calculate that? Courts take two approaches: a. If you can show you lost some chance, we'll give you the full value of your life. Normally happens in cancer cases b. Percentage value of your life (that you lost) if you had thirty percent chance of living, lost twenty, multiply twenty percent times the value of your life 2. If you're a cancer patient, you don't have to show more likely than not you would've lived. Just show more likely than not you suffered a loss of chance (NOT that more likely than not you would've lived. THAT IS WHAT we have to prove more likely than not - show more likely than not that bc of doc's negligence you suffered a loss of chance. 3. Other states, like MS, vary from this, say that you have to show more likely than not the loss of chance was the cause of the death or injury.

In State Farm case, what is the standard the judge lays out to determine when it is and is not excessive: the three Gore factors (Just like three elements to res ipsa, violation of a statute, there are three factors that one looks at to evaluate whether an award of punitive damages is excessive or not)

1. Degree of reprehensibility of the D's misconduct 2. The disparity between the actual or potential harm suffered by the P and the punitive damages award 3. The diff between the punitive damages awarded by the jury and the civil penalties authorized or imposed in comparable cases.

Smith v. Providence Health and Services

1. Docs didn't take proper steps to follow up on his complaints of stroke symptoms, P argues argue that this lost him a chance for treatment that in some cases provides patient with no or reduced complications following stroke. Question is whether OR law permits a P who's suffered an adverse medical outcome to state a common law medical negligence claim by alleging that the D negligently caused a loss of his or her chance of recovery. D says P failed to allege it was more likely than not that the delay in treatment, rather than his underlying condition, caused his permanent disability.

requirements for a violate of a statute:

1. Does P belongs to the class that statute was intended to protect 2. does the P's injury is of a type that the statute was designed to prevent. 3. is it fair/appropriate/wise to adopt this statute.

Imputed contributory negligence has been largely rejected in these areas:

1. Driver and passenger 2. Husband and wife 3. Parent and child 4. Employer and employee 5. Derivative claims - where a claim is held to be derivative in nature, aka loss of consortium or wrongful death, the contributory negligence of a n injured party usually will be imputed ot the P bc P's claim derived from that of the injured party.

violating a statute

1. Duty is to do what the statute says. If you don't, there's breach. Duty of reasonable person no longer in play. 2. Breaking the statute itself constitutes negligence 3. Court will state affirmatively that the duty is to do x, instead of saying that a reasonable prudent person would do in same or similar circumstances. Breach occurs if they didn't do those things.

Breakdown of Perkins v. Texas and New Orleans R. Co.

1. Engineer on the train was speeding - was negligent, had duty to drive the speed limit. Duty is to drive 25 mph. engineer was acting negligently. Couldn't est causation, though. Couldn't prove that the speeding of the train caused the accident - court said accident would've occurred even if they were driving 25 mph. the speeding didn't cause the accident.

extraordinary care:

1. Ex: hotel window up, toddler in the room. Baby falls bc screen wasn't fixed to the frame. With respect to the hotel/common carrier, court said the hotel, being open to the public, have a greater duty to make certain that screen is affixed so people won't fall out. a. Mom had a duty to act as a reasonable prudent parent. Since hotel was a common carrier, had a greater duty. Extraordinary care means they have to anticipate what might happen. 2. Even when you're not a common carrier, some acts are so dangerous that anybody's liable - ex, person carrying a firearm has duty to use utmost/extraordinary care to guard against harm; same with mug of nuclear waste to class. Matters if act's inherently dangerous. 3. Many js are getting away from degrees of care, say that everybody has duty of a reasonable sensible person in same or similar circumstances - circumstances change, but not the duty. 4. Good way to distinguish between ordinary care v. extraordinary care: care you'd take driving your friend to the store v. police officer with a cocked gun

Slocum v. Donahue brief

1. FACTS: Driver pleaded guilty to a motor vehicle homicide in the death of the parents' baby. Before the civil trial instituted by the parents against the driver, the parents signed a settlement agreement with the car maker that gave the car maker a release of liability on any claim. The driver had filed a third-party complaint against the car maker for negligence and breach of warranties of merchantability and fitness for particular purpose. The car maker filed a motion for summary judgment against the driver's claims on the grounds that the settlement with the parents was made in good faith and under Mass. Gen. Laws ch. 231B, § 4, the car maker was released from liability for contribution to the driver. The trial court granted summary judgment to the car maker. 2. ISSUE: Was the driver entitled to indemnification from the car maker? No 3. CONCLUSION: The court affirmed the summary judgment against the driver. It held that there was no evidence of bad faith in the negotiation of the settlement. It further held that when a release is given in good faith to one of two or more persons liable in tort for the same injury it shall discharge the tort-feasor to whom it is given from all liability for contribution to any other tort-feasor.

Knell v. Feltman brief

1. FACTS: Evelyn Langland and her husband were guest passengers in an automobile owned and operated by Kenneth E. Knell. The car in which they were riding collided with a taxicab owned by Ralph L. Feltman and operated by his employee, as a result of which Mrs. Langland was seriously injured. She and her husband sued Feltman to recover damages. After answering, Feltman filed a third-party complaint against Knell, asserting the collision was caused by the contributing or sole negligence of Knell. However, Knell denied this. The court held that contribution was enforceable regarding a tort committed by the concurrent negligence of two or more persons who were not intentional wrongdoers, that judgment against both was not a prerequisite, and that the personal negligence of each was immaterial. 2. ISSUE: Was District of Columbia correct in stating there can be no contribution between concurrent tort-feasors unless P previously has obtained a judgment against both, and unless both were 'vicariously' negligent, i.e., liable under the doctrine of respondeat superior? Yes 3. CONCLUSION: The court concluded that when a tort is committed by the concurrent negligence of two or more persons who are not intentional wrongdoers, contribution should be enforced; that a joint judgment against such tort-feasors is not a prerequisite to contribution between them, and it is immaterial whether they were, or any of them was, personally negligent. In other words, we adopt for the District of Columbia, without exception or reservation. It is true that Feltman charged the Langlands with contributory negligence, although Knell did not. But no evidence of such negligence on their part has been pointed out by the parties, so the court assumed that there was none.

Bruckman v. Pena brief

1. FACTS: Pena was injured in a car accident. About a year later, he was yet again injured in a second collision, resulting to an aggravation of certain injuries that he had sustained in the first collision. An action was thus commenced, and the only Ds named were the owner and driver of the truck involved in the first collision. The lower court in favor of Pena and his parents. Thus, Bruckman sought a reversal, asserting that the trial court erred in one of its instructions to the jury - esp the instruction that if an apportionment of the injuries was not possible, then they would be liable for the entire disability. 2. ISSUE: Did the trial court err in instructing the jury that one driver was liable for all the injuries sustained when two vehicles were involved in the passenger's accident? Yes 3. CONCLUSION: Court reversed and remanded the case. The questioned instruction erroneously placed upon the defense the burden of proving that the injuries could be apportioned. This was, according to the court, an incorrect application of law. The burden of proof is on the plaintiff (Pena) to establish that damages were proximately caused by the negligence of the defendant.

on professional malpractice - why do we call these people the 'professionals,' and what's so special about them that they get their own standard of care?

1. Have technical skills, knowledge that's beyond laypeople's skills, knowledge. a. Bc they have this technical knowledge and skills, are only ones that know about certain things. Can't judge their activity based on a lay standard. b. Bc of this knowledge, when actions're questioned on malpractice cases, need an expert to testify when standard of care's violated This need for an expert in prof cases is primary diff between lay/profs.

res ipsa permits an INFERENCE of negligence that the jury may or may not actually draw. There are at least four positions that have been taken by the courts as to whether the P, by his specific pleading of neglignece, is precluded from relying upon res ipsa:

1. He cannot rely on it at all 2. He may take advantage of res ipsa to the extent that the inference of negligence to be drawn supports the specific allegations 3. That he may rely on res ipsa only if the specific pleading is accompanied by a general allegation of negligence. 4. That it is available without regard to the form of the pelading

the lesson from Daubert: if you get an expert, must make sure that if they're going to give scientific opinions based upon research they've done, rely upon technical expertise, must show its based on respected techniques.

1. In many court cases, you have a Daubert hearing; if you're offering up an expert, court will have a hearing where the expert is called, have to decide if expert will in fact testify, if they're relying upon good science. Rule of evidence: if expert doesn't meet Daubert reqs of scientific evidence, case'll be dismissed bc have no proof of cause in fact. No evidence on what was the cause in fact - no scientific evidence

Res ipsa is a way to get to the jury when P doesn't know what caused the accident.

1. In order for it to be given as a jury instruction to jury, judge must evaluate several factors, must find those factors by a preponderance of the evidence - aka, more likely than not that the D had control, that this is the type of accident that doesn't occur without negligence.

informed consent requires more than getting a signature.

1. In some cases, need to provide an interpreter. How does this resonate with us: when we get a particular case on informed consent, don't make assumptions. Ask the necessary questions of your client, ask did doc tell you this, did they have discussions with you, did they just have you sign a paper. Realize there may be lang barriers, cultural barriers that were the reason why this patient wasn't informed

When defining what transactions affect public interests, court relied on Tunkl factors:

1. It concerns a business of a type generally thought suitable for public regulation 2. The party seeking exculpation is engaged in performing a service of great importance to the public, which is often a matter of practical necessity for some members of the public 3. The party holds himself out as willing to perform this service for any member of the public who seeks it, or at least for any member coming within certain established standards 4. As a result of the essential nature of the service, in the economic setting of the transaction, the party invoking exculpation possesses a decisive advantage of bargaining strength against any member of the public who seeks his services 5. In exercising a superior bargaining power the party confronts the public with a standardized ashesion k of exculpation, and makes no provision whereby a purchaser may pay additional reasonable fees and obtain protection against negligence n. Finally, as a result of the transaction, the person or property of the purchaser is placed under the control of the sller, subject to the risk of carelessness by the seller or his agents

persons that rest outside the est categories of trespasser, invitee, leasee:

1. Kids - courts have been reluctant to apply to child trespassers the same limited duty rules that are applicable to adults. Most have imposed on landowners the duty to exercise a higher standard of care towards kids. Attractive nuisance doctrine - whena landholder sets before young kids a temptation that he has reason to believe will lead them into danger, must use ordinary care to protect them from harm. 2.Persons privileged to enter irrespective of landowner's consent - public employees or officials. They aren't trespassers, since they're privileged to enter. This privilege is independent of any permission, consent or license of the occupier; they would be privileged to enter and could insist upon doing so even if the landholder made an active objection.

Sheehan case: deals with injured person ON the property, not objects on the property. P's on the land. The duty owed by the property owner to the injured person depends on the P's status on the premises. Three categories:

1. Licensee 2. Invitee 3. trespasser this guy was not a licensee or invitee. Ask if he was a trespasser, and does rr owe duty to trespassers.

discussing evidence of negligence ex: Anjou v. Boston Elevated Railway Co.

1. Looking at constructive notice. Why they discuss the color of the peel, etc. 2. Brief a. Rule: D has obligation to keep its station reasonably safe for its passengers. b. Facts: P slipped on a banana peel. c. Issue: Is a railroad company considered negligent and in breach of its duty of care by not keeping its platforms free of debris? Yes d. Conclusion: state SC found for P after testimony indicated peel had been on the platform for a while before P slipped on it; said this was evidence of negligence that should have been submitted to the jury; was evidence that D may not have performed its duty.

Things i must know from Daubert:

1. MUST KNOW: Prelitigation, peer reviewed, know that the step by step must be set out, must be based on some treatise and based upon a method accepted by a respected minority of the profession. 2. Frye test says that a respectable minority is not sufficient; a majority of the scientists must accept it. but fed system no longer abides by Frye test. a. Most states follow fed rule, requires Daubert, but some follow Frye. 3. Parts of this case I must know: the 'more than dobling,' is a rule of tort law. Have rule of evidence. How do you prove something statistically.

Evidence of Negligence - deals with breach. Before the D can be liable, you have to prove they did something negligently. Frequently you won't have actual knowledge, no video or witness. How do you PROVE breach of duty. Judge has told us what the duty is (professional malpractice, etc.) now we're asking how you prove breach.

1. Most of these cases deal with people who own businesses, people hurt on business premises. The owner of a business has a duty to make premises safe for those legitimately on it. a. On exam: this is store's owner. Duty is to use the care of a reasonable prudent store owner to make premises safe for people who are legitimately on the site. b. To satisfy this duty, store must inspect the premises periodically to discover if there are any dangers to the people there. Duty to use reasonable care to INSPECT. 2. When does a breach occur? When the store, in maintaining premises, does something that a reasonable prudent store wouldn't do a. Explain this breach: ex, something on the floor on which a customer fell. Even though the store has a duty to use reasonable care to make the premises safe for customers, they're not an insurer of their customers' safety. They don't guarantee that you won't fall inside. They have to have NOTICE of the dangerous condition in enough time to remove it before fall. If they didn't have enough time to discover it, they'd not be liable. Only way they can be liable is if something was defective, they had enough time. § If P can't show this, they probably aren't going to get to the jury.

Two other types of privileges we discussed (besides self-defense, etc):

1. Necessity (public and private) 2. justification

1. So when two independent sources of negligence join together to cause injury to the P, neither is a 'but for' cause bc the injury would've happened anyway.

1. Neither source in these causes are a but for cause, but can be a substantial factor in causing the injury. When you have independent causes joining together to cause a single injury, must use substantial factor test. In contrast, when there's only one negligent act, CAN use but for test to define substantial factor

There are several common situations where retaining an independent contractor won't insulate the party from vicarious liability:

1. Nondelegable duties 2. Apparent authority 3. Inherently dangerous activities or peculiar risk of harm 4. Illegal activities

concurrent causes: Hill says that two different things can be a cause in fact. Here, both parties were a cause in fact. Accident would not have occurred if either one of them had stopped their actions.

1. Normally when you have 2 people who are negligence, their negligence comes together to create an injury, both are liable for a single injury. 2 people can be a cause in fact of 1 injury. a. If two cars ran into a third car, either one is a cause in fact of the accident, third person's death. Look at parties' negligence, see if they have a but for relationship with the injury

Further notes on disclosure:

1. Note 5: you must disclose a risk of the recommended treatment, risk of doing nothing, risk of alternative treatments, as well as all of their benefits. 2. Normally it's doc that has duty to disclose, NOT hospital. So can't sue a hospital for doc's failure to disclose. Must be a hospital employee for you to sue hospital. 3. If hospital's involved in clinical trial, have to disclose the risks of that trial. If there's a statute, have to disclose that risk under the statute

You could ALSO have a battery case in lack of informed consent case. If doc doesn't give you risk, benefit disclosures you're entitled to under either standard, can bring a lack of informed consent OR battery case.

1. Only jurisdiction that uses lack of informed consent to be a battery intentional tort case (as opposed to negligence case): PA. 2. If you did consent and doc only gave you risk disclosures afterwards, then your case is for lack of informed consent, applying either physicians or patients standard of care. If you consented to treatment and doc didn't give you risk disclosures, claim's for lack of informed consent in 49 states. If didn't consent, and they treated you anyway, then claim is for battery. 3. If you didn't consent at all, claim is for battery - this is in all fifty states. If you operate on the wrong body part, that's battery. If you operate on correct leg, just didn't tell patient the risks, then claim is for lack of informed consent - that's a negligence case. Difference - statute of limitations is longer for negligence case as opposed to a battery case. 4. PA only state that classifies it as battery whether or not you consent. If your cause of action is for informed consent, need an expert witness; don't need an expert if cause is battery.

Wagon Mound cases - foreseeable test of proximate causation

1. Overseas Tankship (U.K.) Ltd. V. Morts Dock & Engineering Co., Ltd. "Wagon Mound No. 1." a. P operated a wharf. D owned a freighter. D's crew left a valve open, oil flowed from into harbor. Caused damage, a few days lighter oil was ignited, fire damaged wharf. b. "It doesn't seem consonant with current ideas of justice or morality that for an act of negligence, however slight or venial, which could have resulted in some trivial foreseeable damages, the actor should be liable for all consequences however unforeseeable and grave, as long as they can be said to be 'direct.'" c. An actor's liability is limited to those harms that result from the risks that made the actor's conduct tortious. 2. Overseas Tankship (U.K.) Ltd. V. Miller Steamship Co. "Wagon Mound No. 2." a. The accident facts are the same as in the preceding case, but this action was brought by the owners of the two ships docked at the wharf that were damaged by the fire. b. "HERE, the findings show that some risk of fire would have been present to the mind of a reasonable man in the shoes of the ship's chief engineer."

different ways to approach the loss of opportunity theory (Smith v. Providence Health):

1. Permit the patient to recover as long as the lost chance was a substantial factor in producing the harm. 2. View patient's injury as the loss of the opportunity for a better outcome, rather than as the death or worse outcome. Here, patients are permitted to recover damages, but only in proportion of the full amount. a. Jury's asked to determine the total amount of damages and the percentage by which the chance of a favorable outcome was reduced, with the damage award being a multiple of those numbers 3. Require the traditional causation requirement. Based on reasonable probability, not on possibilities. Aka, need to have more than 50% chance of recovery.

Yellow Cab C. of D.C., Inc. v. Deslin brief

1. RULE: The right of contribution arises out of a common liability. The rule hinges on the doctrine that general principles of justice require that in the case of a common obligation, the discharge of it by one of the obligors without proportionate payment from the other, gives the latter an advantage to which he is not equitably entitled. Contribution, then, depends upon joint liability. An injured party plaintiff in the suit from which a right of contribution develops must have had a cause of action against the party from whom contribution is sought. 2. FACTS: Appellant cab company sought review of a decision of the trial court which ruled that appellant could not obtain contribution from appellee husband upon a judgment against his wife. The trial court held that the right to contribution arose from a joint liability, and as the husband was not liable in tort to his wife, there was no joint liability between him and appellant as to her. 3. ISSUE: Could a husband be liable for contribution to a tort-judgment creditor who had prevailed at trial against his wife? No 4. CONCLUSION: The court held that contribution depended upon joint liability. An injured party plaintiff in the suit from which a right of contribution developed must have had a cause of action against the party from whom contribution was sought. Here, there was no liability by appellee to his wife. There was no right to action against him and appellant, hence nothing to which a right of contribution could attach.

violating a rule of law

1. Rarely happens 2. inappropriate for trial judge to impose a rule of law. Duty'd normally be to determine if one acted as a reasonable prudent person in a similar circumstance, jury'd decide if there's breach. B4 a court can impose a rule of law, such that if you breach it you're negligent, needs to be some background of exp in this particular area, where it's so common that what court says should be done is acknowledged as being what a reasonable prudent person would do. a. Before judge can impose a rule of law, there should be a background of exp that that's what ordinary people do. If no background, duty is to do what a reasonable person would do in a similar circumstance and then leave it to jury to decide if duty was breached. 3. Under rule of law, duty is to do a specific thing, what the law says. Court will state affirmatively that the duty is to do x, instead of saying that a reasonable prudent person would do in same or similar circumstances. a. Breach occurs if they didn't do those things.

res ipsa loquitor

1. Res ipsa is another way to prove something, mostly deals with breach. a. It's another way to get the case to the jury, another way to look at if evidence is more likely than not negligent. There are some elements: § Whether D had control over the instrumentality that caused injury § whether the accident or injury is the type that ordinarily doesn't occur without the one in control being negligent. b. If P can prove these 2, then normally she goes to jury. Primary thing is getting to the jury.

In re Arbitration Between Polemis and Furness, With & Co., Ltd. - proximate cause ex

1. Respondents chartered boat to appellants. Cargo included petrol in cases. A plank fell into hold, caused an explosion, set fire to the boat and destroyed it. owners claimed the value of the boat from the charterers. Charterers contended that the damages were too remote. "arbitrators have found as a fact that the falling of the plank was due to the negligence of the D's servants. The fire appears to have been directly caused by the plank's falling. Ie, consider that it's immaterial that the causing of the spakr could not have been reasonable anticipated. Aka, damages claimed are not too remote. "Once the act is negligent, the fact that its exact operation was not foreseen is immaterial."

Pokora v. Wabash Ry. Co. rule of law:

1. Rule of law: duty now is established by a rule of law. a. Appellate court said this wasn't an appropriate rule of law to est, said it was inappropriate for trial judge to direct a rule of law. Ordinarily, duty would be to do what a reasonable prudent driver would do. There needs to be some background of exp here so that it is so common that it's common knowledge what should be done. If people had gotten out and looked prior to the judge saying that, judge would've been correct. But judge imposed a rule of law without a background of exp saying that's what people ordinarily do. If that doesn't exist, needs to be left to the standard of what a reasonable person would do in the same or similar circumstance.

dissent opinion in Elbaor:

1. Simply bc jurors may initially expect P to have interests advers to all Ds does not mean that they are incapable of understanding that certain Ds have an incentive for the P to succeed. 2. The trial can't be a "sham of adversity" when the jury, as here, is fully aware of this shift in alliances. 3. Most js allow Mary Carter agreements wen trial courts implement similar procedural safeguards to those adopted here

notes following Cox v. Pearl Investment Co.

1. Suppose a release is signed on the basis of assumed injuries, turns out that injuries are much more extensive than og thought. Ordinarily, injured party must seek to get release set aside on the ground of fraud, mistake, duress. Most courts won't permit P to avoid the terms of the release if the mistake is about the extent of the injury. 2. Distinguish between settlement and judgment. Satisfaction is receiving full compensation for the injury, based either on P's settlement of jury verdict. Claim is satisfied when the judgment is actually paid after the trial or when a settlement reflecting the full amount of P's claim is paid. P can only receive one satisfaction for an injury 3. Release: release is a surrender of the P's claim, which may be for only partial compensation or for no comp at all 4. Covenant not to sue: P doesn't surrender the cause of action, but promises not to sue on it. the protection to D is that if P sues, D will have a counterclaim for breach of the covenant. 5. The early common law rule that the release of one joint tortfeasor releases them all has been extensively changed. Statutes in some states have provided that a release with an express reservation of rights against other tortfeasors doesn't release them. 6. In some js, the release of a negligent agent also releases their principle. Depends on j.

Understand: there is a guidepost, have three factor Gore test mostly used for deciding if punitive damages are excessive. Some js go beyond this, look at other factors (p626, note 5):

1. The character of D's act 2. The nature and extent of the harm to the P and the wealth of the D 3. Likelihood that serious harm'd arisen from the misconduct 4. The degree of D's awareness of the misconduct 5. The profitability of the misconduct 6. THE AMOUNT OF MONEY THAT THE D HAS. The more the D has, the more you need to take to actually punish them (punishment should hurt) (If court does take into consideration the wealth of the D, can do discovery, take interrogatories, subpoena their bank accounts to figure out how much money they have).

Whittaker v. Sandford gives us the elements to use for intentional infliction of emotional distress:

1. The conduct must be intentional or reckless 2. The conduct must be extreme and outrageous 3. There must be a causal connection between the wrongful conduct and the emotional distress 4. The emotional distress must be severe THOSE ARE THE ELEMENTS we need to know for intentional infliction of emotional distress we need to know for the exam

Several factors have been identified as helpful in determining whether an employee has embarked on a slight or substantial deviation:

1. The employee's intent. 2. The nature, time, and place of the deviation. 3. The time consumed in the deviation. 4. The work for which the employee was hired. 5. The incidental acts reasonably expected by the employer. 6. The freedom allowed the employee in performing his job responsibilities

There are two lines of cases concerning an invitor's duty to an invitee injured at the hands of third party criminal conduct. Injured parties assert either that:

1. The invitor has failed to take reasonable measures to reduce the likelihood of dangerous criminal activity posing a danger to invitees or that 2. The invitor's acts in the face of the crim negligently endangered the invitee. Two other lines of cases concern an invitor's duty to an invitee injured at the hands of third-party criminal elements: Injured parties assert either that 1. The invitor has failed to take reasonable measures to reduce the likelihood of dangerous criminal activity posing a danger to invitees or that 2. The invitor's acts in face of the crim negligently endangered the invitee

there are four basic options available in law in connection with how to treat the fact that P's negligence contributed to the happening of an accident:

1. The law could completely bar P's claim - this is called contributory negligence. 2. Law could completely ignore P's culpable conduct. This approach is taken under most worker compensation acts. 3. Law could adopt one of the first two options as a general rule and then set up other rules making exceptions for designated situations. Aka, generally adopt contributory negligence, but allow exceptions. 4. Law could compare P's fault with that of D and reduce P's damages according to the measure of fault. This is called comparative negligence or comparative fault; has been expressly adopted for some actions in most states.

In some states, you can consent to an illegal action - ex, in most states, fighting is illegal, but you can still consent to do it. other js say you can't consent to illegal activity - in that case, you can't sue by using consent as a defense bc you can't consent to illegal activity. NOTE: Even states that generally recognize the validity of consent to an illegal act will not deny recovery to those whom the statute making the conduct illegal was designed to protect. For ex, if P, a 15 year old girl, consents to sex with a 50 year old man in a state with criminal penalties for men who had sex with kids of that age, most courts have held P's consent to be ineffective. The question of when a P's consent should be invalidated bc D violated a criminal statute may depend on several considerations:

1. The policy of denying compensation to an intentiaonl wrongdoer who himself may have committd a crime and been injured as a result of it. 2. The effect of deterring him, and others like him, by denying him recovery if he gets hurt. 3. The effect of potential liability in deterring D and others like him. 4. The fact that P has after all ben intentionally battered by D. 5. The policy expressed by in pari delicto potior est conditio defendentis (in equal guilt, the position of the D is stronger).

There are two ways one can have the intent to commit an intentional tort:

1. To do an act for the purpose of causing an intentional tort 2. To do an act with reasonable certainty that the intentional tort will occur Purpose and substantial certainty are the two ways

In the Sindell Labs case, drug has already been proven defective, have basically proven cause in fact for it. Differs from Summers bc all of Ds aren't before court. Not fair to make them all liable for the whole. However, Innocent P, neg D, still shifts the burden. Notes that follow this case:

1. WI case allocated neg to the Ds that doesn't necessarily be the same as their market share 2. the market of the market share could be the state, city, even the pharmacy 3. Pittman disagrees with the note case that says if og manufacturer made a fraudulent claim in their insert, that that fraud can't be held against them if P took a generic drug. 4. Some states don't allow for market share liability - have chosen to retain in DES cases the trad common law requirement that P prove the identity of the tortfeasor to recover.

review of proximate cause: Several different tests to use when defining if something is/isn't a proximate cause. On test, need to know all the tests and how to apply them:

1. Wagon mound cases - foreseeability tests 2. Railroad/fire cases: immediate and remote test 3. Paranoid schizo after a car accident: eggshell rule 4. Pelomis: direct cause

Two basic issues involved when D asserts that P expressly assumed a risk:

1. Whether the risk that injured P fell within the unambiguous scope of the terms of the agreement. (word 'negligence' doesn't have to be in the agreement as long as it is otherwise clear and unequivocable) 2. Whether the k itself violates public policy and therefore should not be enforced.

If you look at banana peel cases: looking at constructive notice.

1. Why they discuss the color of the peel, whether there was dirt on it? If it's dark, mashed down, dried, then that's indicative of it being there for a long time. 2. Have to show by a preponderance of the evidence, more likely than not, it was there for a long time. We're dealing with probabilities, not certainties, as to how long it was there. P in these banana peel cases has to show more likely than not it was there so long a reasonable prudent person would've discovered it.

But for test

1. You can use the but for test to define the substantial factor except when you have two independent causes that cause an injury, and either alone would've caused the injury. Then you can't use 'but for.' Then you'd have to use substantial factor. 2. Ex: you have two fires, one starts on A's property, another starts on Z's property. The properties are separated. Fires escape from barbecues, escpaes, burns A's yard, then crossed and travels to Z's yard. But someplace on burning down M's house, A's negligently started fire joins with Z's negligently started fire, they create a bigger fire, proceed to burn down M's ouse. Each of those fires, even if they hadn't joined together, would have burnt down the house. Neither fire is a 'but for' cause of the house burning down, bc the other fire would've caused the house to burn down. When two independent sources of negligence would have separately caused the injury, when it joins with another cause that would have caused the injury, neither is a 'but for' cause on the house burning down.

damages ex - Anderson v. Sears, Roebuck and Co.:

1. Young burn victim injured because of D's negligently made/sold heater 2. RULE: The legal standard on which to gauge a jury verdict for remittitur purposes is the "maximum recovery rule." This rule directs the trial judge to determine whether the verdict of the jury exceeds the maximum amount which the jury could reasonably find and if it does, the trial judge may then reduce the verdict to the highest amount that the jury could properly have awarded. Thus, the court's task is to ascertain, by scrutinizing all of the evidence as to each element of damages, what amount would be the maximum the jury could have reasonably awarded.

P has three separate burdens of proof on the issue of negligence:

1. burden of pleading a. P discharges this by alleging sufficient facts in his complaint 2. burden of production a. discharges this burden by convincing the judge that reasonable jurors could find on a more probable than not basis that P's contention is correct. b. Must offer enough evidence to support your motion for case in chief, summary judgment 3. Burden of persuasion a. Discharges this burden by persuading jury that the preponderance of the evidence is in his favor.

discuss ordinary negligence (one of the ways that people can be held liable in negligence)

1. can breach duty of ordinary care (duty = reasonable person) 2. ex cases: water main, golf club, haystack, child on snowmobile. 3. NOTE: Ordinary negligence can be applied to med people - ex, if a hospital didn't have a generator in a hurricane. Just bc a doc is involved doesn't mean it isn't under ordinary negligence. If doc slips while walking down the street, isn't professional malpractice.

Holding in Moore v. Regents

1. doc has to disclose whether he has research or eco interest in your treatment. Normally, only thing that should be taken into consideration for treatment is a patient's own med condition (as well as risks/benefits of treatment, doing nothing, alt treatments). 2. No treatment should be given to benefit the doc. If a doc has an interest in researching on some of your body parts, or is sending you to his lab that he has eco interest in, you want to know that - want to know if your need for this test is based on his desire to make more money or if you really do need it. that's MATERIAL for your decision to have that test, treat with that particular doc.

second thing to get from Smith v. Providence:

1. if it's a cancer case, don't have to show that you probably would've lived, 50.001% causation, because you just can't. so we'll just say it won't be a preponderance of the evidence that you lived, it'll be a preponderance of the evidence that you suffered a loss of chance. So NOW what we're concerned about is that the P has to show more likely than not that the docs may have reduced their chance. Has to show the docs' malpractice more likely than not caused the loss of chance. Can't just be possible that they did it. show through experts that the failure to treat in the proscribed way more likely than not caused the reduction in their chance of recovery, went from 30 to 0 percent chance. a. Some js, like MS, say no, that's too complicated. What we want to do is keep the 50.1%, more likely than not. There, you'd have to show that the misdiagnose was the probable cause of death or current disability. So in MS, you wouldn't be able to win on the probable evidence rule. In MS, then, this case would lose on a loss of chance theory - he'd have to show that he had a 50.01% chance of not having his current disabilities. b. In MS, if you didn't have a more than fifty percent chance of living, a doc couldn't cause your death. c. If you had less than fifty percent change of not have disabilities, in MS, the doc couldn't cause your injuries.

In the state marine case, p632, US SC held that in a maritime case, fed courts should recognize a wrongful death claim. So if the decedent is killed on the sea, his beneficiaries can bring a wrongful death claim. What this case further talks about is what kind of damages can the beneficiaries get. LOOK AT THIS - notes after case. This court didn't decide what kinds of ds can they get in maritime law (there are other cases you can find where courts define the ds beneficiaries can get in maritime law for wrongful death):

1. loss of support, including all the financial support that the decedent would've made to his dependents had he lived 2. the monetary value of services the decedent provided and would have continued to provide but for his wrongful death, including the nurture, training, edu, and guidance that a kid would've received hadn't the parent been wrongfully killed, plus services the decedent performed at home or for his spouse. 3. compensation for loss of society, including a broad range of mutual benefits each family member receives from the others' companionship, comfort and protection 4. damages for funeral expenses in circumstances where the decedent's dependents have paid for the funeral or are liable for its payment. (if beneficiaries pay, part of wrongful death claim; if estate pays, is part of the survival claim). 5. Kids can get ds for the training, instructions that a parent would've given them. 6. Ds based on "grief" NOTE: those categories will normally just apply when an adult is the decedent. What if decedents are kids: then you can calculate lost wages based on average high school graduate, and then you can est how much of those wages would the kid have given to the parents. The big type of ds for kid decedents is the loss of love and affect. In Selders case, the court says that that exists - normally in this j can only get things that a money amount can be applied to; this case changes that, says can get love ds.

Jasko v. F.W. Woolworth Co. is a:

1. method of sale case, not a constructive notice case. a. If the method of sale is such that a reasonable person would not sell a good that way, then don't have to argue constructive notice. Here, they had actual knowledge of their method of sale; having actual knowledge that your method of sale is dangerous, continuing to do it that way, do you fail to do what a reasonable person would do? Here, yes. b. You knew selling pizza on a flimsy plate on a slippery floor would lead to people falling. Is an ACTUAL NOTICE case with the method that you use being negligent

there are four kinds/types of damages in tort law:

1. nominal damages 2. compensatory damages 3. punitive damages 4. equitable relief

If the amount the jury awards is so low it shocks the court, judge can do two things;

1. order a new trial 2. In state court (NOT fed), judge can give an additur, increase the amount how much he wants as long as it doesn't go over the max recovery amount. aka, can add to damages; this will get an appeal, but he can still do it In fed court, judge can ONLY order a new trial

How can people be held liable in negligence (can be liable under any of these)

1. ordinary negligence (all ordinary people) 2. Professional malpractice (applies to professionals; requires expert testimony 3. Violating a rule of law 4. violating a statute

To determine future lost wages (and med expenses):

1. person's life expectancy - note that most people don't work until they die. In such cases, experts will ahve to talk about work-life expectancy, not actual life expectancy (though for med expenses, DO just look at life expectancy) In addition to lost wages, if employer was going to contribute to retirement or 401k, that's calculated in with your lost wages - not only the base amount you would've made in paycheck, but any bonuses or benefits that would have been given, any fringes. Fringe: insurance payments (most emps give this) is an ex. May have lost that, have to pay for it yourself, so maybe should be included as a fringe benefit. Notes 11, 12: inflation of med expenses year by year (p594). Need to be inflated, brought back to present-day price if they are economic values.

Three areas in which a professional's conduct may be questioned (duty is to act as a reasonable, prudent member of the profession would in a reasonable or similar circumstance. If any of these three are broken, there is breach):

1. possession of knowledge or skill (must have the knowledge, skills, and ability of an ordinary member of the profession) 2. exercise of best judgment 3. use of due care If 1 of these 3 are deficient, can be liable for malpractice. Note: whilst must use due care an ordinary member of the profession would use in a similar circumstance, circumstances can change! a. Diff situation in a hospital v. helping somebody on roadside. If doc's in a small town with no Cat scan, maybe he'll have to use an xray or send you to a different hospital.

There's 2 diff theories of lack of informed consent: (on exam, won't know which jurisdiction it is, which of the 2 standard's adopted. Depends on jurisdiction - you'd have to explain both)

1. reasonable prudent physician 2. patient-oriented standard

A. Regardless of whether a jurisdiction adopts patient or physician standard, there are exceptions to disclosure:

1. risks that ought to be known by everyone or are already know to patient; 2. don't disclose when doing so would be detrimental to patient's total care and best interests (ie, if it'd alarm and emotionally upset or apprehensive patient) 3. An emergency, patient can't decide whether treatment should be given a. If patient isn't conscious and it is an emergency, but have to act not right now, talk to next of kin. If you do need to act right now, then act.

Daubert case: there are two types of considerations we have when dealing with statistics as a cause in fact:

1. show that this drug was the sort of drug capable of causing birth defects 2. show that this drug more likely than not caused this patient's birth defects (show that P took the drug, show that the population that took the drug and had birth defects was more than doubled) This case also gives us a rule of evidence, two levels of accepting an expert: 1. must have knowledge in the relevant field 2. Is opinion credible to be believed (is evaluate by looking at if research was done pre-lawsuit, submitted to peer review) If opinion isn't done pre-lawsuit, peer reviewed, scientist must lay out step by step his method, method must be based on some treatise or publication to show it's accepted by a respectable minority of scientists Regarding the rule of evidence: Daubert is a fed rule of evidence; states can come up with their own, follow the Frye test (requires a maj of scientists to approve the method)

degrees of negligence:

1. simple negligence 2. gross negligence 3. willful, reckless, ad wanton negligence 4. slight negligence automobile guest statutes: many states used to have these, now nixed them. Say that if you have a guest in a car, they have to show gross negligence to sue you for negligence.

In Daubert, why is it that the experts' testimony did not est that the drug was the cause in fact of the injury:

1. statistics. Have to show that in the population of women that took the drug, there must be 2 in a thousand. Must double the likelihood that already existed. a. In order to be more likely than not, have to show a doubling - og one in a thousand, has to be 2 in a thousand to get this more likely than not. When you use statistics. P316. b. P317, must MORE THAN DOUBLE risk of birth defects

Breunig v. American Family Ins. Co. takeaways:

1. sudden mental capacity, equivalent in effect to a sudden health event as a stroke, fainting, heart attack, shouldn't be treated under general insanity rule. a. Aka, there's a distinction between permanent mental insanity and sudden mental incapacity. If it's permanent, the person has been diagnosed as schizophrenic and they or caregivers know this, then he can't use insanity defense. Reason why - bottom of p 192. WRITE THESE DOWN. b. If it's a sudden mental incapacity that occurs without warning, person'll be held to standard of care of a person that is undergoing sudden incapacity. i. Here, she had some forewarning - had had episodes before. Thus, she or caregivers should've foreseen that she could be overcome with a delusion. Most courts don't adopt this sudden insanity rule; doesn't matter if it's sudden - still held to the standard of a reasonable person. But this jurisdiction gives allowance for suddenness, so look at what a reasonable non-insane person would've done in a similar circumstance

Takeaways from Bruckman v. Pena (apportionment of damages):

1. the burden of proof is on P to est that damages were proximately caused by the D's negligence. 2. Basic facts: P has wreck with D1, hurt her back. months later, another accident with D2 that also injured her back. now, after both accidents, she has paralysis. Against whom can she collect? Answers: § She has to decide how much of her subsequent pain should be allocated to D1 and D2. If can't show D1 can't show he produced a certain amount of the damages, can't collect from him. § If can't say how much after 2nd accident is due to D1, can't collect from him, but can collect from D2 - eggshell rule will get him (had to accept her as she was). § Can collect from D1 for damages that existed, bills, etc between first and second accident. Only liable for the interim unless can show by a preponderance of the evidence that 75% of her paralysis happened in the interim, before second accident (get an expert to prove this).

another ex of the but for test:

1. two motorcycles are parked on the street, owners are revving engines. The revving of the engines frightens a nearby child, who runs into the street and hit by a car. If either cycle alone, in revving engine, would've caused kid to be frightened, the fact that the noises joined together does not est that either is a but for cause bc kid would've run into the street if only one was revving its engine. v How do you define substantial factor in the hypo motorcycle case, when you have two independent acts? You can only do it by reference to something else; no hard and fast def. So if two fires hypo on exam, if A's fire creates a burn source, a fire, that's three feet long, and that fire joins with another ball of fire from Z's property that's three feet long, and they create a six foot long fire, we can say that both A and Z's fire are a substantial factor. At some point, if the size of Z's fire starts to reduce before it joins, so instead of three feet it's only one feet long, at some point you're going to say that that tiny fire is not a substantial factor in causing the house to burn down

Reynolds bring up the question of when does the negligence probably cause the action:

1. when it greatly multiplies the chance that an injury will occur; when the injury is in the scope of the negligence. Injury is in the scope of negligence if the injury is the thing you'd expect from the negligence. A building blowing up isn't in the scope of having no lights on steps. If building explodes, not having lights doesn't greatly multiply that chance.

policy reasons for holding a permanently insane person liable for his torts:

1. where one of two innocent persons must suffer a loss it should be born by the one who occasioned it 2. to induce those interested in the estate of the insane person to restrain and control him 3. the fear an insantiy defense would lead to false claims of insanity to avoid liability

a. 12(b)(6) motions ask court to dismiss the case for failure to state a factual claim. Note - this rule is the same as a demurrer (English name for this motion, though some states also still use this term) b. Cases can be dismissed for a factual or legal reason.

12(b)(6) motion:

the restatement section 217 and 217 comment e:

2017(b) states that a trespass to chattel may be committed by intentionally using or intermeddling with the chattel in possession of another. comment e defines physical intermeddling as intentionally bringing about a physical contact with the chattel.

Charles is a guest in Heights Hotel. The bathroom in the hotel includes a shower, which is protected by a sliding door made of ordinary glass. While taking a shower, Charles trips and falls on the glass door, causing it to shatter. The shards of glass cut Charles causing serious injuries. It is a standard practice among hotels to use shatterproof, safety glass rather than ordinary glass at shower enclosures. In a negligence action brought by Charles against Heights Hotel, what effect, if any, will Hotel's departure from the industry custom have on Hotel's negligence? A Heights Hotel's departure from this custom is evidence of its negligence. B Heights Hotel's departure from this custom is conclusive proof of its negligence. C Evidence of Heights Hotel's departure from this industry custom is legally irrelevant to the issue of its negligence. D Despite evidence of that Heights Hotel disregarded custom, Heights Hotel may not be subject to liability in negligence because Charles tripped and fell, causing his injuries.

A Correct. A Defendant's departure from custom is evidence that the defendant was negligent, but such departure does not require a finding of negligence. Restatement (Third) § 13. B Incorrect. A defendant's departure from custom does not require a finding of negligence. C Incorrect. The evidence of the defendant's departure from custom is evidence that the jury may take into consideration and is therefore not legally irrelevant. D Incorrect. The fact that Charles tripped and fell does not necessarily shield the defendant from liability for its negligence.

Victim's wallet was stolen by a thief who snatched it and ran. Victim pursues thief on foot and thief realizing he cannot outrun victim, throws the wallet over a chain linked fence into the back yard of owner's property. Victim stops running after thief throws his wallet into owner's yard and opens an unlocked gate, steps 2 feet past the fence onto owner's property recovers his wallet, shuts the gate and leaves. Is the victim liable for trespass to land? A No, because he was likely operating under a privilege which allows him to enter onto another's land to recover his chattel B Incorrect. Many privileges allow entry onto the land of another, including recovery of goods, necessity and consent. C No, because it is not trespass to land to enter another's land to recover your chattel. D Yes, if he did not first seek the owner's permission prior to entry.

A Correct. A defendant who is entitled to immediate possession may recover goods from another's land if the defendant did not cause the intrusion in the first place and if entry is reasonable as to both time and manner. Here, the gate was unlocked, victim steps only 2 feet onto the property recovers the property and immediately leaves. Victim did only what was necessary to recover the chattel and so quite likely acted reasonably. B Incorrect. Many privileges allow entry onto the land of another, including recovery of goods, necessity and consent. C Incorrect. It is an incorrect statement of the law. You are only privileged to enter onto another's land to recover your goods if you did not cause the intrusion in the first place and if entry is reasonable as to both time and manner. The privilege is not an absolute privilege but its existence depends on the reasonableness of the entry. D Incorrect. It is an incorrect statement of law. A defendant who is entitled to immediate possession may recover goods from another's land if the defendant did not cause the intrusion in the first place and if entry is reasonable as to both time and manner. The rules does not require the defendant to seek permission first.

Homeowner and neighbor own unfenced, adjoining pieces of property. Homeowner accidentally begins parking his car on neighbor's property because he is confused about the boundary line. After a few days of this, neighbor drags homeowner's car back onto homeowner's property and unintentionally ruins the transmission as he does so. Homeowner sues neighbor for trespass to chattel. What result? A Neighbor will prevail if he used reasonable force in moving the car. B Homeowner will prevail because he never intended to trespass on neighbor's land. C Neighbor will lose because he caused damage to homeowner's car. D Homeowner will lose because the damage to homeowner's car was unintentional.

A Correct. A landowner may use reasonable force to defend the possession of their property. This means that they may take reasonable steps to remove trespassers or trespassing objects from their land. B Incorrect. Even though a landowner may take reasonable steps to remove trespassers or trespassing objects from their land, the answer choice incorrectly implies that the intent needed for trespass is the intent to trespass. However, the only intent needed for trespass is the intent to cause tangible entry onto land. If that land happens to be in the possession of another at the time you caused tangible entry then you are trespassing whether or not you knew who possessed the land. This is another example of the mistake does not negate intent rule. C Incorrect. If in defending your property you use reasonable force to defend the property then you are not liable for any harm caused in so defending. D Incorrect. Homeowner will not lose because the damage was unintentional. Homeowner will lose because neighbor has a right to use reasonable force to defend his property. So even if the damage to the chattel was intentional as long as the actions constituting defense of property and causing the damage were reasonable homeowner would still not be able to recover.

Store security guard sees person A stealing a watch by placing the watch in their coat on the store's security camera. Security guard immediately leaves her station to question A. When security guard gets to the floor of the store security guard accidentally detains B instead of A. B and A were dressed in the same coat and were about the same physical stature. As soon as the security guard begins to question B it becomes apparent that he detained the wrong person and B is released and free to go. Assuming the security guard made a reasonable mistake what result if B sues the store for false imprisonment? A B will prevail unless he was detained in a manner that was reasonable to detain A. B B will prevail even if he was detained in a manner that was reasonable to detain A. C B will not prevail because he was suspected of shoplifting. D B will not prevail because a merchant may detain anyone at any time without justification once they are on the merchant's premises.

A Correct. A merchant is privileged to detain an individual suspected of shoplifting. The fact that B was stopped instead of A due to the security guard's reasonable mistake does not affect the privilege. Reasonable mistake in exercising a privilege does not negate the privilege. So B would prevail unless B's detention would have been a reasonable one if A was detained. B Incorrect. A merchant is privileged to detain an individual suspected of shoplifting. The fact that B was stopped instead of A due to the security guard's reasonable mistake does not affect the privilege. Reasonable mistake in exercising a privilege does not negate the privilege. So B will not prevail if the manner in which B was detained was a reasonable way to detain A. C Incorrect. A merchant does not have unlimited power to detain a suspected shoplifter. A shoplifter may only be detained in a reasonable fashion. D Incorrect. It is simply an erroneous statement of the law. A shopkeeper may only detain someone reasonably suspected of shoplifting and may do so only in a reasonable manner.

Alice sues Bill alleging negligence. Bill defends, claiming that Alice's negligence also contributed to her injuries. At trial, the factfinder assigns 40% responsibility to Alice and 60% responsibility to Bill. The factfinder also finds that Alice's damages are $100,000. In a pure comparative fault jurisdiction, how much is Alice entitled to recover? A Alice is entitled to recover $60,000. B Alice is entitled to recover $100,000. C Alice is not entitled to recover anything because she was partially responsible. D Alice is entitled to recover $40,000.

A Correct. Alice is entitled to recover only that portion of the judgment that is not attributable to her fault. So the judgment of $100,000 is reduced by 40%, the amount of fault attributed to Alice, which if $40,000. $100,000-$40,000=$60,000, making choice A correct. B Incorrect. In a pure comparative fault system, the plaintiff's judgment is reduced by the amount of fault attributed to her, so she cannot recover the entire $100,000. C Incorrect. In a pure comparative fault system, the plaintiff is not completely barred from recovery; she can recover some of her damages. D Incorrect. Her recovery is reduced by $40,000 (40% of $100,000); her recovery doesn't amount to $40,000.

A state statute fixes a speed limit of 55 miles per hour for a state highway. Driver is driving her car on that highway at a speed of 50 miles per hour. Teenager unexpectedly darts from the sidewalk onto the highway. Despite a proper lookout, Driver is unable to stop in time and strikes Teenager, injuring him. Had Driver been driving at 40 miles per hour, she would have been able to avoid hitting Teenager. In his lawsuit, Teenager alleges that Driver was negligent in driving at a speed faster than 40 miles per hour. At the time the weather had been fine, the road and traffic conditions had been entirely ordinary, and no other circumstances suggested any special danger. Driver asks the court to rule as a matter of law that she did not breach the standard of care. Which statement regarding the court's ruling on the standard of care and Driver's compliance with the statute is correct? A The court would be justified in ruling as a matter of law that Driver did not breach the standard of care by driving at the speed she was driving at the time of the accident due to the favorable weather and road conditions at the time of the accident. B The court would not be justified in ruling as a matter of law that Driver did not breach the standard of care by driving at the speed she was at the time of the accident because of the favorable weather and road conditions at the time of the accident. C If the court rules as a matter of law that compliance with the statute establishes Driver's non-negligence, Driver cannot be held liable in negligence for Teenager's injuries. D If the court refuses to rule as a matter of law that Driver's compliance with the speed limit statute is evidence of non-negligence, the factfinder would then have to conclude that Driver did not breach the standard of care.

A Correct. Although usually compliance with a statute may be used merely as evidence of non-negligence, the court may rule as a matter of law that compliance amounts to non-negligence, such as in the facts of this question when the weather was fine, road conditions are ordinary, and there was nothing to suggest any special dangers. B Incorrect. The court may rule as a matter of law that compliance amounts to non-negligence, such as under the facts of this question when the weather was fine, road conditions are ordinary, and there was nothing to suggest any special dangers. C Incorrect. If the court refuses to rule as a matter of law that Driver's compliance with the speed limit establishes her non-negligence, the factfinder can seek to determine that compliance with the speed limit was or was not negligence as it chooses. See Restatement (Third) Torts § 16 and illustration 1 upon which this question is based. D Incorrect. If the court refuses to rule as a matter of law that Driver's compliance with the speed limit establishes her non-negligence, the factfinder can seek to determine that compliance with the speed limit was or was not negligence as it chooses.

On a dark night, Jim who is 40 years old is walking in a city park that winds along the coastline on a bluff above the water. While walking on a path adjacent to a stone wall marking the edge of the bluff, the ground gives way, and Jim falls to the bottom of the bluff, suffering physical harm. A city ordinance prohibits entry into the park from dusk until dawn, but no physical barrier prevents entry during that time nor is there a sign prohibiting entry. Jim sues City in negligence. Which of the following statements about Jim's status and City's duty is correct? A Jim is a trespasser to whom City owes a duty of reasonable care. B Jim is a trespasser to whom City owes no duty of care. C Jim is a licensee to whom City owes a duty to render the premises reasonably safe. D Jim is a business invitee to whom City owes a duty to render the premises reasonably safe.

A Correct. At the time of his injury, Jim was a trespasser as, pursuant to the ordinance, he was not supposed to be in the park after dark. This question asks about the duty of care owed to a trespasser. A trespasser is a person who has entered another's land without consent or other legal privilege. A possessor of land owes a duty of reasonable care to entrants of the land with regard to natural conditions of the land that pose a risk of harm. Restatement (Third) Torts § 51. B Incorrect. One owes a duty of reasonable care even to a trespasser. This question asks about the duty of care owed to a trespasser. A trespasser is a person who has entered another's land without consent or other legal privilege. A possessor of land owes a duty of reasonable care to entrants of the land with regard to natural conditions of the land that pose a risk of harm. Restatement (Third) Torts § 51. C Incorrect. A licensee is typically defined as a person present on another's land for the licensee's own purpose. An invitee is typically a person present on another land for the purpose of the landowner. Jim is most properly characterized as a trespasser rather than a licensee or an invitee. This question asks about the duty of care owed to a trespasser. A trespasser is a person who has entered another's land without consent or other legal privilege. A possessor of land owes a duty of reasonable care to entrants of the land with regard to natural conditions of the land that pose a risk of harm. Restatement (Third) Torts § 51 D Incorrect. A licensee is typically defined as a person present on another's land for the licensee's own purpose. An invitee is typically a person present on another land for the purpose of the landowner. Jim is most properly characterized as a trespasser rather than a licensee or an invitee. This question asks about the duty of care owed to a trespasser. A trespasser is a person who has entered another's land without consent or other legal privilege. A possessor of land owes a duty of reasonable care to entrants of the land with regard to natural conditions of the land that pose a risk of harm. Restatement (Third) Torts § 51

Law student is angry at Torts professor because professor made student work and professor is always so prepared in class. As professor is walking to his car in his perfectly matching designer shoes, "I am going to hit you with this book for being so prepared." Student did not intend to hit him but just to scare him by swinging the book close to his head and making him flinch. Unfortunately student misjudged the length of the book and the book struck professor's head after professor flinched. Are the student's actions assault and/or battery? A The students actions are both an assault and battery. B The students actions are neither an assault nor a battery. C The students actions are an assault but not a battery. D The students actions are a battery but not an assault.

A Correct. Here the student intended make the professor afraid of imminent contact with the book on his head. Being hit in the head with a book is clearly harmful or offensive. In other words the student committed an assault. Student had the intent to cause the professor fear or apprehension of imminent harmful or offensive contact and the professor's flinch indicates that he did experience some form of fear or apprehension. Therefore assault occurred. Battery also occurred. Even though the Student did not have the intent to cause harmful or offensive contact with the professor, harmful or offensive contact is exactly what ended up happening when the professor was unintentionally hit in the head. The lack of intent to hit does not prevent a battery claim however. This is because the intent to assault will transfer to satisfy the intent requirement of battery via the doctrine of transferred intent. Transferred intent operates whenever the tort intended and the tort committed are within the original writ of trespass (Trespass to Land, Trespass to Chattel, False Imprisonment, Assault and Battery). Whenever a tortfeasor intends any one of the five but ends up committing any other of the five torts then all the torts committed are deemed to have the intent prong satisfied. B Incorrect. Here the student intended make the professor afraid of imminent contact with the book on his head. Being hit in the head with a book is clearly harmful or offensive. In other words the student committed an assault. Student had the intent to cause the professor fear or apprehension of imminent harmful or offensive contact and the professor's flinch indicates that he did experience some form of fear or apprehension. Therefore assault occurred. Battery also occurred. Even though the Student did not have the intent to cause harmful or offensive contact with the professor, harmful or offensive contact is exactly what ended up happening when the professor was unintentionally hit in the head. The lack of intent to hit does not prevent a battery claim however. This is because the intent to assault will transfer to satisfy the intent requirement of battery via the doctrine of transferred intent. Transferred intent operates whenever the tort intended and the tort committed are within the original writ of trespass (Trespass to Land, Trespass to Chattel, False Imprisonment, Assault and Battery). Whenever a tortfeasor intends any one of the five but ends up committing any other of the five torts then all the torts committed are deemed to have the intent prong satisfied. C Incorrect. Here the student intended make the professor afraid of imminent contact with the book on his head. Being hit in the head with a book is clearly harmful or offensive. In other words the student committed an assault. Student had the intent to cause the professor fear or apprehension of imminent harmful or offensive contact and the professor's flinch indicates that he did experience some form of fear or apprehension. Therefore assault occurred. Battery also occurred. Even though the Student did not have the intent to cause harmful or offensive contact with the professor, harmful or offensive contact is exactly what ended up happening when the professor was unintentionally hit in the head. The lack of intent to hit does not prevent a battery claim however. This is because the intent to assault will transfer to satisfy the intent requirement of battery via the doctrine of transferred intent. Transferred intent operates whenever the tort intended and the tort committed are within the original writ of trespass (Trespass to Land, Trespass to Chattel, False Imprisonment, Assault and Battery). Whenever a tortfeasor intends any one of the five but ends up committing any other of the five torts then all the torts committed are deemed to have the intent prong satisfied. D Incorrect. Here the student intended make the professor afraid of imminent contact with the book on his head. Being hit in the head with a book is clearly harmful or offensive. In other words the student committed an assault. Student had the intent to cause the professor fear or apprehension of imminent harmful or offensive contact and the professor's flinch indicates that he did experience some form of fear or apprehension. Therefore assault occurred. Battery also occurred. Even though the Student did not have the intent to cause harmful or offensive contact with the professor, harmful or offensive contact is exactly what ended up happening when the professor was unintentionally hit in the head. The lack of intent to hit does not prevent a battery claim however. This is because the intent to assault will transfer to satisfy the intent requirement of battery via the doctrine of transferred intent. Transferred intent operates whenever the tort intended and the tort committed are within the original writ of trespass (Trespass to Land, Trespass to Chattel, False Imprisonment, Assault and Battery). Whenever a tortfeasor intends any one of the five but ends up committing any other of the five torts then all the torts committed are deemed to have the intent prong satisfied.

For almost 2 centuries Landowner's family has owned a 2 miles stretch of coastal land just landward of a public beach. As the land has been passed down from generation to generation landowner's family has always allowed the public to walk on the land to get beach access even though there are many conspicuous signs saying private property on the land. The newest descendant to own the land has decided that he no longer wants anyone to access the land without permission. One day after this, beachgoer, as other beachgoers have done for centuries walks on the land to get to the beach. Did beachgoer trespass? A No, because a custom existed which permitted entry onto the land that would otherwise be trespass. B Yes, because customary consent may be unilaterally withdrawn without notice. C Yes, because the private property signs meant that entry onto the land was trespass. D No, because once you allow access pursuant to custom it cannot be revoked.

A Correct. If a custom exists which permits entry which otherwise would be a trespass then before entry is actionable the landowner must affirmatively provide notice that the consent is withdrawn. B Incorrect. If a custom exists which permits entry which otherwise would be a trespass then before entry is actionable the landowner must affirmatively provide notice that the consent is withdrawn. C Incorrect. If a custom exists which permits entry which otherwise would be a trespass then before entry is actionable the landowner must affirmatively provide notice that the consent is withdrawn. The signs saying private property were there even as the custom operated before newest descendant decided no one would be allowed to use the land to access the beach. D Incorrect. Unless the customary consent has developed into a prescriptive right or easement then the consent may be withdrawn at any time by the property owner.

Traditionally, contributory negligence as a defense to a negligence action: A was a complete bar to the plaintiff's recovery. B was synonymous with an immunity. C reduced the plaintiff's recovery in appropriate circumstances. D was synonymous with comparative fault.

A Correct. It is a correct statement of the traditional defense of contributory negligence. B Incorrect. Contributory negligence is not synonymous with an immunity. An immunity shields a defendant from liability based on that defendant's status to the community or to the plaintiff, whereas the defense of contributory negligence turns on the conduct of the plaintiff. C Incorrect. Contributory negligence does not merely reduce the plaintiff's recovery; rather it is a complete bar to the plaintiff's ability to recover. D Incorrect. Comparative fault is not necessarily a complete bar to recovery, but rather may merely reduce a plaintiff's ability to recover.

On Friday, employee is credibly threatened by coworker. Coworker says to employee, "I will kill you next Friday if you show up to work then." Employee and three friend's go to Coworker's house the day after Coworker threatened employee and beat Coworker to such an extent that the Coworker will be hospitalized long term and incapable of carrying out his threat. May the employee successfully assert the privilege of self-defense in a battery suit by Coworker against Employee? A No, because the threat by coworker was not one which required immediate action. B Yes, because Coworker credibly threatened employee with death so employee could use up to lethal force in defending himself. C Yes, because employee can preemptively prevent the threat from materializing. D No, because the person asserting self-defense must wait for the attacker to begin attacking before defending.

A Correct. Self-defense or the privileged use of force against another to prevent harm to oneself requires the harm be imminent or immediate or that the imminent or immediate use of force is necessary to prevent future harm. Here the harm is not imminent as Coworker threatened harm in a week. Additionally, there are other avenues such as reporting Coworker to the authorities which are preferable here to the use of force "vigilante justice" by employee. B Incorrect. It is true that when threatened with deadly force or force likely to cause grievous bodily harm a person may use deadly force to defend herself. However, Self-defense or the privileged use of force against another to prevent harm to oneself requires the harm be imminent or immediate or that the imminent or immediate use of force is necessary to prevent future harm. Here the harm is not imminent as Coworker threatened harm in a week. Additionally, there are other avenues such as reporting Coworker to the authorities which are preferable here to the use of force "vigilante justice" by employee. C Incorrect. While it is true that the preemptive use of force is permissible in self-defense, the preemptive use of force must be to negate an imminent threat. For example if someone says to another, "You are dead," and simultaneously reaches into his jacket to grab a gun, the other person my use force to preemptively prevent the threat. They don't have to wait for shots to actually be fired before defending themselves. Here however the preemptive use of force is not to prevent an immediate threat and so I is impermissible. D Incorrect. The preemptive use of force is sometimes permissible in self-defense. However the preemptive use of force must be to negate an imminent threat. For example if someone says to another, "You are dead," and simultaneously reaches into his jacket to grab a gun, the other person my use force to preemptively prevent the threat. They don't have to wait for shots to actually be fired before defending themselves. So the rule of law stated in D "no self-defense until attacker begins to use force," is an erroneous rule of law.

Shopper is picking up a few items at Grocery Store on her way home from work. As she enters the produce section of Grocery Store, she sees Plaintiff lying on the floor, holding her obviously injured hip. Right next to Plaintiff, Shopper observes a dry, gritty, almost entirely black, squishy banana lying on the floor. In Plaintiff's negligence action against Grocery Store alleging Grocery Store's negligence, Shopper testifies to the condition of the banana when she saw it as it lay next to Plaintiff who was on the floor as well. What is the best characterization of Shopper's testimony presented in the trial brought by Plaintiff against Grocery Store for its alleged negligence? A Shopper's testimony is circumstantial evidence of Grocery Store's negligence and requires the factfinder to draw an inference regarding Grocery Store's breach of the standard of care. B Shopper's testimony is direct evidence of Grocery Store's negligence and requires the factfinder to draw an inference regarding Grocery Store's breach of the standard of care. C Shopper's testimony is circumstantial evidence and cannot be used to establish Grocery Store's breach of the standard of care. Incorrect. It is inaccurate to state that circumstantial evidence may be used to establish breach of the standard of care. D Shopper's testimony is direct evidence of Grocery Store's negligence and cannot be used to establish Grocery Store's breach of the standard of care.

A Correct. Shopper's evidence is circumstantial evidence from which the factfinder is asked to infer that Grocery Store breached its standard of care. The fact that the banana was much trampled on, black all over with very little yellow, gritty, sticky, and yucky indicates that the banana had been on the floor for a long time. If the banana had been on the floor for a long time, one can infer that Grocery Store did not keep its premises in a reasonably safe condition. B Incorrect. Shopper's testimony requires the factfinder to infer that Grocery Store was negligent; it is not an account of exactly what happened that lead to the unreasonable condition of its store. This question asks about the use of circumstantial evidence in order to prove a defendant's breach of the standard of care. Direct evidence is evidence that proves the fact sought to be established, e.g., breach of the standard of care. Circumstantial evidence is evidence that requires the factfinder to make one or more inferences to conclude the fact sought to be established. C Incorrect. It is inaccurate to state that circumstantial evidence may be used to establish breach of the standard of care. D Incorrect. Shopper's testimony is circumstantial evidence of negligence, not direct evidence of negligence. This question asks about the use of circumstantial evidence in order to prove a defendant's breach of the standard of care. Direct evidence is evidence that proves the fact sought to be established, e.g., breach of the standard of care. Circumstantial evidence is evidence that requires the factfinder to make one or more inferences to conclude the fact sought to be established.

Shopper insulted store clerk by saying that the store clerk had a disgusting body odor. Shopper intended to insult and embarrass store clerk and said it loud enough for the fellow shoppers in the vicinity to hear. Store clerk was mortified when other shoppers laughed. As a result of the insult, store clerk fainted and hit his head on a display and suffered a concussion. Assume that the shopper intentionally, impermissibly and offensively touched the store clerk while he made the comment but the store clerk was only mildly embarrassed and suffered no additional physical injuries due to the touch. Will store clerk be able to recover? A Store clerk may be able to recover for both the impermissible contact and the mild embarrassment. B Store clerk may not recover for the emotional harm because it was not severe but may recover for the impermissible contact. C Store clerk may not recover for either the emotional harm or the impermissible contact. D Store clerk may not recover for the emotional harm unless the touching was considered extreme and outrageous conduct but may recover for the impermissible touching.

A Correct. The intentional, impermissible and offensive touching will constitute a battery. And the tort of battery unlike the tort of IIED is actionable on only nominal damage as it is a dignitary tort . The requirement of outrageous and extreme conduct and severe harm are only conditions precedent to recovery if the cause of action is Intentional Infliction of Emotional Distress. Here the store clerk will likely have an actionable battery and if the store clerk elects to sue in battery then any and all associated emotional harms are recoverable. It is only when the cause of action is IIED that the requirements of outrageous conduct and severe harm must be satisfied before recovery of emotional harm is permitted. B Incorrect. The intentional, impermissible and offensive touching will constitute a battery. And the tort of battery unlike the tort of IIED is actionable on only nominal damage as it is a dignitary tort. The requirement of outrageous and extreme conduct and severe harm are only conditions precedent to recovery if the cause of action is Intentional Infliction of Emotional Distress. Here the store clerk will likely have an actionable battery and if the store clerk elects to sue in battery then any and all associated emotional harms are recoverable. It is only when the cause of action is IIED that the requirements of outrageous conduct and severe harm must be satisfied before recovery of emotional harm is permitted. If the store clerk elects to sue in battery then the store clerk may recover for any and all associated emotional harm regardless of the lack of severe harm. Again this is because the requirement of severe harm is only an element of an IIED cause of action and not a battery cause of action. C Incorrect. The intentional, impermissible and offensive touching will constitute a battery. And the tort of battery unlike the tort of IIED is actionable on only nominal damage as it is a dignitary tort. The requirement of outrageous and extreme conduct and severe harm are only conditions precedent to recovery if the cause of action is Intentional Infliction of Emotional Distress. Here the store clerk will likely have an actionable battery and if the store clerk elects to sue in battery then any and all associated emotional harms are recoverable. It is only when the cause of action is IIED that the requirements of outrageous conduct and severe harm must be satisfied before recovery of emotional harm is permitted. If the store clerk elects to sue in battery then the store clerk may recover for any and all associated emotional harm regardless of the lack of severe harm. The store clerk will likely recover for the offensive touching and the emotional harm if he pursues a battery cause of action rather than an IIED cause of action. D Incorrect. The intentional, impermissible and offensive touching will constitute a battery. And the tort of battery unlike the tort of IIED is actionable on only nominal damage as it is a dignitary tort. The requirement of outrageous and extreme conduct and severe harm are only conditions precedent to recovery if the cause of action is Intentional Infliction of Emotional Distress. Here the store clerk will likely have an actionable battery and if the store clerk elects to sue in battery then any and all associated emotional harms are recoverable. It is only when the cause of action is IIED that the requirements of outrageous conduct and severe harm must be satisfied before recovery of emotional harm is permitted.

Father was severely beaten by robber and is bringing a claim for assault and battery against robber. Unknown to robber, daughter witnessed father being beaten and suffered emotional distress as a result. If daughter brings a claim for intentional infliction of emotional distress (IIED) against robber, may daughter recover? A Daughter may not recover because robber did not know she witnessed the father's beating. B Daughter may recover only if she experienced bodily injury due to the emotional distress. C Daughter may not recover because she did not suffer bodily injury due to the emotional distress. D Daughter may recover even if robber was unaware of her presence.

A Correct. The rule by which bystanders may recover pursuant to IIED for tortious conduct directed at another is that the tortfeasor must have known or have been substantially certain the bystander witnessed the tortious conduct directed at another person. The reason for this is that IIED is not one of the torts within the original writ of trespass and so the intent does not transfer from the tortious conduct against the father so as to support the intent prong of the daughter's IIED claim in this scenario. In order for transferred intent to apply the tort intended and the tort committed must be within the original writ of trespass (Trespass to Land, Trespass to Chattel, False Imprisonment, Assault and Battery). B Incorrect. When bystander is seeking to recover pursuant to IIED for tortious conduct directed at a close relative, the bystander need not prove that bodily injury occurred contemporaneously with the emotional distress. Here, the daughter and the father are close relatives so she need not demonstrate accompanying physical injury in order to recover assuming all other elements of the tort are met. C Incorrect. The first reason it's incorrect is that when a bystander is seeking to recover pursuant to IIED for tortious conduct directed at a close relative, the bystander need not prove that bodily injury occurred contemporaneously with the emotional distress. Here, the daughter and the father are close relatives so she need not demonstrate accompanying physical injury in order to recover assuming all other elements of the tort are met. Additionally, the answer choice suggests that the reason recovery may not occur is "because" of the lack of bodily injury. This is not the case. Recovery may not be had, "because" there was no intent. There is no need to even check the relational status if there is no intent. Here, there is no intent because IIED is not one of the torts within the original writ of trespass and so the intent does not transfer from the tortious conduct against the father so as to support the intent prong of the daughter's IIED claim in this scenario. In order for transferred intent to apply the tort intended and the tort committed must be within the original writ of trespass (Trespass to Land, Trespass to Chattel, False Imprisonment, Assault and Battery). D Incorrect. Bystanders may recover pursuant to IIED for tortious conduct directed at another only when the tortfeasor knew or was substantially certain the bystander witnessed the tortious conduct directed at another person. The reason for this is that IIED is not one of the torts within the original writ of trespass and so the intent does not transfer from the tortious conduct against the father so as to support the intent prong of the daughter's IIED claim in this scenario. In order for transferred intent to apply, the tort intended and the tort committed must be within the original writ of trespass (Trespass to Land, Trespass to Chattel, False Imprisonment, Assault and Battery).

Doctor while conducting an operation on plaintiff's left eye decides to examine the rest of plaintiff's face and discovers that plaintiff has a deviated nasal septum. At the time of the discovery the plaintiff was under the effect of general anesthesia and unconscious. The doctor decides to correct the deviated septum while the plaintiff rather than do it separately which would require another operation. Which of the following is true? A Doctor likely committed a battery. B Doctor will not be liable unless he performed the nasal septum operation negligently. C Doctor's may assume consent from patients who are under general anesthesia to perform medically warranted procedures. D Doctor likely committed an assault and battery.

A Correct. This is a classic case of battery in tort law. Battery is a dignitary tort and is based on the individual's right to permit or not allow contact with his or her body. Even though the doctor acted with "good" intentions, motive is not the same as intent. Here the doctor committed nonconsensual contact to the person of another because presumably he only had consent to operate on the patient's eye and perform any contact associated with that. If a reasonable person would find the extra contact to be harmful or offensive then battery would be actionable. Even though the damage may be minimal many intentional tort suits survive on nominal damages. The purpose of this is that the decisions become judicial statements on what is permitted and not permitted under the law. B Incorrect. Even if the doctor performed the operation up to the standard of medical care a battery suit might still be actionable. Battery is a dignitary tort and is based on the individual's right to permit or not allow contact with his or her body. Even though the doctor acted with "good" intentions, motive is not the same as intent. Here the doctor committed nonconsensual contact to the person of another because presumably he only had consent to operate on the patient's eye and perform any contact associated with that. If a reasonable person would find the extra contact to be harmful or offensive then battery would be actionable. Even though the damage may be minimal many intentional tort suits survive on nominal damages. The purpose of this is that the decisions become judicial statements on what is permitted and not permitted under the law. C Incorrect. It is an erroneous rule of law. Consent may only be assumed from patients under general anesthesia if the discovered condition is emergent and observed within the course and scope of the consented to procedure. D Incorrect. Assault requires fear or apprehension of imminent harmful and offensive contact. Because fear or apprehension are elements of assault, this tort is actionable only when there is a sensory awareness of the imminent contact. This is impossible here where the patient is unconscious due to general anesthesia.

Gas Co. supplies natural gas for the county and hires Supply Inc. to install a service line to carry gas to a new neighborhood. Supply Inc.'s employees negligently cause a slight leak in one of the gas pipes. The resulting leak causes an explosion that injures Neighbor. In a lawsuit brought by Neighbor against Gas Co., may Gas Co. be subject to vicarious or direct liability? A Gas Co. may be subject to vicarious liability for the negligence of Supply Inc. because Gas Co. hired Supply Inc. to carry out an activity that posed a highly dangerous risk and Supply Inc.'s negligence was a factual cause of Neighbor's harm. B Gas Co. may not be subject to vicarious liability for the negligence of Supply Inc. because Supply Inc. was an independent contractor. C Gas Co. may be subject to direct liability for the negligence of Supply Inc. because Gas Co. hired Supply Inc. and Supply Inc.'s negligence was a factual harm of Neighbor's harm. D Gas Co. may be subject to direct liability for the negligence of Supply Co. because Gas Co. hired Supply Co. to carry out a highly dangerous activity.

A Correct. This is a correct statement of the law regarding the vicarious responsibility of one who hires an independent contractor to carry out an activity that poses a highly dangerous risk and whose negligence in doing so causes injury to the plaintiff. Restatement (Third) Torts § 58. B Incorrect. When hiring an independent contractor to carry out an activity that poses a highly dangerous risk, the general rule of non-liability for the torts of one's independent contractors does not apply. C Incorrect. Defendant will be subject to direct liability if Gas Co. breached the standard of care, which the facts do not suggest it did. These facts concern Gas Co.'s vicarious liability. D Incorrect. Defendant will be subject to direct liability if Gas Co. breached the standard of care, which the facts do not suggest it did. These facts concern Gas Co.'s vicarious liability.

About two years ago, Client was a patient of Ophthalmologist. Client alleges that during the time that he was Ophthalmologist's patient, Ophthalmologist improperly performed LASIK surgery on Client's eyes. The LASIK surgery was supposed to correct Client's poor vision. Instead, the surgery was performed skillfully, but resulted in blindness in both eyes, a risk about which Client was never informed and a reasonable person would not have known. If Client had known of the risk of blindness, he would not have opted for the surgery. Client hires Attorney Matthew to sue Ophthalmologist for medical malpractice. Attorney Matthew fails to file the complaint to initiate the cause of action against Ophthalmologist within the applicable statute of limitations. As a result, the court dismisses Client's case against Ophthalmologist. Client then hires Attorney Becky to represent him in a cause of action against Attorney Matthew for legal malpractice. In the legal malpractice action brought by Client against Attorney Matthew, how will the professional standard of care owed by Attorney Matthew need to be established? A Client will have to establish, by introducing expert testimony, that Attorney Matthew failed to perform as the reasonable prudent attorney in handling Client's medical malpractice action against Ophthalmologist. B Client will be able to establish that Attorney Matthew failed to perform as the reasonable prudent attorney in handling Client's medical malpractice action without expert testimony regarding the standard of care. C Client will be able to establish the standard of care in the legal malpractice action against Attorney Matthew by applying a rule of law. D The factfinder will determine the standard of care in this legal malpractice action without evidentiary guidance.

A Correct. This question asks about establishing the standard of care in a professional negligence action. The general rule is that the professional standard of care must be established by expert testimony. B Incorrect. This question asks about establishing the standard of care in a professional negligence action. Although Client may testify as to his interactions with Attorney Matthew, the factfinder will not determine what the reasonable prudent lawyer would have done in the same or similar circumstances from that testimony. Rather an expert must testify to what the reasonable prudent lawyer would have done in the same or similar circumstances. C Incorrect. The standard of care in a legal malpractice action is not established by rule of law. This question asks about establishing the standard of care in a professional negligence action. D Incorrect. The factfinder will need to receive expert testimony in order to determine the standard of care in a legal malpractice action.

Defendant, driving a car, approaches Plaintiff's car, and properly steps on the brakes. For reasons unrelated to any negligence on Defendant's part, her brakes fail to function. Much to her surprise, Defendant's car continues forward, so she quickly pumps the brakes again, but to no avail. Defendant's car careens into Plaintiff's car, injuring Plaintiff. Upon reflecting calmly on Defendant's brake-failing situation, a better choice for Defendant would have been for her to turn her car quickly to the right. In Plaintiff's lawsuit against Defendant in which he seeks to recover from her in negligence, will Defendant likely be held liable in negligence? A Defendant will likely not be held liable for negligence because in light of her emergency circumstances, she behaved as a reasonable prudent person. B Defendant will likely be held liable in negligence because she should have chosen more wisely by turning the car quickly to the right. C Defendant will likely be held liable in negligence if she caused Plaintiff's injuries. D Defendant will likely not be held liable in negligence because she was reacting to an emergency situation.

A Correct. This question asks about the standard of care in a negligence action when the defendant is acting in an emergency, which requires a person to act reasonably in the emergency situation. B Incorrect. What is required of a person in an emergency situation may be different than what may be required of that person in a non-emergency. See Restatement (Third) Torts § 9 and illustration 1, upon which this question is based. C Incorrect. Merely being the factual cause of another's injuries is insufficient to impose liability in negligence; the negligence of the defendant must cause the plaintiff's injuries. D Incorrect. A person does not avoid liability in negligence merely because she was acting in an emergency. Rather, a person is required to act reasonably in light of the emergency situation.

A state statute requires that the operator of a truck that becomes disabled on a highway promptly put out a warning sign at least 100 feet behind the truck. When a deflated tire disables Chuck's truck, he promptly places out a warning sign right next to the truck rather than at the 100-foot distance. Alice, approaching Chuck's truck from behind, does not see Chuck's warning sign until it is too late for her to stop. Her car strikes the rear of Chuck's truck, and she is injured in the collision. Alice sues Chuck in negligence, and the evidence at trial shows that Alice would have been able to stop in time had the warning sign been set at the 100-foot distance as required by statute. What effect, if any, does Chuck's violation of the statute have on his liability for negligence? A Chuck's violation of the statute may be used to establish his negligence, because the basic purpose of the statute is to prevent accidents of this type and Alice is in the group of persons the statute is designed to protect. B Chuck's violation of the statute is evidence of res ipsa loquitur because the basic purpose of the statute is to prevent accidents of this type and Alice is in the group of persons the statute is designed to protect. C Chuck's violation of the statute has no effect on his liability for negligence because the statute cannot be used to establish the standard of care in Alice's negligence action against Chuck because the statute was not designed expressly to provide a private right of action in negligence. D Chuck's violation of the statute has no effect on his liability for negligence because the standard of care in negligence must be established by proving what the reasonable prudent person in the same or similar circumstances would have done.

A Correct. This response correctly provides the standard for using a statute to establish the standard of care in negligence. Restatement (Third) Torts § 14. This question asks about using a statute's violation to establish the standard of care in a negligence action. B Incorrect. This response refers to the doctrine of res ipsa loquitur, a doctrine that is unrelated to using a statute to establish he standard of care in negligence. C Incorrect. A statute may be used to establish the standard of care in negligence even when the statute was not enacted with the purpose of providing a private right of action in tort. D Incorrect. Proving the reasonable prudent person standard is not the only way that the standard of care can be established in negligence.

Furniture delivery person knocks on homeowner's door and announces, "We have a side table here for delivery." Homeowner lets the delivery person in the house and leads her to the dining room where the side table is to be assembled. Homeowner then leaves the area to go upstairs. While assembling the table in the dining room, the delivery person hears what she thinks is a puppy barking playfully in the basement of the house. While homeowner is upstairs the delivery person walks through the house looking for the basement door, finds the door and enters the basement and finds a toy dog barking. The delivery person laughs and returns to the dining room to assemble the table. While returning to the table the delivery person leaves a small shoe stain on the carpet in the basement. Has the delivery person trespassed? A Yes, because she likely exceeded the scope of the permission granted to her by the homeowner to enter the house. B No, because once the homeowner gave permission to enter the house the delivery person was free to go anywhere in the house. C Yes, because the delivery person caused actual damage when she stained the carpet. D No, if the reason the furniture delivery person entered the basement was to play with the puppy.

A Correct. Trespass to land is actionable if the defendant exceeds the scope of the permission to enter the land. Many examples of this are common with meter readers, delivery workers who are given permission to enter land for a specific purpose. If they exceed the scope of the permission granted then a permissible entry becomes a trespass. Here the facts do not indicate the delivery person was given permission to roam the entire house, rather she was given permission to be in the dining room and to do what was necessary to complete the installation. This likely did not encompass roaming the house and entering the basement to play with a dog. Another classic example of this is when customers enter a store to shop. Clearly they are invited into the shopping area and cannot be trespassing if they remain there. However, if once in the store they enter for example a locked area of the store with a sign saying, "Employees Only," then they morph from permissible entrants to trespassers. B Incorrect. Trespass to land is actionable if the defendant exceeds the scope of the permission to enter the land. Many examples of this are common with meter readers, delivery workers who are given permission to enter land for a specific purpose. If they exceed the scope of the permission granted then a permissible entry becomes a trespass. Here the facts do not indicate the delivery person was given permission to roam the entire house, rather she was given permission to be in the dining room and to do what was necessary to complete the installation. This likely did not encompass roaming the house and entering the basement to play with a dog. Another classic example of this is when customers enter a store to shop. Clearly they are invited into the shopping area and cannot be trespassing if they remain there. However, if once in the store they enter for example a locked area of the store with a sign saying, "Employees Only," then they morph from permissible entrants to trespassers. C Incorrect. It is premised on the incorrect rule that actual damage is necessary for trespass to be actionable. Trespass is actionable on nominal damages only and so the fact that the trespass caused damage or not relates only to the amount of damages but not to the existence of trespass. D Incorrect. It serves as a good review of one aspect of intent. The motivation behind an intentional act has no effect on intent. Even if her motivations to play with a puppy was not "unlawful, trespass is not negated. As long as there was the impermissible intentional entry onto the property of another then trespass exists.

Hiker gets lost in the woods. As hiker is trying to find his way out of the woods he is attacked by a bear. As the hiker is running from the attacking bear he sees a cabin in the woods. The door of the cabin has a conspicuous sign, saying, "Private Property. Keep Out." In desperation after reading the sign the hiker tries the door and it is unlocked. He enters the cabin and shuts the door behind him. About twenty minutes later hiker is rescued and leaves the cabin unharmed exactly as he found it. Owner of the cabin is furious that Hiker entered the cabin without permission and sues the hiker for trespass to land. Is the hiker liable for trespass? A No, hiker is not liable for trespass to land because he was exercising the privilege of private necessity. B Yes, hiker is liable for trespass to land because he intentionally entered the property in the possession of another without consent. C Yes, trespass to land is actionable on nominal damages only and so the fact that the hiker did no harm to the cabin is immaterial. D No, because a person may freely enter anyone's property as long as the entry will reasonably benefit the person.

A Correct. Under the privilege of private necessity a person may enter the land of another without permission in order to avoid serious and imminent harm to the person and the landowner has no right to exclude that person and the person is not liable for any nominal damages associated with trespass but would be liable for actual damages. Here there were no actual damages so there can be no liability. B Incorrect. Under the privilege of private necessity a person may enter the land of another without permission in order to avoid serious and imminent harm to the person and the landowner has no right to exclude that person and the person is not liable for any nominal damages associated with trespass. Here there were no actual damages so there can be no liability. C Incorrect. While it is true that trespass to land is actionable on nominal damages only. When a defendant successfully claims the defense of private necessity there is no liability for any nominal damages. D Incorrect. It is blatantly legally erroneous. It is simply an incorrect rule of law which if correct would prevent the existence of the trespass to land action.

City is 100 square miles and contains almost 60,000 trees along its streets. Barney is driving on a street during a windstorm when a tree on City property adjacent to the street falls on his car. Barney's car is damaged, and he suffers physical injuries. City had planted the tree some 15 years previously, and upon visual inspection by City workers, the tree appeared to be quite healthy. City has trained staff, which inspects each city tree annually, and employees of other City departments are under strict instruction to keep their eyes open for trees in a hazardous condition as they perform their regular duties. However, at the time of the storm, the tree was rotting due to rare disease from its inside out, and the weakened state of this particular tree caused it to fall under the pressure of the storm. Even when a tree does go bad as this one did, the chances of it suddenly falling in circumstances likely to produce serious property or personal injury are quite small. City had no actual knowledge of this tree's condition. Only a much more costly inspection program could have successfully detected the rotting tree that fell on Barney. In a negligence action brought by Barney against City, will Barney likely be able to hold city liable in negligence? A Barney will likely not be able to hold City liable in negligence because the cost to City of detecting the dangerous condition of the tree was prohibitive. B Barney will likely be able to hold City liable in negligence for failing to enact the more costly inspection program if it could have possibly prevented injuries such as the ones sustained by Barney. C Barney will likely not be able to hold City liable in negligence because the tree fell only due to the windstorm. D Barney will likely be able to hold City responsible in negligence because Barney was injured as a result of the tree's dangerous condition.

A Correct. We know from the facts provided that only a "much more costly inspection program could have successfully detected the rotting tree" and that the chance of a diseased tree falling and causing injury to person or property is "quite small." Therefore, guarding against the small chance of harm was cost prohibitive, and City's failure to do so is not unreasonable. B Incorrect. City is not required to enact the more costly inspection program. This question asks about the reasonableness of the care used by City in inspecting its trees. City is not required to guard against all potential injuries. It is required to act reasonably by taking into consideration the likelihood that harm will result, the foreseeable severity of any possible harm, and the burden of guarding against that harm. Restatement (Third) Torts § 3. C Incorrect. The windstorm is a contributing cause of the tree falling, but the windstorm does not necessarily prevent City from being responsible if City had otherwise behaved unreasonably (but it did not). D Incorrect. Imposition of negligence liability requires more than just a showing that City's tree led to Barney's injuries. Imposition of liability in negligence requires a showing that City's breach of the standard of care caused Barney's harm.

Two college students, A and B, were out drinking heavily one night. A third college student, C, who knew the drunk students watched as A and B passed out. C drove B to B's house and left B there. After doing this C thought it would be a great idea to drop the unconscious A on some random stranger's lawn. C pulled up next to an unfenced yard and dragged A onto the lawn. The next morning the irate property owner woke A up and asked A to leave. Did A and/or C trespass? A Neither A nor C trespassed B Only A trespassed C Only C trespassed D Both A and C trespassed

A Incorrect. A could not have trespassed because trespass requires an intentional act to enter or cause tangible entry onto the property of another. A was unconscious and incapable of acting voluntarily therefore A was incapable of intent and cannot have trespassed. C however acted intentionally to cause the impermissible entry of A onto the land of another and therefore C trespassed. It is important to note that a person can be liable for trespass if they cause tangible entry and not only if they actually enter land. For example if a person X throws a tangible object over a person Y's fence into their yard, person X may be liable for trespass even though person X did not actually enter the land of person Y, This is because person X caused tangible entry onto the land of person Y. B Incorrect. A could not have trespassed because trespass requires an intentional act to enter or cause tangible entry onto the property of another. A was unconscious and incapable of acting voluntarily therefore A was incapable of intent and cannot have trespassed. C Correct. A could not have trespassed because trespass requires an intentional act to enter or cause tangible entry onto the property of another. A was unconscious and incapable of acting voluntarily therefore A was incapable of intent and cannot have trespassed. C however acted intentionally to cause the impermissible entry of A onto the land of another and therefore C trespassed. It is important to note that a person can be liable for trespass if they cause tangible entry and not only if they actually enter land. For example if a person X throws a tangible object over a person Y's fence into their yard, person X may be liable for trespass even though person X did not actually enter the land of person Y, This is because person X caused tangible entry onto the land of person Y. D Incorrect. A could not have trespassed because trespass requires an intentional act to enter or cause tangible entry onto the property of another. A was unconscious and incapable of acting voluntarily therefore A was incapable of intent and cannot have trespassed. C however acted intentionally to cause the impermissible entry of A onto the land of another and therefore C trespassed. It is important to note that a person can be liable for trespass if they cause tangible entry and not only if they actually enter land. For example if a person X throws a tangible object over a person Y's fence into their yard, person X may be liable for trespass even though person X did not actually enter the land of person Y, This is because person X caused tangible entry onto the land of person Y.

Phone Co. lays underground cable throughout City. Phone Co. hires Indep as the independent contractor to perform the digging necessary for laying the cable. When the job starts, Indep has no previous digging or construction experience and only several hours of practice using a trenching machine. Prior to hiring Indep, Phone Co. did not inquire into Indep's experience using trenching machines or working around gas pipelines, the presence of which Phone Co. was reasonably certain to arise. During the course of the digging, Indep negligently strikes a gas line, causing an explosion that seriously injuries Tony. In a negligence action brought by Tony against Phone Co., will Phone Co. be subject to liability? A Phone Co. cannot be subject to liability to Tony because Indep was hired as an independent contractor. B Phone Co. may be subject to direct liability to Tony for failing to use reasonable care in selecting a competent contractor. C Phone Co. is subject to liability to Tony for failing to use reasonable care in digging around the gas pipeline because employers are vicariously liable for the tortious conduct of their independent contractors. D Phone Co. may be subject to vicarious liability to Tony for failing to use reasonable care in selecting a competent contractor.

A Incorrect. A defendant is still required to use reasonable care in selecting its independent contractors, which Phone Co. did not do. This question asks about a principle's liability for the tortious conduct of its independent contractors. Although often a defendant is not subject to liability for the tortious conduct of its independent contractors, there are some exceptions to that rule. Restatement (Third) Torts § 55 and illustration 1 upon which this fact pattern is based. B Correct. Tony is seeking to hold Phone Co. directly liable for its negligence in choosing its independent contractor. This question asks about a principle's liability for the tortious conduct of its independent contractors. Although often a defendant is not subject to liability for the tortious conduct of its independent contractors, there are some exceptions to that rule. Restatement (Third) Torts § 55 and illustration 1 upon which this fact pattern is based. C Incorrect. Tony is seeking to hold Phone Co. directly liable, not vicariously liable. This question asks about a principle's liability for the tortious conduct of its independent contractors. Although often a defendant is not subject to liability for the tortious conduct of its independent contractors, there are some exceptions to that rule. Restatement (Third) Torts § 55 and illustration 1 upon which this fact pattern is based. D Incorrect. It refers to Phone Co.'s vicarious liability rather than its direct liability. This question asks about a principle's liability for the tortious conduct of its independent contractors. Although often a defendant is not subject to liability for the tortious conduct of its independent contractors, there are some exceptions to that rule. Restatement (Third) Torts § 55 and illustration 1 upon which this fact pattern is based.

Alice sues Bill alleging negligence. Bill defends, claiming that Alice's negligence also contributed to her injuries. At trial, the factfinder assigns 50% responsibility to Alice and 50% responsibility to Bill. The factfinder also finds that Alice's damages are $100,000. In a modified comparative fault (51% or "not greater than") jurisdiction, what amount, if any, is Alice entitled to recover? A Alice is entitled to recover $50,000 from Bill because Bill only has to pay for his percentage of the judgment attributable to his fault. B Alice is entitled to recover $50,000 from Bill because her recoverable damages are her damages reduced by the amount of fault attributable to her. C Alice is entitled to recover $100,000 from Bill. D Alice is barred from recovering from Bill because she was partially at fault.

A Incorrect. A plaintiff's judgment is reduced, it is reduced by the amount of fault attributable to the plaintiff. This question asks about a plaintiff's recovery in a modified fault (51% or "not greater than") jurisdiction. Plaintiff's recoverable damages are not calculated based upon the defendant's percentage of responsibility. Restatement (Third) Torts § 7 and comment g. B Correct. In a modified not greater than jurisdiction, the plaintiff is not completely barred from recovery unless her fault is greater than that of the defendant. Here, plaintiff's fault is not greater than that of the defendant, so she can recover. However, plaintiff's recoverable damages are the amount of the judgment reduced by the percentage of fault attributed to the plaintiff. So Alice's recovery is $100,000-(50% of $100,000), which is $50,000. C Incorrect. As 50% responsible, Alice is not entitled to recover the entire $100,000. This question asks about a plaintiff's recovery in a modified fault (51% or "not greater than") jurisdiction. D Incorrect. In this modified comparative fault (51% not greater than) jurisdiction, Alice will not be completely barred from recovery.

Abe, who is negligently driving while intoxicated, is stopped at a red light. Betty negligently fails to stop and hits Abe's car in a rear-end collision. Abe sues Betty in negligence for personal injuries he suffered in the collision. In this comparative negligence jurisdiction, what effect will Abe's negligence have on his recovery? A Abe cannot recover in negligence from Betty because he was intoxicated at the time. B Abe's own negligence of being intoxicated will affect his recovery because his intoxication was a factual cause of his own injuries. C Abe's own negligence of being intoxicated does not affect Abe's recovery because Abe's intoxication was not a factual cause of his own injuries. D Abe's own negligence of being intoxicated will affect his recovery because his intoxication was a proximate cause of his own injuries.

A Incorrect. Abe's intoxication will only affect his recovery if his negligence was a factual cause of the accident, and it was not - but for his intoxication, Betty still would have rear-ended his vehicle, so Abe is not a factual cause of the collision. B Incorrect. Abe's intoxication will only affect his recovery if his negligence was a factual cause of the accident, and it was not - but for his intoxication, Betty still would have rear-ended his vehicle, so Abe is not a factual cause of the collision. C Correct. Abe's own negligence will affect his recovery only if it was a factual cause of his injuries. This question asks about the requirement of causation as part of a comparative negligence defense. Restatement (Third) Torts § 4. D Incorrect. Abe's negligence cannot be a proximate cause if it is not first determined to be a factual cause.

Max owns a furniture store and employs Alex to deliver furniture to retail customers. Alex's duties include entering customers' homes to situate items they have purchased. Having entered Customer's home to deliver a sofa, Alex assaults Customer. Prior to employing Alex, Max conducted no check of Alex's background, although a reasonable employer of an employee who would be entering the homes of customers would have conducted such a check. Had Max done so, Max would have discovered criminal convictions for assault and would not have employed Alex to make deliveries. Customer seeks to hold Max directly liable in negligence for Customer's injuries. Can Max be held directly liable for Customer's injuries? A Max cannot be held directly liable for Customer's injuries because an employer cannot be held liable for the intentional tortious acts of its employees. B Max cannot be held directly liable for Customer's injuries because an employer is not responsible for the criminal acts of its employees. C Max may be held directly liable in negligence because employers are always responsible for the wrongful conduct of their employees. D Max may be held directly liable for Customer's injuries because the injuries for which Customer seeks recovery are foreseeable from Max's negligence in not conducting a background check.

A Incorrect. An employer may be subject to direct liability for the wrongful conduct of its employees under some circumstances, such as the one presented by these facts. This question asks about holding an employer liable for its negligence in hiring an employee. An employer may be held directly liable for its employee's wrongful conduct when the employer was negligent in selecting the employee. Restatement (Third) Agency § 7.03. B Incorrect. If an employee's criminal acts are foreseeable from the employers negligence, the employer may be subject to direct liability. This question asks about holding an employer liable for its negligence in hiring an employee. An employer may be held directly liable for its employee's wrongful conduct when the employer was negligent in selecting the employee. Restatement (Third) Agency § 7.03. C Incorrect. It is untrue that an employer is always responsible for the wrongful conduct if its employees. This question asks about holding an employer liable for its negligence in hiring an employee. An employer may be held directly liable for its employee's wrongful conduct when the employer was negligent in selecting the employee. Restatement (Third) Agency § 7.03. D Correct. The employer may be subject to direct liability under these circumstances for his negligence. This question asks about holding an employer liable for its negligence in hiring an employee. An employer may be held directly liable for its employee's wrongful conduct when the employer was negligent in selecting the employee. Restatement (Third) Agency § 7.03.

Tracie and Francis were soccer rivals. One day before a game between their respective teams Tracie whispered to Francis, "I am going to beat you down immediately after this 90 minute game is over if you score. Two minutes into the game Francis scored a goal. Immediately after scoring he experienced nervousness and became fearful of what Tracie would do to him after the game. Surprisingly, Francis fainted as a result of the fear, hit his head on the pitch, and had to be carried off unconscious in a stretcher. Should Francis sue for assault and/or battery? A Francis should sue for assault and battery. B Incorrect. Assault is defined as intentionally causing fear or apprehension of imminent harmful or offensive contact. Here, Tracie intentionally caused fear or apprehension of harmful or offensive, but the contact was not imminent. The threatened contact was at the beginning of a 90 minute game and therefore was not imminent. Good examples of imminence is swinging a bat at someone's face but stopping an inch short of hitting them, making someone jump, etc. Battery is also not actionable because even though Tracie threatened to cause harmful or offensive contact, the contact that happened was not substantially certain to occur. The facts say it wasn't even predictable because it was surprising that contact occurred. C Francis should sue for battery but not assault. D Francis should not sue for assault nor should he sue for battery.

A Incorrect. Assault is defined as intentionally causing fear or apprehension of imminent harmful or offensive contact. Here, Tracie intentionally caused fear or apprehension of harmful or offensive, but the contact was not imminent. The threatened contact was at the beginning of a 90 minute game and therefore was not imminent. Good examples of imminence is swinging a bat at someone's face but stopping an inch short of hitting them, making someone jump, etc. Battery is also not actionable because even though Tracie threatened to cause harmful or offensive contact, the contact that happened was not substantially certain to occur. The facts say it wasn't even predictable because it was surprising that contact occurred. B Incorrect. Assault is defined as intentionally causing fear or apprehension of imminent harmful or offensive contact. Here, Tracie intentionally caused fear or apprehension of harmful or offensive, but the contact was not imminent. The threatened contact was at the beginning of a 90 minute game and therefore was not imminent. Good examples of imminence is swinging a bat at someone's face but stopping an inch short of hitting them, making someone jump, etc. Battery is also not actionable because even though Tracie threatened to cause harmful or offensive contact, the contact that happened was not substantially certain to occur. The facts say it wasn't even predictable because it was surprising that contact occurred. C Incorrect. Assault is defined as intentionally causing fear or apprehension of imminent harmful or offensive contact. Here, Tracie intentionally caused fear or apprehension of harmful or offensive, but the contact was not imminent. The threatened contact was at the beginning of a 90 minute game and therefore was not imminent. Good examples of imminence is swinging a bat at someone's face but stopping an inch short of hitting them, making someone jump, etc. Battery is also not actionable because even though Tracie threatened to cause harmful or offensive contact, the contact that happened was not substantially certain to occur. The facts say it wasn't even predictable because it was surprising that contact occurred. D Correct. Assault is defined as intentionally causing fear or apprehension of imminent harmful or offensive contact. Here, Tracie intentionally caused fear or apprehension of harmful or offensive but the contact was not imminent. The threatened contact was at the beginning of a 90 minute game and therefore was not imminent. Good examples of imminence is swinging a bat at someone's face but stopping an inch short of hitting them, making someone jump, etc. Battery is also not actionable because even though Tracie threatened to cause harmful or offensive contact, the contact that happened was not substantially certain to occur. The facts say it wasn't even predictable because it was surprising that contact occurred.

Sleeping Beauty had been asleep for years. One day Prince Charming strolled into town. He saw Sleeping Beauty and out of the goodness of his heart decided to kiss her. His only motivation in kissing her was to help her because he knew the kiss would wake her up. Two weeks after the kiss, sleeping beauty woke up. She was completely unaware at this point that she had been kissed by the long gone prince. After she woke up she ran into her friend Rapunzel who said to her, "You know Prince Charming came by 2 weeks ago and kissed you and that is the reason you are now awake."Even though she was physically and emotionally unharmed, the unsolicited kiss from Prince charming was something both she and a reasonable person would find offensive. What cause(s) of action are at least actionable from these facts? A Assault and Battery and Intentional Infliction of Emotional Distress B Assault C Intentional Infliction of Emotional Distress D Battery

A Incorrect. Assault requires fear or apprehension of imminent harmful or offensive contact. Because Sleeping Beauty did not sense the kiss before it happened, assault clearly is not actionable. Intentional Infliction of Emotional Distress is likewise not actionable because this tort requires that the tortfeasor intend to cause severe emotional distress. The facts state that Prince Charming's intention (albeit sexist and misplaced) was not to cause harm but to help. B Incorrect. Assault requires fear or apprehension of imminent harmful or offensive contact. Bc Sleeping Beauty didn't sense the kiss before it happened, assault clearly isn't actionable. C Incorrect. Intentional Infliction of Emotional Distress is likewise not actionable because this tort requires that the tortfeasor intend to cause severe emotional distress. The facts state that Prince Charming's intention (albeit sexist and misplaced) was not to cause harm but to help. D Correct. Battery is defined as intentionally causing harmful or offensive contact to the person of another. Unlike assault, which requires fear or apprehension, battery does not require an element of sensory awareness of the contact. Prince Charming unquestionably had the intent to cause contact to Sleeping Beauty so intent is satisfied. He did in fact cause contact and the contact was the type of contact that would have been harmful or offensive to a reasonable person. The tort is therefore satisfied. Additionally, battery is a dignitary tort so Sleeping Beauty may sue in Battery even if there was no actual harm.

Lessee of house was away on vacation when unauthorized person entered his apartment hung out for a few hours but otherwise caused no damage. Upon his return from vacation the lessee sues the unauthorized person for trespass to land. May lessee successfully sue unauthorized entrant for trespass to land? A No, because the lessee was unaware of the unauthorized entry at the time it occurred. B No, because the lessee is not the owner of the house he cannot sue the unauthorized entrant. C No, because there was no damage. D Yes, because even though lessee did not own the house he was in possession of the house.

A Incorrect. Awareness of the trespass as it occurred is not a prerequisite for suing in trespass to land. Compare this to IIED where being aware of the harm as it occurred is an element of bystander liability for IIED. B Incorrect. Standing to sue in trespass to land does not require ownership of the property where the trespass occurred. Only possession of the property is required to sue in trespass and a lessee possesses the property they rent. C Incorrect. Trespass to land does not require actual damage to be actionable as it is actionable on nominal damages only. D Correct. A possessor of real property has standing to sue in trespass to land. Here the lessee even though not the owner, possesses the house because of the lease arrangement.

Law student is angry at Torts professor because professor made student work and professor is always so prepared in class. As professor is walking to his car in his perfectly matching designer shoes student sneaks up behind him and with a Torts casebook, hits professor on the back of his head. Professor slumps unconscious to the ground. The student was so stealthy that the professor never even knew what happened until he regained consciousness. Are the student's action assault and/or battery? A The students actions are both an assault and battery. B The students actions are neither an assault nor a battery. C The students actions are an assault but not a battery. D The students actions are a battery but not an assault.

A Incorrect. Battery is intentionally causing contact to the person of another which is harmful or offensive. Clearly the student intended to cause contact to the professor. He did cause contact. Being hit on the back of the head hard enough to lose consciousness is clearly harmful or offensive. So clearly there is a battery. However, there is no assault. Assault is intentionally causing fear or apprehension of imminent harmful or offensive contact. The facts state that the student was so stealthy that the professor did not know what happened until he woke up. His lack of awareness forecloses fear or apprehension. B Incorrect. Battery is intentionally causing contact to the person of another which is harmful or offensive. Clearly the student intended to cause contact to the professor. He did cause contact. Being hit on the back of the head hard enough to lose consciousness is clearly harmful or offensive. So clearly there is a battery. However, there is no assault. Assault is intentionally causing fear or apprehension of imminent harmful or offensive contact. The facts state that the student was so stealthy that the professor did not know what happened until he woke up. His lack of awareness forecloses fear or apprehension. C Incorrect. Battery is intentionally causing contact to the person of another which is harmful or offensive. Clearly the student intended to cause contact to the professor. He did cause contact. Being hit on the back of the head hard enough to lose consciousness is clearly harmful or offensive. So clearly there is a battery. However, there is no assault. Assault is intentionally causing fear or apprehension of imminent harmful or offensive contact. The facts state that the student was so stealthy that the professor did not know what happened until he woke up. His lack of awareness forecloses fear or apprehension. D Correct. Battery is intentionally causing contact to the person of another which is harmful or offensive. Clearly the student intended to cause contact to the professor. He did cause contact. Being hit on the back of the head hard enough to lose consciousness is clearly harmful or offensive. So clearly there is a battery. However, there is no assault. Assault is intentionally causing fear or apprehension of imminent harmful or offensive contact. The facts state that the student was so stealthy that the professor did not know what happened until he woke up. His lack of awareness forecloses fear or apprehension.

As Pedestrian was walking along the street on the edge of Homeowner's property, Homeowner decided to play a prank on Pedestrian. Homeowner opened the door of his house facing the street where Pedestrian was walking. He stood in the open doorway with his huge ferocious dog and called to Pedestrian, "Hey." As Pedestrian looked across the 15 yards of lawn between the street and the doorway he was petrified, and yelled to Homeowner, "make sure you hold on to that monster." At which point Homeowner yelled ATTACK!!!!!!!! and released the dog The dog sprinted towards Pedestrian baring it's teeth and barking ferociously. Pedestrian froze with fear but just before as the dog reached the edge of the property less than 2 feet from where Pedestrian was in the street it suddenly stopped and moved back away from the property line and sat on the lawn. Relieved Pedestrian walked away scared and almost in shock because the fear he had just experienced. Homeowner knew the dog would not reach Pedestrian because of the buried electronic fence installed in the yard which was connected to the dog's collar and prevented the dog from reaching the street. If Pedestrian sues Homeowner for battery Pedestrian will: A Win because the dog caused him to be fearful of harmful or offensive contact B Win because the dog entered into what could be considered Pedestrian's personal space C Lose even if the dog had touched Pedestrian there was no direct contact between Homeowner and Pedestrian D Lose because Homeowner did not cause any harmful or offensive contact to Pedestrian

A Incorrect. Battery is intentionally causing harmful or offensive bodily contact to the person of another. In order for battery to exist the tortfeasor must have caused to the plaintiff and the contact caused by the actions of the tortfeasor must be harmful or offensive. Here there was no contact with Pedestrian so there can be no battery. While this may be an actionable assault it is not a battery because there was no contact. B Incorrect. Almost contact is not legally sufficient to be contact. There must be actual contact with the plaintiff to be considered battery. While it is true that contact with objects in intimate association with the body is considered contact with the plaintiff, there is no contact here. C Incorrect. Battery does not require direct contact between the plaintiff and defendant to be actionable. Shooting the defendant intentionally, for example, may be a battery even if the plaintiff was not touched by the defendant. As long as the defendant intentionally caused contact to the person of another, the contact element is satisfied. D Correct. There was no contact to Pedestrian.

Law student is angry at Torts professor because professor made student work and professor is always so prepared in class. As professor is walking to his car in his perfectly matching designer shoes student confronts him and says, "Professor, if I do not understand Legal Analysis in two weeks I will strike you as hard as I can with my casebook across your head." The professor immediately becomes afraid and becomes worried that in two weeks he will be subject to harmful and offensive contact. Are the student's actions assault and/or battery? A The students actions are both an assault and battery. B The students actions are neither an assault nor a battery. C The students actions are an assault but not a battery. D The students actions are a battery but not an assault.

A Incorrect. Battery requires contact to be actionable and here there is not contact. The lack of assault is perhaps a little more nuanced. Assault is intentionally causing fear or apprehension of imminent harmful or offensive contact. Here the question states that the professor is immediately afraid or in other words it subtly suggests that his fear is immediate. However, it is to the fear that is subject to the imminence test in assault but the contact one is fearful of. Here the threatened contact was two weeks in the future and so it was that contact and not an imminent contact that the professor was fearful of. This lack of imminent harmful or offensive contact negates the possibility of an assault claim. B Correct. Battery requires contact to be actionable and here there is not contact. The lack of assault is perhaps a little more nuanced. Assault is intentionally causing fear or apprehension of imminent harmful or offensive contact. Here the question states that the professor is immediately afraid or in other words it subtly suggests that his fear is immediate. However, it is to the fear that is subject to the imminence test in assault but the contact one is fearful of. Here the threatened contact was two weeks in the future and so it was that contact and not an imminent contact that the professor was fearful of. This lack of imminent harmful or offensive contact negates the possibility of an assault claim. C Incorrect. Battery requires contact to be actionable and here there is not contact. The lack of assault is perhaps a little more nuanced. Assault is intentionally causing fear or apprehension of imminent harmful or offensive contact. Here the question states that the professor is immediately afraid or in other words it subtly suggests that his fear is immediate. However, it is to the fear that is subject to the imminence test in assault but the contact one is fearful of. Here the threatened contact was two weeks in the future and so it was that contact and not an imminent contact that the professor was fearful of. This lack of imminent harmful or offensive contact negates the possibility of an assault claim. D Incorrect. Battery requires contact to be actionable and here there is not contact. The lack of assault is perhaps a little more nuanced. Assault is intentionally causing fear or apprehension of imminent harmful or offensive contact. Here the question states that the professor is immediately afraid or in other words it subtly suggests that his fear is immediate. However, it is to the fear that is subject to the imminence test in assault but the contact one is fearful of. Here the threatened contact was two weeks in the future and so it was that contact and not an imminent contact that the professor was fearful of. This lack of imminent harmful or offensive contact negates the possibility of an assault claim.

Law student is angry at Torts professor because professor made student work and professor is always so prepared in class. As professor is walking to his car in his perfectly matching designer shoes the student confronts him and screams, "I am going to hit you with this book for being so prepared." Student did not intend to hit him but just to scare him by swinging the book close to his head and making him flinch. If everything went exactly as planned are the student's actions assault and/or battery? A The students actions are both an assault and battery. B The students actions are neither an assault nor a battery. C The students actions are an assault but not a battery. D The students actions are a battery but not an assault.

A Incorrect. Battery requires contact. Here there was no contact so there can be no battery. B Incorrect. While there is no battery because battery requires contact. There is an assault. Assault is the intentionally causing fear or apprehension of imminent harmful or offensive contact. Here the student intended to scare the professor and the professor's flinch indicates he was at fearful or apprehensive of what he perceived to be imminent contact. C Correct. While there is no battery because battery requires contact. There is an assault. Assault is the intentionally causing fear or apprehension of imminent harmful or offensive contact. Here the student intended to scare the professor and the professor's flinch indicates he was at fearful or apprehensive of what he perceived to be imminent contact. D Incorrect. While there is no battery because battery requires contact. There is an assault. Assault is the intentionally causing fear or apprehension of imminent harmful or offensive contact. Here the student intended to scare the professor and the professor's flinch indicates he was at fearful or apprehensive of what he perceived to be imminent contact.

Randy was Angry at Big Brock so Randy hatched a plan. He would sneak up on Brock while Brock was sleeping in his tent and point his rifle into the center of the tent where he knew Brock always slept and shoot, killing Brock. Randy waited until Brock was camping out in the wilderness and he snuck up to the tent. He stuck the barrel of the rifle through the opening of the tent and fired into the center of the tent without looking. It turns out that Brock was nowhere near the tent and Randy ended up shooting only Brock's expensive sleeping bag, completely destroying it. This was curious since Brock never slept with a sleeping bag before and Randy knew this. Brock discovered all that had transpired after he returned to his tent and other campers described what had happened. Brock seeks your advice to recover for the value of the sleeping bag. Which of the following causes of action would be most likely to succeed? A Assault and battery B Conversion because the sleeping bag was pretty much destroyed C Trespass to chattel D Battery but not assault

A Incorrect. Brock was not there, and as a result would not have suffered fear or apprehension of imminent harmful or offensive contact when the gun was pointed through the tent's opening. Because essential elements of the tort of assault are absent there can be no assault. Additionally, battery requires contact with the person of another and Brock was not hit so there can be no battery. B Incorrect. The sleeping bag was destroyed because there was no intent to damage the bag. The facts state that Brock never slept with a bag before and Randy knew that. As a result, Randy could not have had specific intent to destroy the bag nor was he substantially certain that his actions would destroy the bag, which is a chattel. Because the doctrine of transferred intent operates only when both the tort attempted and the tort achieved are one of the five torts within the original writ of trespass (Battery, Assault, False Imprisonment, Trespass to Land and Trespass to Chattel) the intent to batter Brock does not transfer to satisfy the intent element of the tort of conversion. C Correct. The intent to batter will transfer to satisfy the intent element of trespass to chattel. Because the resulting tort involved interference with Brock's chattel or personal property, then trespass to chattel would be most likely to succeed. D Incorrect. Battery requires contact with the person of another, and Brock was not hit so there can be no battery.

Curtis was a guest at Hotel where at 9:00 p.m. one evening there was a power outage that deprived Hotel of electricity. As there was no emergency lighting in his dark room, Curtis fell and injured himself. Curtis sued Hotel for his injuries, alleging that Hotel was negligent in not having inexpensive battery-powered lighting fixtures installed in its guest rooms. Curtis alleges that if his room had such a fixture, he would have avoided injury. The only expert to testify at trial explains that at the time of Curtis's injury, it was not customary in the hotel industry for in-room emergency lighting of this sort to be provided. May Hotel be subject to liability in negligence although if it was in compliance with customary hotel industry standards by not providing in-room emergency lighting? A No, the evidence of Hotel's compliance with customary hotel industry standards, by not providing in-room emergency lighting, is conclusive proof that Hotel was not negligent. B Yes, the evidence of Hotel's compliance with customary hotel industry standards is legally irrelevant in determining whether Hotel was negligent. C Yes, Hotel may be liable for Curtis's injuries despite the industry custom if requiring emergency lighting in hotel rooms is reasonable with respect to the risk of guests injuring themselves during a power outage. D No, Hotel may be liable for Curtis's injuries despite the industry customs because Curtis was injured in one of Hotel's room through no fault of his own.

A Incorrect. Compliance with industry custom is only evidence that the defendant was not negligent; it is not conclusive proof that the defendant was not negligent. B Incorrect. Evidence of a defendant's compliance with custom may be taken into consideration by the factfinder in determining whether on the occasion in question the defendant breached the standard of care. Compliance with custom is relevant evidence, but is not determinative of negligence. C Correct. This question asks about the use of custom as evidence of a defendant's negligence. A defendant's compliance with customary standards does not preclude a finding of negligence. Restatement (Third) Torts § 13. D Incorrect. More must be shown than just that Curtis was injured in Hotel's room through no fault of his own for Hotel to be liable in negligence. There must be a showing that the Hotel breached the standard of care.

A and B are on a first date. A says to B, "I would really like to kiss you." B responds by shrugging her shoulders. A then kisses B. If B sues A for battery which of the following if true would be A's best defense? A A honestly believed B consented by shrugging her shoulders. B A reasonably believed B consented by shrugging her shoulders. C A did not know what the shrug meant but kissed B anyway. D A only intended to make B happy when he decided to kiss her.

A Incorrect. Consent is an objective standard. In other words consent is only a defense to the battery claim if A reasonably believed B consented to the kiss when she shrugged. B Correct. Consent is an objective standard. If a person reasonably believes another consented to an act then consent exists even if subjectively the other person did not consent. C Incorrect. It again relies on A's subjective belief to determine the validity of the consent. However consent is an objective test and exists only when a reasonable person would believe it exists. D Incorrect. Remember that motivation is not the same as intent. A person can really believe they are causing contact to help another, but if that contact would be harmful or offensive to a reasonable person then a battery exists if the contact occurs.

Soccer player asks permission to use soccer field from field owner. Field owner gives consent for soccer player to use soccer field between 4:00 pm and 6:00 pm. After using the field a few times between 4 and 6 pm, soccer player notices no one uses the field after 6:00 pm and decides to use the field even after the time consented to by the owner. May owner sue soccer player for trespass to land due to soccer player's use of the field after 6:00 pm? A No, because field owner consented to use of the field. B Yes, because soccer player exceeded the scope of the consent granted. C No, unless the soccer player harmed the field when using it after 6:00 pm. D Yes, if owner had previously cautioned the soccer player at least once before when soccer player used the field after 6:00 pm.

A Incorrect. Consent is not an all or nothing proposition. Consent to permit conduct which otherwise would be tortious may be limited in scope. For example here the consent to allow use of the property was limited to between 4:00 and 6:00 pm. Any other use would constitute a trespass to land. B Correct. Consent to permit conduct which otherwise would be tortious may be limited in scope. For example here the consent to allow use of the property was limited to between 4:00 and 6:00 pm. Any other use would constitute a trespass to land. Here the scope of the consent was limited to use between 4:00 and 6:00 pm and the soccer player exceeded the scope of that consent by using it at other times. C Incorrect. Trespass does not require actual damages to be actionable. D Incorrect. When consent is limited there is no consent for acts outside the limited scope of consent. There is no need for the owner to warn the soccer player before a trespass is actionable.

E and F have been friends for a long time. For years they have played a game where they hit each other on the shoulder. The game starts by one hitting the other and then the other returns the hit slightly harder. This sequence continues until one person is hit hard enough and "surrenders." One day F hits E on the shoulder in an attempt to start the game. E sues F for battery. What is the likely result of this suit? A E will win because F hit him. B F will win because E consented to being hit. C E will win because he never consented to this specific episode of contact. D F will win because once you consent to being hit you can never recover for being hit in battery.

A Incorrect. Consent may be implied from a course of conduct between the parties. Here these two friends have engaged in similar conduct for years. Unless something changed or was said there was implied consent for similar contact between the two and therefore the battery suit would be unsustainable. B Correct. Consent may be implied from a course of conduct between the parties. Here these two friends have engaged in similar conduct for years. Unless something changed or was said there was implied consent for similar contact between the two and therefore the battery suit would be unsustainable. C Incorrect. Consent may be implied from a course of conduct between the parties. Here these two friends have engaged in similar conduct for years. Unless something changed or was said there was implied consent for similar contact between the two and therefore the battery suit would be unsustainable. D Incorrect. While it is true that consent may be implied from a course of conduct between the parties consent is not absolute or nor is it irrevocable. At any time one of the parties could have said, "I no longer want to play the game," for example and the consent would have been revoked making any future contact a battery.

Car dealer asks landowner if he can use landowner's unoccupied field to store some cars for the night as his lot will be temporarily beyond capacity while new inventory arrives. In reality and unknown to the landowner the car dealer wanted to hide some stolen cars on the landowner's property. When the police discovered the stolen cars on landowner's field they attempted to arrest landowner. Landowner quickly recounted to the police why the cars were there and the police arrested car dealer instead. Landowner sues car dealer for trespass to land for the time the cars were on his field. What result? A Car dealer will prevail because landowner consented to the cars being on his land. B Car dealer will prevail because he did not exceed the scope of the consent given. C Car dealer will lose because he did not obtain valid consent. D Car dealer will lose because the landowner's reputation was actually harmed when the police arrested him.

A Incorrect. Consent obtained by fraud is not valid consent. Therefore because the car dealer lied to the landowner to obtain consent, there really was no consent. B Incorrect. Consent obtained by fraud is not valid consent. Therefore because the car dealer lied to the landowner to obtain consent, there really was no consent. So the issue of scope of consent is not relevant because no consent was obtained. C Correct. Consent obtained by fraud is not valid consent. Therefore because the car dealer lied to the landowner to obtain consent, there really was no consent. D Incorrect. Trespass does not require actual harm to be actionable.

Customer left her waterproof coat on a coat rack at the entrance to a restaurant. Thief decided to steal the coat and keep it for himself. After 3 hours the thief had a change of heart and figured out where customer lived. Thief returned the coat to customer's address unharmed. However because of Thief's actions Customer had to leave the restaurant without her coat. As she was walking home it rained suddenly and heavily completely destroying Customer's expensive portable computer which except for Thief's actions would have been in the waterproof pocket of the coat and would not have been damaged. Does Customer have an action for trespass to chattel or conversion against Thief? A Customer has both an action for conversion and for trespass to chattel against thief. B Customer only has an action for trespass to chattel against thief but not an action for conversion. C Customer only has an action for conversion against thief but not trespass to chattel. D Customer has neither an action for trespass to chattel nor conversion against thief.

A Incorrect. Conversion requires substantial interference by control. As the restatement suggests, the ultimate question is whether the defendant "exercised so much control over the chattel that courts can justly require him to pay its full value." Many of the decisions indicate that the 3 hour time period of control would not be substantial enough to support conversion. The rule is oft stated that in order for trespass to chattel to be actionable the chattel actual harm to the chattel must occur. Here there is no actual harm to the chattel and at first blush the tort would therefore not seem actionable. However, when interference with the chattel causes harm which would not otherwise have occurred if the chattel was not interfered with then the tort is actionable. In this case even though the coal was actually harmed the plaintiff's expensive portable computer was destroyed as a result of the interference with the coat. Therefore trespass to chattel might be actionable but not conversion. B Correct. Conversion requires substantial interference by control. As the restatement suggests, the ultimate question is whether the defendant "exercised so much control over the chattel that courts can justly require him to pay its full value." Many of the decisions indicate that the 3 hour time period of control would not be substantial enough to support conversion. The rule is oft stated that in order for trespass to chattel to be actionable the chattel actual harm to the chattel must occur. Here there is no actual harm to the chattel and at first blush the tort would therefore not seem actionable. However, when interference with the chattel causes harm which would not otherwise have occurred if the chattel was not interfered with then the tort is actionable. In this case even though the coal was actually harmed the plaintiff's expensive portable computer was destroyed as a result of the interference with the coat. Therefore trespass to chattel might be actionable but not conversion. C Incorrect. Conversion requires substantial interference by control. As the restatement suggests, the ultimate question is whether the defendant "exercised so much control over the chattel that courts can justly require him to pay its full value." Many of the decisions indicate that the 3 hour time period of control would not be substantial enough to support conversion. The rule is oft stated that in order for trespass to chattel to be actionable the chattel actual harm to the chattel must occur. Here there is no actual harm to the chattel and at first blush the tort would therefore not seem actionable. However, when interference with the chattel causes harm which would not otherwise have occurred if the chattel was not interfered with then the tort is actionable. In this case even though the coat was actually harmed the plaintiff's expensive portable computer was destroyed as a result of the interference with the coat. Therefore trespass to chattel might be actionable but not conversion. D Incorrect. Conversion requires substantial interference by control. As the restatement suggests, the ultimate question is whether the defendant "exercised so much control over the chattel that courts can justly require him to pay its full value." Many of the decisions indicate that the 3 hour time period of control would not be substantial enough to support conversion. The rule is oft stated that in order for trespass to chattel to be actionable the chattel actual harm to the chattel must occur. Here there is no actual harm to the chattel and at first blush the tort would therefore not seem actionable. However, when interference with the chattel causes harm which would not otherwise have occurred if the chattel was not interfered with then the tort is actionable. In this case even though the coal was actually harmed the plaintiff's expensive portable computer was destroyed as a result of the interference with the coat. Therefore trespass to chattel might be actionable but not conversion.

Plaintiff is injured when Defendant briefly loses control of the car he was driving when he reaches for his cell phone he dropped on the floorboard and hits Plaintiff who was standing on the street corner. Witness was a passenger in the Defendant's car at the time of the accident and saw everything that happened. In the negligence action brought by Plaintiff against Defendant to recover for her injuries, Witness testifies that she was riding in Defendant's car while Defendant was driving, that she heard Defendant's cell ring, that Defendant went to answer his cell phone while he was driving but dropped it on the floorboard of the car, and that when Defendant went to pick up his cell phone while he was driving, he took his eyes off the road for a brief period of time during which he lost control of the car and struck Plaintiff. What is the proper characterization of the evidence presented at Plaintiff's trial against Defendant for Defendant's alleged negligence? A Witness's testimony is circumstantial evidence of Defendant's conduct that may be used to prove that Defendant breached the standard of care. B Witness's testimony is circumstantial evidence of Defendant's conduct that may not be used to prove that Defendant breached the standard of care. C Witness's testimony is direct evidence of Defendant's negligence and does not require the factfinder to draw an inference regarding Defendant's breach of the standard of care. D Witness's testimony is direct evidence of Defendant's negligence and requires the factfinder to draw an inference regarding the Defendant's breach of the standard of care.

A Incorrect. Direct evidence is evidence that proves the fact sought to be established, e.g., breach of the standard of care. Circumstantial evidence is evidence that requires the factfinder to make one or more inferences to conclude the fact sought to be established. Witness's eyewitness testimony of Defendant's conduct leading to him running into Plaintiff with his car is direct evidence of his breach of the standard of care. B Incorrect. Direct evidence is evidence that proves the fact sought to be established, e.g., breach of the standard of care. Circumstantial evidence is evidence that requires the factfinder to make one or more inferences to conclude the fact sought to be established. Witness's eyewitness testimony of Defendant's conduct leading to him running into Plaintiff with his car is direct evidence of his breach of the standard of care. C Correct. Witness's testimony of Defendant's unreasonable conduct while driving is direct evidence of Defendant's negligence (breach of the standard of care). D Incorrect. It defines direct evidence improperly as direct evidence requires that the factfinder make no inference of the fact sought to be proven.

Pedestrian is walking down the street when she notices person about to be hit with a baseball bat by screaming attacker. Pedestrian is a martial artist with years of experience and disarms attacker before he can harm person. In the process of disarming the screaming attacker she twists his arm and pins his arm, throws him to the street and holds him there until the police arrive. Does attacker have a viable suit against pedestrian for battery? A Yes, because pedestrian intentionally caused harmful or offensive contact to the screaming attacker. B No, if pedestrian's actions were reasonable. C Yes, because pedestrian does not have a right to interfere in the affairs of others. D No, because a person may use any force to defend another person from tortious conduct.

A Incorrect. Even though pedestrian's actions satisfy all the elements of battery the suit against pedestrian would not be viable because pedestrian was likely operating under the privilege of defense of others. B Correct. A person may use reasonable force to defend another person from an attack that appears to threaten imminent harm. This privilege operates exactly like the privilege of self defense. C Incorrect. It is an incorrect statement of the law. A person may use reasonable force to defend another person from an attack that appears to threaten imminent harm. This privilege operates exactly like the privilege of self defense. D Incorrect. A person may use reasonable force to defend another person from an attack that appears to threaten imminent harm. This answer choice says, "any force," is permissible in defense of others but only reasonable force is permitted.

Employee worked as the maintenance manager for a 500 acre estate. The entire estate was fenced. One day the owner of the estate got angry at employee. He locked employee inside the estate and called employee on the telephone, "I am angry at you, and so you won't be allowed to leave the estate for a week." Employee was forced to stay in the estate for the week. During the week, and with the permission of the owner of the estate, the employee lounged in the fancy house on the estate and consumed the fancy food and drink. The employee's view was, "As long as I am forced to stay here I might as well make the best of it and enjoy the level of luxury that I am normally not privy to." In his suit for false imprisonment against the owner: A Employee will not prevail because the grounds were more luxurious than he was used to B Employee will prevail even though the grounds were more luxurious than he was used to C Employee will not prevail because 500 acres is too large an area to be considered imprisonment D Employee will prevail because the confinement was for more than a short time

A Incorrect. False imprisonment is the intentional confinement of the plaintiff within identifiable boundaries. Confinement implies that the plaintiff is held against his or her will. Here, the luxury aspect of it might reduce the harm suffered by the employee and hence the damages compared to if he was confined, for example, in a filthy small cell with no food and water. But false imprisonment exists simply because he was restrained against his will. Unless there is authority of justification or another privilege existing, there is a right to be free from confinement. In other words, false imprisonment is a dignitary tort and it is the impermissible restraint of the employee in this case which is the basis of the action. B Correct. False imprisonment is the intentional confinement of the plaintiff within identifiable boundaries. Confinement implies that the plaintiff is held against his or her will. Here, the luxury aspect of it might reduce the harm suffered by the employee and hence the damages compared to if he was confined, for example, in a filthy small cell with no food and water. But false imprisonment exists simply because he was restrained against his will. Unless there is authority of justification or another privilege existing, there is a right to be free from confinement. In other words, false imprisonment is a dignitary tort and it is the impermissible restraint of the employee in this case which is the basis of the action C Incorrect. Generally, the size of the confined area does not affect the existence of the tort. Again, confinement in a large area such as a three city blocks may not be as harmful as confinement in a small space where the person can't stand, for example. However, it is the fact that confinement occurred which is the basis of the torts. It is important to note that the cases all recognize that at some point in time the area of confinement is so large that it can no longer be considered confinement. Cases have recognized that confinement within the country of Taiwan is too large and area for false imprisonment. Similarly, confinement on Earth and in North America would likewise seem to involve way too large areas for the tort to be actionable. D Incorrect. It implies that there must be some minimum length of confinement before the tort can be actionable, but that is not the case. So while it is correct that there was confinement and it was for more than a short time, the tort is not actionable "because" the confinement was for more than a short time. Theoretically, any length of time of impermissible intentional restraint supports an action for false imprisonment. The longer the restrain the higher the damages usually, but restraint "for a long time" is not a necessary element of the tort.

Homeowner hears on the news that there is a dangerous criminal on the loose in her neighborhood. She is on edge as a result and hyper vigilant. One evening at dusk she is taking out her trash to the street and is startled by her neighbor who walked up behind her to give her some pie. Thinking it was the criminal approaching her she swings a large piece of wood at the neighbor causing him a pretty painful broken arm. Neighbor sues homeowner for battery. Homeowner argues that even though she committed a battery there should be no liability because she reasonably mistook the neighbor for the dangerous criminal and she defended herself accordingly. Will Homeowner's defense succeed? A No, because she made a mistake. B Yes, if factfinder concludes she made a reasonable mistake. C No, because she used force in defending herself. D Yes, whether or not her mistake was reasonable.

A Incorrect. Here the homeowner is not arguing mistake negates intent but she is arguing that she had the right to defend herself or to exercise the privilege of self-defense. While it is true that mistake does not negate intent, the commission of a reasonable mistake in the exercise of a privilege does not negate the privilege. In contrast to the previous question homeowner is not seeking to make the legally impossible argument that mistake negates intent, rather she is arguing that she was privileged to defend herself. A is wrong because reasonable mistake does not negate the privilege. So even though homeowner committed a battery there is no liability because of the privilege. B Correct. Here the homeowner is not arguing mistake negates intent, but she is arguing that she had the right to defend herself or to exercise the privilege of self-defense. While it is true that mistake does not negate intent, the commission of a reasonable mistake in the exercise of a privilege does not negate the privilege. In contrast to the previous question homeowner is not seeking to make the legally impossible argument that mistake negates intent, rather she is arguing that she was privileged to defend herself. B is correct because reasonable mistake does not negate the privilege. So even though homeowner committed a battery there is no liability because of the privilege of self-defense. C Incorrect. It is simply an incorrect statement of law. People are allowed to use force in defending themselves as long as the force used is reasonable. D Incorrect. Here the homeowner is not arguing mistake negates intent but she is arguing that she had the right to defend herself or to exercise the privilege of self-defense. While it is true that mistake does not negate intent, the commission of a reasonable mistake in the exercise of a privilege does not negate the privilege. In contrast to the previous question homeowner is not seeking to make the legally impossible argument that mistake negates intent, rather she is arguing that she was privileged to defend herself. D is wrong because while reasonable mistake does not negate the privilege an unreasonable mistake will negate the privilege.

Political refugee lived in a war torn, impoverished country. Political refugee and his family were under threat of death. Humanitarian worker said to political refugee, "Seems like you have a choice. You can either stay here and face certain death or come with me and I will transport you and your family to a safe country and you are free to go once we get there." Political refugee and his family decided to go to safe country with humanitarian worker. Upon arriving in safe country by ship, humanitarian worker refuses to let Political refugee and her family disembark. When political refugee asked to leave, humanitarian worker said, "Sure if you can swim the shark infested mile from the ship to shore." After spending 3 days on board without being allowed to leave, political refugee and her family escaped in a life boat. They are now seeking damages based on false imprisonment against the humanitarian worker. Is Political refugee's claim sustainable? A No, because political refugee voluntarily decided to go with humanitarian worker. B Yes, even though political refugee voluntarily decided to go with humanitarian worker. C No, because 3 days on board is not a long enough confinement to be actionable. D Yes, because the confinement exceeded one day.

A Incorrect. Humanitarian worker induced political refugee to accept confinement based on an express or implied promise to end the confinement. A defendant who induces a person to accept confinement in reliance on a promise or implicit promise to end the confinement on demand may be obliged to take affirmative action to end the confinement when it is demanded. B Correct. Humanitarian worker induced political refugee to accept confinement based on an express or implied promise to end the confinement. A defendant who induces a person to accept confinement in reliance on a promise or implicit promise to end the confinement on demand may be obliged to take affirmative action to end the confinement when it is demanded. These facts mirror the case of Whittaker v. Sanford, 85 A. 399 (1912). C Incorrect. There is no minimum confinement length required for the tort to be actionable. D Incorrect. There is no minimum confinement length required for the tort to be actionable.

B suddenly and violently attacks A with a machete. A evades the machete and kicks B in the stomach causing B to stumble down some stairs and to collide with C who unknown to A was coming up the stairs. The collision causes C to suffer a torn ligament in his knee. If A raises the self-defense would A be liable to C? A A would be liable to C because C never used any force against A so A had no privilege which permitted force against C. B A would not be liable to C because A acted reasonably in defending herself against B. C A would be liable to C because A had to absolutely ensure her use of force against B would not cause harm to anyone besides B. D A would not be liable even if A acted unreasonably in defending herself against B.

A Incorrect. If a bystander is hurt by a person's use of defensive force against another then there is no liability as long as the defensive force was reasonable. B Correct. If a bystander is hurt by a person's use of defensive force against another then there is no liability as long as the defensive force was reasonable. C Incorrect. If a bystander is hurt by a person's use of defensive force against another then there is no liability as long as the defensive force was reasonable. Choice C also recognizes that very often the use of force in self-defense occurs in a chaotic and dynamic setting. It often involves reflexive and quick decisions on the part of the defender and it would be unrealistic to expect the defender to calculate, determine and then apply the precise and exact force perfectly necessary to under these conditions the exact amount of force needed to precisely defend themselves and do nothing else. D Incorrect. The gist of the privilege to defend oneself is that the defender behaved in a way that was reasonable in the circumstances. It is this reasonable behavior that negates liability for otherwise tortious conduct. If A behaved unreasonably in defending herself and that unreasonable behavior caused harm to a bystander, then A might be liable in negligence to C

Alice sues Bill alleging negligence. Bill defends, claiming that Alice's negligence also contributed to her injuries. At trial, the factfinder assigns 50% responsibility to both Alice and Bill. The factfinder also finds that Alice's damages are $100,000. In a modified comparative fault (50% or "not as great as") jurisdiction, what amount, if any, is Alice entitle to recover from Bill? A Alice is entitled to recover $50,000 from Bill because Bill only has to pay for his percentage of her damages attributable to his fault. B Alice is entitled to recover $50,000 from Bill because her recoverable damages are the judgment reduced by the amount of fault attributable to her. C Alice is entitled to recover $100,000 from Bill. D Alice is completely barred from recovering from Bill because she was 50% at fault.

A Incorrect. If a plaintiff's judgment is reduced, it is reduced by the amount of fault attributable to the plaintiff. Plaintiff's recoverable damages are not calculated based upon the defendant's percentage of responsibility. Restatement (Third) Torts § 7 and comment g. This question asks about a plaintiff's recovery in a modified fault (50% or "not as great as") jurisdiction. B Incorrect. In a modified not as great as jurisdiction, the plaintiff is completely barred from recovery if her fault is as great as that of the defendant. This question asks about a plaintiff's recovery in a modified fault (50% or "not as great as") jurisdiction. C Incorrect. Alice's fault is as great as the defendant's, and she is therefore not entitled to recover from Bill. This question asks about a plaintiff's recovery in a modified fault (50% or "not as great as") jurisdiction. D Correct. Plaintiff's fault is as great as that of the defendant, so she is barred from recovering, making D the correct answer because in a modified fault (50% or "not as great as") jurisdiction, when Plaintiff's fault is as great as that of the defendant, she is barred from recovering.

Alice sues Bill alleging negligence. Bill defends, claiming that Alice's negligence also contributed to her injuries. At trial, the factfinder assigns 50% responsibility to both Alice and Bill. The factfinder also finds that Alice's damages are $100,000. In a traditional contributory negligence jurisdiction, what amount, if any, is Alice entitled to recover? A Alice is entitled to recover $50,000 from Bill because Bill only has to pay for his percentage of the judgment attributable to his fault. B Alice is entitled to recover $50,000 from Bill because her recoverable damages are her damages reduced by the amount of fault attributable to her. C Alice is entitled to recover $100,000 from Bill. D Alice is completely barred from recovering from Bill because she was partially at fault.

A Incorrect. In a contributory negligence jurisdiction, a plaintiff's recovery is not reduced. B Incorrect. In a contributory negligence jurisdiction, if plaintiff is at fault at all, she is barred from recovery. C Incorrect. In a contributory negligence jurisdiction, if plaintiff is at fault at all, she is barred from recovery. D Correct. If a plaintiff's negligence contributes at all to her injuries, she is completely barred from recovery.

About two years ago, Client was a patient of Ophthalmologist. Client alleges that during the time that he was Ophthalmologist's patient, Ophthalmologist improperly performed LASIK surgery on Client's eyes. The LASIK surgery was supposed to correct Client's poor vision. Instead, the surgery was performed skillfully, but resulted in blindness in both eyes, a risk about which Client was never informed and a reasonable person would not have known. If Client had known of the risk of blindness, he would not have opted for the surgery. Client hires Attorney Matthew to sue Ophthalmologist for medical malpractice. Attorney Matthew fails to file the complaint to initiate the cause of action against Ophthalmologist within the applicable statute of limitations. As a result, the court dismisses Client's case against Ophthalmologist. Client then hires Attorney Becky to represent him in a cause of action against Attorney Matthew for legal malpractice. In the legal malpractice action brought by Client against Attorney Matthew, what duty applies to Attorney Matthew? A Attorney Matthew had a duty to act as the average lawyer under the same or similar circumstances. B Attorney Matthew had a duty to act as the reasonable prudent lawyer under the same or similar circumstances. C Attorney Matthew had a duty to act as the reasonable prudent lawyer in the same or similar locality. D Attorney Matthew had a duty to act as the reasonable lawyer with Attorney Matthew's same training and experience.

A Incorrect. In a professional negligence action, such as the lawsuit brought against Attorney Matthew, a lawyer, has a duty to act as the reasonably prudent lawyer, not as the average lawyer. B Correct. It provides the correct standard of care in a professional negligence action. An attorney has a duty to act as the reasonable prudent attorney in the same or similar circumstances. C Attorney Matthew had a duty to act as the reasonable prudent lawyer in the same or similar locality. D Incorrect. The professional standard of care does not take on the training or experience of the defendant. Rather it is an objective standard based on the reasonably prudent professional under the same or similar circumstances.

A sues B and C in negligence. B and C defend, claiming that A's negligence contributed to A's injuries. At trial, the jury assigns 40% responsibility to A, 40% responsibility to B, and 20% responsibility to C. It also determines that A's damages are $100,000. In a pure comparative fault jurisdiction, what amount, if any, may A recover? A A may not recover for her injuries because the jury determined that A was at fault. B A may recover from B and C for a total of $100,000. C A may recover from B and C for a total of $60,000. D Incorrect. A's recovery is reduced by $40,000; $40,000 is not the amount that A is entitled to receive.

A Incorrect. In a pure comparative fault jurisdiction, the plaintiff is not barred from recovery if her negligence has contributed to her injuries. B Incorrect. In a pure comparative fault jurisdiction, the plaintiff's damages are reduced by the percentage of fault attributable to her, so A may not recover the entire $100,000. C Correct. A's damages are reduced by 40% of $100,000 which is $40,000. $100,000-$40,000=$60,000. D Incorrect. A's recovery is reduced by $40,000; $40,000 is not the amount that A is entitled to receive.

Alice sues Bill alleging negligence. Bill defends, claiming that Alice's negligence also contributed to her injuries. At trial, the factfinder assigns 50% responsibility to both Alice and Bill. The factfinder also finds that Alice's damages are $100,000. In this pure comparative fault jurisdiction, what amount, if any, is Alice entitled to recover? A Alice is entitled to recover $50,000 from Bill because Bill only has to pay for his percentage of the judgment attributable to his fault. B Alice is entitled to recover $50,000 from Bill because her recoverable damages are her damages reduced by the amount of fault attributable to her. C Alice is entitled to recover $100,000 from Bill. D Alice is completely barred from recovering from Bill because she was partially at fault.

A Incorrect. In a pure comparative fault jurisdiction, the plaintiff's recovery is reduced by the amount of fault attributable to the plaintiff. Plaintiff's recoverable damages are not calculated based upon the defendant's percentage of responsibility. Restatement (Third) Torts § 7 and comment g. B Correct. Plaintiff's recoverable damages in a pure comparative fault jurisdiction are reduced based upon the percentage of fault attributed to the plaintiff. C Incorrect. In a pure comparative fault jurisdiction, as 50% responsible, Alice is not entitled to recover the entire $100,000. D Incorrect. In a pure comparative fault jurisdiction, Alice is not completely barred from recovery.

A city ordinance requires that all apartments be equipped with fire sprinklers in each sleeping area and in each stairwell leading to an occupied area. Apartment Owner, who owns an apartment building and leases apartments, hires Ivan, an independent electrical and plumbing contractor, to install fire sprinklers throughout the building. Ivan does not install a fire sprinkler in every stairwell as specified by the ordinance. Tenant, a tenant in Apartment Owner's building, is injured in a fire that started in a stairwell without sprinklers. Tenant would have escaped the fire without injury if a fire sprinkler had been in place. In a lawsuit brought by Tenant against Apartment Owner for injuries sustained in the fire, may Apartment Owner be subject to vicarious liability? A Apartment Owner may not be subject to vicarious liability for Tenant's injuries because Ivan was an independent contractor. B Apartment Owner may not be subject to vicarious liability to Tenant for Ivan's conduct unless Apartment Owner was negligent in hiring Ivan. C Incorrect. The mere hiring of an independent contractor does not subject Apartment Owner to vicarious liability for Ivan's conduct. D Correct. It is a correct statement of vicarious liability. This question asks about the liability if a defendant who hires an independent contractor to perform an activity that is imposed on the defendant by statute or regulation. See Restatement (Third) Torts § 63 and illustration 1 upon which this fact pattern is based

A Incorrect. In a situation where the defendant hires an independent contractor to perform a task required of the defendant by statute, the general rule of non-liability for the tortious conduct of one's independent contractors does not apply. B Incorrect. It misstates the rule; a defendant may be subject to vicarious liability in the absence of negligent hiring of an independent contractor. C Incorrect. The mere hiring of an independent contractor does not subject Apartment Owner to vicarious liability for Ivan's conduct. D Correct. It is a correct statement of vicarious liability. This question asks about the liability if a defendant who hires an independent contractor to perform an activity that is imposed on the defendant by statute or regulation. See Restatement (Third) Torts § 63 and illustration 1 upon which this fact pattern is based

As part of a game of truth or dare Melissa walked toward a perfect stranger, and when she was about 10 feet from the stranger she asked the stranger the time. As the stranger looked up from his watch, Melissa removed an unloaded gun from her pocket, pointed it at the stranger, and pulled the trigger. As he saw this, the stranger yelled in terror, jumped back, and raised his hands. Melissa laughed uncontrollably at the stranger's reaction and then told him it was a simple prank and the gun was unloaded. Will Melissa be liable for assault and battery? A Melissa will not be liable for assault because the gun was not loaded. B Incorrect. It attempts to confuse you by hinting at the "close enough to the body is the body" rule from battery, but that rule is inapplicable here because in order for it to be invoked, contact has to be made with an object in intimate association with the body. Also, shooting a bullet at someone will qualify for harmful or offensive contact. This answer choice is incorrect because it also tangentially invokes the erroneous rule that direct contact is all that can cause a battery. Again, battery may occur without direct contact between the tortfeasor and the plaintiff. For example, if tortfeasor intentionally throws a stone that hits the plaintiff, that contact is a battery. Similarly, assault is not negated because of distance unless the threatened contact is impossible. For example, you are on the phone with someone you know is in New York City and you are in Kansas. The person screams, "Duck. I just threw a rock at you." Fear or apprehension of being hit by the rock is unreasonable and not support an assault claim. C Correct. In assault, it is the apparent ability to effectuate harmful or offensive contact rather than the actual ability to cause contact that is legally sufficient. For example, here, even though the gun was not loaded, and as a result there was no actual ability to effectuate the harmful or offensive contact ,it reasonably appeared to the stranger (apparent ability) that harmful or offensive contact was imminent. If the victim of the tort reasonably believed harmful or offensive contact was imminent, then it is irrelevant to the tort whether there was actually the ability to bring about the contact. The tricky part of this question is that many students erroneously assume that no assault is possible unless there is a battery. That is incorrect. Assault is an independent tort which does not depend on the existence of a battery and vice versa. D Melissa will be liable for battery if she is liable for assault because assault can never occur independently of battery.

A Incorrect. In assault, it is the apparent ability to effectuate harmful or offensive contact rather than the actual ability to cause contact that is legally sufficient. For example, here, even though the gun was not loaded, and as a result there was no actual ability to effectuate the harmful or offensive contact, it reasonably appeared to the stranger (apparent ability) that harmful or offensive contact was imminent. If the victim of the tort reasonably believed harmful or offensive contact was imminent, then it is irrelevant to the tort whether there was actually the ability to bring about the contact. B Incorrect. It attempts to confuse you by hinting at the "close enough to the body is the body" rule from battery, but that rule is inapplicable here because in order for it to be invoked, contact has to be made with an object in intimate association with the body. Also, shooting a bullet at someone will qualify for harmful or offensive contact. This answer choice is incorrect because it also tangentially invokes the erroneous rule that direct contact is all that can cause a battery. Again, battery may occur without direct contact between the tortfeasor and the plaintiff. For example, if tortfeasor intentionally throws a stone that hits the plaintiff, that contact is a battery. Similarly, assault is not negated because of distance unless the threatened contact is impossible. For example, you are on the phone with someone you know is in New York City and you are in Kansas. The person screams, "Duck. I just threw a rock at you." Fear or apprehension of being hit by the rock is unreasonable and not support an assault claim. C Correct. In assault, it is the apparent ability to effectuate harmful or offensive contact rather than the actual ability to cause contact that is legally sufficient. For example, here, even though the gun was not loaded, and as a result there was no actual ability to effectuate the harmful or offensive contact ,it reasonably appeared to the stranger (apparent ability) that harmful or offensive contact was imminent. If the victim of the tort reasonably believed harmful or offensive contact was imminent, then it is irrelevant to the tort whether there was actually the ability to bring about the contact. The tricky part of this question is that many students erroneously assume that no assault is possible unless there is a battery. That is incorrect. Assault is an independent tort which does not depend on the existence of a battery and vice versa. D Incorrect. The tricky part of this question is that many students erroneously assume that no assault is possible unless there is a battery. That is incorrect. Assault is an independent tort which does not depend on the existence of a battery and vice versa.

Most airline flights from the East Coast of the United States to Western Europe depart at about 8:00 pm United States Time. Because of the 5 hour time difference this means that they leave the United States when it is approximately 1:00 am in Western Europe. The average 8 hour flying time means that passengers land in Western Europe at approximately 9:00 am the following morning. Approximately 2 hours after takeoff the cabin lights are dimmed and passengers are asked to keep the window shades closed to facilitate passengers sleeping. During this phase of a flight as a flight attendant was walking in the aisles with a coffee pot full of hot coffee the plane experienced significant turbulence. In order to steady himself and prevent the coffee from spilling over many passengers he reached down and steadied himself by intentionally putting his left hand on the shoulder of a passenger for a few seconds. The passenger who was awake when this happened was deeply offended by the flight attendant's touch. If passenger sues flight attendant for battery will Passenger prevail? A Passenger will prevail because battery requires harmful OR offensive not harmful AND offensive contact, and even though he was not harmed the passenger was offended. B Passenger will not prevail because even though the passenger found the contact offensive it will likely be deemed not harmful or offensive as a matter of law. C Passenger will prevail because the flight attendant's touch was intentional. D Passenger will not prevail because the flight attendant was acting with a good motive.

A Incorrect. In order for contact to be considered harmful or offensive, it must be such that a reasonable person would find the contact harmful or offensive. This answer choice relies exclusively on the fact that the plaintiff found the contact harmful or offensive without indicating whether a reasonable person would also find the contact harmful or offensive. B Correct. It addresses the so called "crowded world theory." The crowded world theory basically permits certain unconsented to contact which is necessary for social existence to be considered legally "not harmful or offensive as a matter of law." There are many examples of this, such as bumping into someone on a crowded subway as you make your way to a seat or pushing someone to prevent them from blocking an emergency exit during an emergency in a building. Usually, the crowded world theory is stated in terms of social usage and conflicting rights. So contacts deemed necessary for social function are usually not deemed harmful or offensive. Here it is likely the flight attendant's decision to touch a passenger's shoulder during turbulence to prevent hot coffee landing on many passengers will be considered not harmful or offensive as a result of the crowded world theory. C Incorrect. The mere fact that a touch was intentional does not make it tortious in terms of battery. The other elements of the rule require the touch to also be harmful or offensive to a reasonable person. D Incorrect. Intent and motive are not the same. Good intentions do not negate intent, but as mentioned previously they may be relevant in terms of whether or not to award punitive damages.

Insane person perceives caregiver as a threat even though caregiver is not a threat. As a result insane person violently attacks caregiver and confines caregiver in a restroom for 3 days. Caregiver sues insane person for battery and false imprisonment. Will insane person be liable? A Insane person will not be liable because insane people cannot have intent. B Insane person will be liable unless he acted like the reasonable insane person under the circumstances. C Insane man will be liable bc insanity does not negate intent. D Insane person will not be liable if he made a mistake in defending himself.

A Incorrect. Insane people are deemed to have intent exactly like the sane and insanity is not generally a defense to intentional torts. Regardless of the motivation for the insane person's actions, even if those actions were fuelled by insanity related involuntary delusions, if the insane person intended to hit the caregiver then the law is that insanity is not defense to intentional tort B Incorrect. It is incorrect for a number of reasons. The first is that intent is a subjective standard not an objective standard and the answer key judges the insane person's actions by a reasonable person standard which is incorrect. Furthermore, even in negligence law the reasonable person standard does not account for insanity. The insane person will be held to the standard of the reasonable (not insane) person. C Correct. The general rule of law in tort is that insanity does not negate intent. As long as the insane person is capable of intent, for example in this case, intending to hit the caregiver who the insane person erroneously perceived as a threat, the insanity is irrelevant in establishing intent. D Incorrect. Only reasonable mistakes in defending yourself are permissible. There is no indication in the facts that the mistake was reasonable. In fact the opposite is likely and the insanity caused an unreasonable perception of threat where none existed.

Bully intends to cause victim to suffer severe emotional distress. As a result Bully wants to make victim look weak in front of other classmates. As victim is approaching his locker between classes Bully sneaks up behind victim and gently says, "Hey." As victim turns around Bully screams right in his face causing victim to lurch backwards out of fear and surprise and to hit the back of his head on the locker from fear of what the victim thought was an imminent physical attack on him. Assume the prank by the bully is not considered extreme and outrageous and the bully never intended for the victim to suffer physical harm. How may the victim recover for the injuries suffered when he hit his head? A Sue for battery because Bully intended extreme emotional distress and even though that tort is not actionable the intent form the emotional distress tort will transfer rendering battery actionable. B Sue for battery because even though Bully did not intend physical harm it would have been a foreseeable result to a reasonable person. C Sue for assault and battery because the bully was probably substantially certain that his actions would cause apprehension of harmful or offensive contact and the intent from assault will transfer to battery rendering the tort of battery complete. D Sue for trespass to chattel because the victim's head hit the locker causing damage to the locker and the intent for trespass to chattels will transfer to make assault and battery actionable.

A Incorrect. Intent does not transfer to any tort from Intentional Infliction of Emotional Distress. Intent only transfers between the five torts which were within the original writ of trespass, (Trespass to Land, Trespass to Chattel, False Imprisonment, Assault and Battery). B Incorrect. Mere foreseeability is not enough to be substantial certainty and furthermore this answer choice uses the reasonable person standard which is not appropriate for determining the subjective state of mind required to be shown for intentional torts. C Correct. The bully likely intended to scare the victim if he yelled in his face. Even if his ultimate specific intent was to embarrass the victim in front of his peers the Bully likely knew or at the very least was substantially certain that the victim would be fearful or apprehensive of contact after he screamed. Once this substantial certainty is established for assault, intent is established for all the five torts within the original writ of trespass, (Trespass to Land, Trespass to Chattel, False Imprisonment, Assault and Battery). Therefore even if the bully never intended harmful or offensive contact the intent from assault would transfer satisfying the intent requirement of battery and because harmful or offensive contact occurred, battery would be actionable. D Incorrect. The facts clearly state that the bully never intended for the victim to suffer physical harm, which in turn likely means the bully never intended the victim to damage the locker. Because there was no intent to intermeddle of interfere with a chattel in the possession of another then there will be no intent available for transfer if this is the cause of action chosen. Additionally, it is ambiguous whether a locker in a school is a chattel or real property.

Two students at the end of the semester got a D grade in torts. One student was incredibly irate about the grade and the other was blissfully happy he did not get an F. The blissfully happy student decided to celebrate. In order to celebrate she went to the store and bought a bow and arrow. She waited until the classroom was full and began shooting suction cup tipped, wooden arrows into the air in the classroom with a sign on them that said, "Joy, Joy I will work harder because now I know that your grades in law school are proportional to the amount of work you do." Her only goal in shooting the arrows was to share her profound discovery about the correlation between grades and work in law school. However, the arrows bounced off the walls and hit many, many people causing them to experience offensive contact. The irate student got the same kind of bow and arrow that the other student got and began shooting the students who did better than he did. Using his 10 years of archery lessons, he methodically aimed the suction cupped arrows at the individual students and let fly. The students he aimed at experienced offensive contact from the arrows. Which of the following about the students' intent is true? A The happy student acted with intent if a reasonable person would have been substantially certain that shooting the arrows in a crowded classroom would result in harmful or offensive contact. B The happy student did not act with intent because she did not act with the specific purpose of causing harmful or offensive contact. C The irate student acted with intent because he acted with the specific purpose of causing harmful and offensive contact. D The irate student acted with intent because a reasonable person would have been substantially certain the arrows would cause harmful or offensive contact.

A Incorrect. Intent is subjective and not objective. Unlike in negligence where we impute the mindset of the reasonable person onto the defendant, in intent we need to show that the specific defendant possessed the requisite knowledge level such that they specifically would be substantially certain that their actions would produce a tortious result, in this case harmful or offensive contact. In this case, if we can prove that the happy student had a sufficient knowledge level such that they were substantially certain that ricochets, etc. would occur hitting people, then we may be closer to establishing intent via the substantial certainty test. But the mere fact that a reasonable person would be substantially certain is not enough to establish substantial certainty/intent for this specific defendant. B Incorrect. Intent may be established by demonstrating that the defendant acted with the specific purpose of causing a tortious result. We know that is not the case here because the happy student acted with no other specific purpose than to share something he had learned after a semester in law school. However, even if someone does not act with specific intent, she can still be found to have acted intentionally if she was substantially certain her actions would produce a tortious result. So intent may be established via the substantial certainty test even if there is no specific intent. C Correct. Here the intent of the unhappy student may be classified as specific intent. The facts state the student "methodically aimed at other students," and that he had 10 years of archery lessons. What this shows is that the unhappy student is taking deliberate actions to bring about the tortious result of harmful or offensive contact. This action with a deliberate purpose to achieve a tortious result is called specific intent. D Incorrect. Intent is subjective and not objective. Unlike in negligence where we impute the mindset of the reasonable person onto the defendant, in intent we need to show that the specific defendant possessed the requisite knowledge level such that they specifically would be substantially certain that their actions would produce a tortious result, in this case harmful or offensive contact. A further reason this question is not the best answer is because the facts demonstrate the irate student had specific intent so there is no need to argue that she was substantially certain.

Shopper insulted store clerk by saying that the store clerk had a disgusting body odor. Shopper intended to insult and embarrass store clerk and said it loud enough for the fellow shoppers in the vicinity to hear. Store clerk was mortified when other shoppers laughed. As a result of the insult, store clerk fainted and hit his head on a display and suffered a concussion. If store clerk brought an action for Intentional Infliction of Emotional Distress against shopper, would store clerk recover? A Store clerk would recover because he intended to cause emotional harm and did in fact cause emotional harm. B Store clerk would likely not recover because the shoppers comment is legally insufficient to support a cause of action for IIED. C Store clerk would recover if the harm suffered was indeed severe. D Store clerk would not recover if he is considered the equivalent of an innkeeper.

A Incorrect. Intentional infliction of emotional distress requires more than an act intended to cause severe emotional distress and the resulting distress. In order to be actionable, not only must the tortfeasor intend severe emotional harm but his actions which bring about the severe harm must also be "extreme and outrageous." The restatement has defined this standard as acts which are beyond all bounds tolerated by a civilized society. Mere insults have been held not to fall within the category of extreme and outrageous. Therefore even though the shopper intended to cause emotional harm and acted deliberately to achieve that purpose and did in fact achieve the intended result the insult is legally insufficient to sustain the action. B Correct. Intentional infliction of emotional distress requires more than an act intended to cause severe emotional distress and the resulting distress. In order to be actionable, not only must the tortfeasor intend severe emotional harm but his actions which bring about the severe harm must also be "extreme and outrageous." The restatement has defined this standard as acts which are beyond all bounds tolerated by a civilized society. Mere insults have been held not to fall within the category of extreme and outrageous. Therefore even though the shopper intended to cause emotional harm and acted deliberately to achieve that purpose and did in fact achieve the intended result the insult is legally insufficient to sustain the action. C Incorrect. Intentional Infliction of Emotional Distress is defined as actionable when (1) the defendant causes severe emotional distress, (2) intentionally by (3) extreme and outrageous conduct. This answer choice focuses on the severity of the harm. While this element is likely met the element of outrageous conduct has not been satisfied because in order to be considered extreme and outrageous the conduct must be beyond all bounds tolerated by a civilized society. Mere insults have been held not to fall within the category of extreme and outrageous. Therefore even though shopper's harm is likely to be considered severe the store clerk would not recover because the conduct was not extreme and outrageous. D Incorrect. D invokes, albeit incorrectly for this fact pattern, the common carrier exception to the requirement for extreme and outrageous conduct on the defendant's part. That exception recognizes that common carriers and inn keepers had a special duty to passengers and customers respectively and could be liable for intentional emotional harm even if the behavior was not really extreme and outrageous. Here this rule is inapplicable because it is the store clerk (who can arguably be considered an innkeeper's agent if we analogize the storeowner to an innkeeper) who is seeking to recover. The common carrier exception applied only to attempts to recover against an innkeeper or common carrier.

In a crowded subway two people are standing next to each other because there are no available seats. Person 1 looks down and notices that Person 2 has shiny, new shoes. Person 1 thinking that it would be funny and a good natured prank, decides to step on person 2's shoes to scuff them a little. As he does so the train slows suddenly and instead of just scuffing the shoes with light contact Person 1 ends up stomping Person 2's shoe with a lot of force. Person 2 suffers injury to his toes but there is no actual damage to the shoe. For what intentional tort, if any, may Person 1 be liable? A Person 1 is not liable in intentional tort because he did not intend to injure Person 2. B Person 1 may be liable for trespass to chattel. C Person 1 may be liable for battery. D Person 1 may be liable for both battery and trespass to chattel.

A Incorrect. Intentional tort liability is not based on an intent to injure or an intent to commit a tort. Rather, it is based on an intent to do an act, which act turns out to result in harm which the law deems compensable. For example, a trespasser who intentionally steps on another's land believing the land to be her own is still liable for trespass because she had the intent to step on that land even thought there was no intent to trespass. B Incorrect. While the shoe clearly is a chattel, in order to be actionable, trespass to chattel requires that actual damage be done to the chattel. The facts state there was no actual damage to the shoe. C Correct. Even though there was no intent to commit a battery, there was the intent to commit a trespass to chattel by scuffing the shoe. Even though the shoe ended up not being scuffed, because harmful or offensive contact occurred (damage to toes), the intent to commit trespass to chattel transfers via the doctrine of transferred intent to satisfy all the elements of the tort of battery. D Incorrect. Even though there was no intent to commit a battery, there was the intent to commit a trespass to chattel by scuffing the shoe. Even though the shoe ended up not being scuffed, because harmful or offensive contact occurred (damage to toes), the intent to commit trespass to chattel transfers via the doctrine of transferred intent to satisfy all the elements of the tort of battery. While the shoe clearly is a chattel, in order to be actionable, trespass to chattel requires that actual damage be done to the chattel. The facts state there was no actual damage to the shoe.

Driver was angry at corporate employee who he says cut him off the previous week in traffic. Driver followed employee to his place of employment and observed that employee parked in the same spot on the third floor of the same parking garage every morning. Every other spot was always occupied in the garage. Driver waited until the garage was empty of people but full of parked cars and placed a powerful explosive device under Driver's car. The device unsurprisingly to Driver was so powerful it caused the destruction of the employee's car but also caused the complete destruction of the five cars on either side of the employee's car to suffer damage. May the other people whose cars were damaged maintain a conversion action against driver? A No, bc driver only had intent to hit to harm employee's car. B Yes, if driver was substantially certain the other people's cars would be completely destroyed. C Yes, if driver knew that it was reasonably foreseeable the other people's cars would have been. D No, even if the driver was substantially certain the other people's cars would have been destroyed.

A Incorrect. It assumes that the only way for intent to be established is via the specific intent test. A person may not specifically intend harm but if they were substantially certain their voluntary act would result in harm then intent is established as a matter of law. B Correct. If the driver did not specifically intend harm to the other people's cars he might still be found to have had the requisite intent if he was substantially certain the harm would result. In these facts the driver knew every other spot was always occupied. The driver was not surprised that the explosive device he planted for the specific purpose of destroying employee's car was so powerful it also destroyed five cars on either side. So even though he did not specifically intend to damage the other people's cars if the facts are such that he was substantially certain the bomb was powerful enough to cause the destruction of these other cars then even if he was indifferent about their destruction he will be deemed to have intended their destruction as a matter of law. C Incorrect. The tort of conversion is an intentional tort which requires specific intent or substantial certainty to be actionable. If the driver could reasonably foresee the harm that does not establish intent. The degree of certainty is what separates intent from negligence. Because negligence is objective, if a reasonable person would foresee the harm then we say that the driver foresaw it as a matter of law and could be liable in a negligence cause of action. However, because intent is subjective, the driver is only deemed to have intended the results in the absence of specific intent if driver was substantially certain the results would occur. D Incorrect. If the driver did not specifically intend harm to the other people's cars he might still be found to have had the requisite intent if he was substantially certain the harm would result. In these facts the driver knew every other spot was always occupied. The driver was not surprised that the explosive device he planted for the specific purpose of destroying employee's car was so powerful it also destroyed five cars on either side. So even though he did not specifically intend to damage the other people's cars if the facts are such that he was substantially certain the bomb was powerful enough to cause the destruction of these other cars then even if he was indifferent about their destruction he will be deemed to have intended their destruction as a matter of law.

Car dealer asks landowner if he can use landowner's unoccupied field to store some blue cars for the night as his lot will be temporarily beyond capacity while new inventory arrives. In reality and unknown to the landowner the car dealer planned to store some red cars on the land. When the landowner awoke and saw the red cars on the land he decided to sue the car dealer for trespass to land. What result? A Car dealer will prevail because landowner consented to the cars being on his land. B Car dealer will prevail if landowner would have allowed red cars anyway. C Car dealer will lose because he did not obtain valid consent. D Car dealer will lose because the landowner's reputation was actually harmed when the police arrested him.

A Incorrect. It at first seems like a correct answer because of the exception to the "Consent obtained by fraud is not valid consent rule." Fraud does not vitiate consent when the fraud is about a collateral or relatively unimportant matter. Here we aren't sure if the color of the car is a collateral matter. B Correct. Because fraud does not vitiate consent when the fraud is about a collateral matter. If the land owner would have allowed red cars to be on the land if asked then it appears as though the color of the car is not important or material. Rather it is collateral and therefore the consent is not invalidated. C Incorrect. It is unclear whether the color was a material or collateral matter. D Incorrect. It is legally irrelevant. The existence of actual damage is immaterial to the maintainability of a trespass for land action.

Hotel Owner contacts Pipe Co. to deliver and install a large pipe to display a sign for its hotel. Hotel Owner thoroughly vets Pipe Co., and Pipe Co.'s representative sends Hotel Owner information showing that Pipe Co. has extensive experience in installing pipes like this and that Pipe Co. is bonded for this purpose. When Pipe Co.'s workers attempt the installation by using their truck, hooks, chains, and additional poles for stabilization, the pipe falls, bouncing on a passing car and injuring Driver. In a lawsuit brought by Driver against Hotel Owner for injuries sustained when the pipe fell on his car, may Hotel Owner be subject to direct or vicarious liability? A Hotel Owner may be subject to direct liability in negligence because he failed to use reasonable care in hiring Pipe Co. B Hotel Owner may not be subject to direct liability for the actions of Pipe Co. because Hotel Owner did not owe a duty of care with respect to the manner in which Pipe Co. performed its work and did not retain control over any part of its work. C Hotel Owner may be subject to vicarious liability in negligence because he hired Pipe Co. and Pipe Co.'s negligence factually caused injuries to Driver. D Hotel Owner may be subject to vicarious liability for Driver's injuries because employers are responsible for the torts of their independent contractors.

A Incorrect. It contradicts the facts provided, which suggest that Hotel Owner used due care in hiring a pipe installation company for this job. B Correct. It is a correct statement regarding the direct liability of one who hires an independent contractor and does not retain control over any part of the contractor's work. Restatement (Third) Torts § 56. C Incorrect. A defendant who hires an independent contractor is not subject to vicarious liability for physical harm caused by the tortious conduct of the contractor. Restatement (Third) Torts § 57. D Incorrect. A defendant who hires an independent contractor is not subject to vicarious liability for physical harm caused by the tortious conduct of the contractor. Restatement (Third) Torts § 57.

A pointed a gun at B scaring B, and without authority or justification, A ordered B into a room and locked the door for 2 hours. B was awake the whole time and worried about what would happen until A released him unharmed. When A initially pointed gun at B, B was standing next to C, and C dove for cover thinking the gun was pointed at her. In diving for cover C collided violently with D causing D to spill coffee on E's computer, completely destroying the computer. E is no longer interested in retaining title to the computer but simply wants A to pay the market value of the chattel at the time the chattel was destroyed and to transfer title of the computer to A. Who may maintain an action against A? A Only B but not C, D and E may maintain an action against A B Only B and C but not D and E may maintain an action against A C Only B, C and D but not E may maintain an action against A D B, C, D and E may all maintain actions against A

A Incorrect. It erroneously fails to apply the doctrine of transferred intent and the rule of extended liability in intentional tort. Transferred intent operates whenever the tort intended and the tort committed are within the original writ of trespass (Trespass to Land, Trespass to Chattel, False Imprisonment, Assault and Battery). Whenever a tortfeasor intends any one of the five but ends up committing any other of the five torts then all the torts committed are deemed to have the intent prong satisfied. For example in the above scenario A intended to falsely imprison B, but while committing false imprisonment he also caused B to get scared. Therefore A has committed both an assault and a false imprisonment against B. Additionally, not only does intent transfer between torts but it transfers between people as well. So even though A was intending only to falsely imprison B, he also caused C to get scared and therefore has committed assault against C. Similarly, even though A's only intent was to falsely imprison B, his tortious conduct scared C and resulted in a violent collision with D (harmful or offensive contact) and as a result D has an actionable battery claim against A. However A's intent to falsely imprison B caused the complete destruction of E's computer (a chattel). Now rather than suing for the harm caused to the chattel as in the previous question E is seeking to have A pay the entire market price of the computer when it was destroyed. This means that rather than trespass to chattel against A, E is seeking to recover pursuant to conversion. Conversion unlike trespass to chattel is not one of the torts within the original writ of trespass and therefore transferred intent will not support E's action against A. Also relevant to this hypothetical is the rule of limitless causation in intentional tort. A tortfeasor is usually liable for all the harm caused by their tortious conduct whether or not the harm was foreseeable. While this rule will help C and D recover it will not help E recover here. The reason is a little complicated. Because transferred intent does not apply, in order to recover E would have to prove that A was at least substantially certain that trying to imprison B would cause the destruction of E's computer. Because A's intent to imprison does not transfer to a conversion action and the facts do not indicate that A was substantially certain to destroy E's computer when he tried to imprison B, the intent prong of the conversion suit would fail and E would not be able to recover against A for the loss of the computer. It is important to remember that even though an intentional tortfeasor is potentially liable for all the harm they cause, recovery for harm is possible only if there is a maintainable cause of action to recover for that harm. In this case E cannot maintain a conversion cause of action. B Incorrect. It erroneously fails to apply the doctrine of transferred intent and the rule of extended liability in intentional tort. Transferred intent operates whenever the tort intended and the tort committed are within the original writ of trespass (Trespass to Land, Trespass to Chattel, False Imprisonment, Assault and Battery). Whenever a tortfeasor intends any one of the five but ends up committing any other of the five torts then all the torts committed are deemed to have the intent prong satisfied. For example in the above scenario A intended to falsely imprison B, but while committing false imprisonment he also caused B to get scared. Therefore A has committed both an assault and a false imprisonment against B. Additionally, not only does intent transfer between torts but it transfers between people as well. So even though A was intending only to falsely imprison B, he also caused C to get scared and therefore has committed assault against C. Similarly, even though A's only intent was to falsely imprison B, his tortious conduct scared C and resulted in a violent collision with D (harmful or offensive contact) and as a result D has an actionable battery claim against A. However A's intent to falsely imprison B caused the complete destruction of E's computer (a chattel). Now rather than suing for the harm caused to the chattel as in the previous question E is seeking to have A pay the entire market price of the computer when it was destroyed. This means that rather than trespass to chattel against A, E is seeking to recover pursuant to conversion. Conversion unlike trespass to chattel is not one of the torts within the original writ of trespass and therefore transferred intent will not support E's action against A. Also relevant to this hypothetical is the rule of limitless causation in intentional tort. A tortfeasor is usually liable for all the harm caused by their tortious conduct whether or not the harm was foreseeable. While this rule will help C and D recover it will not help E recover here. The reason is a little complicated. Because transferred intent does not apply, in order to recover E would have to prove that A was at least substantially certain that trying to imprison B would cause the destruction of E's computer. Because A's intent to imprison does not transfer to a conversion action and the facts do not indicate that A was substantially certain to destroy E's computer when he tried to imprison B, the intent prong of the conversion suit would fail and E would not be able to recover against A for the loss of the computer. It is important to remember that even though an intentional tortfeasor is potentially liable for all the harm they cause, recovery for harm is possible only if there is a maintainable cause of action to recover for that harm. In this case E cannot maintain a conversion cause of action. C Correct. It correctly applies the doctrine of transferred intent and the rule of extended liability in intentional tort. Transferred intent operates whenever the tort intended and the tort committed are within the original writ of trespass (Trespass to Land, Trespass to Chattel, False Imprisonment, Assault and Battery). Whenever a tortfeasor intends any one of the five but ends up committing any other of the five torts then all the torts committed are deemed to have the intent prong satisfied. For example in the above scenario A intended to falsely imprison B, but while committing false imprisonment he also caused B to get scared. Therefore A has committed both an assault and a false imprisonment against B. Additionally, not only does intent transfer between torts but it transfers between people as well. So even though A was intending only to falsely imprison B, he also caused C to get scared and therefore has committed assault against C. Similarly, even though A's only intent was to falsely imprison B, his tortious conduct scared C and resulted in a violent collision with D (harmful or offensive contact) and as a result D has an actionable battery claim against A. However A's intent to falsely imprison B caused the complete destruction of E's computer (a chattel). Now rather than suing for the harm caused to the chattel as in the previous question E is seeking to have A pay the entire market price of the computer when it was destroyed. This means that rather than trespass to chattel against A, E is seeking to recover pursuant to conversion. Conversion unlike trespass to chattel is not one of the torts within the original writ of trespass and therefore transferred intent will not support E's action against A. Also relevant to this hypothetical is the rule of limitless causation in intentional tort. A tortfeasor is usually liable for all the harm caused by their tortious conduct whether or not the harm was foreseeable. While this rule will help C and D recover it will not help E recover here. The reason is a little complicated. Because transferred intent does not apply, in order to recover E would have to prove that A was at least substantially certain that trying to imprison B would cause the destruction of E's computer. Because A's intent to imprison does not transfer to a conversion action and the facts do not indicate that A was substantially certain to destroy E's computer when he tried to imprison B, the intent prong of the conversion suit would fail and E would not be able to recover against A for the loss of the computer. It is important to remember that even though an intentional tortfeasor is potentially liable for all the harm they cause, recovery for harm is possible only if there is a maintainable cause of action to recover for that harm. In this case E cannot maintain a conversion cause of action. D Incorrect. It erroneously fails to apply the doctrine of transferred intent and the rule of extended liability in intentional tort. Transferred intent operates whenever the tort intended and the tort committed are within the original writ of trespass (Trespass to Land, Trespass to Chattel, False Imprisonment, Assault and Battery). Whenever a tortfeasor intends any one of the five but ends up committing any other of the five torts then all the torts committed are deemed to have the intent prong satisfied. For example in the above scenario A intended to falsely imprison B, but while committing false imprisonment he also caused B to get scared. Therefore A has committed both an assault and a false imprisonment against B. Additionally, not only does intent transfer between torts but it transfers between people as well. So even though A was intending only to falsely imprison B, he also caused C to get scared and therefore has committed assault against C. Similarly, even though A's only intent was to falsely imprison B, his tortious conduct scared C and resulted in a violent collision with D (harmful or offensive contact) and as a result D has an actionable battery claim against A. However A's intent to falsely imprison B caused the complete destruction of E's computer (a chattel). Now rather than suing for the harm caused to the chattel as in the previous question E is seeking to have A pay the entire market price of the computer when it was destroyed. This means that rather than trespass to chattel against A, E is seeking to recover pursuant to conversion. Conversion unlike trespass to chattel is not one of the torts within the original writ of trespass and therefore transferred intent will not support E's action against A. Also relevant to this hypothetical is the rule of limitless causation in intentional tort. A tortfeasor is usually liable for all the harm caused by their tortious conduct whether or not the harm was foreseeable. While this rule will help C and D recover it will not help E recover here. The reason is a little complicated. Because transferred intent does not apply, in order to recover E would have to prove that A was at least substantially certain that trying to imprison B would cause the destruction of E's computer. Because A's intent to imprison does not transfer to a conversion action and the facts do not indicate that A was substantially certain to destroy E's computer when he tried to imprison B, the intent prong of the conversion suit would fail and E would not be able to recover against A for the loss of the computer. It is important to remember that even though an intentional tortfeasor is potentially liable for all the harm they cause, recovery for harm is possible only if there is a maintainable cause of action to recover for that harm. In this case E cannot maintain a conversion cause of action.

A pointed a gun at B scaring B and without authority or justification, A ordered B into a room and locked the door for 2 hours. B was awake the whole time and worried about what would happen until A released him unharmed. When A initially pointed gun at B, B was standing next to C and C dove for cover thinking the gun was pointed at her. In diving for cover C collided violently with D causing D to spill coffee on E's computer. E subsequently had to pay to have the computer repaired. Who may maintain an action against A? A Only B but not C, D and E may maintain an action against A B Only B and C but not D and E may maintain an action against A C Only B, C and D but not E may maintain an action against A D B, C, D and E may all maintain actions against A

A Incorrect. It erroneously fails to apply the doctrine of transferred intent and the rule of extended liability in intentional tort. Transferred intent operates whenever the tort intended and the tort committed are within the original writ of trespass (Trespass to Land, Trespass to Chattel, False Imprisonment, Assault and Battery). Whenever a tortfeasor intends any one of the five but ends up committing any other of the five torts then all the torts committed are deemed to have the intent prong satisfied. For example, in the above scenario, A intended to falsely imprison B, but while committing false imprisonment he also caused B to get scared. Therefore, A has committed both an assault and a false imprisonment against B. Additionally, not only does intent transfer between torts but it transfers between people as well. So even though A was intending only to falsely imprison B, he also caused C to get scared and therefore has committed assault against C. Similarly, even though A's only intent was to falsely imprison B, his tortious conduct scared C and resulted in a violent collision with D (harmful or offensive contact), and as a result D has an actionable battery claim against A. Likewise, A's intent to falsely imprison B caused damage to E's chattel (computer), which had to be repaired, and so A is also liable to E for trespass to chattel. Also relevant to this hypothetical is the rule of limitless causation in intentional tort. A tortfeasor is usually liable for all the harm caused by their tortious conduct whether or not the harm was foreseeable. This is an important distinction between intentional tort and negligence. In negligence the tortfeasor is liable only for the foreseeable harm their tortious conduct causes. B Incorrect. It erroneously fails to apply the doctrine of transferred intent and the rule of extended liability in intentional tort. Transferred intent operates whenever the tort intended and the tort committed are within the original writ of trespass (Trespass to Land, Trespass to Chattel, False Imprisonment, Assault and Battery). Whenever a tortfeasor intends any one of the five but ends up committing any other of the five torts then all the torts committed are deemed to have the intent prong satisfied. For example, in the above scenario, A intended to falsely imprison B, but while committing false imprisonment he also caused B to get scared. Therefore, A has committed both an assault and a false imprisonment against B. Additionally, not only does intent transfer between torts but it transfers between people as well. So even though A was intending only to falsely imprison B, he also caused C to get scared and therefore has committed assault against C. Similarly, even though A's only intent was to falsely imprison B, his tortious conduct scared C and resulted in a violent collision with D (harmful or offensive contact), and as a result D has an actionable battery claim against A. Likewise, A's intent to falsely imprison B caused damage to E's chattel (computer), which had to be repaired, and so A is also liable to E for trespass to chattel. Also relevant to this hypothetical is the rule of limitless causation in intentional tort. A tortfeasor is usually liable for all the harm caused by their tortious conduct whether or not the harm was foreseeable. This is an important distinction between intentional tort and negligence. In negligence the tortfeasor is liable only for the foreseeable harm their tortious conduct causes C Incorrect. It erroneously fails to apply the doctrine of transferred intent and the rule of extended liability in intentional tort. Transferred intent operates whenever the tort intended and the tort committed are within the original writ of trespass (Trespass to Land, Trespass to Chattel, False Imprisonment, Assault and Battery). Whenever a tortfeasor intends any one of the five but ends up committing any other of the five torts then all the torts committed are deemed to have the intent prong satisfied. For example, in the above scenario, A intended to falsely imprison B, but while committing false imprisonment he also caused B to get scared. Therefore, A has committed both an assault and a false imprisonment against B. Additionally, not only does intent transfer between torts but it transfers between people as well. So even though A was intending only to falsely imprison B, he also caused C to get scared and therefore has committed assault against C. Similarly, even though A's only intent was to falsely imprison B, his tortious conduct scared C and resulted in a violent collision with D (harmful or offensive contact), and as a result D has an actionable battery claim against A. Likewise, A's intent to falsely imprison B caused damage to E's chattel (computer), which had to be repaired, and so A is also liable to E for trespass to chattel. Also relevant to this hypothetical is the rule of limitless causation in intentional tort. A tortfeasor is usually liable for all the harm caused by their tortious conduct whether or not the harm was foreseeable. This is an important distinction between intentional tort and negligence. In negligence the tortfeasor is liable only for the foreseeable harm their tortious conduct causes. D Correct. It correctly applies the doctrines of transferred intent and the rule of extended liability in tort. Transferred intent operates whenever the tort intended and the tort committed are within the original writ of trespass (Trespass to Land, Trespass to Chattel, False Imprisonment, Assault and Battery). Whenever a tortfeasor intends any one of the five but ends up committing any other of the five torts then all the torts committed are deemed to have the intent prong satisfied. For example, in the above scenario, A intended to falsely imprison B, but while committing false imprisonment he also caused B to get scared. Therefore, A has committed both an assault and a false imprisonment against B. Additionally, not only does intent transfer between torts but it transfers between people as well. So even though A was intending only to falsely imprison B, he also caused C to get scared and therefore has committed assault against C. Similarly, even though A's only intent was to falsely imprison B, his tortious conduct scared C and resulted in a violent collision with D (harmful or offensive contact), and as a result D has an actionable battery claim against A. Likewise, A's intent to falsely imprison B caused damage to E's chattel (computer), which had to be repaired, and so A is also liable to E for trespass to chattel. Also relevant to this hypothetical is the rule of limitless causation in intentional tort. A tortfeasor is usually liable for all the harm caused by their tortious conduct whether or not the harm was foreseeable. This is an important distinction between intentional tort and negligence. In negligence the tortfeasor is liable only for the foreseeable harm their tortious conduct causes. So in this scenario B, C, D and E may maintain actions against A.

While asleep one night, homeowner hears a noise coming from a window in the house. She immediately goes down stairs armed with a handgun. In the dark she notices a broken window and a person climbing through the window. Without asking questions she aims at the intruder and fires, severely injuring the intruder in the process. It turns out that the intruder was homeowner's cousin who was intoxicated was simply breaking into the house to leave a present for the family. Will homeowner be able to claim the privilege of self-defense in a suit by cousin against homeowner? A No, because there was no force directed at homeowner so the use of force was not authorized in defense. B No, because deadly force is never authorized as a defense unless you are certain that the attacker will use or has used deadly force against you. C Yes, because the castle doctrine allows a person to use deadly force to protect their occupied dwelling at night. D Yes, because unauthorized entries into a dwelling are never permissible.

A Incorrect. It is incorrect because of the castle doctrine. The castle doctrine allows a person to protect their occupied dwelling at night by using deadly force without asking any questions. This is also a classic example from tort law of where the use of deadly force is allowed without requiring the defender to retreat or to attempt to avoid the use of deadly force before resorting to it. The common law presumption is that anyone entering an occupied dwelling at night does so for the purpose of causing death or serious bodily harm. B Incorrect. It is incorrect because of the castle doctrine. The castle doctrine allows a person to protect their occupied dwelling at night by using deadly force without asking any questions. This is also a classic example from tort law of where the use of deadly force is allowed without requiring the defender to retreat or to attempt to avoid the use of deadly force before resorting to it. The common law presumption is that anyone entering an occupied dwelling at night does so for the purpose of causing death or serious bodily harm. C Correct. It is correct because of the castle doctrine. The castle doctrine allows a person to protect their occupied dwelling at night by using deadly force without asking any questions. This is also a classic example from tort law of where the use of deadly force is allowed without requiring the defender to retreat or to attempt to avoid the use of deadly force before resorting to it. The common law presumption is that anyone entering an occupied dwelling at night does so for the purpose of causing death or serious bodily harm. D Incorrect. Unauthorized entries into dwellings are sometimes permissible under the doctrine of necessity. The doctrine of necessity allows an individual's property in exceptional circumstances, to be used or destroyed if it is necessary to prevent a greater harm.

Twin 1 has an identical twin brother, Twin 2. Twin 1 is a violent person while Twin 2 is a docile, benevolent individual. Twin 1 and Twin 2 always dressed in identical fashion however. One day jogger was confronted by Twin 1 who told jogger, "If I ever run into you again I will kill you instantly." Jogger kept on jogging and as he turned the corner he saw Twin 2 standing on the side of the road. Twin 2 approached jogger quickly to inquire how jogger's day was going. Thinking Twin 2 was Twin 1, jogger began throwing large rocks at Twin 2 hitting him many times under the mistaken belief he had to defend himself as his own life was in danger. Jogger stopped throwing stones and ran away quickly as soon as Twin 2 stopped approaching him. In the lawsuit Twin 2 v. Jogger for battery, jogger claimed that he reasonably mistook Twin 2 for Twin 1 and acted as any reasonable person would have in the situation and defended himself. Which of the following legal arguments is correct? A Twin 2's argument that the reasonableness of jogger's actions is irrelevant in intentional tort law because intent is subjective B Jogger's argument that if a reasonable person would have done what he did he should be exempt from liability C Twin 2's argument that he never intended the jogger any harm and thus the jogger has no basis for applying the self-defense doctrine D Even if jogger continued to throw stones at Twin 2 after Twin 2 ceased approaching him he would not be liable because there is no limit to the level of force permitted in defending oneself.

A Incorrect. It is incorrect for a reason that confuses many torts students. There are two aspects to establishing liability in intentional tort. The first is that the plaintiff must establish all the elements of the tort exist. In this case the tort is battery. One element of the tort of battery and all intentional torts is intent. Whether the defendant possessed the requisite intent is absolutely a subjective standard. In other words intent is proven by showing that the particular defendant had the specific intent or was substantially certain. The knowledge of the reasonable person is not imputed as a matter of law onto the defendant in establishing the defendant's intent. In negligence however the defendant is deemed to possess all the knowledge of a reasonable person. In this context the proof of intent is said to be subjective (specific to the defendant) and the proof of negligence is said to be objective (not dependent on the defendant's particular knowledge etc). But once the plaintiff has established that all the elements of the tort have been met liability is not automatic. The defendant could admit that all elements of the tort have been met but if the defendant then proves the existence of a privilege the defendant is absolved from liability despite all the elements of the tort being met. One such privilege is self-defense. A person is privileged to use reasonable force to defend themselves. In contrast to the intent element reasonableness of the defendant's actions is the touchstone of whether or not a privilege can be asserted. Another rule is that if a person makes a reasonable mistake in exercising a privilege the privilege remains valid. Here, the facts state the twins were identical and dressed in the same clothes. Jogger reasonably mistook Twin 2 for Twin 1 and defended himself from a perceived death threat. Twin 2's argument about the irrelevance of reasonableness in intentional tort is therefore overbroad. While it is irrelevant in establishing intent it is very relevant in establishing whether or not a privilege can be claimed. B Correct. It is correct for a reason that confuses many tort students. There are two aspects to establishing liability in intentional tort. The first is that the plaintiff must establish all the elements of the tort exist. In this case the tort is battery. One element of the tort of battery and all intentional torts is intent. Whether the defendant possessed the requisite intent is absolutely a subjective standard. In other words intent is proven by showing that the particular defendant had the specific intent or was substantially certain. The knowledge of the reasonable person is not imputed as a matter of law onto the defendant in establishing the defendant's intent. In negligence however the defendant is deemed to possess all the knowledge of a reasonable person. In this context the proof of intent is said to be subjective (specific to the defendant) and the proof of negligence is said to be objective (not dependent on the defendant's particular knowledge etc). But once the plaintiff has established that all the elements of the tort have been met liability is not automatic. The defendant could admit that all elements of the tort have been met but if the defendant then proves the existence of a privilege the defendant is absolved from liability despite all the elements of the tort being met. One such privilege is self-defense. A person is privileged to use reasonable force to defend themselves. In contrast to the intent element reasonableness of the defendant's actions is the touchstone of whether or not a privilege can be asserted. Another rule is that if a person makes a reasonable mistake in exercising a privilege the privilege remains valid. Here, the facts state the twins were identical and dressed in the same clothes. Jogger reasonably mistook Twin 2 for Twin 1 and defended himself from a perceived death threat. Jogger's argument that if a reasonable person would have acted as he did he is exempt from liability is exactly right in terms of establishing a privilege. C Incorrect. A reasonable mistake in applying a privilege does not negate the privilege. Here jogger reasonably mistook Twin 2 for Twin 1. Jogger after the death threat was permitted to defend himself from Twin 1. The fact that he reasonably mistook Twin 2 for Twin 1 does not negate the privilege. D Incorrect. It runs contrary to the rule of self-defense that once the threat is negated the continued use of force against is impermissible. This is often stated as, Self-defense is permissible but retaliation is not." In other words jogger could use self-defense to prevent what he perceived as harm from Twin 1, but as long as the threat was negated (Twin 2 ceased his approach) the need to defend oneself ceased to exist and the use permission to use force to do so likewise ceased.

Homeowner hears on the news that there is a dangerous criminal on the loose in her neighborhood. She is on edge as a result and hyper vigilant. One evening at dusk she is taking out her trash to the street and is startled by her neighbor who walked up behind her to give her some pie. Thinking it was the criminal approaching her she swings a large piece of wood at the neighbor causing him a pretty painful broken arm. Neighbor sues homeowner for battery. Homeowner argues that the intent element of the tort is missing because she never intended to harm the neighbor but thought it was a criminal. Will Homeowner's defense be successful? A Yes, because mistake negates intent. B Yes, but only if she can prove her mistake was reasonable. C No, because mistake does not negate intent. D Yes, if she did not use excessive force when swinging the large piece of wood.

A Incorrect. It states an incorrect rule of law. Mistake does not negate intent is the correct rule of law. Here the homeowner is arguing that the intent element of the tort is missing so mistake is irrelevant. B Incorrect. It states an incorrect rule of law. Mistake does not negate intent even if it is a reasonable mistake. Here the homeowner is arguing that the intent element of the tort is missing so mistake is irrelevant. C Correct. It states the correct rule of law. Mistake does not negate intent. D Incorrect. The issue of force has nothing to do with intent. The issue of force may be relevant in assessing the reasonableness of self-defense but it is irrelevant to intent.

Patron was trying to enter a large exclusive hotel in the middle of a city. The representatives of the hotel met patron at the door and casually informed him that he was not permitted to be on the hotel property. After that, they forcibly excluded him from the hotel property every time he tried to enter. If patron files a false imprisonment claim against the hotel what is the hotel's best defense? A False imprisonment can never occur on private property. B False imprisonment cannot occur because property peaceably informed patron he would be a trespasser the first time patron tried to enter. C Exclusion is never false imprisonment. D The remainder of the city other than the hotel was too large to be considered false imprisonment.

A Incorrect. It's an incorrect statement of hte law. In fact, many false imprisonment claims involve confinement on private property B Incorrect. It tries to distract the student from the false imprisonment issue with a statement of law related to property owner's rights to exclude uninvited guests. Property owner's rights are not at issue in this question. C Correct. It restates an established rule of law. That rule is that exclusion is never false imprisonment. Here, patron was excluded from entering, and exclusion is almost the opposite of confinement. D Incorrect. At some point in time an area becomes too large for confinement within to be considered false imprisonment. D seems like a good choice. However, we are not told how large the city is and, additionally, because the minimum size required before the "size of area is too large" defense can be invoked is not absolute, but rather decided on a case by case basis, C is the better answer choice because it provides the opportunity to use an established rule of law as a defense.

A martial arts expert with incredibly fast reflexes and catlike agility was walking along the street when a stranger approached her. Stranger knew of Martial Arts Expert's reputation for speed and wanted to see how fast Martial Arts Expert really was. Stranger was 3 feet in front of Martial arts expert when he attempted to strike her shoulder with his hand. Even though it was moving quickly, Martial Arts Expert saw the stranger's hand and arm moving as if it was in slow motion. In her mind she was calm and experienced no fear whatsoever. Not only was the stranger laughably slow but she would not have gotten hurt if the strike made contact with her. She merely thought, "I'm just going to move out of the way of his hand because I don't want any strangers touching me." She moved out of the way of the strike and stranger fell. Is stranger liable for assault? A No, because stranger did not attempt or intend to cause assault but only attempted to cause a battery so the element of intent is missing. B No, because Martial Arts Expert was never fearful of being hit C Yes, because Martial Arts Expert was aware of an imminent unconsented to contact which would be harmful or offensive to a reasonable person D Yes, because the stranger could actually have reached the Martial Arts Expert

A Incorrect. It's incorrect because of the concept of transferred intent. Where a tortfeasor intends any of the five torts within the original writ of trespass (Trespass to Land, Trespass to Chattel, False Imprisonment, Assault and Battery) and alternatively or additionally ends up committing any of the other five torts within the writ of trespass, then intent exists as a matter of law for any tort committed. So in this case, even if the tortfeasor can prove he did not intend to cause fear or apprehension (Assault) but only cause contact (battery), if in fact he caused Maria to be fearful or apprehensive of imminent harmful or offensive contact, then he will also be deemed to have intended assault. Intent for one of the five is intent for all of the five. B Incorrect. Assault does not require fear AND apprehension but fear OR apprehension. Either fear or apprehension is legally sufficient for the tort. Apprehension is defined as the mere anticipation or awareness of an unconsented to touching. Remember, unconsented to touching qualifies as offensive if we remember that battery is a dignitary tort. So, martial arts expert's awareness that an unconsented, offensive touching would occur unless she took evasive action is enough for apprehension to be established. Here, the facts say that the stranger's hand was moving quickly so even though it appeared to be moving slowly to martial arts expert, the contact was imminent. C Correct. Assault does not require fear but fear OR Apprehension and Apprehension is defined as the mere anticipation or awareness of an unconsented to touching. Remember, unconsented to touching qualifies as offensive if we remember that battery is a dignitary tort. So, martial arts expert's awareness that an unconsented, offensive touching would occur unless she took evasive action is enough for apprehension to be established. Here, the facts say that the stranger's hand was moving quickly so even though it appeared to be moving slowly to martial arts expert, the contact was imminent. D Incorrect. Assault does not require the tortfeasor to have the actual ability to carry out the imminent harmful or offensive contact but only the apparent ability to do so.

About two years ago, Client was a patient of Ophthalmologist. Client alleges that during the time that he was Ophthalmologist's patient, Ophthalmologist improperly performed LASIK surgery on Client's eyes. The LASIK surgery was supposed to correct Client's poor vision. Instead, the surgery was performed skillfully, but resulted in blindness in both eyes, a risk about which Client was never informed and a reasonable person would not have known. If Client had known of the risk of blindness, he would not have opted for the surgery. Client hires Attorney Matthew to sue Ophthalmologist for medical malpractice. Attorney Matthew fails to file the complaint to initiate the cause of action against Ophthalmologist within the applicable statute of limitations. As a result, the court dismisses Client's case against Ophthalmologist. Client then hires Attorney Becky to represent him in a cause of action against Attorney Matthew for legal malpractice. In the legal malpractice action brought by Client against Attorney Matthew, how, if at all, must Client prove factual causation? A Client will not need to prove factual causation in this legal malpractice action. B To prove factual causation in the legal malpractice action, Client will need to prove that he possibly would have prevailed in the underlying medical malpractice action. C To prove factual causation in the legal malpractice action, Client will need to prove that he more likely than not would have prevailed in the underlying medical malpractice action. D To prove factual causation in the legal malpractice action, Client will need to prove that Attorney Matthew breached the standard of care in not filing the complaint before the statute of limitations ran.

A Incorrect. Legal malpractice is a professional negligence action. Therefore, as is the case for all negligence actions, Client is required to prove factual causation. B Incorrect. Client will have to prove that Attorney Matthew's breach of the standard of care probably (more likely than not) resulted in his injury. A possibility is insufficient to establish factual causation, so B is incorrect. C Correct. Client will have to prove that he probably (i.e., more likely than not) would have prevailed in the underlying medical malpractice action. D Incorrect. It defines only breach of the standard of care and does not define factual causation.

Prankster yells in an urgent voice, as loudly as possible, at person standing on the edge of a forest, "Snake!!!!!!!!" While doing this he is pointing at the feet of person. Person jumps violently and hits his head when he lands. Would Prankster be liable for assault and battery? A If there really was a snake near the feet of person Prankster would be liable for assault and battery. B If Prankster reasonably believed there was a snake at the feet of person but there wasn't one Prankster would be liable for assault and battery. C Because mistake does not negate intent Prankster would be liable for assault and battery even if Prankster reasonably but mistakenly believed there was a snake near the feet of person. D Because reasonable mistake in exercising a privilege does not negate the privilege Prankster would not be liable if Prankster reasonably but mistakenly believed there was a snake near the feet of person.

A Incorrect. Let's be clear Prankster had the intent to cause person to be startled and move away quickly from the snake. Prankster therefore had the intent to cause person fear or apprehension of imminent harmful or offensive contact. Person jumped violently because he was put in fear of imminent harmful or offensive contact (the contact with snake) so all the elements of assault are satisfied. Because Prankster intended assault but also ended up causing harmful or offensive contact then transferred intent results in all the elements of battery being satisfied as well. So Prankster did commit an assault and battery, but if the snake was really there Prankster would not be liable because he would be acting reasonably in the circumstances to warn person and therefore the privilege of defense of others would apply to negate any liability for the technical assault and battery. B Incorrect. Let's be clear Prankster had the intent to cause person to be startled and move away quickly from the snake. Prankster therefore had the intent to cause person fear or apprehension of imminent harmful or offensive contact. Person jumped violently because he was put in fear of imminent harmful or offensive contact (the contact with snake) so all the elements of assault are satisfied. Because Prankster intended assault but also ended up causing harmful or offensive contact then transferred intent results in all the elements of battery being satisfied as well. So Prankster did commit an assault and battery, but if the snake was really there Prankster would not be liable because he would be acting reasonably in the circumstances to warn person and therefore the privilege of defense of others would apply to negate any liability for the technical assault and battery. If Parker reasonably believed the snake was there he would also not be liable because he was exercising the privilege of defense of others. A reasonable mistake in exercising a privilege does not negate the privilege. C Incorrect. Let's be clear Prankster had the intent to cause person to be startled and move away quickly from the snake. Prankster therefore had the intent to cause person fear or apprehension of imminent harmful or offensive contact. Person jumped violently because he was put in fear of imminent harmful or offensive contact (the contact with snake) so all the elements of assault are satisfied. Because Prankster intended assault but also ended up causing harmful or offensive contact then transferred intent results in all the elements of battery being satisfied as well. So Prankster did commit an assault and battery, but if the snake was really there Prankster would not be liable because he would be acting reasonably in the circumstances to warn person and therefore the privilege of defense of others would apply to negate any liability for the technical assault and battery. Even though mistake does not negate intent Prankster would not be liable because he could argue that there actually was intent which he would not debate, but the reasonable mistake negated his liability for the assault and battery he otherwise committed. Because Prankster could apply reasonable mistake to the privilege, the fact that reasonable mistake does not negate intent will still not result in liability. D Correct. Let's be clear Prankster had the intent to cause person to be startled and move away quickly from the snake. Prankster therefore had the intent to cause person fear or apprehension of imminent harmful or offensive contact. Person jumped violently because he was put in fear of imminent harmful or offensive contact (the contact with snake) so all the elements of assault are satisfied. Because Prankster intended assault but also ended up causing harmful or offensive contact then transferred intent results in all the elements of battery being satisfied as well. So Prankster did commit an assault and battery, but if the snake was really there Prankster would not be liable because he would be acting reasonably in the circumstances to warn person and therefore the privilege of defense of others would apply to negate any liability for the technical assault and battery. If Parker reasonably believed the snake was there he would also not be liable because he was exercising the privilege of defense of others. A reasonable mistake in exercising a privilege does not negate the privilege.

A major fire is approaching city. Between city and fire are five houses. These five houses will act as a bridge to communicate the fire into the city if they ignite. As a result the mayor of city orders the 5 houses destroyed despite the protests of the homeowners before the fire reaches them. The destruction of the houses prevents the destruction of the entire city. Homeowners sue the mayor and the city for trespass to land. Will homeowners prevail? A Homeowners will prevail because their private property was destroyed without consent. B Homeowners will prevail but liability would only exist for actual harm because the mayor and city were acting under the doctrine of necessity. C Homeowners will not prevail and there will be no liability because of the doctrine of public necessity. D Homeowners will not prevail because it was a government official who made the decision that necessity warranted destruction of their houses.

A Incorrect. Many doctrines permit, destruction of private property without tort compensation. Abatement of nuisances and the doctrine of public necessity are some examples. Public necessity may be stated as follows: "A defendant who damages, destroys, or uses the plaintiff's property in the reasonable belief that by doing so he can avoid or minimize serious and immediate harm to the public is protected against liability for intentional torts by a complete privilege." B Incorrect. Public necessity is defined as follows, "A defendant who damages, destroys, or uses the plaintiff's property in the reasonable belief that by doing so he can avoid or minimize serious and immediate harm to the public is protected against liability for intentional torts by a complete privilege." In contrast to private necessity the doctrine of public necessity is a complete privilege and the defendant if properly acting under the privilege is completely free of liability, even liability for actual damage. C Correct. Public necessity is defined as follows, "A defendant who damages, destroys, or uses the plaintiff's property in the reasonable belief that by doing so he can avoid or minimize serious and immediate harm to the public is protected against liability for intentional torts by a complete privilege." In contrast to private necessity the doctrine of public necessity is a complete privilege and the defendant if properly acting under the privilege is completely free of liability, even liability for actual damage. D Incorrect. It is not dispositive that a government official made the decision to destroy the houses. Public necessity protects also a private defendant who destroys another's property while acting under the privilege. This is likely because some situations are so emergent that the opportunity to protect the public will have expired if official government approval is required before destruction of the property.

Insane person erroneously believes that individual is sick and needs help. Insane person believes that individual can be cured by hitting the person over the head with a baseball bat. Insane person hits individual in the head with a baseball bat to cure the person and causes the person serious harm. Did the insane person commit a battery? A No, because the insane person was mistaken in believing the baseball bat would cure the person. B Yes, because the insane person intended no harm but only intended to help. C No, because an insane person cannot have intent. D Yes, because the insane person intentionally caused contact which was harmful or offensive to the person of another.

A Incorrect. Mistake does not negate intent B Incorrect. Motive and intent are not the same. A person with the best of intentions in touching a person may none the less be liable for battery if a reasonable person would find the contact harmful or offensive. C Incorrect. It is legally incorrect. Insane people are deemed to have intent exactly like sane people. D Correct. The insane person is liable for intentional torts exactly like a sane person would be and here the insane person acted intentionally to cause contact to the individual which was harmful or offensive.

In all jurisdictions that have replaced the traditional defense of contributory negligence with comparative fault: A comparative fault schemes were adopted judicially. B proof of the plaintiff's comparative fault may operate to reduce the plaintiff's recovery by the percentage of fault attributable to her. C comparative fault schemes were adopted legislatively. D proof of the plaintiff's comparative fault is a complete bar to the plaintiff's recovery.

A Incorrect. Not all jurisdictions with a comparative fault scheme in place have adopted them by judicial opinion. B Correct. It is a correct definition of comparative fault. C Incorrect. Not all jurisdictions with a comparative fault scheme have adopted them legislatively. D Incorrect. Although sometimes a comparative fault scheme will operate as a complete bar to recovery, it is not true that a comparative fault scheme always operates as a complete bar to recovery for the plaintiff.

A law student rented a furnished apartment pursuant to a yearlong lease. Two months into the lease her landlord began to solicit her advice about legal matters, but she refused to provide it. The landlord then demanded that she vacate the apartment immediately. The landlord also engaged in a pattern of harassment, calling the student at home every evening and entering her apartment without her consent during times when she was at school. During these unauthorized visits landlord removed the handles from the bathroom and kitchen sinks, but did not touch anything belonging to the student. The student is still living in the apartment and sues the landlord for trespass to land. Is the student likely to prevail? A No, because she has no standing to sue for trespass. B No, because the landlord caused no damage to her property. C Yes, but will recovery compensatory damages only. D Yes, and may obtain injunctive relief, compensatory damages, and punitive damages.

A Incorrect. Possession and not ownership of real property is what provides the basis for standing to sue in trespass. Here the student is in possession of the apartment pursuant to a lease agreement. B Incorrect. The fact that the landlord did not cause damage to the property of the student is not relevant. Trespass is actionable on only nominal damages and protects the right of the possessor to determine entry onto her property. C Incorrect. In intentional tort actions the full range of appropriate remedies is available. Here there would be compensatory damages for the changing of the locks etc. and for the harm caused the student by these actions, there may be injunctive relief to enjoin the landlord from continuing the unpermitted entries and punitive damages may also be available if the landlord's actions are egregious enough or considered retaliatory in nature. D Correct. In intentional tort actions the full range of appropriate remedies is available. Here there would be compensatory damages for the changing of the locks etc. and for the harm caused the student by these actions, there may be injunctive relief to enjoin the landlord from continuing the unpermitted entries and punitive damages may also be available if the landlord's actions are egregious enough or considered retaliatory in nature.

Sam received a text message with a video from an undercover agent pretending to be a gang member. Video showed a distant relative of Sam being severely beaten by other gang members a few days earlier. Distant relative has a viable cause of action for assault and battery against gang members. Upon seeing the video Sam got sick, vomited quite a few times, and was diagnosed with high blood pressure as a result of the video. He also experienced severely disabling emotional distress as a result of the video. May Sam maintain an action for intentional infliction of emotional distress against the gang members based on his reaction to seeing the video? A Probably, because beating someone is considered extreme and outrageous conduct. B Probably, even if the gang members were not substantially certain Sam would see the video. C Probably, not because Sam is not a close relative to the victim of the tortious conduct. D Probably not, because Sam was not sensorily and contemporaneously present during the beating of his distant relative.

A Incorrect. The answer choice correctly states that physically attacking someone is extreme and outrageous conduct; however, in this situation that is not enough. Initially, because intent does not transfer in the case of IIED, the gang members would have to at least have been substantially certain Sam would see the video. The facts do not support this as it suggests an undercover agent sent Sam the video. B Incorrect. The lack of substantial certainty would render the tort unsustainable because the intent prong would not be satisfied. As IIED is an intentional tort which is not subject to the doctrine of transferred intent, the intent must be established independently for this tort as it won't transfer from the assault and battery. As a result, if the gang members were not substantially certain Sam would see the video, there would not be intent, and Sam would not be able to maintain an action. C Incorrect. It is also tricky because it tries to erroneously suggest the application of the bystander rule. The bystander rule states that a bystander or third party may recover for IIED based on tortious conduct to a close relative if the tortfeasors were substantially certain the bystander would witness the tortious conduct, and in this situation the bystander does not have to suffer bodily harm. When the bystander is not a close relative of the victim, the bystander the emotional distress must be accompanied by bodily harm to recover under a theory of IIED. The fact that Sam is not a close relative of the victim in this case is irrelevant because Sam has suffered bodily harm (vomiting, high blood pressure, etc), so C is incorrect. D Correct. One of the nuances of bystander recovery pursuant to IIED based on tortious conduct directed at another person is that the bystander must have observed the conduct as it is happening. According to the Dobbs Hornbook § 307 the line is drawn, "against recovery by excluding those who are not present and who have not had an immediate sensory perception of the injury." Here, Sam was not present and he received the video "a few days" after the "primary injury." Therefore, he would not be able to maintain the IIED action based on the tortious conduct directed to his distant relative.

An old farmhouse contains some old furniture that the owner stores there. No one lives there. Recently homeless people have been breaking in and using the property as a shelter. In response the owner of the farmhouse boarded up all the windows, placed a fence around the property and posted conspicuous, "No Trespassing. Keep Out" signs around the property. Additionally the owner rigged spring guns to fire when anyone opened an internal door. One day a curious person walks into the farmhouse opens a door and is shot in the leg by a spring gun. The curious person sues the farmhouse owner for battery. Which of the following statement is correct? A The castle doctrine protects the farmhouse owner from liability. B The castle doctrine does not apply. C Deadly force to protect your property is always appropriate after warnings are posted and here the signs clearly warned people to stay out. D Deadly force to protect unoccupied property is never permitted.

A Incorrect. The castle doctrine only operates when an intruder attempts to enter an occupied dwelling at night. It does not apply to an old farmhouse which cannot be a dwelling because no one lives there. B Correct. The castle doctrine only operates when an intruder attempts to enter an occupied dwelling at night. It does not apply to an old farmhouse in which no one lives. C Incorrect. Warnings alone do not allow the use of deadly force where it otherwise is impermissible. D Incorrect. It invokes the complexity of tort law. The correct rule is that only reasonable force may be used to protect property. It would be a dangerous over generalization to say that deadly force is never reasonable to protect unoccupied property. For example at airshows military aircraft are protected by soldiers who are authorized to use deadly force to prevent your entry onto the property. Any number of high value places may not traditionally be occupied in the traditional sense but it may be reasonable to use deadly force to protect them. Fort Knox is another example, maybe even banks at night. Whether something is reasonable is determined on a case by case basis.

Thief knowing that child was only 8 years old and home alone knocks on the door of child's house. Child opens door and thief befriends child. After a while thief tells child that he will buy child candy if child allows thief to come in and to take away anything thief wants. Child, excited about the prospect of candy, allows thief in. Thief takes away many chattels from the house and gives child candy as promised. Was the child's consent valid? A Yes. The child's consent was validly obtained because thief bought the candy as promised. B No. The consent was invalid because only the owner of a dwelling can give permission to enter. C Yes. The child's consent was validly given because the thief did not place the child in duress in order to obtain consent. D No. The child is incapable of giving consent.

A Incorrect. The common law rule is that minors are incapable of giving consent. The child was eight years old and therefore incapable of giving consent. B Incorrect. Any person in possession of premises can give consent to enter. C Incorrect. Minors are incapable of giving consent whether or not they are placed in duress. D Correct. As a minor the child is incapable of giving consent.

A commercial aircraft pilot had an argument with her friend. Her friend lives 300 miles from an airport that the commercial pilot usually flies out of. One morning the commercial pilot realized that her flight path for that day would take her really close to the property of her friend. On that day the pilot lifted off from the airport piloting a sleek modern Boeing 787 aircraft. The aircraft climbed to an altitude of 39,000 feet where it could not be seen or heard from people on the ground. At that height and traveling at approximately 550 miles per hour the pilot diverted the plane ever so slightly so it passed directly over her friend's house. Her friend was simply unaware that the aircraft flew over her property as it did so. After the pilot landed she called her friend and said, "I was so angry at you I flew my airplane at 39,000 feet directly over your property." What result if the friend sues the pilot for trespass to land? A The friend will prevail because of the cujus est solum est uque ad coleum doctrine. B The pilot will prevail because the cujus est solum est uque ad coleum doctrine has been overruled. C The friend will not prevail as air travel above 5000 feet is not trespass to land as a matter of law. D The pilot will prevail because the airplane was not in the immediate reaches of the land and did not interfere with the use and enjoyment of the land.

A Incorrect. The cujus est solum est uque ad coleum doctrine is no longer an absolute. This doctrine literally means that one who owns the land owns the area above the land all the way to the heavens. With the advent of artillery and aircraft overflight this doctrine has yielded to the realities of modern weaponry and air travel but it hasn't been overruled. The general restatement rule now is that aircraft overflight can only be considered trespass if the overflight occurs in the immediate reaches of the land and interferes with the use and enjoyment of the land. Here the friend was simply unaware of the aircraft overflight and the plane was at an altitude where it could not be seen or heard from land so it clearly does not satisfy the elements of "immediate reaches" and "interfering with use and enjoyment." The cujus est solum est uque ad coleum doctrine is no longer dispositive in and of itself. B The pilot will prevail because the cujus est solum est uque ad coleum doctrine has been overruled. C Incorrect. There is no fixed altitude above which the cujus est solum est uque ad coleum is said to no longer apply as a matter of law. This doctrine literally means that one who owns the land owns the area above the land all the way to the heavens. With the advent of artillery and aircraft overflight this doctrine has yielded to the realities of modern weaponry and air travel but it hasn't been overruled. The general restatement rule now is that aircraft overflight can only be considered trespass if the overflight occurs in the immediate reaches of the land and interferes with the use and enjoyment of the land. No altitude (5000 ft in this answer choice) is talismanic or a bright line rule beyond which the doctrine is deemed to always yield to the realities of modern weaponry and air travel. D Correct. The general restatement rule now is that aircraft overflight can only be considered trespass if the overflight occurs in the immediate reaches of the land and interferes with the use and enjoyment of the land. Here the friend was simply unaware of the aircraft overflight and the plane was at an altitude where it could not be seen or heard from land so it clearly does not satisfy the elements of "immediate reaches" and "interfering with use and enjoyment." The cujus est solum est uque ad coleum doctrine is no longer dispositive in and of itself and so overflights are no longer trespass as a matter of law.

One day Darcy, a high school teacher, takes her students to watch an ice hockey game starring the local semi-professional team. Aware that the students are in attendance at the game that day, the team owner invites Darcy and the kids onto the ice during intermission. As Darcy steps onto the ice, a hockey puck flies through the air, hitting her in the face and knocking out her front teeth. Darcy sues the team in negligence. In a jurisdiction that recognizes the traditional defense of assumption of the risk, what effect will the defense have on Darcy's claim? A Darcy's recovery in negligence will be reduced by the percentage of fault attributable to her if the defendant can prove that Darcy voluntarily encountered the risk of being hit in the mouth by a hockey puck and was aware that she might have her front teeth knocked out. B Darcy's recovery in negligence will be reduced by the percentage of fault attributable to her if the defendant can prove that a reasonable person would not have voluntarily encountered the risk of being hit in the mouth by a hockey puck. C Darcy's recovery in negligence will be barred if the defendant can prove that a reasonable person would not have voluntarily encountered the risk of being hit in the mouth by a hockey puck. D Darcy's recovery in negligence will be barred if the defendant can prove that Darcy voluntarily encountered the risk of being hit in the mouth by a hockey puck and was aware that she might have her front teeth knocked out.

A Incorrect. The defense of assumption of the risk acts as a complete bar to recovery and does not merely reduce the plaintiff's recovery. B Incorrect. It provides a definition of comparative fault rather than assumption of the risk. This question asks about the traditional defense of assumption of the risk in a negligence action. C Incorrect. It provides a definition of contributory negligence rather than assumption of the risk. This question asks about the traditional defense of assumption of the risk in a negligence action. D Correct. It provides the correct definition of assumption of the risk.

Hiker gets lost in the woods. As hiker is trying to find his way out of the woods he is attacked by a bear. As the hiker is running from the attacking bear he sees a cabin in the woods. The door of the cabin has a conspicuous sign, saying, "Private Property. Keep Out." In desperation after reading the sign the hiker tries the door and it is unlocked. He enters the cabin and shuts the door behind him. Hiker is trapped in the cabin for weeks before rescue arrives and the bear is scared off. During the time he was in the cabin, hiker had to destroy wooden furniture to keep a fire going in order to prevent himself from freezing to death. He also had to make a small hole in the roof to allow snow to melt and drip into the cabin so he could have drinking water. And finally, he had to use food stored in the cabinets in order to survive. After rescue the owner of the cabin sues hiker for trespass to land after the hiker refuses to pay for the damage done to cabin owner's property. Is the hiker liable? A No, hiker is not liable for trespass to land because he was exercising the privilege of private necessity. B Yes, hiker is liable for trespass to land because he intentionally entered the property in the possession of another without consent. C Yes, because hiker caused actual damage to the property. D No, because a person may freely enter anyone's property as long as the entry will reasonably benefit the person.

A Incorrect. The defense of private necessity allows you to enter and use another's property when there is a serious an imminent threat to your person but it only prevents liability for nominal damages. While the privilege allows defendant to use the property of another in an emergency to save defendant's life and it trumps the landowner's right to exclude defendant during the emergency, the defendant must pay for any actual harm done. B Incorrect. The privilege of private necessity allows defendant to enter property of another without consent. The entry is considered a trespass but the defendant will not be liable as long as no actual damage is caused. So the mere intentional entry onto the land does not trigger liability as suggested in answer choice B. C Correct. The defense of private necessity allows you to enter and use another's property when there is a serious an imminent threat to your person but it only prevents liability for nominal damages. While the privilege allows defendant to use the property of another in an emergency to save defendant's life and it trumps the landowner's right to exclude defendant during the emergency, the defendant must pay for any actual harm done. Here there was harm done to furniture, the roof and supplies were consumed so there was actual harm and therefore liability will attach. D Incorrect. It is blatantly legally erroneous. It is simply an incorrect rule of law which if correct would prevent the existence of the trespass to land action.

In a lawsuit brought by Plaintiff against Defendant alleging negligence, Plaintiff seeks to use Defendant's violation of a statute to help establish his prima facie case. The court conducts an inquiry to determine: (1) whether Plaintiff is a member of the class the legislature intended to protect in enacting the statute; (2) whether the hazard about which Plaintiff is complaining is one the legislature intended to prevent; and (3) whether tort liability is appropriate. How is conducting this three step analysis important to Plaintiff's claim against Defendant? A It establishes whether Plaintiff may use the doctrine of res ipsa loquitur. B It establishes the effect of proof of the statute's violation. C It establishes whether the statute's violation can be used to establish negligence (also known as negligence per se). D It establishes whether a Rule of Law may be used to establish negligence.

A Incorrect. The doctrine of res ipsa loquitur applies when there is an accident that ordinarily does not occur in the absence of negligence, the defendant was in exclusive control of the instrumentality of harm, and there is no other evidence of the defendant's negligence. This question asks you to identify the legally relevant inquiry when trying to use a statute's violation to establish a defendant's breach of the standard of care. B Incorrect. The effect of the proof is a separate determination than whether a statute can be used to establish the standard of care. This question asks you to identify the legally relevant inquiry when trying to use a statute's violation to establish a defendant's breach of the standard of care. C Correct. The determination is one of whether a statute can be used to establish a defendant's breach of the standard of care (i.e., negligence). D Incorrect. Use of a Rule of Law to establish negligence is a result of a judicial pronouncement and does not implicate use of a statute. This question asks you to identify the legally relevant inquiry when trying to use a statute's violation to establish a defendant's breach of the standard of care.

As Plaintiff is walking down the street, a chair falls out of Warehouse's window, striking Plaintiff on the head. Plaintiff seeks to sue Warehouse in negligence and wants to employ the doctrine of res ipsa loquitur to do so. When using the doctrine of res ipsa loquitur to prove a Warehouse's negligence, Plaintiff: A most likely has additional direct evidence of the defendant's negligence. B most likely has additional circumstantial evidence of the defendant's negligence. C must prove that the accident does not ordinarily occur in the absence of negligence. D must specifically identify what the defendant's negligent act entailed.

A Incorrect. The doctrine of res ipsa loquitur is employed when a Plaintiff has no other evidence of the defendant's breach of the standard of care. B Incorrect. The doctrine of res ipsa loquitur is employed when a Plaintiff has no other evidence of the defendant's breach of the standard of care. C Correct. Res ipsa loquitur requires proof that the accident is the type of accident that does not ordinarily occur in the absence of negligence. Restatement (Third) Torts § 17. D Incorrect. The doctrine of res ipsa loquitur is used when the plaintiff cannot identify the defendant's specific act of negligence.

Passenger was angry at taxi driver because passenger thought he was charged too much. They both began arguing outside the car at passenger's destination. In a fit of anger passenger attempted to kick taxi driver. Taxi driver blocked the blow by putting his tablet computer in the path of passenger's foot. Although taxi driver was unharmed his tablet computer was destroyed in the process. If taxi driver sues passenger for trespass to chattel what is the likely result? A Taxi driver will lose because passenger did not have the intent to interfere with his chattel. B Taxi driver will prevail even though passenger never intended to harm his chattel. C Taxi driver will lose because the chattel was destroyed and so the only appropriate cause of action is conversion. D Taxi driver will prevail because intent is not required for trespass to chattel or conversion.

A Incorrect. The doctrine of transferred intent applies to all the torts within the original writ of trespass. As a reminder these torts are Trespass to Chattel, Trespass to Land, False Imprisonment, Assault and Battery (TTFAB). The facts clearly indicate that the passenger attempted to commit a battery on the taxi driver. What happened however, was the destruction of the tablet computer. Because a tablet computer is personal as opposed to real property it is considered a chattel. As a result the intent from the battery attempt will transfer so that trespass to chattel will be satisfied. Trespass to chattel is intentionally interfering with the plaintiff's possession of a chattel in a way that causes recognizable harm. Based on that definition the only element of the tort which is unsatisfied is the intentional aspect. However the intent prong is satisfied because of transferred intent and as a result trespass to chattel is actionable. B Correct. The doctrine of transferred intent applies to all the torts within the original writ of trespass. As a reminder these torts are Trespass to Chattel, Trespass to Land, False Imprisonment, Assault and Battery (TTFAB). The facts clearly indicate that the passenger attempted to commit a battery on the taxi driver. What happened however, was the destruction of the tablet computer. Because a tablet computer is personal as opposed to real property it is considered a chattel. As a result the intent from the battery attempt will transfer so that trespass to chattel will be satisfied. Trespass to chattel is intentionally interfering with the plaintiff's possession of a chattel in a way that causes recognizable harm. Based on that definition the only element of the tort which is unsatisfied is the intentional aspect. However the intent prong is satisfied because of transferred intent and as a result trespass to chattel is actionable. C Incorrect. There is a very critical distinction between trespass to chattel and conversion. Transferred intent does not apply to conversion but it applies to trespass to chattel. Because there was no intent to harm the tablet here the intent element of conversion is missing and this tort cannot therefore be actionable under the facts. D Incorrect. Both trespass to chattel and conversion are intentional torts and therefore require intent.

Defendant, driving a car, approaches Plaintiff's car, and properly steps on the brakes. Defendant's brakes fail to function because she carelessly maintained the brakes. Defendant's car continues forward. In this emergency, Defendant chooses to step on the brakes again, and they fail again, causing Defendant's car to strike Plaintiff's car, injuring Plaintiff. Will Defendant likely be held liable for negligence? A Defendant will likely not be held liable for negligence because in light of her emergency circumstances she behaved as a reasonable prudent person. B Defendant will likely be held liable in negligence because she should have chosen better by turning the car quickly to the right. C Defendant will likely be held liable in negligence because her brakes failed because she unreasonably failed to properly maintain her brakes. D Defendant will likely not be held liable in negligence because she was reacting to an emergency situation.

A Incorrect. The emergency situation that Defendant was faced with was caused by her prior negligence. Her original negligence of not properly maintaining her brakes is therefore the cause of Plaintiff's injuries, and Defendant may be subject to liability. B Incorrect. This question asks about a person who finds herself in an emergency situation, which was created by her own negligence. Defendant has negligently failed to maintain her brakes. C Correct. This question asks about a person who finds herself in an emergency situation, which was created by her own negligence. Defendant has negligently failed to maintain her brakes. D Incorrect. This question asks about a person who finds herself in an emergency situation, which was created by her own negligence. Defendant has negligently failed to maintain her brakes.

Electric Company has strung power lines along the bank of a local river approximately 20 feet above the water's level. Recreational boaters, with masts sometimes exceeding the power lines by no more than one foot, often sail up and down the river. Polly is a guest on the boat of Friend, who is unfamiliar with this particular segment of the river. The height of the mast of Friend's sailboat reaches 21 feet above the water line. As the boat approaches the shore, Polly (who does not see the power line) is holding the mast when it comes into contact with the power line above. Due to the contact, Polly suffers severe electrical burns. In Polly's negligence action against Electric Company, the evidence shows that the likelihood of contact between sailboat masts and the power lines in this segment of the river is considerable, the severity of injuries when such contacts occur will probably be extreme, and the cost of raising the height of the power lines another 3-4 feet higher would be moderate. Can Electric Company be held liable in negligence? A Electric Company can be held liable in negligence to Polly for failing to raise the height of its power lines because Electric Company installed the power lines. B Electric Company can be held liable in negligence to Polly for failing to raise the height of its power lines because Electric Company's cost of raising the power lines 3-4 feet higher is moderate as weighed against the considerable likelihood of contact between a mast and the lines and the severity of probable potential injuries. C Electric Company cannot be held liable in negligence to Polly for her injuries because her injuries were caused by Friend's negligence, not Electric Company's negligence. D Electric Company cannot be held liable in negligence to Polly because it was Friend's sailboat, not Polly's sailboat.

A Incorrect. The fact that Electric Company installed the power lines is not enough to hold Electric Company liable in negligence. This question asks whether Electric Company failed to exercise reasonable care. Restatement (Third) Torts § 3 explains that the primary factors to consider in determining whether a party was negligent are the foreseeable likelihood that the person's conduct will result in harm, the foreseeable severity of any harm that may ensue, and the burden of precautions to eliminate or reduce the risk of harm. B Correct. The facts tell us that the cost of guarding against the harm was moderate as compared against the considerable likelihood of a severe injury. This question asks whether Electric Company failed to exercise reasonable care. Restatement (Third) Torts § 3 explains that the primary factors to consider in determining whether a party was negligent are the foreseeable likelihood that the person's conduct will result in harm, the foreseeable severity of any harm that may ensue, and the burden of precautions to eliminate or reduce the risk of harm. C Incorrect. There is no indication in the facts that Friend was negligent. Also, even if Friend were negligent, Friend's negligence would not preclude Electric Company from being held liable in negligence as well. D Incorrect. Electric Company's liability is not limited just to the owner of the sailboat. Polly is a foreseeable plaintiff and can hold Electric Company liable for its negligence without regard to whether Polly owned the sailboat.

Company provides a company-owned car to Harry for personal and business use. While Harry is on vacation, Harry is speeding while driving the car. He loses control, destroying a roadside billboard owned by Announcements, Inc. Announcements sues Company, claiming that Company negligently entrusted the car to Harry. Announcements claims that despite knowing of Harry's history of speeding, other moving violations, and a number of accidents, Company provided Harry with the car. Should Company be held liable in negligence for the destruction of Announcement's billboard? A Company should not be held liable to Announcements for Harry's destruction of their billboard because Harry was on vacation at the time of the accident. B Company should not be held liable in negligence because it was Harry's negligence that was the sole proximate cause of Announcement's injuries. C Company should not be held liable because Announcement has not suffered a harm that is compensable in negligence law. D Company should be held liable in negligence because one of the risks that made Company negligent was that Harry would drive poorly and cause an accident.

A Incorrect. The fact that Harry was on vacation has no bearing on whether Company's negligence resulted in injury to Announcements. Harry being on vacation might be relevant if this were a question of Company's vicarious liability. This question asks about the direct liability in negligence of an employer. B Incorrect. There may be more than one proximate cause of an injury. Holding Company liable does not preclude a finding that Harry is responsible as well. C Incorrect. Damage to property is compensable in negligence law. D Correct. The risk that made Company negligent was that Harry would drive poorly and cause an accident, thereby giving rise to Company's direct liability.

Soda Company, a company that provides beverages to local businesses, employs Delivery Person and has provided Delivery Person with detailed instructions on exactly how to conduct its business throughout the workday. Delivery Person is delivering beverages to SuperStore. Teenager is shopping at SuperStore when, while hauling sodas into SuperStore, Delivery Person carelessly runs over Teenager's foot, spraining and bruising it. In a negligence action brought against Soda Company on Teenager's behalf for the injuries sustained when Delivery Person ran over his foot, may Soda Company be held vicariously liable? A Soda Company may not be held vicariously liable for Delivery Person's negligence because Delivery Person was an independent contractor at the time of the accident. B Correct. An employer may be held vicariously liable in negligence for its employee's tortious conduct. C Incorrect. Soda Company can be liable for Delivery Person's negligence as long as it took place within the scope of employment, even if he was told to do otherwise. This question asks about an employer's vicarious responsibility for the tortious behavior of its employees. D Soda Company may be held vicariously liable for Delivery Person's negligence if Delivery Person was on a frolic.

A Incorrect. The facts suggest that Delivery Person was an employee, not an independent contractor. An independent contractor has the liberty of controlling the details of his or her own work. The facts tell us that Soda Company has provided for specific details about Delivery Person's workday procedures. B Correct. An employer may be held vicariously liable in negligence for its employee's tortious conduct. C Incorrect. Soda Company can be liable for Delivery Person's negligence as long as it took place within the scope of employment, even if he was told to do otherwise. This question asks about an employer's vicarious responsibility for the tortious behavior of its employees. D Incorrect. An employer is not liable for the torts of its employees while the employees are on a frolic. This question asks about an employer's vicarious responsibility for the tortious behavior of its employees.

Patient gives consent to be treated by surgeon for a minor surgery on plaintiff's left elbow. While plaintiff is unconscious from the anesthetic and during the course of the surgery on the left elbow the surgeon decides to examine the patient's entire body. Surgeon finds nothing else wrong with patient. Patient sues the doctor in battery. What is the best defense of the doctor? A A patient who consents to surgical treatment on one part of their body automatically consents to examination and treatment of their entire body. B Because of superior knowledge a treating physician may examine and touch any part of an unconscious patient's body. C During the surgery, the doctor realizes the patient suffered from a condition that typically rapidly spreads to all parts of the body and only a careful and painful examination of the patient's entire body would reveal whether or not other parts of the body were affected needing immediate treatment. D The doctor felt that it would save time to examine the patient at this instant rather than schedule a separate appointment.

A Incorrect. The human body has some of the highest privacy standards. Absent an acute or emergent conditions a doctor must ask consent to examine parts of the body other than the part the patient gave consent to be examined. B Incorrect. The human body has some of the highest privacy standards. Absent an acute or emergent conditions a doctor must ask consent to examine parts of the body other than the part the patient gave consent to be examined. The unconscious patient has no less rights than the conscious patient. C Correct. If during surgery the doctor realizes that the patient suffers from a condition that rapidly spreads to other parts of the body and if the patient needed immediate treatment to prevent spread of the disease or the doctor discovers the need for immediate treatment during surgery then the doctor would be acting reasonably if she treated the patient's condition. The doctor would not be committing a battery if it was reasonable to assume consent for treatment would have been given if the patient were conscious and had the opportunity to consent. D Incorrect. Unless during the surgery when the patient is unconscious the doctor realizes that an emergent condition requiring immediate and further treatment to areas other than the doctor obtained consent to treat then the doctor must wait and seek consent for further treatment from the awake patient.

Carl negligently injures himself in an automobile accident. Carl seeks medical treatment from Doctor, who negligently aggravates Carl's injury. In a suit in which Carl seeks to recover from Doctor for the part of Carl's injuries caused by Doctor's medical malpractice, will Carl be able to recover from Doctor? A Carl cannot recover from Doctor because Carl's negligence caused the automobile accident in which he was initially injured. B Carl may hold Doctor liable for all of his injuries, including the injuries he sustained in the accident Carl caused. C Carl may hold Doctor liable only if Doctor intended to cause Carl harm in providing him with medical care. D Carl may recover for Doctor's negligence for the part of Carl's injury attributable to Doctor's negligence notwithstanding the fact that Carl's negligence produced the very condition Doctor undertook to treat.

A Incorrect. The plaintiff's negligence is not a bar to recovery when the defendant undertakes to treat the plaintiff for an injury caused by plaintiff's own negligence. This question asks about the liability of a defendant who undertakes to treat a condition caused by the plaintiff's negligence. In such a case, the factfinder does not consider the plaintiff's negligence in creating the condition that the defendant was employed to remedy. See Restatement (Third) Torts § 7 and illustration 8, upon which this fact pattern was based. B Incorrect. Doctor is only responsible for the aggravation of Carl's injury that is attributable to Doctor's negligence. This question asks about the liability of a defendant who undertakes to treat a condition caused by the plaintiff's negligence. In such a case, the factfinder does not consider the plaintiff's negligence in creating the condition that the defendant was employed to remedy. See Restatement (Third) Torts § 7 and illustration 8, upon which this fact pattern was based. C Incorrect. Doctor can be liable for negligently caused injuries; his liability is not dependent upon him having intended to cause injury. This question asks about the liability of a defendant who undertakes to treat a condition caused by the plaintiff's negligence. In such a case, the factfinder does not consider the plaintiff's negligence in creating the condition that the defendant was employed to remedy. See Restatement (Third) Torts § 7 and illustration 8, upon which this fact pattern was based. D Correct. This question asks about the liability of a defendant who undertakes to treat a condition caused by the plaintiff's negligence. In such a case, the factfinder does not consider the plaintiff's negligence in creating the condition that the defendant was employed to remedy. See Restatement (Third) Torts § 7 and illustration 8, upon which this fact pattern was based.

Psychotherapist is counseling adult patient as patient is going through a divorce. After three or four sessions patient expresses a romantic interest in psychotherapist. Psychotherapist asks patient if patient is sure about how he feels. Patient responds that he is sure and psychotherapist and patient engage in a sexual relationship. Does the patient have a potential breach of fiduciary duty case against the psychotherapist? A No, because the patient is adult and consented. B No, because the psychotherapist asked the patient if he was sure about his decision. C Yes, because patients of psychotherapists are incapable of giving consent. D Yes, because psychotherapists are in a position of power over their patients and as a result consent of the patient for a sexual relationship is not valid.

A Incorrect. The position of power and mental control psychotherapists usually have over their patients the consent of the patient to a sexual relationship is not a bar to the breach of fiduciary action based on the sexual relationship. B Incorrect. Asking the patient if he was sure is not dispositive given the difference in power of the patient and the psychotherapist. C Incorrect. It is an overbroad statement. Patients of psychotherapists are capable of giving consent to many things. However, included in those things is not the consent for a sexual relationship with the psychotherapist because of the level of influence the psychotherapist is assumed to have over the patient. D Correct. Psychotherapists are deemed to have such a huge influence on the mental state of their patients that any consent by the patient for sexual relationship with the psychotherapist is deemed invalid.

Parker parks his car at the top of a driveway, which is on an incline. Two minutes later, the car rolls down the incline and injures Pedestrian who was on the sidewalk. In suing Parker, Pedestrian seeks to rely on res ipsa loquitur in order to prove Parker's negligence. Which of the following statements about res ipsa loquitur is correct? A Relying on res ipsa loquitur in these circumstances would be improper if there are other possible explanations for why the car rolled. B If able to employ the doctrine of res ipsa loquitur, the jury is required to infer that Parker was negligent. C If permitted to employ the doctrine of res ipsa loquitur, the jury may not infer that Parker was negligent. D If permitted to employ the doctrine of res ipsa loquitur, the jury may infer that Parker was negligent.

A Incorrect. The possibility of alternative explanations for the accident do not preclude use of the doctrine of res ipsa loquitur. B Incorrect. The fact finder is not required to find that Parker was negligent. If allowed to use the doctrine of res ipsa loquitur, the jury is permitted but is not required to infer that the defendant was negligent. C Incorrect. It states the opposite of the effect of using res ipsa loquitur. If allowed to use the doctrine of res ipsa loquitur, the jury is permitted but is not required to infer that the defendant was negligent. D Correct. This question asks about the use of res ipsa loquitur to establish a defendant's negligence. If allowed to use the doctrine of res ipsa loquitur, the jury is permitted but is not required to infer that the defendant was negligent. See Restatement (Third) Torts § 17 and illustration 2, upon which this question was based.

Private boater is out on the water for a leisure day when the weather radio announces the development of a sudden but strong storm in the immediate vicinity of the boater. The weather radio says that all boats in the area must seek immediate cover and be fastened to a dock if possible. Hearing the warning, private boater ties his boat up to the nearest dock. The dock is owned by landowner who sees the boat tie up and decides that he does not want the boat tied to HER dock. Landowner unties private boat from his dock just as the storm approaches and private boat drifts off into the storm colliding with other boats causing damage to other boats. Owners of other boats sue private landowner for damages caused by private boat he refused to let stay on his dock. Is landowner liable for damage? A No, landowner is not liable for damage because she has the absolute right to exclude others from his property. B Yes, landowner is liable only if she acted negligently. C No, landowner is not liable because private boater did not ask permission to use the dock. D Yes, landowner is liable because during private necessity the landowner's right to exclude people from his property yields to the person claiming necessity's right to use the property.

A Incorrect. The privilege of private necessity operates to suspend the landowner's right to exclude others claiming necessity from her land. B Incorrect. There is no need for negligence on the part of the landowner for liability. If the landowner denies use of property during private necessity and that denial causes harm then the landowner is liable for that harm. C Incorrect. An element of the private necessity privilege is that a serious n imminent threat exist. Given the emergent nature of private necessity cases it would be nonsensical to make a condition precedent to claiming the privilege obtaining permission from the landowner. D Correct. In times of private necessity the landowner has no right to exclude a person operating under the privilege from access to her property.

Farmer is a country farmer who has a lower IQ than most individuals. However, he is also unusually physically fit, much physically bigger and stronger than the average person. Lately, as part of his trade as a farmer, he has been stacking hay on his property. However, he has been doing so in a way that a farmer of ordinary intelligence would know will cause the hay to combust spontaneously into flames. Defendant's hay does in fact combust causing a fire, which spreads to Neighbor's property causing property damage. In the negligence action brought by Neighbor against Farmer, how should the court instruct the jury regarding the reasonable prudent person standard of care? A The reasonable prudent person standard will take into account that Farmer has a lower IQ than most individuals. B The reasonable prudent person standard will not take into account that Farmer was physically bigger and stronger than the average person. C The reasonable prudent person standard will not take into account that Farmer has a lower IQ than most individuals. D The reasonable prudent person standard is subjective and based on Farmer's qualities.

A Incorrect. The reasonable prudent person standard typically does not take into account the mental incapacities of the defendant. B Incorrect. The standard does typically take into account the physical incapacities of the actor. C Correct. The reasonable prudent person standard does not take into account mental incapacities of the defendant. D Incorrect. The standard is an objective one, not subjective. This question asks about the qualities and characteristics incorporated by the reasonable prudent person standard in negligence actions. The standard is always an objective standard.

Practical joker familiar to shopper pulls out a small feather from his pocket and while laughing tells shopper, "I am going to tickle you with this feather now," and approaches. Shopper has told practical joker before not to do this exact same thing. Practical joker approaches shopper with feather slowly, in an unthreatening manner and begins to lightly tickle shopper on shopper's arm. Shopper warns practical joker to stop. When practical joker refuses to stop shopper shoots practical joker in the leg with a powerful handgun causing permanent injury. Assuming the tickling would be considered a battery, which of the following statements is correct? A Because shopper warned practical joker first and practical joker did not heed the warning shopper is entitled to use any force to stop the battery. B Shopper exceeded the scope of the privilege of self-defense. C Because practical joker's actions constitute a battery shopper was privileged to use any force to defend himself. D Practical joker as the instigator of the altercation is barred from recovering damages from shopper for injuries to practical joker's leg.

A Incorrect. The rule in tort law is that the use of force to defend oneself is limited to the force reasonably necessary in the situation. The issuance of a warning does not negate the "only reasonable force rule." The amount of force reasonable in self-defense is directly related to the magnitude of the threat. The use of deadly force or force likely to cause grievous bodily harm is only permitted when the person asserting the privilege of self-defense reasonably believes they are being attacked with similar force. Here the tickling of an arm with a feather is not the equivalent of deadly force and so deadly force in self-defense is impermissible. B Correct. The rule in tort law is that the use of force to defend oneself is limited to the force reasonably necessary in the situation. The amount of force reasonable in self-defense is directly related to the magnitude of the threat. The use of deadly force or force likely to cause grievous bodily harm is only permitted when the person asserting the privilege of self-defense reasonably believes they are being attacked with similar force. Here the tickling of an arm with a feather is not the equivalent of deadly force and so deadly force in self-defense is impermissible. The self-defense privilege is not a privilege which allows unlimited use of force in any situation. Rather the scope of the force permitted in exercising the privilege is directly related to the magnitude of the force used by the attacker. Shooting someone in the leg is beyond the scope of permissible force levels for being tickled on the arm. C Incorrect. The amount of force reasonable in self-defense is directly related to the magnitude of the threat. The use of deadly force or force likely to cause grievous bodily harm is only permitted when the person asserting the privilege of self-defense reasonably believes they are being attacked with similar force. Here the tickling of an arm with a feather is not the equivalent of deadly force and so deadly force in self-defense is impermissible. The self-defense privilege is not a privilege which allows unlimited use of force in any situation. Rather the scope of the force permitted in exercising the privilege is directly related to the magnitude of the force used by the attacker. Shooting someone in the leg is beyond the scope of permissible force levels for being tickled on the arm. The mere fact that the tickling constituted a battery does not permit the defender to automatically use deadly force. Remember a battery may be established by "harmful OR offensive contact." Here the tickling was merely offensive and so the permissible use of force clearly did not include shooting someone in the leg. D Incorrect. Where a defender exceeds the scope of the privilege by using excessive force to defend himself against a perceived threat, the defender may be liable to the attacker for any harm caused by the excessive use of force. For example if the shopper initially swatted the practical joker's hand away that might be considered an acceptable use of force and the shopper would not be liable for battery. However in this case the shopper will likely be liable for the harm to the practical joker's leg caused by the shooting which would likely be considered an excessive use of force given the situation.

Intoxicated bar patron fell asleep in a booth at the bar. Security guard lifted the patron up over his shoulder and took him out to a shed behind the bar. He dropped him on the floor of the shed, breaking the patron's hand in the process. The security guard locked the door and if the patron had woken up he would not have been able to leave the shed. Patron was so inebriated that he did not wake up when this happened. About 4 hours later the security guard lifted the still sleeping patron out of the shed and deposited him on a bus bench. Patron wakes up with a throbbing and swollen hand and asks the guard about it. The guard told the patron what happened. In a false imprisonment suit by patron against security guard, will the patron recover? A The patron will not recover because patron was unaware of the confinement as it was happening. B Correct. The rule of false imprisonment cases is that in order to recover the victim must either have been aware of the confinement or suffer actual harm during the confinement. In this case, although patron was unaware of the confinement because he was asleep, he suffered actual harm when his hand was broken. In fact, the emerging rule is that it is only in the cases where the victim suffers only dignitary and not actual harm is the awareness of confinement a necessary element of the tort. Here, patron suffered actual harm (broken hand) and, as a result, recovery is permissible even though he was unaware of the confinement. C The patron will not recover if the security guard's motive for imprisoning the patron in the shed was to protect the patron. D The patron will recover only if the security guard was aware that he did not have the authority or right to imprison the patron.

A Incorrect. The rule of false imprisonment cases is that in order to recover the victim must either have been aware of the confinement or suffer actual harm during the confinement. In this case, although patron was unaware of the confinement because he was asleep, he suffered actual harm when his hand was broken. B Correct. The rule of false imprisonment cases is that in order to recover the victim must either have been aware of the confinement or suffer actual harm during the confinement. In this case, although patron was unaware of the confinement because he was asleep, he suffered actual harm when his hand was broken. In fact, the emerging rule is that it is only in the cases where the victim suffers only dignitary and not actual harm is the awareness of confinement a necessary element of the tort. Here, patron suffered actual harm (broken hand) and, as a result, recovery is permissible even though he was unaware of the confinement. C Incorrect. It is a reminder that motive is irrelevant to intent. A good motive does not absolve the tortfeasor of otherwise tortious conduct. D Incorrect. It is a reminder that motive is irrelevant to intent. A good motive does not absolve the tortfeasor of otherwise tortious conduct. This answer choice incorrectly hinges liability on the security guard's awareness that his conduct is impermissible. This is not a requirement for liability in intentional tort. On some basic level this makes sense because if no one could be liable if they personally were unaware they were breaking the law that would be too subjective a standard for the imposition of liability.

Jill loans her car for the evening to Friend who needs the car for social purposes. Jill knows that Friend's driver's license was suspended a month previously on account of repeated instances of reckless driving. In the course of the evening, Friend drives the car negligently and injures Pedestrian. In a cause of action against Jill brought by Pedestrian for recovery of her injuries, can Jill be held liable for negligence? A Jill cannot be held liable in negligence because Friend was driving the car and he was therefore the sole proximate cause for Pedestrian's injuries. B Jill cannot be held liable in negligence because an adult cannot be held liable for the tortious conduct of another adult. C Jill may be held liable to Pedestrian in negligence because car owners are responsible for injuries caused by their vehicles. D Jill may be held liable to Pedestrian in negligence because the reasonable prudent person would likely not have lent her car to Friend, knowing that Friend's license was suspended for repeated instances of reckless driving.

A Incorrect. There can be more than one proximate cause of a plaintiff's injury. Friend's negligence does not preclude a finding that Jill was negligent as well. B Incorrect. There can be more than one proximate cause of a plaintiff's injury. Even though Friend is an adult, Friend's negligence does not preclude a finding that Jill was negligent as well. C Incorrect. This question asks about negligent entrustment. Car owners are not necessarily liable for every injury caused by their vehicles. D Correct. This question asks about negligent entrustment. See Restatement (Third) Torts § 19 and illustration 1 upon which this fact pattern is based.

Mom is the mother of Toddler, who is almost two years old. Mom and Toddler are visiting Friend's quaint rustic cabin. One morning when Friend has gone to run errands, Mom and Toddler are in the kitchen, a lovely room lit by a kerosene lamp sitting on a kitchen table. When Mom leaves the kitchen for an hour to read a book, Toddler knocks over the lantern, starting a fire that damages Friend's cabin. In a negligence action brought by Friend against Mom, is Mom likely to be held liable in negligence? A Mom is not likely to be held liable to Friend in negligence because Toddler started the fire, not Mom. B Mom is not likely to be held liable in negligence to Friend because Friend assumed the risk by his ownership of the cabin. C Mom is likely to be held liable in negligence because parents are vicariously responsible for the damage caused by their children. D Mom is likely to be held liable in negligence to Friend because Mom failed to act as the reasonable prudent person when she left Toddler alone for an hour next to a kerosene lamp.

A Incorrect. There often is not just one factual cause of an injury, and this fact pattern is an example. Yes, Toddler started the fire by knocking over the lamp, but Mom is a factual cause of the fire as well: but for Mom leaving her small child unsupervised, Toddler would not have knocked over the lamp. Therefore, Mom is a cause of the fire as well, even though Toddler started the fire. B Incorrect. There are no facts to suggest that Friend assumed the risk of this fire. Assumption of the risk requires voluntarily encountering a known, specific risk and appreciating the magnitude of the potential harm that may result. Ownership of the property that is damaged does not amount to assumption of the risk. C Incorrect. It is incorrect to say that parents are always responsible for the damage caused by their children. This question is not asking whether the parent should be vicariously liable for the damages caused by their children, but rather whether this parent can be subject to liability for her own negligence which resulted in her child causing property damage to another. D Correct. It is unreasonable to leave a small child alone next to a kerosene lamp for an hour merely to go read a book, and the resultant fire was reasonably foreseeable from Mom having done so.

Mom is the mother of Toddler, who is almost two years old. Mom and Toddler are visiting Friend's quaint rustic cabin. One morning when Friend has gone to run errands, Mom and Toddler are in the kitchen, a lovely room lit by a kerosene lamp sitting on a kitchen table. Toddler knocks the lantern over during a less than 10-second period during which Mom has turned her back in order to take a boiling pot off the stove. The knocked over lantern starts a fire that damages Friend's cabin. In a negligence action brought by Friend against Mom, is Mom likely to be held liable in negligence? A Mom is likely not to be held liable in negligence because Toddler caused the fire, not her. B Mom is likely liable for negligence if she failed to act as the reasonable prudent person when she left Toddler alone next to a kerosene lamp. C Mom is likely not liable in negligence if turning her back for that brief period of time was reasonable. D Mom is likely liable in negligence because parents are vicariously responsible for the damage caused by their children.

A Incorrect. There often is not just one factual cause of an injury, and this fact pattern is an example. Yes, Toddler started the fire by knocking over the lamp, but Mom is a factual cause of the fire as well: but for Mom leaving her small child unsupervised, Toddler would not have knocked over the lamp. Therefore, Mom is a cause of the fire as well, even though Toddler was the most immediate start of the fire. B Incorrect. This question asks whether Mom's conduct was reasonable when she takes her eyes off of her small child for a very brief period of time to do something important (and something that could be dangerous if not taken care of immediately). This question is included to show that the determination of reasonable conduct varies, depending on the facts. C Correct. Mom will not be liable in negligence absent a showing that her conduct was unreasonable and her unreasonable conduct caused the fire. Because she took her eyes off her child for such a short period of time and did so to eliminate a dangerous situation, Mom's conduct may be reasonable under the circumstances, making C correct. B is an attractive option, but C is the better answer. D Incorrect. It is a misstatement of law; it is untrue that parents are always responsible in negligence for the damage caused by their children.

After drinking himself silly at a bar, drunk decides to walk home. Drunk sees a car and decides that it would be fun to scratch the car with his keys. Drunk scratches the car repeatedly, destroying the paint job in the process. The next morning drunk has no recollection of the incident and when he is sued by the car owner for trespass to chattel drunk defends by saying the following, "Hey I did not even know what I was doing, if I was sober I would never have done that." Is drunk's statement a valid defense? A Yes. If drunk cannot remember what he was doing to the car he cannot be liable. B Yes. If drunk was not really in control of himself because of the alcohol he was not acting intentionally. C No. Any harm caused by an intentional tort is compensable in damages. D No. Voluntary intoxication is not a valid defense to intentional tort.

A Incorrect. There simply is no defense for not remembering an intentional act which caused harm. B Incorrect. The fact that drunk was not in control of himself due to inebriation does not render his acts involuntary or unintentional. Voluntary inebriation is not a defense to intentional tort. C Incorrect. It is an incorrect statement of law. The previous questions demonstrate that there are many defenses which negate liability for acts that would otherwise result in liability. Public necessity, self-defense etc. D Correct. Voluntary intoxication is not a defense to intentional tort.

Company hires Janitor to provide maintenance and janitorial services for its apartment building. When it hires Janitor, Company knows that Janitor has a record of inappropriate sexually aggressive conduct toward women. In his role as janitor, Janitor has frequent interactions with Tenant, one of the building's occupants. One evening, after his workday is over, Janitor knocks on Tenant's door. Being familiar with him because he is the janitor at apartment complex, Tenant lets him in, whereupon Janitor rapes her. Tenant sues Company in negligence, seeking to hold it liable for her injuries. Can Company be held liable for negligence? A Company may not be held liable in negligence because Janitor's rape of Tenant was an intervening criminal act. B Company may be held liable in negligence in hiring Janitor for a job in which Janitor would have access to female tenants within their apartments. C Company may not be held liable in negligence because Janitor's assault of Tenant was outside the scope of his employment. D Company may be held liable in negligence because employers are liable in negligence for the harm caused by their employees.

A Incorrect. This question asks about an employer's direct negligence in hiring a foreseeably dangerous employee. Although intervening criminal acts may often cut off an earlier act of negligence, when the harm caused is the very risk that makes the initial act unreasonable, the initial actor may be subject to liability. Here, Company is negligent for hiring someone who presents a foreseeable risk of harm to female tenants. The intervening criminal act was foreseeable to Company by virtue of its negligence. B Correct. Janitor was a foreseeably dangerous employee, for which Company may be held directly liable. C Incorrect. It addresses whether the employer can be vicariously liable rather than directly liable for its own negligence. This question asks about an employer's direct negligence in hiring a foreseeably dangerous employee. D Incorrect. It is too broad of a statement to say that employers are always liable for harm caused by their employees.

About two years ago, Client was a patient of Ophthalmologist. Client alleges that during the time that he was Ophthalmologist's patient, Ophthalmologist improperly performed LASIK surgery on Client's eyes. The LASIK surgery was supposed to correct Client's poor vision. Instead, the surgery was performed skillfully, but resulted in blindness in both eyes, a risk about which Client was never informed and a reasonable person would not have known. If Client had known of the risk of blindness, he would not have opted for the surgery. Client hires Attorney Matthew to sue Ophthalmologist for medical malpractice. Regarding the underlying medical malpractice action, which of the following statements concerning the Ophthalmologist's liability is correct? A Ophthalmologist's failure to inform Client of the material risks and available alternatives to LASIK treatment does not constitute a breach of the standard of care in negligence because every patient should know that any eye surgery could lead to blindness. B Ophthalmologist's failure to inform Client of the material risks and available alternatives to LASIK treatment does not constitute a breach of the standard of care in negligence because the reasonable patient should know that any eye surgery could result in blindness. C Ophthalmologist cannot be subject to liability in negligence because he performed the surgery skillfully. D Ophthalmologist's failure to inform Client of the material risks and available alternatives to LASIK treatment is a breach of the standard of care in negligence.

A Incorrect. This question asks about the informed consent doctrine in negligence law. A physician can be subject to liability in negligence if he fails to inform his patient of the material risks and alternative treatments to the patient's treatment protocol. Failure to inform constitutes a breach of the standard of care in negligence. B Incorrect. It is not likely that the reasonable patient should know that any eye surgery may result in blindness. This question asks about the informed consent doctrine in negligence law. A physician can be subject to liability in negligence if he fails to inform his patient of the material risks and alternative treatments to the patient's treatment protocol. Failure to inform constitutes a breach of the standard of care in negligence. C Incorrect. A negligence action based on lack of informed consent may be available without regard to whether the doctor performed the procedure skillfully. D Correct. This question asks about the informed consent doctrine in negligence law. A physician can be subject to liability in negligence if he fails to inform his patient of the material risks and alternative treatments to the patient's treatment protocol. Failure to inform constitutes a breach of the standard of care in negligence.

Susie Sickley, a single mother of three young boys, has been ill for several weeks, and much to Susie's delight, her neighbor Mary has volunteered to bring by dinner for Susie and her family and to tidy up their home. When Mary arrives at Susie's home, she finds the place a mess. After tending to Susie upstairs and tidying up the boys' bedrooms, Mary realizes that she has to use the restroom very badly. For as long as Susie has lived in her home, Susie has known that the toilet seat in one of the upstairs bathrooms is not attached very securely. Susie has not yet made the effort to get the toilet seat fixed because the boys are typically the only ones who use that particular bathroom, and when they do so, they never complain about the seat being loose. Mary enters the nearest bathroom (the one with the loose toilet seat) and finds the toilet seat in a disgusting condition. Seeing only a couple of squares of toilet paper, no cleaning supplies in sight, and in dire need of relieving herself, Mary decides to kick off her shoes and stand on the toilet seat in order to empty her bladder. Soon after she manages to perch herself atop the toilet seat, the seat unexpectedly breaks loose, and Mary falls, breaking her hip and an arm. In a negligence action brought by Mary against Susie for the injuries she sustained in the upstairs bathroom, what duty, if any, did Susie owe Mary? A Susie owed Mary the duty to warn about the broken toilet seat because Mary was an invitee. B Susie owed Mary the duty to warn about the broken toilet seat because Mary was a licensee. C Susie did not owe Mary a duty of care because Mary was a trespasser. D Susie did not owe Mary a duty of care because Mary was contributorily negligent.

A Incorrect. This question asks about the standard of care in negligence that an occupier of land owes to visitors on the premises. Although the Restatement (Third) of Torts § 51 takes an integrated approach of reasonable care of owners and occupiers of land, this question is based on those jurisdictions that still take a more traditional approach regarding the duty of care owed to one injured on a defendant's premises. Historically, the duty of care owed varies according to the legal status of the plaintiff at the time of the injury. A social visitor, as Mary, is typically classified as a licensee, and a landowner owes a licensee the duty to warn of hidden dangers about which the landowner is aware. An invitee is a person on the land owner's property for the land owner's business. Because Susie is not properly characterized as an invitee, A is incorrect. B Correct. This question asks about the standard of care in negligence that an occupier of land owes to visitors on the premises. Although the Restatement (Third) of Torts § 51 takes an integrated approach of reasonable care of owners and occupiers of land, this question is based on those jurisdictions that still take a more traditional approach regarding the duty of care owed to one injured on a defendant's premises. Historically, the duty of care owed varies according to the legal status of the plaintiff at the time of the injury. An invitee is a person on the land owner's property for the land owner's business. A social visitor, as Mary, is typically classified as a licensee, and a landowner owes a licensee the duty to warn of hidden dangers about which the landowner is aware. C Incorrect. This answer is incorrect for at least two reasons: (1) Mary is likely not properly characterized as a trespasser and (2) even if she is a trespasser, she is owed a duty not to wantonly or willing injure her rather than owing no duty of care at all as this answer choice suggests. D Incorrect. A landowner still owes a duty of care to licensees, even if the licensee is negligent.

A state administrative regulation prohibits railroad trains from blocking highway crossings for more than 10 minutes. Railway allows one of its trains to remain in a highway crossing for 30 minutes. Fifteen minutes into this 30-minute period, Driver, who is driving his car on the highway, fails to notice the train until it is too late and collides with it, suffering an injury. Driver sues Railway in negligence. At trial, Driver concedes that his negligence should reduce his recovery under comparative negligence principles, but claims that Railway is negligent per se for violating the regulation. The evidence at trial reveals that the history of the regulation and of the agency's findings accompanying the regulation show that the only purpose of the regulation is to encourage the free flow of traffic and prevent traffic delays. What effect, if any, does Railroad's violation of the regulation have on Railroad's liability for negligence? A Driver should be able to use evidence of the regulation's violation to establish that Railway's negligence because it is a state administrative regulation applicable to railroad trains. B Driver should not be permitted to use the regulation's violation to establish Railway's negligence because the prevention of personal injuries is not part of the regulation's purpose. C Driver may not use the regulation's violation as proof of Railway's negligence per se because Driver has conceded his own negligence. D Driver should not be permitted to use evidence of the regulation's violation to establish that Railway was negligent because administrative regulations may not be used to establish the standard of care in a negligence action.

A Incorrect. This question asks about the use of a regulation to establish the standard of care in negligence. A statute may be used to establish the standard of care in a negligence action when the statute was designed to protect against the type of accident about which the plaintiff is complaining and the plaintiff is within the class of persons the statute is designed to protect. Because the regulation was not enacted to prevent a driver from colliding with a train, its violation cannot be used to establish the standard of care in this action. B Correct. The facts explain that the regulation's only purpose was to encourage the free flow of traffic and prevent traffic delays. Therefore, it was not enacted to prevent the type of harm about which Driver is complaining, a car colliding with a train. C Incorrect. The fact that Driver has conceded his own negligence does not impact whether the Driver can use the regulation to establish Railroad's negligence. This question asks about the use of a regulation to establish the standard of care in negligence. A statute may be used to establish the standard of care in a negligence action when the statute was designed to protect against the type of accident about which the plaintiff is complaining and the plaintiff is within the class of persons the statute is designed to protect. D Incorrect. Statutes, regulations, and ordinances may be used to establish the standard of care in a negligence action where appropriate. A statute may be used to establish the standard of care in a negligence action when the statute was designed to protect against the type of accident about which the plaintiff is complaining and the plaintiff is within the class of persons the statute is designed to protect.

A state statute requires that all slow-moving vehicles drive as far to the right on the highway as possible. The statute's primary purpose is to minimize the safety hazards posed by vehicles whose slow speed can interfere with the traffic flow. Driver is driving slowly on a highway heading north and is violating the statute by driving in the fast lane on the left. Passenger is riding along in her car. Unexpectedly, a car heading south on the highway crosses the median line striking Driver's car and injuring Passenger. Had Driver been in the lane on the far right, her car would have avoided contact with the other car. Passenger sues Driver, seeking to establish her negligence by proving her violation of the statute. What effect, if any, may proof of Driver's violation of the statute have in a negligence action brought by Passenger? A Driver's violation of the statute is negligence per se because the purpose of the statute is to protect highway safety. B Driver's violation of the statute may be used to establish the standard of care because any statute may be used to establish the standard of care in a negligence action. C Passenger will likely not be able to use the statute to prove Driver's negligence because the statute was designed to prevent accidents between cars moving in the same direction, not to prevent accidents between cars moving in opposite directions. D Passenger will likely be able to use the statute to establish the standard of care in negligence because if she had not been violating the statute, Passenger would not have been injured.

A Incorrect. This response mischaracterizes the facts provided. The statute's purpose was to minimize safety hazards posed by vehicles whose slow speed interferes with traffic flow, not to prevent personal injury. B Incorrect. It is not true that any statute may be used to establish the standard of care in a negligence action. A statute may be used to establish the standard of care in a negligence action when the statute was designed to protect against the type of accident about which the plaintiff is complaining and the plaintiff is within the class of persons the statute is designed to protect. C Correct. The statute was not designed to prevent the type of accident that occurred in this case and therefore cannot be used to establish the standard of care. Restatement (Third) Torts § 14. D Incorrect. Even though the violation of the statute is causally linked to the plaintiff's harm, the plaintiff still must show that the violation of statute is appropriate to use to establish the standard of care in negligence. Causation alone is insufficient to impose liability in negligence.

Tonya lay in wait for Michael. She hated him and her hate made her certifiably insane and this time she was going to get him. As Michael cruised along the highway Tonya hid in the bushes at the side of the road and planned to ram a huge piece of metal into the rear wheel of Michael's motor bike. The plan was a sudden stop, causing Michael to fly over the front of the bike and to get severely cut as he dragged along the road. As Tonya rammed the wood into the wheel Michael screamed in terror and then flew over the bike and got severely injured from his contact with the road and suffered a severely disabling emotional response. His bike was also completely destroyed in the process. Tonya did not intend to scare him or to cause severe emotional harm or to wreck his bike. What causes of action does Michael likely have? A Michael likely has a cause of action for battery only. B Michael likely has a cause of action for assault and battery. C Incorrect. Tonya clearly intended to cause a battery, but also ended up scaring Michael as evidenced by his "scream in terror." Therefore because of the doctrine of transferred intent Tonya's intent to commit battery will also result in an assault because fear or apprehension of harmful or offensive contact also resulted. So assault and battery are likely actionable. However, IIED is not one of the torts to or from which intent transfers. Because Tonya did not have the independent intent to cause a severely disabling emotional response there can be no viable cause of action for IIED. D Michael likely has a cause of action for battery, assault, intentional infliction of emotional distress and conversion.

A Incorrect. Tonya clearly intended to cause a battery, but also ended up scaring Michael as evidenced by his "scream in terror." Therefore because of the doctrine of transferred intent Tonya's intent to commit battery will also result in an assault because fear or apprehension of harmful or offensive contact also resulted. B Correct. Tonya clearly intended to cause a battery, but also ended up scaring Michael as evidenced by his "scream in terror." Therefore because of the doctrine of transferred intent Tonya's intent to commit battery will also result in an assault because fear or apprehension of harmful or offensive contact also resulted. C Incorrect. Tonya clearly intended to cause a battery, but also ended up scaring Michael as evidenced by his "scream in terror." Therefore because of the doctrine of transferred intent Tonya's intent to commit battery will also result in an assault because fear or apprehension of harmful or offensive contact also resulted. So assault and battery are likely actionable. However, IIED is not one of the torts to or from which intent transfers. Because Tonya did not have the independent intent to cause a severely disabling emotional response there can be no viable cause of action for IIED. D Incorrect. Tonya clearly intended to cause a battery, but also ended up scaring Michael as evidenced by his "scream in terror." Therefore because of the doctrine of transferred intent Tonya's intent to commit battery will also result in an assault because fear or apprehension of harmful or offensive contact also resulted. So assault and battery are likely actionable. However, IIED is not one of the torts to or from which intent transfers. Because Tonya did not have the independent intent to cause a severely disabling emotional response there can be no viable cause of action for IIED. Similarly conversion is not one of the torts to or from which intent transfers. As a result there can be no viable claim for conversion based on these facts.

Customer left her coat on a coat rack at the entrance to a restaurant. Thief decided to steal the coat and keep it for himself. After 5 months the thief had a change of heart and figured out where customer lived. Thief returned the coat to customer's address unharmed. Does Customer have an action for trespass to chattel or conversion against Thief? A Customer has both an action for conversion and for trespass to chattel against thief. B Customer only has an action for trespass to chattel against thief but not an action for conversion. C Customer only has an action for conversion against thief but not trespass to chattel. D Customer has neither an action for trespass to chattel nor conversion against thief.

A Incorrect. Trespass to chattel is intentionally interfering with the plaintiff's possession in a way that causes recognizable harm. Conversion is intentionally exercising dominion or control over another's chattel in a way that in inconsistent with the other's rights and deprives the other of the chattel. While the length of time of the deprivation is relevant in determining whether interference with a chattel is a trespass to chattel or if it is substantial enough to be conversion 5 months would typically be enough to be considered dominion or control for conversion. One clear distinction between conversion and trespass to chattels is that the former is actionable even if the chattel is unharmed but the latter requires actual harm to the chattel in order to be actionable. Here the chattel was returned "unharmed" and because there was no actual harm to the chattel there can be no action for trespass to chattel. B Incorrect. Trespass to chattel is intentionally interfering with the plaintiff's possession in a way that causes recognizable harm. Conversion is intentionally exercising dominion or control over another's chattel in a way that in inconsistent with the other's rights and deprives the other of the chattel. While the length of time of the deprivation is relevant in determining whether interference with a chattel is a trespass to chattel or if it is substantial enough to be conversion 5 months would typically be enough to be considered dominion or control for conversion. One clear distinction between conversion and trespass to chattels is that the former is actionable even if the chattel is unharmed but the latter requires actual harm to the chattel in order to be actionable. Here the chattel was returned "unharmed" and because there was no actual harm to the chattel there can be no action for trespass to chattel. C Correct. Trespass to chattel is intentionally interfering with the plaintiff's possession in a way that causes recognizable harm. Conversion is intentionally exercising dominion or control over another's chattel in a way that in inconsistent with the other's rights and deprives the other of the chattel. While the length of time of the deprivation is relevant in determining whether interference with a chattel is a trespass to chattel or if it is substantial enough to be conversion 5 months would typically be enough to be considered dominion or control for conversion. One clear distinction between conversion and trespass to chattels is that the former is actionable even if the chattel is unharmed but the latter requires actual harm to the chattel in order to be actionable. Here the chattel was returned "unharmed" and because there was no actual harm to the chattel there can be no action for trespass to chattel. In contrast 5 months is likely to be considered dominion and control over the chattel and even in the absence of actual harm Customer could demand that Thief keep the chattel and pay Customer the market value of the chattel instead. This would be a conversion. D Incorrect. Trespass to chattel is intentionally interfering with the plaintiff's possession in a way that causes recognizable harm. Conversion is intentionally exercising dominion or control over another's chattel in a way that in inconsistent with the other's rights and deprives the other of the chattel. While the length of time of the deprivation is relevant in determining whether interference with a chattel is a trespass to chattel or if it is substantial enough to be conversion 5 months would typically be enough to be considered dominion or control for conversion. One clear distinction between conversion and trespass to chattels is that the former is actionable even if the chattel is unharmed but the latter requires actual harm to the chattel in order to be actionable. Here the chattel was returned "unharmed" and because there was no actual harm to the chattel there can be no action for trespass to chattel. In contrast 5 months is likely to be considered dominion and control over the chattel and even in the absence of actual harm Customer could demand that Thief keep the chattel and pay Customer the market value of the chattel instead. This would be a conversion.

Tourist was walking along public road in a town he was visiting. Tourist approached a fenced parcel of land. As tourist approached the fenced area he saw many signs that read, "Private Property. KEEP OUT." Believing no one was looking tourist jumped the fence to examine a flower he saw on the land. As soon as tourist landed on the other side of the fence landowner yelled at tourist and tourist immediately jumped back over the fence onto the public roadway causing no harm to landowner's property. If landowner sues tourist for trespass to land will landowner win or lose? A Landowner will win because tourist knew that the land was owned by the landowner. B Landowner will lose because tourist did not spend enough time on landowner's land to be considered a trespass to land. C Landowner will win because tourist intentionally entered onto land owner's land. D Landowner will lose because tourist did not harm landowner's land.

A Incorrect. Trespass to land is intentionally entering or causing direct and tangible entry onto the land in the possession of another. The only intent required is the intent to enter onto land. The trespasser does not have to know that the land is in the possession of another. All that is required is intentional entry onto land and the land you enter is in another's possession. The required intent is not the intent to trespass just the intent to enter land. B Incorrect. Trespass to land requires a tangible entry and the duration of the entry is not material to establishing the tort. The duration of the entry is relevant only to the damage award. C Correct. Trespass to land is intentionally entering or causing direct and tangible entry onto the land in the possession of another. The only intent required is the intent to enter onto land. The trespasser does not have to know that the land is in the possession of another. All that is required is intentional entry onto land and the land you enter is in another's possession. The required intent is not the intent to trespass just the intent to enter land. Here the trespasser intended to enter land in the possession of another and therefore there was trespass to land. D Incorrect. Actual harm to land is not required for the tort to be actionable. Trespass to land is actionable on nominal damages even though no actual harm was done. The mere entry onto the land of another is damage enough to support the action of trespass to land.

houses. Chain link fences separate the houses. Every night light artist goes into his back yard and intentionally shines lights of all different colors on the lawns of her neighbors as part of a light show. The neighbors do not appreciate the lights on their property because they find them distracting and annoying. In an action for trespass to land against light artist will neighbors likely win or lose? A Neighbors will likely win because light artist intentionally causes light to enter on to their real property. B Neighbors will likely lose because the light is not tangible. C Neighbors will likely win if the light decreases their property value. D Neighbors will lose because the light causes no actual harm.

A Incorrect. Trespass to land requires a tangible entry onto the land of another and light is traditionally not considered tangible. B Correct. Trespass to land requires a tangible entry onto the land of another and light is not considered tangible. C Incorrect. In order for trespass to land to be actionable there must be tangible entry onto the land of another and light is not considered tangible. There may be a suit available for nuisance however. D Incorrect. It is true that the light causes no actual harm but this cannot be the right answer because actual harm is not a condition precedent for recovery in a trespass to land action.

Hiker was hiking in the woods. As hiker approached a bend in the path a large tree branch snapped, hitting hiker and sending her airborne for about 30 yards. She landed relatively unhurt but in the process was thrown over the unmarked property line of landowner, landing on landowner's land damaging some shrubs. If landowner sues for trespass to land, will landowner win or lose? A Landowner will win because hiker tangibly entered onto her property. B Hiker will win because her entry onto landowner's property was not volitional. C Landowner will lose because the price of shrubs pales in comparison to human life. D Hiker will lose because her entry onto the land caused actual damage.

A Incorrect. Trespass to land requires an intentional entry in order to be actionable. Here the hiker did not act intentionally to enter the land because she took no voluntary action to enter the land. Instead she was hurled there by a snapping tree branch. B Correct. The entry onto the land was non-volitional. Intent requires a volitional or voluntary act to bring about a tortious result. Here there was no volitional act on the part of the hiker to enter the landowner's property. C Incorrect. It is a pure distractor and is incorrect because the question has nothing to with actual harm or relative value of life and shrubs. There simply was no intent because intent requires a voluntary act and therefore there can be no intentional tort. D Incorrect. Regardless of the damage caused there can be no intentional tort because there simply was no intent because intent requires a voluntary act and the hiker was involuntarily thrown onto the landowner's property.

Shoplifter is seen placing an item in her coat pocket. As soon as shoplifter exits the store she is stopped by store security guards, who quietly and discreetly ask her to stop and empty her pockets. Did the guards act lawfully? A Incorrect. Very often shoplifting can't be confirmed unless the person leaves the store. Before exiting the property the suspected shoplifter may be able to claim that they intended to pay for the item before exiting the store. B Correct. It states the rule of law known as the shopkeeper's privilege. A shopkeeper may reasonably detain a suspected shoplifter. Any unreasonable detention, whether in terms of use of force, duration or distance from the store is not privileged however. C Incorrect. Merchants or their agents may reasonably detain suspected shoplifters. D Incorrect. Merchants and their agents are permitted to use only reasonable force to apprehend a suspected shoplifter.

A Incorrect. Very often shoplifting can't be confirmed unless the person leaves the store. Before exiting the property the suspected shoplifter may be able to claim that they intended to pay for the item before exiting the store. B Correct. It states the rule of law known as the shopkeeper's privilege. A shopkeeper may reasonably detain a suspected shoplifter. Any unreasonable detention, whether in terms of use of force, duration or distance from the store is not privileged however. C Incorrect. Merchants or their agents may reasonably detain suspected shoplifters. D Incorrect. Merchants and their agents are permitted to use only reasonable force to apprehend a suspected shoplifter.

Pedestrian was walking along the street on the edge of Homeowner's property. Unknown to Homeowner the door to his house was open and he was unaware that Pedestrian was on the street. In an instant Homeowner's massive and ferocious dog sped through the door and charged towards Pedestrian. As Pedestrian looked across the 15 yards of lawn between the street and the doorway he was petrified, and screamed in horror as the dog bound towards him. The dog sprinted towards Pedestrian baring it's teeth and barking ferociously. Pedestrian froze with fear but just before as the dog reached the edge of the property less than 2 feet from where Pedestrian was in the street it suddenly stopped and moved back away from the property line and sat on the lawn. Relieved, Pedestrian walked away scared and almost in shock because the fear he had just experienced. Homeowner knew the dog would not reach Pedestrian because of the buried electronic fence installed in the yard which was connected to the dog's collar and prevented the dog from reaching the street. Did homeowner assault Pedestrian? A Win, because the dog caused him to be fearful or apprehensive of harmful or offensive contact B Yes, if it was careless of Homeowner to not check the door C No, because Homeowner did not act with intent D No, because the dog did not actually make contact with the Pedestrian

A Incorrect. While it is true that intent, when either there is either the specific intent to cause a tortious result or the defendant is substantially certain that a tortious result (harmful or offensive contact in the case of battery) would occur, the facts say that "unknown to Homeowner" the door was left open so he was not acting deliberately to cause harmful or offensive contact to Pedestrian, neither could he have been substantially certain it would occur if he was unaware the door was open. B Incorrect. Carelessness is not the same as intent. And assault requires intent. Carelessness is often said to be the essence of negligence. If a reasonable person would have checked the door and Homeowner didn't then Homeowner would be acting unreasonably as a matter of law even if he did not realize the door was unlocked. If you fail to act like a reasonable person would have then you are negligent. Intent however requires that the actual defendant is substantially certain of the result. It requires more than mere carelessness or possibility of harm. C Correct. Homeowner was neither substantially certain his dog would cause harmful or offensive contact to Pedestrian nor did he act with the deliberate purpose of bringing about that result he cannot be said to have acted with intent. All the intentional torts require intent. D Incorrect. Assault does not require contact. So it is incorrect to say there was no assault because there was no contact. Contact is not an element of assault.

General release:

A hit you. You entered into a release with A. settlement agreement says that P releases the D and any other person that may be responsible for the accident. Any and all persons that may be liable are released by that general release. aka, by releasing one, you say that you release also any and all persons that may be responsible.

Trespass to land - The simple thing: if you own or are in possession of a property, you can claim trespass if somebody has entered the property without the owner or the possessor's consent. That's a trespass.

A mere stepping of a foot upon property or land is a trespass, regardless of whether they do any damage. You can sue, get nominal damages to vindicate your right to own and possess the land.

Comparative negligence: a system that's in place in 46 states; purpose is to compare the negligence of all the parties of the lawsuit. P's negligence is examined, and any Ds' negligence is examined. And a jury will assign a percentage to each party.

A percentage goes to the P, D1, D2, etc. those percentages are taken into consideration if the jury comes back with a verdict; if is 100k amount in damages, if P is 10% negligence, then ten percent of that 100k she can't recoup from anyone. The other 90k will be distributed amongst the Ds. When in a comparative negligence state, the negligence of ALL parties will be compared.

Judge can also give a remittitur in the state system, lower the amount.

A state judge can grant both additur and remittitur UNLESS state constitution says that one of them is unconstitutional. On fed level, judge can only give remittitur.

Duty mostly stated on exam: duty to act as a reasonable prudent person in the same or similar circumstance.

A. Breach occurs when one doesn't act like that. evaluate breach using gravity of harm, likelihood of harm, burden of precaution. Likelihood envisions foreseeability, possibility.

Which of the following statements about factual and proximate causation is correct? A. Proximate causation determinations in negligence actions establish whether to shield a defendant from liability even though the defendant's breach of the standard of care was a factual cause of the plaintiff's harm. B. Proximate causation determinations are unique to negligence actions and are not made in intentional torts or strict liability actions. C. Legal causation and factual causation are synonymous terms. D. Factual causation and proximate causation are synonymous terms.

A. Correct. Causation in a negligence action is comprised of factual causation (which is also called but for causation or factual causation) as well as legal causation (which is often termed proximate causation). Legal causation shields a defendant from liability even though they are a factual cause of the complained of harm. B. Incorrect. Most often an issue in negligence actions, legal or proximate causation determinations may also arise in intentional torts or in strict liability. C. Incorrect. Legal causation and factual causation are not synonymous terms. D. Incorrect. Factual causation and proximate causation are not synonymous terms.

Drug Co. negligently manufactures and sells a defective contraceptive pill, which fails to prevent Yolanda's pregnancy. Yolanda gives birth to Child A, who grows up and gives birth to Child B. Child B grows up to be a motorcycle enthusiast, who negligently causes a motorcycle accident that injures Charles. Charles seeks to recover from Drug Co. in a negligence action, alleging that Drug Co.'s negligence was a factual cause of Charles's injury. Will Charles's negligence action likely succeed or fail? A. Charles is correct that Drug Co. is a factual cause of Charles's injury, but his negligence action will likely fail for lack of proximate cause. B. Charles is incorrect that Drug Co. is a factual cause of Charles's injury and therefore cannot be subject to liability in negligence to Charles. C. Charles is correct that Drug Co. is a factual cause of Charles's injury, and therefore his negligence action against Drug Co. will likely succeed. D. Charles is incorrect that Drug Co. is a factual cause of Charles's injury, but his negligence action against Drug Co. may still succeed.

A. Correct. Even though Drug Co. is a factual cause of Charles's injuries, his injuries are likely too remote from Drug Company's negligence in order for its negligence to be a proximate cause of his injuries. This question asks about proximate cause determinations in a products liability action grounded in negligence. B. Incorrect. Drug Co. is a factual cause of Charles's injury. C. Incorrect. A successful negligence action requires proof of factual causation and proximate causation. Proof of factual causation alone is insufficient. D. Incorrect. Drug Co. is a factual cause of Charles's injury, and if it were not, it could not be subject to liability in negligence.

Connie is carelessly changing lanes in her car when her car is struck by Bill who is reaching for his phone, which has fallen under the passenger side seat while he was driving. Adrian, who is in the vicinity on her motorcycle waiting patiently at the intersection for the light to turn green, is struck by Connie's car and suffers serious physical injury as well as damage to her bike. She sues both Connie and Bill for their negligence. The jury determines that Adrian's damages are $100,000 and concludes that Bill and Connie are both liable. It assigns 30% responsibility to Bill, and 70% responsibility to Connie. In a jurisdiction that subjects defendants to joint and several liability, what amount, if any, is Adrian entitled to recover from Bill and/or Connie? A. If he chooses, Adrian may recover $100,000 from Bill alone. B. Adrian is entitled to recover only $30,000 from Bill and only $70,000 from Connie for a total of $100,000. C. Adrian is entitled to recover $100,000 from both Bill and Connie for a total of $200,000. D. If he chooses, Adrian may recover up to $30,000 from Connie and $70,000 from Bill.

A. Correct. Joint and several liability means that each tortfeasor is 100% responsible for the entire judgment and the plaintiff is free to recover her full judgment from any one or combination of them. Therefore, Adrian can recover the entire judgment from Bill alone. B. Incorrect. It describes several responsibility, where each defendant is only responsible for the percentage of the judgment that represents their apportionment of fault. C. Incorrect. Although Adrian can recover from either tortfeasor, she is only entitled to recover up to the amount of her judgment. She cannot recover 100% of the judgment from each tortfeasor as that would be a double recovery for her. D. Incorrect. Joint and several liability means that each tortfeasor is 100% responsible for the entire judgment and the plaintiff is free to recover her full judgment from any one or combination of them.

In a negligence action brought by Plaintiff Betty against Defendant Alice and Defendant Carl, the jury apportions liability based on the negligence of the parties as follows: Betty is 20% at fault; Alice is 60% at fault; and Carl is 20% at fault. The jury also determines that Plaintiff Betty sustained $100,000 in compensatory damages. The parties are in a modified "not greater than" comparative fault jurisdiction that provides that defendants are severally liable, where the plaintiff's fault is compared to the combined fault of all defendants. What amount may Betty recover from Alice or Carl? A. Betty may recover 60% of $100,000 from Alice and 20% of $100,000 from Carl. B. Betty may recover 80% of $100,000 from either Alice or Carl or a combination of both. C. Betty may recover 60% of $80,000 from Alice and 20% of $80,000 from Carl. D. Betty may recover 50% of $100,000 from Carl and 50% of $100,000 from Alice.

A. Correct. Plaintiff Betty's fault is compared to the combined fault of all defendants; Betty is not barred from recovery because her fault is not greater than the combined fault of the two defendants. Therefore, she will be entitled to recover damages reduced by the percentage of fault attributable to her -- $100,000-$20,000=$80,000. B. Incorrect. It describes a joint and several liability rather than just several liability. Defendants in this jurisdiction are only severally liable, meaning responsible only for their percentage of the damages. C. Incorrect. The defendant's percentage of fault is based upon the entire judgment, which is $100,000. D. Incorrect. It assigns incorrect percentages to the defendants.

In a negligence action brought by Plaintiff Betty against Defendant Alice and Defendant Carl, the jury apportions liability based on the negligence of the parties as follows: Betty is 20% at fault; Alice is 60% at fault; and Carl is 20% at fault. The jury also determines that Plaintiff Betty sustained $100,000 in compensatory damages. In a pure comparative fault jurisdiction that provides that defendants are severally liable, what amount, if any, may Betty recover? A. Betty may recover up to 60% of $100,000 from Alice and up to 20% of $100,000 from Carl. B. Betty may recover up to 60% of $80,000 from Alice and up to 20% of $80,000 from Carl. C. Betty may recover 60% of $80,000 from Alice or Carl or a combination of both. D. Betty may not recover at all.

A. Correct. Plaintiff can recover 60% of the $100,000 from Alice and 20% of $100,000 from Carl. This question asks about the several liability of defendants in a pure comparative fault jurisdiction. In jurisdictions with such schemes, the defendant is only responsible for the percentage of the damages that are attributable to it. B. Incorrect. The defendants are responsible for their percentage of fault of the entire judgment, which is $100,000, not $80,000. This question asks about the several liability of defendants in a pure comparative fault jurisdiction. In jurisdictions with such schemes, the defendant is only responsible for the percentage of the damages that are attributable to it. C. Incorrect. The defendants are responsible for their percentage of fault of the entire judgment, which is $100,000, not $80,000. C is additionally incorrect because several liability makes the defendants responsible for only their share of fault, and C provides a joint and several liability scheme, where each defendant is responsible for the entire judgment, not just their individual percentage. D. Incorrect. In a pure comparative fault system, Betty is not barred from recovery; her judgment is just reduced by the amount of fault attributable to her, which is 20% of $100,000 or $20,000. She is entitled to recover $100,000-$20,000=$80,000.

A. That Fanny could be injured was foreseeable from George's negligence and therefore George can be subject to liability. Correct. Pursuant to the eggshell skull plaintiff rule, a plaintiff's physical injuries are foreseeable. B. George cannot be held liable for Fanny's injuries because he could not have foreseen that she was suffering from an asymptomatic herniated disc. C. George may be subject to liability for the collision itself, but not for the extensive back injuries because the extensive back injuries were not foreseeable. D. George may only be held liable to Fanny for injuries that he intended to cause.

A. Correct. Pursuant to the eggshell skull plaintiff rule, a plaintiff's physical injuries are foreseeable. B. Incorrect. A defendant is subject to liability if his tortious conduct causes harm to a person that, because of a preexisting physical condition, is of a greater magnitude than might reasonably be expected. See Restatement (Third) Torts § 31 and illustration 1 upon which this fact pattern is based. C. Incorrect. The defendant is liable for all injuries that result, not just those that would have been suffered by one without the preexisting condition. D. Incorrect. It refers to intended injuries rather than those negligently inflicted.

Toddler, a two-year-old child suffering from a bacterial infection, received a routine childhood vaccination with a vaccine manufactured by Vaco. Shortly after receiving the vaccine, Toddler spiked a very high fever, went into respiratory arrest, and died. Toddler's estate sues Vaco in negligence. At trial, Toddler's estate provides competent evidence that Toddler died due to a combination of the infection and the vaccine and, if either toddler had not had an infection or if Toddler had not been vaccinated, death would not have ensued. Regarding the factual causation element of the negligence action, has Toddler's estate satisfied its burden of production to prove that the vaccine was a factual cause of Toddler's death? A. Toddler's estate has satisfied its burden of production to show that the vaccine was a factual cause of Toddler's death. B. Toddler's estate has not satisfied its burden of production to show that the vaccine was a factual cause of Toddler's death. C. Toddler's estate cannot satisfy its burden of production to show that the vaccine was a factual cause of Toddler's death because the infection contributed to Toddler's death. D. Toddler's estate cannot satisfy its burden of production to show that the vaccine was a factual cause of Toddler's death because the infection was the sole factual cause of Toddler's death.

A. Correct. See Restatement (Third) Torts § 26 and illustration 4, upon which this problem is based. The estate's evidence that Toddler died due to a combination of the infection and the vaccine coupled with proof that either toddler had not had an infection or if Toddler had not been vaccinated, death would not have ensued, is sufficient to prove factual causation. B. Incorrect. Plaintiff is only required to show that the defendant's tortious conduct was more likely than not a factual cause of the harm and Toddler's estate has satisfied that requirement. C. Incorrect. There can be more than one factual cause of any one event, and that the infection was also a factual cause does not preclude a showing that the vaccine was an additional factual cause. D. Incorrect. The facts do not suggest that the infection was the sole factual cause, but rather that the infection contributed along with other factors to result in Toddler's death.

Hunter finishes his day working in the field and stops at a friend's house on his walk home from work. His friend's nine-year-old daughter, Katie, greets Hunter, who hands his loaded shotgun to her as he enters the house. The shotgun is neither especially heavy nor unwieldy. Nonetheless, Katie drops the shotgun, which lands on her toe, breaking it. In a negligence action against Hunter for Katie's broken toe, may Hunter be held liable in negligence? A. Hunter may not be held liable in negligence for Katie's broken toe because the risk that makes Hunter negligent is that Katie might shoot something or some person with the gun, not that she would drop it on her toe and hurt herself. B Hunter may be held liable in negligence for Katie's broken toe because he negligently provided her with the shotgun. C. Hunter may not be held liable because Hunter's act of giving Katie the gun is not a factual cause of Katie's broken toe. D. Hunter may not be held liable for Katie's broken toe if the manner in which she ended up injuring herself was unforeseeable.

A. Correct. This question asks about proximate causation determinations. Generally speaking, a defendant will be liable in negligence for those harms that result from the risks that make the actor's conduct tortious. See Restatement (Third) Torts § 29 and illustration 3, upon which this fact pattern is based. A broken toe is not what makes giving a child a rifle risky. B. Incorrect. This question asks about proximate causation determinations. Generally speaking, a defendant will be liable in negligence for those harms that result from the risks that make the actor's conduct tortious. See Restatement (Third) Torts § 29 and illustration 3, upon which this fact pattern is based. A broken toe is not what makes giving a child a rifle risky. So proof of his negligence in providing her with the gun is not enough; she must also prove that harm that occurred resulted from what made his conduct risky in the first place. C. Incorrect. Hunter's act of giving Katie the gun was the factual of cause of her injury; but for him giving her the gun, she would not have dropped it on her foot. This question asks about proximate causation determinations. Generally speaking, a defendant will be liable in negligence for those harms that result from the risks that make the actor's conduct tortious. See Restatement (Third) Torts § 29 and illustration 3, upon which this fact pattern is based. A broken toe is not what makes giving a child a rifle risky. D. Incorrect. This question asks about proximate causation determinations. Generally speaking, a defendant will be liable in negligence for those harms that result from the risks that make the actor's conduct tortious. See Restatement (Third) Torts § 29 and illustration 3, upon which this fact pattern is based. It does not have to be established that Hunter could have foreseen the manner in which she would hurt herself.

Alice was exposed to two different solvents while working in a laboratory. Each solvent contained a toxic chemical. One contained Chemical B; the other contained Chemical C. After developing the disease decolis several years later, Alice sues the manufacturers of each solvent, claiming that the manufacturers were negligent for including a toxic chemical in their solvents. Alice's evidence, presented by competent expert testimony based on valid scientific evidence reveals that the mechanism by which decolis develops is different for Chemical C exposure and for Chemical B exposure and that exposure to one or the other but not both is the most likely explanation for Alice's decolis. However, Alice cannot prove whether Chemical B or Chemical C caused her decolis. After presentation of her case, the defendants (manufacturers of Chemical B and manufacturer of Chemical C) both move for summary judgment on the issue of factual causation. How should the court rule on the defendants' requests for summary judgment? A. The court should deny the motion for summary judgment and rule that the burden of proof on agent-disease factual causation is shifted to the manufacturers of Chemical B and Chemical C. B. The court should grant the motion for summary judgment because Alice has failed to establish which defendant is the factual cause of her decolis. C. The court should deny the motion for summary judgment because the defendants in negligence action always have the burden of proof on the issue of causation. D. The court should grant the motion for summary judgment so that Alice can engage in further discovery on the issue of causation.

A. Correct. This question raises the issue of proof problems in proving negligence when there are multiple actors. Usually, the plaintiff has the burden of proof for every element of the prima facie case, but when the plaintiff sues multiple tortfeasors and cannot reasonably be expected to prove which one caused her harm, the burden of proof on the issue of factual causation shifts to the defendants. Restatement (Third) Torts § 28. B. Incorrect. In circumstances such as these, the burden shifts to the defendants. Usually, the plaintiff has the burden of proof for every element of the prima facie case, but when the plaintiff sues multiple tortfeasors and cannot reasonably be expected to prove which one caused her harm, the burden of proof on the issue of factual causation shifts to the defendants. Restatement (Third) Torts § 28. C. Incorrect. The burden of proof on all elements of the prima facie case typically falls on the plaintiff. D. Incorrect. The facts indicate that further discovery will not help Alice determine which manufacturer caused her injury.

Rosie and Vincent do not know each other and were camping independently in a heavily forested campground. Each one had a campfire, and each one negligently failed to ensure that the fire was extinguished upon retiring for the night. Due to unusually dry forest conditions and a stiff wind, both campfires escaped their sites. The two fires, burning out of control, eventually joined together and engulfed Company's hunting lodge, destroying it. Either fire alone would have destroyed the lodge. In a lawsuit brought by Company against both Rosie and Vincent alleging negligence, which of the following statements is correct regarding factual causation? A Each of Rosie's and Vincent's negligence is a factual cause of the destruction of Company's hunting lodge. B. Neither Rosie's nor Vincent's negligence is a factual cause of the destruction of Company's hunting lodge. C. Rosie's negligence is the sole factual cause of the destruction of Company's hunting lodge. D. Vincent's negligence is the sole factual cause of the destruction of Company's hunting lodge.

A. Correct. When multiple acts occur and each alone would have been a factual cause in the absence of the other act, each act is a factual cause of the harm. This is a question about factual causation when two independent acts of negligence join together to create one harm where either independent act alone would have been sufficient in creating the harm. Restatement (Third) Torts § 27. B. Incorrect. When multiple acts occur and each alone would have been a factual cause in the absence of the other act, each act is a factual cause of the harm. This is a question about factual causation when two independent acts of negligence join together to create one harm where either independent act alone would have been sufficient in creating the harm. Restatement (Third) Torts § 27. C. Incorrect. Both negligent acts combined together; therefore, both are a factual cause. This is a question about factual causation when two independent acts of negligence join together to create one harm where either independent act alone would have been sufficient in creating the harm. Restatement (Third) Torts § 27. D. Incorrect. Both negligent acts combined together; therefore, both are a factual cause. This is a question about factual causation when two independent acts of negligence join together to create one harm where either independent act alone would have been sufficient in creating the harm. Restatement (Third) Torts § 27.

another theory similar to public policy: burden shifting

A. Ex if you leave a gun in the street, kid picks it up, mom takes it away, but kid gets it again, responsibility has shifted to the mother. B. The theory: does a third party take control of the risk such that the court says it's now their problem, not the og tortfeasor's problem. Libility is cut off if so. C. This theory is like public policy - way we can say that despite everything, can throw this in the works to try to stop liability.

Connie is carelessly changing lanes in her car when her car is struck by Bill who is reaching for his phone, which has fallen under the passenger side seat while he was driving. Adrian, who is in the vicinity on her motorcycle waiting patiently at the intersection for the light to turn green, is struck by Connie's car and suffers serious physical injury as well as damage to her bike. She sues both Connie and Bill for their negligence. The jury determines that Adrian's damages are $100,000 and concludes that Bill and Connie are both liable. It assigns 30% responsibility to Bill, and 70% responsibility to Connie. In a jurisdiction that subjects defendants to joint and several liability, if Adrian recovers $100,000 from Bill alone, what amount is Bill entitled to collect from Connie? A. Bill is entitled to collect $100,000 in contribution from Connie. B. Bill is entitled to collect $100,000 in satisfaction from Connie. C. Bill is entitled to collect $70,000 in contribution from Connie. D. Bill is entitled to collect $70,000 in satisfaction from Connie.

A. Incorrect. A tortfeasor who pays more than its fair share is entitled to contribution from other jointly and severally liable tortfeasors. Therefore, Bill is entitled to collect 70% of the $100,000 from the tortfeasor determined to be 70% at fault, Connie. B. Incorrect. Bill is not entitled to the full $100,000. his recovery from Connie for her share is contribution, the right to have a joint tortfeasor pay in proportion to the fault apportioned to it, not satisfaction (which is a payment in full of one's debt). C. Correct. A tortfeasor who pays more than its fair share is entitled to contribution from other jointly and severally liable tortfeasors. Therefore, Bill is entitled to collect 70% of the $100,000 from the tortfeasor determined to be 70% at fault, Connie. D. Incorrect. His recovery from Connie for her share is contribution, the right to have a joint tortfeasor pay in proportion to the fault apportioned to it, not satisfaction (which is a payment in full of one's debt).

Lilly was a guest at Motel, which was located in a neighborhood where significant violent crime existed. After Lilly returned to her hotel room, Don was able to gain entrance to Lilly's room because the lock on the door was of the simple residential type that could be easily defeated with a credit card. After gaining entrance to Lilly's room, Don sexually assaulted her. Lilly sues Motel, claiming that it was negligent in providing inadequate locks for its guest rooms. May Hotel be held liable in negligence? A. Motel may not be held liable in negligence because Don's criminal act was intentional and therefore breaks the chain of causation. B. Motel may not be held liable because Don's criminal act was an intervening cause. C. Motel may be held liable because Don's act was unforeseeable, unusual, and highly culpable. D. Motel may be held liable in negligence because Don's criminal act was foreseeable from its negligence.

A. Incorrect. Don's criminal act resulted from the risk that made Motel's installation of inadequate locks negligent. When an independent act is also a factual cause of a harm, the defendant's liability is limited to those harms that result from the risks that make the defendant's conduct negligent. See Restatement (Third) Torts § 34. This question asks about intervening causes and proximate causation determinations. When an intervening cause is unforeseeable, unusual, or highly culpable, the initial tortfeasor is no longer a proximate cause of the plaintiff's harm. B. Incorrect. Not all criminal acts are superseding causes which sever the chain of causation. This question asks about intervening causes and proximate causation determinations. When an intervening cause is unforeseeable, unusual, or highly culpable, the initial tortfeasor is no longer a proximate cause of the plaintiff's harm. C. Incorrect. It misstates the principle; it would be correct if it said that Motel may not be liable if Don's act was unforeseeable, unusual, and highly culpable. This question asks about intervening causes and proximate causation determinations. When an intervening cause is unforeseeable, unusual, or highly culpable, the initial tortfeasor is no longer a proximate cause of the plaintiff's harm. D. Correct. The initial tortfeasor may be held liable when the criminal act was foreseeable as it was here. This question asks about intervening causes and proximate causation determinations. When an intervening cause is unforeseeable, unusual, or highly culpable, the initial tortfeasor is no longer a proximate cause of the plaintiff's harm.

Owner owns an apartment building and employs Manager as its on-site manager. Owner reasonably believes that Manager's temperament is suited to the duties of on-site apartment management based on Owner's inquiries to Manager's prior employers. Owner does not check with law enforcement authorities. Had Owner done so, Owner would have discovered that Manager has been convicted of income-tax evasion, but Owner would have hired him in any event because Owner would not have viewed that conviction as material to Manager's duties as an on-site manager, a job which does not involve tax-compliance tasks. One day, Tenant complains to Manager about the lack of heat in Tenet's apartment. When Manager visits Tenant about the complaint, Manager becomes enraged and assaults Tenant. In a lawsuit brought by Tenant against Owner for his negligence, may owner be subject to liability for Manager's assault of Tenant? A. Owner may be subject to liability for Manager's assault of Tenant because Manager was Owner's employee. B. Owner may not be subject to liability to Tenant as a consequence of Manger's assault on Tenant because an employer cannot be subject to liability for the tortious conduct of its employees. C. Owner may not be subject to liability to Tenant as a consequence of Manager's assault on Tenant because Owner's failure to detect Manager's tax-evasion conviction was not a factual cause of Tenant's harm and did not increase the risk of an assault by Manager against Tenant. D. Owner may be subject to liability for Manger's assault of Tenant because Owner was negligent in failing to discover accurate information about Manager's temperament.

A. Incorrect. Employers are not automatically liable for the tortious conduct of their employees as this answer choice suggests. B. Incorrect. Under certain circumstances a lawyer may be subject to liability for the tortious conduct of its employees. C. Correct. The facts tell you that even if Owner had known of the tax evasion conviction, Owner would have hired Manger anyway. Also, knowing about the tax evasion conviction did not increase the risk of an assault by Manager against Tenant. D. Incorrect. It mischaracterizes the facts. The facts suggest that Owner was diligent in discovering information about Manager's temperament.

In a negligence action brought by Plaintiff Betty against Defendant Alice and Defendant Carl, the jury apportions liability based on the negligence of the parties as follows: Betty is 20% at fault; Alice is 60% at fault; and Carl is 20% at fault. The jury also determines that Plaintiff Betty sustained $100,000 in compensatory damages. The parties are in a contributory negligence jurisdiction that provides that defendants are jointly and severally liable to plaintiffs. What amount, if any, may Betty recover? A. Betty may recover up to 60% of $100,000 from Alice and up to 20% of $100,000 from Carl. B. Betty may recover up to 60% of $80,000 from Alice and up to 20% of $80,000 from Carl. C. Betty may recover 100% of $100,000 from either Alice or Carl or a combination of both. D. Betty may not recover at all.

A. Incorrect. In a contributory negligence jurisdiction, the plaintiff cannot recover if she was at fault at all. B. Incorrect. In a contributory negligence jurisdiction, the plaintiff cannot recover if she was at fault at all. C. Incorrect. In a contributory negligence jurisdiction, the plaintiff cannot recover if she was at fault at all. D. Correct. In a contributory negligence, a plaintiff who was at fault at all is complete barred from recovery.

In a negligence action brought by Plaintiff Betty against Defendant Alice and Defendant Carl, the jury apportions liability based on the negligence of the parties as follows: Betty is 20% at fault; Alice is 60% at fault; and Carl is 20% at fault. The jury also determines that Plaintiff Betty sustained $100,000 in compensatory damages. The parties are in a modified "not as great as" comparative fault jurisdiction that provides that defendants are jointly and severally liable and the plaintiff's fault is compared to each defendant separately. What amount may Betty recover from Carl or Alice? A. Betty may recover 100% of $80,000 from Carl or Alice or a combination of both. B. Betty may recover 100% of $100,000 from Carl or Alice or a combination of both. C. Betty may not recover from Carl, but can recover 100% of $80,000 from Alice. D. Betty may not recover from Carl, but can recover 100% of $100,000 from Alice.

A. Incorrect. In this case, Plaintiff Betty's fault is as great as Defendant Carl's fault, both having 20%. Therefore, Betty is barred from recovering from Carl. The question asks about a modified comparative fault jurisdiction where the plaintiff is barred from recovery if plaintiff's percentage of fault is as great as or greater than each defendant compared separately. B. Incorrect. Plaintiff Betty's fault is as great as the fault of Defendant Carl's fault, both having 20%. Therefore, Betty is barred from recovering from Carl. The question asks about a modified comparative fault jurisdiction where the plaintiff is barred from recovery if plaintiff's percentage of fault is as great as or greater than each defendant compared separately. C. Correct. Because Defendant Alice is jointly and severally liable for Betty's recoverable damages of $80,000, Alice is responsible for the entire amount. The question asks about a modified comparative fault jurisdiction where the plaintiff is barred from recovery if plaintiff's percentage of fault is as great as or greater than each defendant compared separately. D. Incorrect. Because her fault is not as great as the fault of Defendant Alice, Betty can recover from Alice. Because of the joint and several liability model for those defendant's at fault, Betty is entitled to recover her damages reduced by the percentage of fault attributed to her: $100,000-$20,000=$80,000. The question asks about a modified comparative fault jurisdiction where the plaintiff is barred from recovery if plaintiff's percentage of fault is as great as or greater than each defendant compared separately.

Visitor was a customer of Barge, when while on the deck of Barge he accidentally fell into the water as the barge approached the pier. Employee of the barge hears Visitor's cries as he yells from the water that he cannot swim. Employee threw a heavy line (used to tie the barge to the pier) in the direction of the cries. The line came within two feet of the spot of Visitor, but Visitor did not grab the lines. Visitor was carried away from the pier by the outgoing tide and downed. At the time of this incident, the employee was aware that several life preservers were on a rack located near the front of the barge where Visitor fell overboard. The preservers remained in the rack and were not used. If the preservers had been used, Victim probably would not have drowned. Visitor's estate sues Barge in negligence, alleging that the failure to use the life preservers was the factual cause of Victim's drowning. Is Visitor's estate correct that Barge's failure to use the life preservers was a factual cause of Victim's drowning? A. No because the failure to use the life preservers is an omission and an omission cannot be the basis for a negligence action. Incorrect. It is an incorrect statement of law to assert that an omission or a failure to act cannot form the basis of a negligence action. As a customer of Barge, Barge likely had an affirmative duty to act under the circumstances and its failure to do so may be the basis of liability in tort. B. No because it is possible that Victim would have drowned even if the life preservers had been thrown out to him. C. Yes because not throwing the preservers to Victim was a substantial factor in bringing about Victim drowning. D. Yes because Victim drowned when he was a customer of Barge. Incorrect. The fact that Victim is a customer of Barge does not necessarily subject Barge to liability in negligence to Victim.

A. Incorrect. It is an incorrect statement of law to assert that an omission or a failure to act cannot form the basis of a negligence action. As a customer of Barge, Barge likely had an affirmative duty to act under the circumstances and its failure to do so may be the basis of liability in tort. B. Incorrect. The Barge's failure to throw the life preservers may be a factual cause of Victim's drowning even if there is evidence that victim would have possibly suffered the same harm had the preservers been thrown to him. Where the negligence of the defendant greatly multiplies the changes of accident to the plaintiff and is of a character naturally leading to it occurrence, the mere possibility that it might have happened without the negligence is not sufficient to break the factual cause chain of cause and effect. C. Correct. Negligence (breach of the standard of care) is a factual cause of a plaintiff's injury if it is more likely than not that the harm would have been averted but for the negligence of the defendant. D. Incorrect. The fact that Victim is a customer of Barge does not necessarily subject Barge to liability in negligence to Victim.

In a negligence action, what is the plaintiff required to prove in order to establish proximate (or legal) causation? A .The plaintiff is required to prove that but for the defendant's breach of the standard of care, the plaintiff would not have suffered the complained of harm. B. The plaintiff is required to prove that the defendant should have reasonably foreseen, as a risk of her conduct, the general consequences or type of harm suffered by the plaintiff. C. The plaintiff is required to prove that the defendant should have reasonably foreseen, as a risk of her conduct, the general consequences of type of harm suffered by the plaintiff and the manner in which the harm came about. D. The plaintiff is required to prove that the defendant should have reasonably foreseen, as a risk of her conduct, the general consequences of type of harm suffered by the plaintiff, the extent of the harm, and the manner in which the harm came about.

A. Incorrect. It provides the definition off actual causation. This is a question to test your knowledge of proximate or legal causation analysis. B. Correct. It is a correct statement of proximate or legal causation. The plaintiff is required to prove that the defendant should have reasonably foreseen, as a risk of her conduct, the general consequences or type of harm suffered by the plaintiff. C. Incorrect. The defendant is not required to have foreseen the manner in which the harm came about. This is a question to test your knowledge of proximate or legal causation analysis. D. Incorrect. The defendant is not required to have foreseen the manner in which the harm came about or the extent of the harm. This is a question to test your knowledge of proximate or legal causation analysis.

Jeff is driving 35 miles per hour on a city street with a speed limit of 25 miles per hour with Nate as his passenger. But for his speeding, Jeff would not have been at the very place on the road when, without warning, a tree crashes onto Jeff's car, injuring Nate. In a negligence action brought by Nate against Jeff, should Jeff be liable for negligence? A. Jeff should not be liable in negligence because Jeff's speeding is not a factual cause of Nate's harm. B. Jeff should be liable in negligence because Jeff's negligent conduct of speeding increased the risk of the type of harm suffered by Nate. C. Jeff should not be liable in negligence because Jeff's negligent conduct of speeding did not increase the risk of the type of harm suffered by Nate. D Jeff should not be liable in negligence because a passenger of a car cannot successfully sue the driver of the car in negligence.

A. Incorrect. Jeff's speeding is a factual cause of Nate's harm. This question focuses on a defendant's liability in negligence when tortious conduct of the defendant does not generally increase the risk of the type of harm suffered by the plaintiff. See Restatement (Third) Torts § 30 and illustration 1 upon which this hypothetical is based. B. Incorrect. Jeff's negligent conduct of speeding did not increase the risk of harm (being struck by a tree) suffered by Nate. This question focuses on a defendant's liability in negligence when tortious conduct of the defendant does not generally increase the risk of the type of harm suffered by the plaintiff. See Restatement (Third) Torts § 30 and illustration 1 upon which this hypothetical is based. C. Correct. Jeff's negligent conduct of speeding did not increase the risk of the type of harm suffered by Nate. This question focuses on a defendant's liability in negligence when tortious conduct of the defendant does not generally increase the risk of the type of harm suffered by the plaintiff. See Restatement (Third) Torts § 30 and illustration 1 upon which this hypothetical is based. D. Incorrect. It is not true that a passenger cannot sue a driver for injuries from a car accident. This question focuses on a defendant's liability in negligence when tortious conduct of the defendant does not generally increase the risk of the type of harm suffered by the plaintiff. See Restatement (Third) Torts § 30 and illustration 1 upon which this hypothetical is based.

Jessica was driving her car, which was manufactured by Motor Company, when the voltage regulator in the car failed due to its negligent installation. The failure caused the battery fluid to boil, which produced toxic fumes that reached the interior of the car. Jessica suffered chronic vocal-cord damage as a result. Prior to this occurrence, Jessica was a popular singer who earned several millions of dollars each year. Jessica sues Motor Company for its negligence to recover for her injuries. Will Jessica be able to recover for her injuries? A. Jessica will only be able to recover for injuries that the ordinary person would have suffered due to the same occurrence, as Jessica being a popular singer was unforeseeable to Motor Company. B. Motor Company is only subject to liability for damages to the vehicle itself, not for injuries to a person. C. All of Jessica's lost earnings due to her vocal-cord injury are within the scope of Motor Company's liability because the type of injuries Jessica suffered were foreseeable to Motor Company from its negligence. D. None of Jessica's lost earnings due to her vocal-cord injury are within the scope of Motor Company's liability because those injuries were not foreseeable to Motor Company from its negligence.

A. Incorrect. Motor Company will be responsible for all the damages that flow from the injuries to Jessica's vocal cords. Restatement (Third) Torts § 31 and illustration 3 upon which this fact pattern is based. This is an application of the eggshell skull plaintiff rule. B. Incorrect. Motor Company may be subject to liability for damages to people and property. C. Correct. Jessica's lost earnings due to her vocal-cord injury are within the scope of Motor Company's liability were foreseeable to Motor Company from its negligence. This is an application of the eggshell skull plaintiff rule. D. Incorrect. Motor Company may still be subject to liability even for damages that are of a greater magnitude than might be expected. This is an application of the eggshell skull plaintiff rule.

Lincoln's airplane was seriously damaged when he was forced to land it without its retractable gear in the down position. Lincoln sues Dow Aircraft, the manufacturer of the plane. Lincoln claims that Dow Aircraft neglected to include instructions in its service manual of the need to be sure of a minimum clearance between two parts in the landing gear assembly when reassembling the gear after routine servicing. Lincoln's plane was serviced at Dow because Dow needed to complete unrelated warranty work to the aircraft. The Dow mechanic who worked on Lincoln's plane was fired shortly after the work was completed for repeatedly failing to consult service manuals. In a cause of action against Dow Aircraft and the mechanic, Lincoln includes a claim based on the mechanic's negligence. Which of the following statements regarding factual cause of the harm to Lincoln's plane is correct? A. The omitted instruction by itself was a factual cause of the harm to Lincoln's plane. B. The mechanic's negligence in failing to consult the manual was by itself a factual cause of the harm to Lincoln's plane. C. Because neither the missing instructions nor the mechanic's failure to consult the manual were a factual cause of Lincoln's harm, Lincoln will not be able to establish that Dow Aircraft was a factual cause of the damage to his plane. D. If Lincoln can prove both that Dow Aircraft providing the omitted instructions and a mechanic properly consulting the service manual would have prevented the damage to Lincoln's plane, Dow Aircraft's multiple acts of negligence are a factual cause of the plane's damage.

A. Incorrect. Neither one of these acts alone would have produced the injury to Lincoln's plane. The question asks about proving factual causation when there are multiple insufficient acts of negligence committed by a defendant. B. Incorrect. Neither one of these acts alone would have produced the injury to Lincoln's plane. The question asks about proving factual causation when there are multiple insufficient acts of negligence committed by a defendant. C. Incorrect. The factfinder can consider both insufficient acts of negligence together even though alone they would not be adequate. The question asks about proving factual causation when there are multiple insufficient acts of negligence committed by a defendant. D. Correct. See Restatement (Third) Torts § 26 and illustration 3, upon which this hypothetical is based.

Driver is driving his car at 60 miles per hour on a stretch of highway with a 50 mile per hour speed limit when he runs into Pedestrian, seriously injuring her. In her negligence action against Driver to recover for her injuries, Pedestrian alleges that Driver was negligent in exceeding the speed limit. In order to establish factual causation (also known as causation-in-fact) as part of her negligence action, what will Pedestrian need to prove? A, Driver ran into her when he was speeding. B. Driver would not have hit her or would have caused her less harm if he had not been exceeding the speed limit at the time she was hit. C. It was foreseeable to Driver that while speeding, he would run into a person walking along the highway thereby causing physical injury. D. It was foreseeable to Driver that while speeding he would run into Pedestrian, causing her injuries.

A. Incorrect. Plaintiff needs to prove not only that Driver was speeding, but that the accident would not have occurred but for Driver speeding. This question asks about establishing factual causation (also known as factual causation or but for causation) in a negligence action. It is essential to a plaintiff's negligence action that she prove that the harm about which she is complaining would not have occurred absent the negligent conduct. Restatement (Third) Torts § 26. B. Correct. This question asks about establishing factual causation (also known as factual causation or but for causation) in a negligence action. It is essential to a plaintiff's negligence action that she prove that the harm about which she is complaining would not have occurred absent the negligent conduct. Restatement (Third) Torts § 26. C. Incorrect. It addresses the standard of care (or duty) required of Driver rather than the factual causation element of Pedestrian's prima facie case. This question asks about establishing factual causation (also known as factual causation or but for causation) in a negligence action. It is essential to a plaintiff's negligence action that she prove that the harm about which she is complaining would not have occurred absent the negligent conduct. Restatement (Third) Torts § 26. D. Incorrect. It addresses the standard of care (or duty) required of Driver rather than the factual causation element of Pedestrian's prima facie case. This question asks about establishing factual causation (also known as factual causation or but for causation) in a negligence action. It is essential to a plaintiff's negligence action that she prove that the harm about which she is complaining would not have occurred absent the negligent conduct. Restatement (Third) Torts § 26.

Hoda, a diabetic, required special orthopedic shoes. Podiatrist negligently fitted Hoda with her orthopedic shoes. The improperly fitted shoes caused blistering on Hoda's feet. Hoda saw Dr. Primary about the blistering, but Dr. Primary decided nothing need be done. However, because of her diabetic condition, Hoda developed gangrene in one of her feet and had to have her leg amputated. Hoda sued Podiatrist for his negligence in fitting her with the shoes, seeking to recover damages for the amputation of her leg. Podiatrist contends that if Dr. Primary had not been negligent in failing to take prompt action to treat Hoda's blistering, Hoda would not have lost her leg and Podiatrist therefore should not be liable to Hoda for the loss of her leg. Which of the following statements is correct? A. Podiatrist should not be liable to Hoda for the loss of her leg because Dr. Primary's negligence was a superseding cause. B Podiatrist may be liable to Hoda for the loss of her leg because Dr. Primary's negligence was a foreseeable intervening event. C. If Podiatrist is liable to Hoda for the loss of her leg, then Dr. Primary cannot also be liable. D. If Dr. Primary is labile to Hoda for the loss of her leg, the Podiatrist cannot also be liable.

A. Incorrect. Podiatrist can still be liable when there is an intervening cause that results from a person's attempt to render aid. B. Correct. A person may be liable to another when their negligence was a foreseeable intervening event. This question asks about the liability of a negligent defendant when there is an intervening negligent act that contributes to the plaintiff's harm. C. Incorrect. Both physicians may be subject to liability for their negligent treatment of Hoda. D. Incorrect. Both physicians may be subject to liability for their negligent treatment of Hoda.

Art owns a convenience store located within an exclusive, gated community. Sarah, who lives in the community, tripped on a curb in the store's parking lot one night after emerging from the store, suffering a skull fracture in the fall. Sarah sues Art for negligence in failing to have lights illuminating the parking lot, which without the lights is pitch dark. If the factfinder determines that Arthur's failure to light the parking lot was negligent and a factual cause of Sarah's harm, which of the following statements about causation is correct? A. Sarah has established the causation element of the negligence action against Arthur. B. The factfinder is likely to determine that Art's negligence is not a proximate cause of Sarah's harm because the risk of someone falling in the parking lot due to darkness is not among the risks that made Art negligent for failing to provide lighting. C. The factfinder is likely to determine that Art is a proximate cause of Sarah's harm as well because the risk of someone falling in the parking lot due to darkness is among the risks that made Art negligent for failing to provide lighting. D. The court must grant judgment in favor of Sarah.

A. Incorrect. The causation component in a negligence action requires proof of proximate or legal causation as well. Based on the facts provided, Sarah has merely established the factual causation component of causation and still has to establish proximate causation for the entire causation element to be established. B. Incorrect. The risk of people falling is what makes failing to light a dark parking lot risky. C. Correct. This question asks about proximate causation determinations once the factfinder has established that the defendant's negligence was a factual cause of plaintiff's injury. The statement that the factfinder is likely to determine that Art is the proximate cause of Sarah's harm as well because the risk of someone falling in the parking lot due to darkness is among the risks that made Art negligent for failing to provide lighting. D. Incorrect. The court is not required to grant summary judgment when the factfinder has found that the defendant's negligence was a factual cause of the plaintiff's injury. Proof of proximate causation is also required.

Apple County maintained a nature park that included paths along its canyons and gorges. Park Ranger, employed by the county, became aware that a stream had dangerously eroded the support for a path, but neglected to close the path or to post warnings. A group of campers pitched camp after dark near the path. One of the campers, Charles, left the group to go on a walk. Charles fell into a gorge when the weakness in the support for the path gave way. The other members of the camping party heard what sounded like a falling object and called out to Charles, but received no response. Sean proceeded with a flashlight to investigate and discovered that Charles had fallen into the bottom of the gorge and appeared to be unconscious. In his attempt to descend to the bottom of the gorge, Sean lost his footing, fell, and suffered harm. Sean sues Apple County and Park Ranger in negligence for his injuries. Who, if anyone, may be liable to Sean for his injuries? A. Neither Apple County nor Park Ranger will be liable to Sean for his injuries because he chose to help Charles. B. Neither Apple County nor Park Ranger may be held liable to Sean for his injuries because Charles's decision to go on a walk was not foreseeable to Apple County or Park Ranger. C. Both Apple County and Park Ranger may be liable to Sean for his injuries because his rescue was a foreseeable consequence of their negligence. D Any cause of action Sean chooses to bring must be brought solely against Charles because Charles was in need of rescue.

A. Incorrect. The scope of liability of a defendant who creates peril from which a person must be rescued includes injuries to the reasonable rescuer, even if they willing choose to effectuate the rescue. This question asks about the ability of rescuers to recover from an original tortfeasor. B. Incorrect. The camper's decision does not rise to the level of a supervening event. This question asks about the ability of rescuers to recover from an original tortfeasor. In the famous words of Judge Cardozo, "danger invites rescue." C. Correct. When a person's tortious conduct imperils another the actor's liability generally includes injury suffered by the rescuer in effectuating a reasonable rescue. Restatement (Third) Torts § 32. This question asks about the ability of rescuers to recover from an original tortfeasor. In the famous words of Judge Cardozo, "danger invites rescue." D. Incorrect. The initial tortfeasor who creates the situation from which a person needs to be rescued remains liable to the rescuer. This question asks about the ability of rescuers to recover from an original tortfeasor. In the famous words of Judge Cardozo, "danger invites rescue."

Polly, a passenger on Railroad, is attempting to board a train while carrying a bulky and apparently fragile package. Bob, an employee of Railroad, attempts to assist Polly in boarding the crowded train, but does so in a careless fashion that creates a likelihood that Polly will drop the package or otherwise damage it. Polly does drop the package, which contains explosives, although there is nothing in the appearance of the package that would have so indicated. The package explodes upon impact and the force of the explosion knocks over a platform scale some 30 feet away. The scale falls on a waiting passenger, Heathcliff, injuring him. Heathcliff sues Bob and Railroad, alleging negligence. Is Heathcliff's harm within the scope of Bob's liability or Railroad's liability for negligence? A. Heathcliff's harm is within the scope of Bob's liability for his negligence because his injury is the type that made Bob's careless conduct risky. B. Heathcliff's harm is within the scope of Railroad's liability for negligence because Heathcliff was a foreseeable plaintiff as he was a passenger of Railroad on the platform. C. Heathcliff's harm is not within the scope of Railroad's liability for negligence because the risks posed by Bob's negligence were possible damage to the package, not the results of concussive forces due to an explosion. D. Heathcliff's harm is not within the scope of Railroad's liability because Railroad cannot be subject to liability for the wrongdoing of its employees.

A. Incorrect. The type of harm that make's Bob's conduct risky was harm to the apparently fragile package from it falling, not a violent concussion knocking over scales onto people on the platform. B. Correct. Heathcliff was a foreseeable plaintiff as he was a passenger of Railroad. The facts are loosely based on the famous Palsgraf opinion. Most courts now agree that the physical injuries to a passenger who was injured while standing on Railroad's platform as a result of Railroad's negligence would be foreseeable to Railroad. C. Incorrect. The risks posed by Bob's negligence were to the package, not the results of concussive forces due to an explosion. D. Incorrect. Railroad may be liable for the negligence of its employees as long as it was within the scope of their employment.

Drug Manufacturers (a total of seven companies) manufactured, produced, and marketed a medication, DRG, designed to prevent miscarriages in pregnancy. The drug was administered to pregnant women. Unfortunately, women who ingested DRG, thereby exposing their unborn daughters to DRG, gave birth to daughters who developed cancerous vaginal and cervical growths. The unique type of cancer these daughters developed was a fast-spreading and deadly disease requiring radical surgery to prevent its spread. Daughter's mother ingested DRG, exposing Daughter to the drug prior to her birth. The evidence also shows that this unique type of cancer is most probably caused by exposure to DRG. Daughter has developed the cancerous vaginal and cervical growths, which are only developed by women like her whose moms ingested DRG. Daughter has sued Drug Manufacturers, all of whom manufactured, produced, and marketed DRG at the time that Daughter's mother ingested the drug. Drug Manufacturers represent 90% of the market share of manufacturers of DRG. Daughter seeks to hold them responsible in negligence for their manufacture of DRG and Daughter's resultant injuries. However, Daughter is unable to prove with certainty which one of the seven defendants actually manufactured the DRG that her mother ingested. Drug Manufacturers have moved for summary judgment, arguing that Daughter's cause of action against them should be dismissed for failing to establish factual causation. In a jurisdiction that recognizes market share liability, should the court grant defendants' motion for summary judgment? A Yes, evidence that Drug Manufacturers produced the drug to which Daughter was exposed is insufficient to establish factual case. B. No, Drug Manufacturers produced the drug at the time that Plaintiff was exposed to DRG in utero and therefore are responsible for her injuries. C. No, as long as Defendants represent a substantial share of the market of those manufacturing DRG and at the time Daughter's mother ingested DRG, each defendant may be responsible. D. Yes, defendants cannot be subject to liability for injuries caused by a drug to which Plaintiff was exposed prior to her birth.

A. Incorrect. This question asks about factual causation problems in a market share liability jurisdiction when the plaintiff has brought before the court a substantial share of the market of defendants who were involved in the manufacture of a harmful product. Although in the ordinary case, the evidence that Drug Manufacturers possibly manufactured the DRG pill would be insufficient proof of factual causation, in a jurisdiction recognizing market share liability, the case should not be dismissed for lack of factual cause. B. Incorrect. More evidence is needed to hold Drug Manufacturers liable in negligence than just the fact that they manufactured the same drug at the relevant time. C. Correct. For an example of a market share liability jurisdiction, see Sindell v. Abbott Laboratories, 26 Cal.3d 588 (1980), in which the Supreme Court of California permitted a lawsuit to proceed against a manufacturer of a drug in the absence of proof of factual causation when the plaintiff had brought before the court defendants representing 90% of the market share. D. Incorrect. It is untrue that a manufacturer of a drug may not be held liable in negligence for exposures to its drugs that occurred before birth.

Alice was exposed to two different cleaners while working in a laboratory. Each cleaner contained a toxic chemical. One contained Chemical B; the other contained Chemical C. After developing the dreadful disease decolis several years later, Alice sues the manufacturers of each solvent, claiming that the manufacturers were negligent for including a toxic chemical in their solvents. Alice's evidence, presented by competent expert testimony based on valid scientific evidence reveals that the increased risk of contracting decolis from the dose of Chemical B to which she was exposed is insufficient to permit a finding of factual causation. Similarly, the increased risk of decolis from exposure to Chemical C is insufficient to permit a finding of factual causation. However, Alice's evidence reveals that while Chemical B and Chemical C operate independently, the combined risk of contracting decolis due to exposure to both is sufficient to permit a finding of factual causation. Which manufacturer is the factual cause of Alice's decolis? A. Neither manufacturer is a factual cause of Alice's decolis because neither was sufficient alone to cause the disease in Alice. B. Each of the manufacturers is a factual cause of Alice's decolis because the combined exposure to both was sufficient to cause her to develop the disease. C. Neither manufacturer is a factual cause of Alice's decolis because, but for Alice's exposure to both, she would have not become sick. D. Neither manufacturer is a factual cause of Alice's decolis because she has failed to satisfy her burden of production on the issue of factual causation.

A. Incorrect. This question asks about factual causation when there are two insufficient, independent causes. The rule is that when there are two insufficient, independent causes, each is a factual cause of the plaintiff's harm. B. Correct. The rule is that when there are two insufficient, independent causes, each is a factual cause. C. Incorrect. But for the negligence of both manufacturers, Alice would not have become sick. Therefore, both are a factual cause of her injury. This question asks about factual causation when there are two insufficient, independent causes. The rule is that when there are two insufficient, independent causes, each is a factual cause of the plaintiff's harm. D. Incorrect. Alice has satisfied her burden of proof on both manufacturer's factual causation.

Pedestrian was injured by a sofa that somehow fell from an upper-story hotel room during the celebration of an NCAA basketball championship. Pedestrian sues all of the occupants of the 47 rooms from which the sofa might have been thrown. All 47 defendants move for summary on the issue of factual causation. How should the court rule on the 47 defendants' motion for summary judgment based on lack of proof of factual causation? A. The court should deny the motion for summary judgment and rule that the burden of proof on the issue of factual causation is shifted to the 47 defendants to prove that each was not responsible for the falling sofa. B. The court should grant the motion for summary judgment because the burden of proof remains on the plaintiff to prove factual causation in a negligence action. C. The court should grant the motion for summary judgment because the burden shifting on the issue of factual causation is unavailable to Pedestrian without proof that each of the defendants acted tortiously. D The court should deny the motion for summary judgment because Pedestrian was injured through no fault of his own.

A. Incorrect. This question asks about shifting the burden of proof on the issue of causation of fact to the defendant in the absence of a showing that all defendants engaged in tortious behavior. See Restatement (Third) Torts § 28 and illustration 6, upon which this question is based. Without a showing that all actors who may have harmed the plaintiff acted tortiously, the rationale for invoking alternative liability is absent. B. Incorrect. The burden of proof can be shifted to the defendant when there is a showing that they all acted tortiously. This question asks about shifting the burden of proof on the issue of causation of fact to the defendant in the absence of a showing that all defendants engaged in tortious behavior. C. Correct. Each defendant must be shown to have breached the standard of care before burden shifting on the issue of causation becomes available. This question asks about shifting the burden of proof on the issue of causation of fact to the defendant in the absence of a showing that all defendants engaged in tortious behavior. D. Incorrect. The plaintiff's lack of fault has no bearing on the issue of proving factual causation. This question asks about shifting the burden of proof on the issue of causation of fact to the defendant in the absence of a showing that all defendants engaged in tortious behavior.

Toddler, a two-year-old child suffering from a bacterial infection, received a routine childhood vaccination with a vaccine manufactured by Vaco. Shortly after receiving the vaccine, Toddler spiked a very high fever, went into respiratory arrest, and died. Toddler's estate sues Vaco in negligence. At trial, Toddler's estate provides competent evidence that Toddler died due to a combination of the infection and the vaccine, and if either toddler had not had an infection or if Toddler had not been vaccinated, death would not have ensued. In its defense, Vaco presents evidence that the infection and its drug do not react together, but rather are entirely independent. Therefore, Toddler's death could only have been caused by the infection or the drug, but not a combination of both. The factfinder is persuaded by the defense's evidence and finds that the infection more likely than not caused the death of Toddler rather than the vaccine. Which of the following statements regarding the factual causation element of the negligence action is correct? A. Toddler's estate has not provided evidence on the issue of factual causation. B. Toddler's estate has satisfied its burden of persuasion that the vaccine was a factual cause of Toddler's death. C Toddler's estate has failed to satisfy its burden of persuasion that the vaccine was a factual cause of Toddler's death. D. Vaco may be held liable for the death of Toddler even without proof of factual causation.

A. Incorrect. Toddler's estate has put forth evidence of factual causation; it put on evidence that it was a combination of the vaccine and the infection. B. Incorrect. Toddler's estate has failed to persuade the jury that Toddler died because of a combination the vaccine and the infection. The jury was persuaded that Toddler's death was caused more likely than not by the infection. C. Correct. The factfinder concluded that the infection more likely than not caused the death of Toddler rather than the vaccine. Toddler's estate put on evidence from which the factfinder could have concluded that the death was caused by the vaccine, so Toddler's estate satisfied its burden of production. This question is more about the burden of persuasion rather than the burden of production. Had Toddler's estate satisfied its burden of persuasion, the factfinder would have found that the vaccine more likely than not caused the death of Toddler, but the factfinder did not. D. Incorrect. Proof of factual causation is critical to all negligence actions. The case against Vaco has failed for lack of proof of factual causation when the factfinder determines that it was the infection and not the vaccine that more likely than not lead to Toddler's death.

Alice, Bill, and Charles, each acting independently but simultaneously, negligently lean on Polly's car, which is parked at an overlook at the edge of a mountain. Their combined force results in Polly's car rolling over the edge of the cliff and plummeting down the mountain to its destruction. The force exerted by either Alice, Bill, or Charles would have been insufficient alone to propel Polly's car over the edge, but the combined force of any two of them would have been sufficient. In a lawsuit brought by Polly against Alice, Bill, and Charles based on negligence, who is the factual cause of the destruction of Polly's car? A. Neither Alice, Bill, nor Charles is a factual cause of the destruction of Polly's car because the force exerted by each of them alone would have been insufficient to move Polly's car down the mountain. B. Neither Alice, Bill, nor Charles is a factual cause of the destruction of Polly's car because the combined force of any two of them would have been sufficient. C. Two and only two of Alice, Bill, and Charles are a factual cause of the destruction of Polly's car because the combined force of any two of them would have been sufficient. D. Alice, Bill, and Charles are each a factual cause of the destruction of Polly's car because their combined force resulted in the destruction Polly's car.

A. Incorrect. When multiple acts combine, each which alone would have been insufficient, each is a factual cause of the harm. B. Incorrect. The fact that only two of them would have been sufficient does not preclude a finding of factual causation of each of them. See Restatement (Third) Torts § 27 and illustration 3, upon which this hypothetical is based. C. Incorrect. The fact that only two of them would have been sufficient does not preclude a finding of factual causation of each of them. This question asks about multiple, independent, insufficient causes of a harm. D. Correct. When multiple acts combine, each which alone would have been insufficient, each is a factual cause of the harm. Therefore, Alice, Bill, and Charles are each a factual cause of the destruction of Polly's car.

Late one night, Pete places his loaded pistol on the coffee table in the family room of his home that he shares with his wife and two young children. The next afternoon Pete's 10-year-old son, Nolan, is spending time in the family room with his friend, Buddy. Nolan picks up the gun and plays with it, accidentally causing it to discharge. Buddy is injured by the gunshot. In a negligence action against Pete brought on behalf of Buddy, can Pete be held liable for negligence? A. Pete cannot be held liable in negligence for injuries to Buddy because Pete never intended for Buddy or anyone else to be injured. B. Incorrect. Nolan's fault in causing the gun to discharge does not shield Pete from liability for his previous negligent act. This question asks about conduct that is negligent because of the prospect of improper conduct by another person. The rule is that conduct of the defendant can lack reasonable care if it foreseeably combines with improper conduct of another. Restatement (Third) Torts § 19. C. Pete may be held liable in negligence because parents are liable for injuries caused by their children. D. Pete may be held liable in negligence because Pete carelessly provided Nolan access to the gun in circumstances in which it was foreseeable that Nolan might use it improperly.

A. Pete cannot be held liable in negligence for injuries to Buddy because Pete never intended for Buddy or anyone else to be injured. B. Incorrect. Nolan's fault in causing the gun to discharge does not shield Pete from liability for his previous negligent act. This question asks about conduct that is negligent because of the prospect of improper conduct by another person. The rule is that conduct of the defendant can lack reasonable care if it foreseeably combines with improper conduct of another. Restatement (Third) Torts § 19. C. Incorrect. It is not true that parents are always liable for the injuries caused by their children. This question asks about conduct that is negligent because of the prospect of improper conduct by another person. The rule is that conduct of the defendant can lack reasonable care if it foreseeably combines with improper conduct of another. Restatement (Third) Torts § 19. D. Correct. When Pete carelessly leaves his gun accessible to his kid, it is foreseeable that one of his kids may misuse it thereby causing an injury. This question asks about conduct that is negligent because of the prospect of improper conduct by another person. The rule is that conduct of the defendant can lack reasonable care if it foreseeably combines with improper conduct of another. Restatement (Third) Torts § 19.

to prove a personal injury negligence, you HAVE to have both proximate causation and cause in fact.

A. Proximate cause is a policy determination. It's just a concept that the court uses to cut off cause in fact, to limit cause in fact. Cause in fact has been est, accident wouldn't have occurred if D's acts didn't occur.

k relationship with professional malpractice:

A. There must be a k before torts liabilities can be attached. Tort duties come into play only when a prof has explicitly or implicitly entered into a k with you, accepted you as a client/patient. 1. Doesn't have to be written - can accept you via their actions. When they accept, tort duties attach as a function of law. 2. To have a k claim, must show doc made a specific promise or produce a certain result. Breach of contract - promise a certain result or treatment, don't provide it. can't sue for breach of k if it's just a therapeudic assurance ("I'm going to make your nose better"). 3. What we know: a. Needs to be a doc/prof client/patient relationship that exists b4 tort duty can be imposed. We also know the 3 tort duties that are the duty of a prof (if there's a deficiency in any of these, that's the basis for a tort lawsuit on malpractice).

If a P wants to use a criminal statute, must convince judge that P's in the class of people the statute was intended to protect and that P's injury is a type it was designed to prevent - If P does prove those things, instead of using reasonable prudent person standard judge will ask if D did or did not do what the statute says.

A. agency) B. P attorney says 'D violated x statute, judge I want you to take it and incorporate it into the duty." If adopted, judge just asks jury to decide if D followed statute. Breach = when statute's broken. 1. Jury has less discretion.

only state that's kept its automobile guest statute:

AL

McGuire v. Almy - nurse attacked by mental patient. Assault and battery. An insane person is liable for his torts. Here, says that where an insane person does intentional damage to the person or property of another he's liable in the same circumstances in which a normal person would be liable. This means that in so far as a particular intent would be necessary to render a normal person liable. The insane person, to be liable, must have been capable of entertaining that same intent and must have entertained it in fact. Here, the jury could find that D was capable of entertaining and did entertain an intent to strike and injure P and that she acted upon that intent.

Agreement in US is basically unanimous: mentally disabled persons can be held responsible for their intentional torts as longa s P can prove that they formed the requisite intent. Several js make an exception to general rule: institutionalized insane people who can't appreciate the consequences of their conduct aren't liable for injuries to those employed for their care. Mental illness may prevent the specific kind of intent necessary for certain torts - ex, for slander, requires proof that D knew that statement was false. A negligent supervision action may lie against those responsible for insane person, but only if they have assumed a caretaking responsibility - familial rel is not enough. Voluntary intoxication, in comparison, does not negate intent.

Negligent entrustment: all of the vicarious liability circumstances should be carefully distinguished from cases in which the P is arguing that the employer/bailor was itself negligent in some way.

Aka, may be negligent in entrusting the chattel to the bailee. Ex: you know a driver is freqneutly drunk, but still lend him your car. Some states recognized the "first party" negligent entrustment actions that permit the driver entrustee who should not have been entrusted with the car to make a claim against the owner entruster.

Rogers v. Board of Road Com'rs for Kent County: "Failure to remove a thing placed on the land pursuant to a license or other privilege, section 160: will be trespass."

Aka, trespass can occur when one "overstays" their welcome. Note, though: If you're a public place, can't prevent entry bc of discrimination

"mistake" is not a defense for trespass, either for land or chattel. Ex: the man who shot the dog thinking it's a wolf committed a trespass of chattel - his mistake was not a valid defense.

Another ex: you may think that you're on your mom's land; if it's not, the mistake won't protect you, bc it'll still be a trespass

Family car doctrine: family member who owns the car will be vicariously liable for the person who is using the car for a family purpose (like going to grocery store, movies, do something else).

Another theory that's closely related: if somebody is driving your car while you're in the car, you're going to be liable for the driver's negligence. Most courts have gotten away from this theory.

Garratt v. Dailey: question is if kid committed a battery. What's battery:

Any harmful or offensive touching. The touching doesn't have to just be harmful, offensive is ok - ex of offensive: offensive to a reasonable prudent person; if toucher knows person is super sensitive, though, that's still battery; if he doesn't know, not battery.

What if in wrongful death claim an unborn kid is the one that's killed?

Applies diff in diff states. Many states won't let an unborn kid be such that the parents can sue the tortfeasor for kid's death. Normally the kid must be born before parents can bring wrongful death claim. Some states say if the kid is violable when killed, can bring claim. Minority of states say even if the kid isn't viable, can bring claim. It just varies from state to state. If it's a child that dies, most statutes will allow parents to be in the priority list. But when they do come up in the ranks, they need to not only show they were the parents of the decedent but also receiving some financial benefit from decedent.

Borders v. Roseberry: In common law, one rule adopted was that if one leases to someone else, the leasee assumes a quasi-ownership of the property, and they therefore, if anyone owes a duty to a guest, it would be them and not the leasor. Aka, when you give up land, you give up the liability to be sued. This rule has been translated into restatement, and there are exceptions!

Apply to us: once people who own your apartment, they're no longer liable to you or your guest bc of some dangerous condition on the property. p guest can sue the tenant, but not the landlord. Exceptions: 1. If landlord knows there's a dangerous condition on the property when they rent it but leasee doesn't know it. But when tenant knows, now they become responsible for it. 2. Condition that's on the premise when it was leased is dangerous to people outside the premises - aka a tree, running water, anything that at the time of the lease was dangerous to people outside of the property 3. Premises leased for admission to the public - if landlord leases land that has dangerous condition on them at the time of the lease and the land is leased to the public, landlord is liable 4. If landlord keeps control of common areas, will be liable for any injury that happens from a dangerous condition in those common areas. 5. If the landlord contracts to repair, and they don't attempt to repair, liable for not repairing. 6. If landlord says they'll repair, and negligently do it.

As bar owner was closing up for the night adult drunk patron walked into the bar and asked for a drink. Bar owner apologized and said that he was closing up and would not be serving anyone else. Drunk patron then told Bar Owner that if he would serve him three more drinks, Bar Owner could have his antique car which was parked in the parking lot of the bar. Bar owner walked outside looked at the car and agreed. After the third drink Bar Owner led drunk patron outside of the bar, locked up and then Bar owner drove away in drunk patron's car. If bar tender was reasonably unaware that bar patron was drunk during this period, what result if drunk patron subsequently sues Bar owner for conversion? A Drunk patron will prevail if he was so drunk he lacked the capacity to actually consent to Bar owner taking his car. B Drunk patron will not prevail even if he was so drunk he lacked the capacity to consent to Bar owner taking his car. C Bar owner will prevail even if Drunk patron was so drunk he lacked the capacity to consent because incapacity does not negate consent. D Bar owner will prevail even if Drunk patron was so drunk he lacked the capacity to consent because only the incapacity of minority can negate consent.

As bar owner was closing up for the night adult drunk patron walked into the bar and asked for a drink. Bar owner apologized and said that he was closing up and would not be serving anyone else. Drunk patron then told Bar Owner that if he would serve him three more drinks, Bar Owner could have his antique car which was parked in the parking lot of the bar. Bar owner walked outside looked at the car and agreed. After the third drink Bar Owner led drunk patron outside of the bar, locked up and then Bar owner drove away in drunk patron's car. If bar tender was reasonably unaware that bar patron was drunk during this period, what result if drunk patron subsequently sues Bar owner for conversion? A Incorrect. When a person lacks the capacity to give consent and the person to whom the consent is given should reasonably be aware of the incapacity there is not valid consent. Here however the person to whom consent was given was reasonably unaware of the Drunk patron's incapacity. When the person to whom consent is given is reasonably unaware that the consenting person did not have the capacity to consent the consent is valid even though the consenting person actually lacked the capacity to consent. B Correct. When a person lacks the capacity to give consent and the person to whom the consent is given should reasonably be aware of the incapacity there is not valid consent. Here however the person to whom consent was given was reasonably unaware of the Drunk patron's incapacity. When the person to whom consent is given is reasonably unaware that the consenting person did not have the capacity to consent the consent is valid even though the consenting person actually lacked the capacity to consent. C Incorrect. It is based on overbroad rule. Incapacity does negate consent in many instances. D Incorrect. There are many incapacities which can negate consent apart from minority.

punitive damages ex - State Farm Mutual Automobile Ins. Co. v. Campbell

Asking whether having 145mill in punitive damages, and just 1mill in compensatory damages, is unconstitutional This j gives courts reviewing punitive damages three guideposts to consider: 1. The degree of reprehensibility of the D's misconduct; 2. The disparity between the actual or potential harm suffered by the P and the punitive damages award; 3. The diff between the punitive damages awarded by the jury and the civil penalties authorized or imposed in comparable cases. Here, the damage were meant to comment on State Farm's nationwide policies. "States does not have a legit concern in imposing punitive damages to punish a d D for unlawful acts committed outside of the stat'e j." Also: courts awarded punitive damages to punish and deter conduct that bore no relation to the Campbells' harm.

Statutes of limitations and repose ex: Teeters v. Currey

Asks whether the statute of limitations begins to run from the date of the injury or from the date of the discovery of the injury P sued three years, five months and nine days after the operation, but approximately eleven months after discovering her pregnancy. Statute says suit must be brought within a year after the cause of action. Our question: when does the cause of action accrue

If the decedent was also neg, either contributorily or comparative, then those defenses will be used against him when his estate brings his claim for survival. In the wrongful death claim, which is derivative of survival claim, his neg will be used against him there, too.

Assumption of the risk can also be used against him. these defenses apply to both wrongful death and survival. But in wrongful death claim, the beneficiaries' neg also applies, can be used to lower award. Ex if they were apart of the accident, her part of it will reduce her recovery.

Borders v. Roseberry: trad common law rule concerning owners who lease their land to another, to the tenant. Issue: what duty does the owner owe to the tenant and the tenant's guest.

At common law, which still exists in most states, no duty is owed by the landlord or the owner unless the state has adopted the six restatement exceptions discussed here.

Breakdown of Bierczynski v. Rogers:

B was sued, even though his car did not hit the P. why was he held responsible for the entire damage: both drivers had a duty, both breached it. were both drivers a cause in fact: substantial factor or but for test. Both were substantial factors - egging each other one, engaged in the race. But for them racing, the accident would not have occurred. So both are a cause in fact of the accident. • B is held liable for the entire accident. He was acting in concert. If you negligently act in concern with someone else, and you both cause in fact an injury, you will b ejonitly and severally liable for that injury - means that each pays the whole. P can sue one or both, she can collect all of it from one of she can collect half from each, or she can collect x percentage from one, y percent from the other. P has the ball in her court.

(Statute of limitations) Sometimes the statute is tolled, stops running - esp for minors. If you're a minor that suffers med malpractice, statute is tolled til you get to your majority.

BUT sometimes, in some js, even if you're a minor, state will cap the outer limits - say you can wait til you reach maj, but this waiting cannot exceed more than seven years after the accident. Aka, whether you're a minor or not, have seven years. That's helping docs. •One year statute of limitations, in this j, is a way they're helping docs

Another way to find one person vicariously liable: Bailment.

Bailment implies a person who owns property allowing someone else to use that property. In Malchose, the bailment is further divided into the Family Car Doctrine. Parents basically allow the son the exclusive use of the car while he's at college. Question is who owns the car. Court here said parents still own it bc they have title to the car.

Every intentional torts: has intent + something else. Two elements to intent. Add the elements of the other thing as well. Exs:

Battery under RS: offensive or harmful touching Assault: to place someone in an imminent apprehension of an imminent battery.

The discovery rule, first applied in med malpractice cases has been extended in many js to apply to any action based on a latent injury.

Bc fixing the time, under discovery rule, that P "should have discovered" something usually is dependent on fact finding by the jury, the discovery rule makes it unlikely that the D can obtain summary judgment based on the statute of limitations.

a minor's comparative negligence can be assigned against them and reduce their recovery by the percentage fault that they contributed to their own damages.

Bt in some situations of a minor, a minor's negligence won't be used, like statutory rape. Also, if a prisoner commits suicide, prisoner's neg won't be used against a guard that allowed him to commit suicide. Aka, special categories where a person's neg won't be used against them - minor in sex abuse, prisoner commits suicide bc guard didn't use reasonable care to prevent it.

The only caveat to the rescuer doctrine is that he must not be negligent, reckless in rescuing. He likes 'reckless' better. You may well be negligent in an emergency situation, if they're panicked and pulling on your neck.

But at some point, the action of the rescuer can cut off the liability of the D. in that case, the rescuer becomes a superceding, intervening cause of his own negligence if he acts recklessly

In Kelly, the host served liquor to a visibly intoxicated guest in their home. Guest drives home, injures somebody. Injured person sues the host, saying he was negligent when he served driver alcohol. Direct cause: but for you serving him liquor, no accident. Proximate cause: foreseeability test is est. foreseeable that if you make somebody more drunk, and they drive away, they'll hurt someone.

But court says we don't care if foreseeable, direct cause tests are est. for public policy reasons, we think this host shouldn't be liable. Aka, social host owes no duty to the P on the road; alt: a duty is owed, but no liability bc no proximate cause. COULD analyze under Palsgraf duty analysis, but here just proximate cause.

Legislative control of the amount recovered: sometimes the legislature, as a separate branch of gov, will dictate stuff about the amounts of damages. One way the legislature, state house and state senate or fed gov, will try to cap certain damages, is capping non-economic damages. Normally they won't try to cap economic damages bc are normally the amount that P really needs to replace wages, med expenses, etc. so normally she gets as much eco damages as jury awards.

But with pain and suffering, many js cap the amount of money the p can get for all pain and suffering. Some will cap non-med harm in med negligence cases at 500k. this is done through tort reform (tort reform caused by a group, like docs, argue that med malpractice insurance is too expensive, is too expensive bc too many frivolous lawsuits. To deal with that complain, legislature in many states says we'll help you out, cap pain and suffering damages. If we cap, fewer lawyers will take these cases, fewer frivolous lawsuits, the premiums will go down. When they do that, try to cap, usually the state SC will hold that the statute that tried to cap those damages are unconstitutional as a violation of equal protection. (only one state puts a limit on ALL damage in a med case: VA)

Ex of intent in an intentional tort: Garratt v. Dailey

Can an infant be liable for an alleged battery? Pulled chair, allegedly deliberately, out from under P. Battery def: the intentional infliction of a harmful bodily contact upon another. A battery would be est if it was proven that when the kid moved the chair he knew with substantial certainty that the P would attempt to sit down where the chair had been. Without such knowledge, there'd be nothing wrongful about his act in moving the chair, and no liability. CONVERSELY: if the court finds that the kid had such knowledge the necessary intent will be est, and the P will be entitled to recover, even though there wasn't a purpose to injure or embarrass the P. On remand of this case, time frame was considered - kid had quickly moved the chair and sat in it and he knew, with substantial certainty, that at the time she would attempt to sit in the place where the chair had been.

Note 17: normally neither contributory or comparative neg can be used as a defense by a D that committed an intentional tort.

Can't use P's negligence against her to reduce their recovery.

Bailments: Malchose v. Kalfell

Car wreck, P sues parents of D, who owned his car. The family car doctrine places liability on the owner of a vehicle for negligent operation by a person using the vehicle with the express or implied consent of the owner for purposes of the business or pleasure of the owner's family. Owner isn't liable for his own negligence; he's vicariously liable for the tortious acts of the driver. To be liable, the head of the household need not own the vehicle, but must furnish it for the use, pleasure, and business of himself or a member of the family. Here, the doctrine applied. Under the common law in most states, a bailment doesn't make a bailor vicariously liable for the acts of the bailee in the use of the chattel. The common law rule on bailments has ben altered in many states by decision or by statues, esp in respect to autos.

The fact that you acted in good faith is irrelevant to conversion. If you misdeliver it, still a converter. If thief sells your ring, can sue both seller and buyer for conversion. Few exceptions when good faith does matter in conversion:

Certain eco transactions where one is allowed to receive stolen goods without having to pay the fmv; aka, people who run garages aren't liable for taking a stolen car. Another situation where one not liable for conversion: where one in good faith takes possession of a property - ex, you know there's a Van Gough at a gargage sale, person sells it to you for 25; if you tell them you didn't know, and you did, ....if they sell if afterwards, can't sue the next buyer because the lying person obtained good title, but voidadble, and remains voidable unless a good faith purchaser buys it, and then it turns into a good faith title. Third situation: where one deals in goods of the kind - ex, if you take your bike to a shop for repair, while they have it they mistakenly sell it, they deal in goods of the kind, and when somebody buys it, they bought it in good faith, buyer won't be held liable for conversion. Can only sue the bike shop. If a good faith purchaser buys a converted good from someone, and can't get a title bc were stolen, they're a converter. However, if the title is obtained by fraud (aka, buying Leonardo at a garage sale), the right to rescind fo fraud is terminated when the good faith purchaser acquires both title and possession. there is thus a distinction between stolen goods, as to which the legal title doesn't pass, and a bona fide purchaser is therefore not protected, and goods obtained by fraud, as to which the title does pass. UCC protects a bona fide purchaser who buys from a merchant.

Champerty v. Maintenance:

Champerty is an agreement whereby a person with no interest in a lawsuit agrees to aid in or carry on its litigation in consideration of a share of the proceeds. 'Maintenance' is meddling n a suit by assisting either party with means to carry it on. both were illegal at common law.

Montgomery Ward & Co., Inc. v. Anderson: D moved to prohibit P from presenting the total amount she was billed as proof of her medical expenses, asked that her evidence be limited to the actual amount she'd be responsible. P said she'd reached an agreement with hospital that the bill would be discounted by half. P asserted that the collateral-source rule would prohibit D from introducing evidence of the discount.

Collateral source rule: a trial court must "exclude evidence of payments received by an injured party from sources 'collateral' to the wrongdoer, such as private insurance or government benefits." • A way to restate this rule: tortfeasor's responsibility to compensate for all harm that he causes, not just the net loss that the injured party receives.

discuss comparative negligence in the light of Coney:

Comparative negligence means that if you have the manufacturer as a player, the employee, and the employer, and all three is negligent, then in a comparative negligence j, then jury should attribute a percentage fault to manufacturer, employer, employee. Comparative negligence: the possible Ds can only be liable for their percentage fault share, which the jury will determine. BUT: some js, even though they have decided to use comparative negligence, will say that that's a diff issue than j and s liability. Even though manufacturer is x percent at fault, manufacturer is y percent at fault, emp can't get all of it, but she can get the either percent from either of them if they are j and s liable - can sue one for the whole amount. Manufacturer says if you have gone to a state where the rule is comparative neg, there should be no more j and s liability, even if we created an individisble injury. Since j and s liability was created as a quid pro quo for people subject to contributory negligence, once you take away contributory negligence and use contributory negligence, so should j and s, the scheme becomes several liability. If you do away with P's contributory negligence and go to comparative neg, should get away with j and s liability. This court says no - if they're joint tortfeasors acting together to make an indivisible injury, can collect all from just one of them.

Ex of cases that detail consent:

Consent: O'Brien case - lady sees others get a vaccination. Court said she consented by silent to getting the vaccine. Ex, go into doc office, doc examines, you consented by just being there an not objecting. Case top of p109: foreign guy asks not to amputate his hand, miscommunication tween him and doc, man won - in language barrier cases, doc needs to do more, consent through silent not ok. Hackbart case: just bc you play sports, you do not consent to being fouled, hit - although it occurs, you don't consent. You DO consent to the normal contact that's implicit in the rules of the game when played appropriately. What about a course of roughhouse practical joking between the parties in the past: jury decides if the parties had been perpetuating practical jokes in a way that the D could believe P would accept the act. Williams, p111: deals with this concept: the lady went to doc with problem with ear, consents to operation on one ear, but when he's operating on right ear realizes nothing's wrong with it, finds out left ear is diseased, so commits the operation on the left ear. He didn't have consent for that. If he had operated on the rt ear and committed malpractice, the cause of action would be negligence. However, in operating where he didn't have consent, he committed a battery - not harmful contact, but was offensive. What some courts will do is that if he's helped you, will subtract the detriment from the benefit to give you the overage, if there is any, for ds. Consent to operate on one part of the body is not consent to operate on another part of the body. Also get that if you get a benefit from the med battery, your ds may be the diff tween the advantage and the detriment. De May v. Roberts, p117: consent given in false pretenses is not valid. She was under the impression he was a doc. He mislead her. He had a duty to disclose it to her. She consented, but she can withdraw her consent after the fact - if she does that, then there's a battery.

nominal damages:

Consist of a small sum of money awarded to P to vindicate rights, make the judgment available as a matter of record to prevent the D from acquiring prescriptive rights; the amount of the award, so long as it is trivial, is unimportant

expenses of litigation: everybody pays their own attorney unless there's a statute that allows for attorney's fees for the winning party, usually the P. normally, most personal injury lawyers are paid on contingency or by an hourly rate.

Contingency fees can be 33%, half; some js will cap them. • Contingency fee: arrangement that P attorney has with client - if you win, collect, we will get a certain percentage. P also has to pay deposition fees, cost of experts, in ADDITION to the contingency fee. • P attorneys normally do contingency fees. Hourly fee is normally paid by the D, and through insurance cos.

if Ds are intentional tortfeasors, are punished by not being able to get contribution from each other

Contribution only applies to negligence. If one pays more than his share, can't get contribution from the other.

introduction to "defenses:" Contributory negligence:

Contributory neg says yes D was negligent but so was P. Historically meant that both were negligent, both were a cause in fact and proximate cause of the accident. Both breached their duty. In pure contributory negligence j, if P is just 1% negligent, can't collect at all from D. only about 4 states currently have this rule. AL is one of them (to ameliorate: P gets to use j and s, last clear change (if both P and D were negligent, if D had the last clear chance to avoid an accident, when P no longer had that chance, he cannot use P's contributory negligence against her to destroy her recovery.) in pure contributory state, if D has clear chance, cannot use P's negligence to avoid recovery to the P - has to pay 100% of P's damages.)

Campbell v. Weathers D's defense: he didn't owe a duty bc P wasn't an invitee bc he hadn't bought anything in the store. What's an invitee: someone explicitly invited on property for purposes of the business. Here, was a café, shop; invitation was to come in and buy something. Invitation is for the purpose to come in and buy something. But this person didn't buy anything. Bc he didn't buy anything, D says that when you were injured, we didn't owe you a duty. D tried to say that he was a trespasser.

Court says dude was an invitee. If you go into a place with the intent to browse, the current intent to buy something, with the possibility that you will buy something, that's the clearest form of an invitee. Court says just bc they're browsing, you wouldn't be mad if he came back later and bought something. if one comes in with intent to buy something now, or later, is invitee. If someone comes in with intent to just lollygag or loiter, perhaps he is a trespasser bc he has no intent or future intent to buy anything. Then perhaps would be a trespasser - depends on his intent; if has no intent to buy anything, becomes a licensee. This case emphasizes that one's status can change on the premises - can be an invitee on one part of the premise, but if he leaves, he becomes either a trespasser or a licensee.

Joint enterprise: Popejoy v. Steinle

D had a crash with P. D died, then her husband. P tried to sue husband's estate, stating that the D and husband were engaged in a joint venture when D went on her trip. P's claim is premised on idea that husband and D were engaged in securing an appreciable business asset for their family business at the time of the accident Daughter testified her dadhad no ownership interest in the cows she and mom raised. Daughter was the only one who had an actual financial interest in the sale of the cow that was going to be bought at the time of the accident. Note the specific nature of the inquiry on joint enterprise here: testimony from witnesses, docs reflection the family's financial dealings, expert testimony from CPAs Their theory as a means of imposing vicarious liability is ost commonly applied in car accident cases.

defenses based on P's conduct: contributory negligence. contributory negligence ex: Butterfield v. Forrester

D had put up pole in the road. P was riding his horse violently, rode into it; if hadn't been riding hard, might have seen and avoided it. • Court completely barred P's claim - this approach to how to deal with the situation in which P's negligence contributed to his or her accident is called contributory negligence

regarding (damages) harm to property:

D has to pay for repairs or replacement or fair market value, depending on how badly the damage is Stocks and damages, things that fluctuate: if D does something that means P can't sell when at the highest point on the market, damages may be the highest intermediate value, not the average point.

Taylor v. Vallelunga: P had emotional distress from watching her dad be beaten. No physical disability or injury resulted from the distress. Could not prove intentionality of the infliction of distress - did not know she could see them, that she would be traumatized from it. Generally, committing murder or suicide isn't a tort against an eyewitness; but may may be if the act is directed at the P or if D knew that extreme emotional distress was substantially certain to follow Regarding intent: many js require that the conduct be directed at the P or take place in the presence of the P, with the D's awareness. The restatement section 46(2) would allow recovery if:

D knows of bystander's presence AND 1. The conduct was directed at a member of bstander's immediate family or 2. Bystander suffers bodily harm as a result of her distress. HOWEVER: Some js have permitted recovery even though P wasn't present.

Talmage v. Smith

D's conduct was directed towards one kid, but it hit another. His intent was transferred. That transfer was sufficient to est battery. Doctrine of Transferred Intent. This doctrine only applies to certain intentional torts: battery, assault, false imprisonment, trespass to land, and trespass to chattels. Must be one of these five.

litigation financing: while a litigation is pending, some Ps will get money from cos related to the anticipated payment of a judgment.

Depending on the agreement, it may be characterized as an assignment, a loan, or as an investment in the suit. whether the state will permit enforcement of such an agreement varies by the wording of the agreement and state law.

Talmage v. Smith D threw stick at boys on shed. Question: did he intend to hit them or scare them. "under these circumstances, the fact that the injury resulted to another than was intended doesn't relieve the D from responsibility." Ex of Doctrine of Transferred Intent.

Doctrine of transferred intent: where D tries to hit A but instead hits B. Thus D is liable when he shoots to frighten A (assault) and the bullet unforeseeably hits a stranger (battery).

a. Facts: P was D's tenant; was sliding the glass door of his tub open to get out. P apparently assumed it was of tempered safety glass. The glass shattered, P was injured. b. Issue: Is common practice enough to create a duty? c. Rule: common practice not necessarily a compelling test of negligence. d. Analysis: Was for jury to decide if this common practice had transformed tub into a part of the apartment that could no longer be considered safe. 1. Takeaway: custom to do something doesn't dispositively est that someone's negligent for not following the custom. 2. Following custom is NOT dispositive. What's usually done may be evidence of what ought to be done, but what to be done is fixed by a standard of reasonable prudence, whether it's usually complied with Doing what people commonly do by way of custom may mean that you're not negligent, but it's not dispositive. Issue's ALWAYS what would a reasonable person do - that may or may not mean following a custom.

Does Common practice institute a duty - Trimarco v. Klein

rejection of categories of trespasser, invitee, leasee: Rowland v. Christian.

Doesn't matter if person is licensee or invitee, same duty is owed to them. Court says no need to treat people with diff statuses differently. Some even include trespassers in this reasoning (except for criminal trespassers). • Reason why court has merged these categories: bc people normally don't clean up based upon your status. • Many states have said that the same duty is owed to anyone, regardless of your supposedly status (only exception is the criminal trespasser).

in patient-oriented standard in informed consent: you _____ need an expert to tell you what facts are material under this standard.

Don't. jury will determine this. Under physician standard, you need an expert to tell you what risks a reasonable prudent doc would disclose in that jurisdiction. Now in a patient jurisdiction, there are some things you still need an expert for: to tell you risks, how severe risk is (3, 6% etc), but once they testify, then jury says 'we think that is/isn't material." Benefits of patient standard: patient should have final say over their body.

Davies v. Mann

Donkey is in the road. Later commentators rationalized the holding here that what has come to be known as the doctrine of last clear chance - the idea is that if the D had the opportunity to avoid the accident after the opportunity was no longer available to the P, the D is the one who should bear the loss. Some js have restricted its use to cases where P was merely inattentive to danger; others allow its use if the D should have discovered that the P was helpless. • The last clear chance doctrine has been called a "transitional doctrine," a way station on the road to comparative negligence.

Ranson v. Kitmer:

Ds killed P's dog bc thought it was a wolf; was a mistake, paid the dog's value

In tree-on-road case, the duty is for landowner to use reasonable care to prevent its trees from falling onto a public highway: question is - did they use reasonable care? What did the case say about whether this tree owner did/didn't use reasonable care?

Duty was to use reasonable care. Reasonable care is defined by doing what a reasonable prudent person would do in same or similar circumstance. Question is: what would a reasonable prudent person have done, or should have done to discover that the tree was rotten? Inspect the tree. What does inspecting the tree entail? Reasonable prudent person would not bore. Not discussed in case, but might have a duty to inspect his tree. Would that mean just driving by tree, looking for dead branches? This will be a jury question. Issue: what would reasonable prudent person do tot inspect the tree. "self-help" rule: if your neighbor's tree is hanging over into your land, your property line, in some js you can cut it off. Need to be careful - gotta live with your neighbor.

Ellie, who was employed as an asbestos insulation installer for many years, is killed when a defect in a ground-fault interrupter (GFI) in her bathroom fails. A GFI is designed to protect a person from a dangerous or lethal electrical shock hazard by breaking the electrical current if the electrical device comes into significant contact with water. The GFI in Ellie's bathroom failed to break the electrical circuit after an electrical razor fell in the bathtub, resulting in Ellie's electrocution and death. The autopsy reveals that Ellie died from the electrocution. It also shows that she was in the early stages of mesothelioma, which would have killed her eventually. Mesothelioma is a rare and almost invariably fatal form of cancer that studies have established is caused only by exposure to asbestos. In a lawsuit against the manufacturer of the GFI, what is the factual cause of Ellie's death? A. The defect in the GFI is not a factual cause of Ellie's death because she was suffering from a fatal form of cancer at the time and would have died from it eventually. B. The defect in the GFI is a factual cause of Ellie's death. C. The defect in the GFI and the asbestos in the insulation were both a factual cause of Ellie's death. D. Neither the GFI nor the asbestos in the insulation was a factual cause of Ellie's death.

Ellie, who was employed as an asbestos insulation installer for many years, is killed when a defect in a ground-fault interrupter (GFI) in her bathroom fails. A GFI is designed to protect a person from a dangerous or lethal electrical shock hazard by breaking the electrical current if the electrical device comes into significant contact with water. The GFI in Ellie's A. Incorrect. The autopsy evidence established that she was killed by electrocution, not by the mesothelioma. B. Correct. For the GFI defect, Ellie would not have died from electrocution when she did. C. Incorrect. There is not evidence that she died from a combination of the cancer and the electrocution. D. Incorrect. The evidence establishes that, but for the electrocution, Ellie would not have died at that time. See Restatement (Third) Torts § 27 and illustration 6, upon which this hypothetical is based.

i. Facts: D employed a chauffeur. Robber got in D's taxi, told D to keep going or he'd blow his brains out. D put car outta 1st gear, pulled emergency brake, jumped out while motor was running. Car kept moving, ended up slightly hurting P. ii. Issue: was driver negligent by fleeing when his life was threatened iii. Rule: "the law...doesn't hold one in an emergency to the level of judgment required of him under circumstances where he has an opportunity for deliberate action...he's not required to exercise the unerring judgment which'd be expected of him if he weren't confronted with an emergency requiring prompt action." iv. Analysis: in this case, the driver's life was being threatened, he had a short time to act. It's actually just remarkable he had the presence of mind to put emergency brake on. He acted in a split second in a bad circumstance, cannot be held liable.

Emergency standard of care ex - Cordas v. Peerless Transportation Co.

for proximate cause, different js use diff tests.

Even in just one j, may use one test for one thing and another test for another thing

Regarding statute of limitations - cause of action accrues when you discover SOME injury, not necessarily the full manifestation of the injury.

Ex, woman with an IUD developed PID was told IUD caused it - she should've gone to lawyer then and said I need to sue, but bc just an infection, she ignored it, but she didn't know those types of infections can lead to infertility. NOW she wants to sue, but her statue of limitations has run bc she knew years beforehand that the IUD was causing some problem. Shouldn't have waited til the full mainfestationof the disease appeared years later. Ex asbestos cases: you know asbestos is causing your lung problems, but what if you let that statute of limitations run and don't bring suit until years later when you discover you have cancer? If discovery rule applies, will apply when you know or should have know you had cancer, not that you had asbestos. They're two separate conditions, each have their own statute of limitations.

Punitive damages ex - Cheatham v. Pohle

Ex-husband committed revenge porn. A claim for compensatory damages can be sustained only if it is accompanied by a viable claim for compensatory damages. •Intentional torts: punitive ds are genearlly permitted when the D has committed an intentional tort.

Another ex that shows one's status can change on the premises - Whelan.

Ex: Whelan. Some man goes to get a box from back of store after he'd made his purchase. He was a licensee - had their consent. They weren't liable - he was a licensee, and their duty was only to warn of latent defects. Here, they didn't know that the bulb had shot, so not liable. Status can change depending on when the injury occurred

Hardy v. LaBelle: says that to confine someone against their will, you need to do it by locking the door, threatening them with a gun, threatening a family member with a gun, but if you just PERSUADE them to go to a back room, say that if you don't come I'll call the police, they get to decide. If you say come or I'll fire you, they can leave. False imprisonment means that you have NO CHOICE. Just a threat of negative consequence doesn't take away your free will.

Ex: fear of losing one's job is a powerful incentive, but it doesn't render behavior involuntary. Not the type of physical restraint that would warranty the conclusion that you've been falsely imprisoned. You still have a choice.

• O'shea case emphasizes that sometimes a car can have a dual purpose. If being used for work, stopping for repairs may not take the driver outside the scope of employment. Another important aspect we need to know: the distinction between a frolic and slight detour.

Ex: truck delivery driver stops at bf's house on way to make deliveries. As employer, can argue it was a folic when accident happened, taking care of personal business, so outside of scope of employment. Ex: if employee stops to pee, can argue that's a slight detour, taking care of basic things that the employer should have expected. Ex: taking your lunch break you're not outside of scope of employment, usually. That's a v basic need. Ex: you go to a work conference, your purpose is to do professional development - you're acting in scope of your employment. If, at the conference, you leave the conference site and go to a bar, then you may still be in scope of employment. Aka, scope of employment is broader than just working inside an office building.

With regard to most conditions on land that arise in the state of nature, most courts have held that there's no duty on the landowner to protect people outside the premises.

Exception regarding trees - agreement that owner is liable for negligence if he knewos, or should have known, that the tree is defective and fails to take reasonable precautions.

A. Most cases can be analyzed as a breach case or as a duty case 1. Duty formula: "did D have a duty to do x." Aka, if case talks about what P or D should know, that's a duty case 2. Breach formula: "did D, having the duty to act as a reasonable sensible person would in a similar circumstance, breach their duty by x"

Exemplifying Negligence - is it a duty case or a breach case?

Types of assumption of risk: express, implicit.

Express: ks where P releases D from D's negligence AND P's negligence unless they are against public policy or interest (certain industries, like hospitals, utilities, where exculpatory clauses would be deemed contrary to policy). But generally, those ks will be upheld. •Implied assumption of the risk: 1. Where the D really doesn't owe a duty to the P, and the P's injury will be deemed to be assumed as an assumed risk - ex, regular turbulence in an airplane. Airline has no duty, no way to avoid it, no liability. 2. If a duty was owed, you assume the risk by getting on an airplane. 3. (category) secondary assumption of the risk - things that the P knew the risk of, appreciated the magnitude of the risk, voluntarily encountered the risk. Two exs: a person who runs into a building to save their child - deemed to have assumed the risk (knew it, should appreciate magnitude, voluntarily encountered it), their conduct was reasonable. Second ex: person runs into a burning building to save their PC. Their conduct was not reasonable, but it is still an assumption of the risk, met those three factors. So: secondary can be reasonable or unreasonable.

Enright v. Eli Lilly & Co. Notes:

Exs where this the public policy took away liability: A. Employer of P who hurt their family member with on-job exposure to asbestos B. Woman's sues husband's urologist bc she couldn't have kids C. Pharmacy filled a hydrocode prescription, person later had a crash under the influence of that drug D. Makers of video game not liable bc a man who played them went on a killing spree

(Brief) Campbell v. Weathers

FACTS: A customer of a cigar and lunch shop was injured as the result of falling down on an open trap door on the way to the restroom. He filed a case for damages against the operator of the lunch shop, the owner of the building, and the manager of the building. A demurrer was filed by the defendants and granted by the trial court. The case was appealed to the Supreme Court of Kansas. ISSUE: Was the customer entitled to damages? Yes CONCLUSION: The court found that the customer was an invitee at all times. The court rejected the operator's argument that customer lost his status as an invitee simply bc he had not made a purchase prior to his injury on the single occasion. The court also noted that there was a box directly in front of the hole which obstructed the customers view. The court stated the rule that a storekeeper who placed racks of merchandise about a railing view of customers was negligent. However, with respect to the owner of the building and the building manager, the court found no actionable negligence. The court found a landlord was not liable where the intervening act or acts of the tenant over which the landlord had no control constituted the procuring cause of the injury.

(Brief) Wehlan v. Van Natta

FACTS: After paying for his purchase at appellee's grocery store, appellant asked if appellee had any spare boxes. Appellee directed him to a back room, which appellant found to be dark. While looking for a box, appellant fell down a stair. • ISSUE: Did P's status change from invitee to licensee after he went into the storage room to obtain a box? Yes • CONCLUSION: Court held that Appellant ceased being an invitee in appellee's store when the particular business purpose of the invitation, shopping, ended, and became a licensee when he proceeded to the back room with permission. In addition, appellant was contributorily negligent in continuing to look for the box when he realized he could not see well in the room. •Trial court held that the appellant was a liceensee at the time of fall, to whom appellee owed no duty to provide a safe place, save and except to have abstinaed from doing any intentional or willful act endangering his safety or knowingly letting appellant run upon a hidden peril.

False imprisonment - Big Town Nursing Home, Inc. v. Newman: P was in nursing home, tried to escape multiple times. What is false imprisonment:

False imprisonment is the direct restraint of a person of the physical liberty of another without adequate legal justification. Here, there plenty of evident to hold that P was falsely imprisoned. Mere refusal to admit someone is not false imprisonment. If one exit of a room or building is locked with P inside, but another reasonable means of exist is open, there's no imprisonment. The means of escape is unreasonable if it involves exposure of the person, material harm, or danger of substantial harm to another, or P just doesn't know of its existence. No actual damages need to be proven for battery, assault, false imprisonment. Similarly, all these are exclusively intentional torts (not negligent torts)

Wagner v. State: Woman attacked by mentally ill man in a store, who had been brought there by state employees. P argues that the insane man's attack couldn't legally constitute a battery because that intentional tort requires the actor to intend harm through his deliberate contact, which he wasn't mentally competent to form. D argues that it is battery and that the only intent required in battery is just the intent to make contact; the contact must be harmful and offensive by law, but the actor need not intend harm as long as he intended contact. Court agrees with state, that only intent to make contact is necessary In order for a contact to constitute a battery at civil law, two elements must be fulfilled.

First, the contact must have been deliberate. Second, the contact must have been harmful or offensive at law. This court holds that the actor needn't intent that his contact be harmful or offensive in order to commit a battery as long as he deliberately made the contact and as long as that contact satisfies our legal test for what is harmful or offensive. "we agree that not all intentional contacts are batteries, and that the contact must be harmful or offensive to be actionable. We don't agree, however, that the actor must appreciate that his act is harmful or offensive for his contact to constitute a battery." "so long as D intended to make that contact, and so long as that contact was one to which P had not give her consent, either expressly or by implicated, he committed a battery." This court's conclusion that P must prove only that D intended to touch P, resulting in offense or harm, is sometimes referred to as "single" intent. Other courts have required what's sometimes called "Dual" intent: P must prove that D intended to offend or harm P by causing bodily contact.

Western Union Telegraph Co. v. Hill is an assault case. Every battery has an assault, but every assault isn't necessarily a battery. What it takes to cause assault: an unlawful attempt to commit a battery/an intentional, unlawful offer to touch the person of another in a rude or angry manner. Here, he tried to touch her but didn't. For assault:

For assault, the victim must have a reasonable apprehension of contact, and it's not necessary that the D have the actual ability to carry out the threatened contact. The def of assault in the criminal law of many states differs from tortious assault. Some states require proof of fear rather than just apprehension on the part of the victim, some require neither, and some require that the D have the actual ability to contact the victim rather than just the apparent ability.

Pokora v. Wabash Ry. Co. takeaway:

For judge to decide what duty is, have to build upon a basis of ordinary action. Can't just say 'this is what you should've done;' needs to be some kind of background of exp that that's what people normally do. If there's no background, just tell jury that the duty is to do what a reasonable prudent person would do in a similar circumstance.

In pure contributory negligence js, P's recovery is reduced by the percentage fault attributable to P

Four states that continue to apply the common law doctrine that contributory negligence is a complete bar to recovery: AL, MD, NC, VA

Cox v. Pearl is about release. In a release, you:

Free D from any liability for the accident, say I will not sue you. but if you release someone, usually becomes a k. their consideration could be money. But once you release someone, you can't sue them again • Normally, there's a separate rule that goes along with this: if Ds are j and s liable, if you release one of them, you release the other one. If one settles, that automatically releases the other one. That's the common law rule. If not j and s liable, don't get to this. Must be joint tortfeasers are j and s liable. • Some states have gotten away from this, say it depends on the intent of the parties - did they intend to release both Ds. So this case says if they didn't intend to do it, then they can settle for one and still sue the other. Measn: in your release agreement, say that you retain the right to sue the other D. • Covenant not to sue is another name for this - way to not squirrel around with releases at all. •This case says we don't care what you call it; if you put in your agreement that you retain the right to sue the other D, your intent is est, and you can proceed to sue him.

1. Inherently dangerous activity rule 2. Adult activity rule 3. Rule of 7

From reading Robinson case, how many different duties/rules do courts of different jurisdictions apply to children?

Statutes of repose: limit potential liability by limiting the time during which a cause of action can arise. Have been enacted primarily in area of liability for architects and engineers. A few states with statutes of repose have made special exceptions for particular products, such as asbestos or DES, that involve a long latency period between exposure and manifestation of injury

Generally, the tolling provisions of the statutes of limitation don't apply to statutes of repose. Ex - a ten year statute of repose not extended by the fact that P motorist was a minor. Statutes that allow tort claims against governmental entities frequently provide, in addition to a statute of limitation, that a notice-of-claim be filed with the appropriate gov agency in a particular time frame. P must meet both deadlines - both provide notice to agency and for filing in court.

Bc punitive damages are punishment, they have a different standard of review:

Have to prove the behavior was intentional or reckless/in disregard - must be proven by CLEAR AND CONVINCING EVIDENCE (when the D acted he acted with such certainty that we know he would cause harm without a moment's hesitation. Without a moment's hesitation is the standard. Simple negligence is driving 65mph in 55 - more likely than not. The other standard: beyond a reasonable doubt - 99.9. clear and convincing - 85%. Simple negligece - 50.1%).

Enright v. Groves: Woman seized and taken to jail for violation of dog leash law/not producing her license. False arrest arises when one is taken into custody by a person who claims but doesn't have proper legal authority. Accordingly, a claim for false arrest won't lie if an officer has a valid warrant or probably cause to believe that an offense has been committed and that the person who was arrested committed it. Convinction of the crime for which one is specifically arrested is a complete defense to a subsequent claim of false arrest

Here, court found that police's demand for P's license was not a lawful order and that refusal to comply therewith wasn't therefore an offense in and of itself. Police therefore weren't entitled to use force in arresting P. Note: not necessary for police to be the one to assert authority of law in false imprisonment. A private citizen, though, who physically (NOT if just provide info) aids a police man in making a false arrest can be liable for false imprisonment. If, however, police requests assistance, private man won't be liable unless he knows the arrest is unlawful

Richardson v. Chapman deals with the same elements of damages in the Anderson case. But the issue is a little bit different - what if jury awards too much money?

Here, the court gave the remittitur. Though the 11 mill too much, considering the 9.5 high-end calculation given by economist. Court basically saying the jury's determination needs to be close to what the expert says. Court reduced 11 mill that was awrded to pretty close to 9.5 given by expert by way of a remittitur. • Bottom line of this case: get a good expert, get him or her to cost out each expense and use the correct inflation rate. • Second part of this case: gave second Ps 100k for her pain and suffering - most of that was mental distress. Judge says 100k is too much, I'm going to cut it in half - that's the amount a reasonable jury would have awarded. If you don't accept 50k, I'm going to order a new trial. So P had a choice: accept reduction, or take a new trial. • Dissenting opinion says: it's for jury to decide how much P gets. Unless they're totally outside of the ballpark, we need to accept it. here, don't be biased just bc she has emotional pain. • The remittitur is the main idea of this case. Don't like reduction, we get a new trial.

For the Eggshell rule, all states are agreed upon the rule when unforeseeable consequences follow from a physical injury to the P.

However, a few courts have limited the test to pre-existing physical conditions. Like MS.

regarding collateral sources: When it's P's private insurance co, for which she paid premiums, or her worker's comp, often those carriers will have a subrogation clause that says if you get money from a D in a case about which we've paid money, you have to repay us.

Ie, P isn't usually getting a windfall; if has any of her bills paid by insurance, workers comp, they'll usually want reimbursement from any money she gets from D. •Many js have changed the collateral source rule under tort reform, now allow collateral sources to be deducted.

Salevan case breakdown: Baseball is an artificial condition. Anything you build on your land is artificial, you'll owe a duty. Common law rule - distinction between natural and artificial conditions with the tree exception. Trend against this: towards foreseeability

If a pond owner, for ex, can foresee that it will overflow (look at history, what's happened in the past), will fall in this trend; should've put up a retaining wall, drain pond. In this case, bc was an artificial condition, D had a duty to use reasonable care to keep balls from getting outside the park.

Derdiarian case stands for:

If an injury that within the risk of harm occurs, then its foreseeable, and proximate cause will be est. freakish accidents that occur will not cut off proximate cause. Intervening causes: sometimes an intervening cause (ie, the car), can cut off the D's negligence in not having a barricade. But not if the intervening act was negligent as opposed to intentionally done, and not if the risk or injury that materialized is in the realm of risk in the og negligent act.

Bruckman v. Pena: apportionment of damages. P has wreck with D1, hurt her back. months later, another accident with D2 that also injured her back. now, after both accidents, she has paralysis. Against whom can she collect? She has to decide how much of her subsequent pain should be allocated to D1 and D2.

If can't show D1 can't show he produced a certain amount of the damages, can't collect from him. if can't say how much after second accident is due to D1, can't collect from him, but can collect from D2 - eggshell rule will get him (had to accept her as she was). Can collect from D1 for damages that existed, bills, etc between first and second accident. Only liable for the interim unless can show by a preponderance of the evidence that 75% of her paralysis happened in the interim, before second accident (get an expert to prove this).

for proximate cause: Foreseeability is an evaluative standard. If you want a more definite definition: something is foreseeable from the D's negligence if, to a reasonable prudent person, this thing that was allegedly foreseeable was deemed to be possible.

If could've been seen as a possible risk, then it's foreseeable. Doesn't have to be probable, just possible.

Regarding loss of chance in legal malpractice cases: in most js, P must prove that he would've won the case, how much the award would have been, and that the award was collectible.

If former criminal wants to sue his lawyer: most js require him to show that he actually is innocent of underlying criminal defense.

False imprisonment: What is a reasonable means of escape: depends. You're not required to escape in ways that embarrass you, like crawling through a sewer. Can't expose you to harm or danger or embarrassment. Note: You can commit a false imprisonment both negligently and intentionally:

If it's a negligent confinement, the person has to show actual damages to their property or themselves. Ex you blocked me in my house, I couldn't go to work, lost my job. If it's intentional, the mere fact that they confine you against your will is compensable; the longer your confined, the more your damages will be, the more you can get compensation. Can also get punitive damages in this case.

a. This appeal will be based upon a legal mistake that was made. i. Filer can raise an appeal for ANY issue, like sj motion, that was made - say that x motion should have been granted along the way. ii. If appellate court decides a mistake was indeed made, will usually send it back, order court to grant the motion that it orders it to. b. In the appeal, filer can tackle any issue they raised b4 trial court - they gave trial court to opportunity to 1strespond. Only when they think lower court got it wrong can they appeal.

If losing side's post-trial motions are denied, filer can appeal to appellate court above trial court.

Pagelsdorf: the initial rule was that landlord didn't owe a duty to the tenant or guest of tenant unless one of six exceptions. But now, this case says in this j a duty will be owed to both tenant and tenant's guest for any dangerous conditions on the premises that are not corrected/landlord owes a duty to use reasonable care to guard against an unreasonable risk of harm to tenant and tenant's guest.

If tenant never informs landlord of dangerous condition in the apartment, probably not going to be found liable, though - court here says the question becomes if the lease gives them a right to inspect periodically. That's why, in this case, the court found for the landlord.

a. If everybody says the same thing, and there is no dispute, one party's owed summary judgment according to the law. b. Why aren't you entitled to summary judgment if there's a genuine issue of material fact? Because a jury is supposed to determine disputed facts c. If there's dispute, judge will assume that jury could believe P's witnesses and evaluate if they do believe P's witnesses if they could find for P.

If there're witnesses saying different things, there's a genuine issue of material fact and Rule 56 motion'll be denied

Outhouse case: did not voluntarily encounter the risk. She had no other place to go to the bathroom. However: you could be negligent if a reasonable prudent person would have used that floor even if you didn't do it voluntarily. Ie, contributory/comparative negligence may apply even though hthis defense doesn't.

If you inadvertently encounter a danger, maybe comparative/contributory negligence should just be used, esp if you didn't know the magnitude of the risk, but assumption of the risk should not be used against you if you had no choice.

P553, restatement second section 332: refers to invitees as "business invitees." In recognition of the understanding that most invitee cases involve businesses.

If you invite refrigerator repair person over, he's an invitee. Have a duty to guard against unreasonable harm to him for both passive and active conditions. So when you compare an invitee to a licensee: if it's an invitee, no distinction between passive and active conditions - owner must use reasonable care to guard invitee from harm. What's the distinction: in the case where the man went to get a box, fell, owner had no liability bc didn't know light was blown. If that man was invitee, would've had a duty to KNOW. you know by doing an inspection. For licensees, no duty to do a prior inspection. If invitee, had duty to look in there before he went in there.

Issue: what duty does the D owe to the P. it depends on the status of the P while she's on the D's property. Taylor v. Olsen: branch fell in the road, P ran over it. so what duty did the landowner owe to the P? How is rural v. urban areas relevant? Depends on it is a natural or artificial condition

In General: landowner does not owe a duty to P outside of premise if hurt by a natural condition. If hurt by an artificial condition, DOES owe a duty to someone outside of the premise. Distinction between natural and artificial. If you own property, something naturally flows or falls from it, you owe no duty to person who is injured. But if artificial, that you created, it flows from your land OUTSDIE of your land and hurt someone, you have a duty to use the care of a reasonable prudent person. Trend is moving away from this artificial v. natural condition rule. Trend is towards foreseeability, whether the D foresaw or should have foreseen a risk of harm to someone outside of the premise from a condition on the premise.

(Gore factor) "Degree of reprehensibility of the D's conduct:"

In State Farm case, co. didn't make a reasonable settlement when we could have. Drug it out, made it go to trial, now that you have a big judgment we'll only cover a little bit. If you'd have just settled when you could have, I wouldn't have this payment or distress. That's called a Bad Faith Insurance Claim. If they don't settle for a reasonable amount when it's requested. Insurance co didn't jump on reasonable settlement in this case just because they hoped by dragging it out to make the P go down more. So how is this dragging stuff out reprehensible to the policy holder: they'd have to pay more, suffer emotional distress. Reprehensible - they've paid their premiums, but co is jerking them around, telling them they can go to trial and win. Now what did SC say about taking into consideration regarding other Ps in other states with other types of insurance: can we take into consideration those other cos bad conduct: can't punish someone for a case that's not under review at the time. Need to look at what they did to this particular person - can't award money for bad conduct that occurred to other Ps; can only award oney based upon reprehensibility f conduct to this P. But many states let you take into consideration that they're doing the same thing to other people in diff states - just can't award money based on what they are doing to others. Ie, looking at this you can prove it wasn't negligent what you ddi to me - it was intentional, bc you're doing it to others. But your damages aren't based on what you do to others - must be awarded on what you did to me. Reprehensibility: the more the thing was intentionally done, done to make a profit, the more the D lied about it, tried to hide it, the longevity of their doing it all adds to the reprehensibility of it

Trespass to land: Dougherty v. Step Every unauthorized and unlawful entry into the close of another is trespass. Bc the cause of action is for exclusive possession of the land, it belongs to the party with the right of possession, whether that be owner or tenant.

In a trespass action, P will be awarded nominal damages if there are no actual/compensatory damages When a trespass is found, the fact that D's conduct was socially useful or even beneficial to P doesn't affect liability. Ex, if somebody trims your trees without permission Intangible intrusions, like smoke, odor, light, and noise aren't typically actionable under trespass theory

Regarding implied assumption of risk: The scope of the risk assumed must be addressed. Courts, unfavorably disposed to assumption of risk, place a narrow gloss on 'risk.' Ex, while watching football game, being trampled on by unruly fans not a risk inherent in or an ordinary part of watching a game.

In addition to knowledge of the risk, P must proceed to encounter it voluntarily. Ex, if road is slippery but you have no other way out, it's not voluntary. The fact that P protested against D's conduct is evidence that he doesn't consent to assume the risk. But if, afterwards, he has reluctantly accepted the situation, some js say he has waived the protest. Many js had adopted statutes that provide protection from tort suits to particular categories of activities or Ds. Ex, landowners that open up their property for public recreational use; spectators at competitive-sports events

Have shorter statutes of limitations for intentional torts than negligence torts.

In intentional tort, usually P's contributory or comparative neg isn't used against them.

Another way to be vicariously liable: automobile guest statutes. Here, the owner doesn't have to be a family member - this rel is just based on the ownership of the car.

In many js, if you own a car and give somebody else permission to drive it, your ownership is enough to make you vicariously liable to the person who's driving the car

Statute of limitations is a complete bar to actions that don't meet its time limits; it is in no way dependent on the merits of the case. For actions sounding in tort, most states impose a two or three year limit. Even within each state, there is variation depending on the type of action that's brought. Since damage is an essential element of a cause of action based on negligence, most courts have held that the statute begins to run when there has been an actual injury to P's person or property.

In most cases, that is easy to determine and not a subject of dispute between the parties. some categories of cases have yet arisen where the date of injury isn't easily ascertainable or where the strict application of the time-of-injury rule works an apparent injustice In some professional malpractice cases, courts have found that the statute of limitations didn't begin to run til the course of treatment was complete. This is sometimes referred to as a continuing tort. Ex: statute of limitations on legal malpractice claim doesn't start to run til resolution of appeal against client in underlying suit/til attorney had made his last act for client.

Regarding discovery rule: Reasonable minds can differ on when she knew or should have known that the injury existed. Ex, if patient started having stomach pain, she should maybe have known something was wrong, even if didn't go to hospital. Knew or should have known of injury: is it that you knew or should have know that you are injured, OR that the doc performed malpractice? What's the exact requirement.

In some js, she must know not only that she had injury, but that there was a breach of doc's duty. She must know all four elements before statute starts to tick under the discovery rule. In some js, she only has to know that an injury occurs (saying: if you know you're hurt, you now need to go get an assessment from your doc, another doc, or your lawyer).

three ways to calculate a statute of limitations (can use discovery rule):

In some js, starts to run when know about injury. In other js, starts when P knows all four elements of negligence. Alt way: when the injury occurs (LISt ALL THREE ON EXAM - three way to calculate statute).

regarding comparative neg: There are some js, a minority, that says her negligence must be looked at not in combination with Ds, but you look at them separately.

In that case, compare her 48 to D's 2, and she couldn't collect from that D. Maj just says look at combined percentage of Ds

Regarding IIED: Does threatening to beat someone meet elements two, "extreme and outrageous?" what is extreme and outrageous? Restatement section 46: "liability has been found only where the conduct has been so outrageous in character, and so extreme in degree, as to go beyond all possible bounds of decency, and to be regarded as atrocious, and utterly intolerable in a civilized society." If you can say that, and have it be a kneejerk reaction, that's met. He says that calling someone a racial slur one time probably doesn't meet that test. But if you are constantly calling someone a racial slur, that may well meet this test.

In the State Rubbish case, p58: that conduct met the test. Slocum case, insulting a shopper: doesn't meet the test. Might be different if a racial thing In Harris v. Jones: employment situation - supervisor constantly ridiculing a man for his speech impediment. That's probably borderline extreme and outrageous. P's problem in this case was that he couldn't prove he suffered extreme emotional distress, so court never definitively really said if the conduct was extreme and outrageous. Should've had an expert to say that his emotional distress after the mocking became much worse, aka his troubles weren't all related to his prior disturbances. There are a whole bunch of policy issues that can come into play when deciding if something is extreme outrageous - need to note this on exam. There are other things that come into play, too - might have a diff case if P is especially vulnerable, a pregnant woman or a small kid, or elderly. Look at the vulnerability of the P also when deciding if something is extreme or outrageous. This includes people you know are especially sensitive, like people who are mentally ill. Collecting agencies: yes you owe the money, but for them to show up at your house, calling your employer, is extreme Proof of severe emotional distress: sometimes you need expert testimony that the person has suffered a detrimental effect of the intentionally inflicted stress. Some don't, just want to hear from P what happened to them Ex of a sensitive P: woman who was super in love with her horses - they knew that she was sensitive about them.

Western Union telegraph case: clock man lunges towards the woman - doesn't touch her, but came close. He showed apparent ability to touch her. even though you have the intent to do a touching, and the P has a reasonable apprehension of an imminent touching, another thing must exist: the perpetrator must have the apparent ability to complete the battery.

In this case, there's big discussion about whether his arm could reach across the counter. If there was no way he could have ever have reached her, then we have no assault - yes she had a reasonable apprehension, but he didn't have the ability to do it, not reasonable to think he could. Must have the apparent ability to complete the battery.

a. EXCEPTION to the application of child's standard of care): when kids do inherently dangerous activity, will be held to adult standard of care i. How do we know when something's inherently dangerous? Look at gravity, likelihood of harm. ii. Act's inherently dangerous if it exposes others to harm that's above an ordinary risk of harm when going about daily activities. Ex, if you're fumigating a store, and employ a child to do that, that child's going to be held to an adult standard because fumigation has the potential to create harm, is inherently dangerous

Inherently Dangerous Activity Rule under child's standard of care:

Maloney v. Rath Adds something else to the general rule of no liability for general contractors: there are exceptions: one of the exceptions is that if the duty is non-delegable, then the one who engages an indep contractor will be vicariously liable for the negligence of the indep contractor. Here, this means that the P sends her car to a brake repair shop, brakes don't get fixed right. If the activity they repaired exposed the public to a high degree of risk, the engager will be vicariously liable bc the duty you owed was non-delegable. So there's a class of risk that's so important to be guarded against that the court'll hold the duty you have can't be delegated to an indep contractor. Reason why: incentivize you to hire a good, competent repairman. How do we know if something is non-delegable?

Is a haphazard, ad-hoc decision the court may make. One way: activity constitute a grave risk of bodily harm or death. Two restatement provisions that may be relevant: 423 and 434

Borders v. Roseberry breakdown

Is landlord under obligation to a social guest of his tenant to remedy a known, potentially dangerous condition? It's the tenant who, as possessor, has initially the burden of maintaining the premises in a safe condition. The rel of landlord and tenant is not in itself sufficient to make the landlord liable for tenant's tortious acts. Exceptions: undisclosed dangerous conditions known to lessor and unknown to lessee; conditions dangerous to persons outside the premises; premises leased for admission to the public; parts of land retained in lessor's control which lessee is entitled to use - ex passageways in an office building; where lessor contracts to repair; negligence by lessor in making repairs - when lessor does attempt to make repairs and fails to use reasonable care he's liable for injuries that result - this comes into play when lessee lacks knowledge that the repairs have not been made or have been made negligently Courts agree that lessor must disclose all known concealed dangerous conditions existing at the time of the transfer of possession. the liability extends not only to the tenant, and the members of his family, but to his employees, his social guests and others on the premises in his right, and to a subtenant to whom he leases the premises. Most courts find it sufficient that the lessor has info that would lead a reasonable person to conclude that the danger may exist, and that if he does he must disclose such info to the tenant The mod approach: derives a lessor's liability from failure to perform a k to repair when on notice of the need for repair or where lessor undertakes repairs voluntarily. Lessors are thus liable for resulting injuries to the tenant, members of tenant's family and their guests

Joint enterprise: one way one can be vicariously liable is via respondeat superior, employer-employee rel. another way if through gen contractor. Yet another way is through joint enterprise

Joint enterprise: a group of people acting together. The negligence of one will be considered the negligence of all. P can sue the joint enterprise as an entity, or sue all of its members Popejoy case: P says mom, dad, and daughter were part of a joint enterprise. Court said no, dad's not a part of it, so he's not vicariously liable for mom's negligence. Found that there was no agreement between them to go and buy a cow

defense of property: can use reasonable to defend your property. the gen rule is that you can nev use deadly force unless you're being exposed to a serious or deadly risk of harm. Ex, if somebody steals your rolex, you're not supposed to shoot them as he's walking out the door. That's common law - castle doctrine may counter this. In common law, can wrestle him to the ground, but can't shoot him. but once again, there may be a statute that lets you shoot bulglars or castle doctrine. Some js have the castle doctrine to protect your house or car. 'what is reasonable force' is a jury question. You can use reasonable force to defend yourself against an aggressor OR defend a third person (usually a family member). Can use deadly force if person is coming at you or family with deadly force. If you go to defend a third person, better make certain they are in fact the victim and not the aggressor; you'll then be an aggressor if you defend them.

Katko case uses common law - can't use deadly force to defend your proepryt. If can't use deadly force when you're not personally there, can't use it when you're not there, hide dynamite under the door. Notes say that in some js if you post on your property that there's a dangerous dog, if you post then you're not liable - others say no, a posting won't protect you if dog kills a trespasser. Katko says that can't use a spring gun bc going to be civilly sued for battery unless the person was exposing your or your fam to deadly force.

Before repair man comes, you have to scour the house, ferret out dangerous conditions. Not so for a friend coming over to watch tv - just say 'I didn't know;' don't have to do an inspection. For invitees, don't always have to clean u pdangers - warning may be ok if a warning would be sufficient (thought this isn't frequently the case) Barring discrimination, the minute you tell someone to get out, they're a trespasser

Licensee: anyone who's on your property with your permission. Invitee is basically a leasee who's there to confer some benefit on you (buying your goods, repairing your fridge). Licensee, in contrast, may be there for their own pleasure Note 4, p556: with respect to licensees, the hist rule was that if the condition was open and obvious, you didn't even have to warn them and make it safe. NOW, the duty is to use reasonable care to detect them - if a warning wouldn't be enough for the licensee, you have to repair the condition.

Barmore v. Elmore Fraternity bro comes over to pay dues; while there, son of homeowner comes out and stabs him. what's P's status? Licensee.

Licensee: he's a social guest. Not a trespasser bc he has permission. Not invitee bc he's there for a purpose for which the owner opens up the property (usually for a business purpose). Here, just gratuitously allows P to come over to pay his dues.

degrees of care: Those who deal with things that are known to be dangerous, such as explosives or electricity, must exercise more care than one who is merely walking down the street.

Likewise, who have accepted a special responsibility toward others, as in the case of a common carrier, must exercise more care.

(Gore factor) "The disparity between the actual or potential harm suffered by the P and the punitive damages award:"

Look at compensatory damages that was awarded to the P, even if the P is the insured, compare it to the amount of punitive damages. If great disparity, not great. What's an appropriate ratio. In State Farm case, court says that we don't give a hard and fast rule, but we give you guideposts. Guidepost would be a single digit ratio. Should not be more than ten times the compensatory amount. When above 10:1, it's suspect. Court also says we think that a better ratio, beyond this, is actually 4:1. Didn't definitively say this, but that's the push of their ruling.

Med expenses and lost wages in the future need to be inflated and discounted to the present value. The jury is supposed to award the present value. Gotta have an expert to pick an inflation rate.

Lost wages: if making 10k this year, inflation may mean she makes 10.5 next year, so on, bumps up each year. Expert has to do that and get a total, and then discount all those increases back to the present value, use a discount rate so that the jury will award an amount TODAY that if the P will invest at the inflation rate she will have the stream of money each year that will be equivalent to the money she'd earn each year if she was able to work. Bonuses also have to be counted, as well as promotions. Then discount back to the present by applying the appropriate interest rate so the jury will know if you give her x and invest at this rate, she'll be able to draw from this amount the amount that's calculated for each year for the next y years. • Do the same for med expenses. Count them out, discount back to present value, say if you invest at this interest rate, you will have a stream of money over the future that will allow you to get anything you need to for your med treatment. • Inflated then discounted back to present value for both lost wages and med expenses.

Ways states have approached market share liability

MI says Ds unable to exculpate themselves were j and s, on per capita rather than market share basis. Courts have had to decide how many of the Ds must be before the court, what P must prove about her efforts to identify the manufacturer of the DES that injured her, whether D can avoid liability by proving that it didn't manufacture the DES taken by the P's mother, who'll absorb the shares of Ds who aren't present or are judgment proof (P? all Ds sued by P? Ds impleaded by other Ps? Ds who were unable to prove they weren't the manufacturer?), how the Ds will share the liability (per capita? Risk contribution? Market share?), what the relevant market is

A general rule developed in many js that contributory negligence won't be imputed unless negligence could be imputed.

Many courts then accepted the converse of this proposition: if negligence can be imputed, contributory negligence will be also. This is the so-called "both-ways" theory. This theory has been under slow but steady attack.

Damages for physical harm to land or chattels is closely tied in with the concept of value or what the property is worth. If destroyed, measure is its entire value at the time and place of the tor. If it's damaged, but not destroyed, the measure of damages are measured by the difference in value before and after the injury. The standard set for value is the market value of the property.

Market value usually is defined as what the property in question could probably have sold for on the open market, in the ordinary course of voluntary sale by a leisurely seller to a willing buyer. Market value ordinariiy is determined on the basis of the market at the place where the wrong occurred. When there is no available market at that place, there is resort to the nearest market, with allowance made for the costs of transportation to and from it. Market value is ordinarily determined as of the time of the wrong.

Regarding contributory negligence: can P's fault after the injury increase her percentage fault?

Mitigation of damages concept or avoidable consequences come into play after the injury. Pre-injury: can reduce her damages based on her percentage fault; after the accident, can further reduce her damages by mitigation of damages (P cannot recoup from D any damages afte rhte accident that she could have avoided by using the care of a reasonable prudent person. If refuses to go to hospital after car wreck, testimony later determines the scar she has could've been avoided if she'd gone to the doc, D's not liable for that, her damages may be reduced. So any dmages after an accident that P fails to mitigate are not recoupable from the D. Applies to surgeries, too - if you don't get surgery, continue to be disabled, can't get adamages bc had you got surgery you would not now be disabled. Must act as a reasonable person; if you don't, can't collect those damages from D. question for surgeries: would a reasonable prudent person in the same or similar circumstance have had the surgery, despite any risks attacked to it. if so, can't collect from not doing it. before the accident caused by the D, things like contributory negligenc and comparative neg are affirmative defenses that must be pled in the answer to the complaint - same with failure to mitigate damages. If you don't allege them, you lose that defense. But once properly alleged, are affirmative defenses that D has the burden of pleading, proof, persuasion (preponderance of the evidence) •It doesn't matter what damages stem from the D's conduct; P has a duty to mitigate it to the same degree as a reasonable person in the same or similar circumstance would do to mitigate it

wrongful death ds:

Most js calculate damages by looking to pecuniary losses caused by the death - aka, loss to beneficiaries statute. Measured by looking at the joint life expectancy of the decendent and each beneficiary. Most js recognize that the economic loss suffered by a kid due to the death of a parent includes loss of training, education, guidance, and nurture as well as direct financial support. Note: although most states measure the damages based on the loss to the beneficiaries, some use the loss to the decedent's estate. However, a growing no of courts have taken the route here and allowed recovery for "loss of companionship" or consortium of a dead family member. Some js have built in caps on ds for nonpecuniary losses. About half of the states permit punitive ds to be awarded in a wrongful death action if the case involves intentional or reckless conduct

Note 13: sometimes employer is immune bc of workers comp rules, sometimes parent is immune, fed gov is immune bc of governmental immunity. Cannot assign percentage fault to an immune party?

Most js say no, have to assign 100 to any other Ds. There is a provision: can look at another D's negligence, wehtehr an employer, gov, parent, and say 'they were the ones who caused the accident and not me; their negligence was the sole cause of the accident, not anything that I did.' Aka, use them to argue for yourself if you're the D; zero percent goes to me (just can't say 30 to me, 70 percent to them); can use to exonerate to zero fault, but not to ALLOCATE fault.

Conversion: twin of TTC. Conversion is a more substantial interference and/or dispossession, so that the only remedy is for you to pay for it. Ex: Mr. Dobbs case

Mr. Dobbs papers: they just made copies of his papers. Have they converted his papers? No, he can still use them. Taking an image is not the type of substantial interference that would make him pay substantial ds. May not even be TTC bc didn't disposses them, or harm them. This case also tells us that there are certain docs that if you use them will be trespass: things that have intellectual property value, like architect drawings, a novel, prof's class notes.

equitable relief:

Much less common damages; if a tort bestows an unjust enrichment on the perpetrator at the expense of the person wrongs, that person may claim a disgorgement of the enrichment

For most intentional torts, court will award nominal damages even if no actual damages were proven, and if P does prove actual damages, D's liable for them as well. At common law, parents are:

NOT liable for the torts of their kids unless P can show some fault on the part of the parents in, ex, supervision

Note: decisions in negligence cases are based on a preponderance of evidence, NOT beyond a shadow of a doubt - jury finds one outcome more LIKELY.

Negligence cases: rest on a failure to use intentional care (accidents aren't intentional).

distinguish family car doctrine, imputed contributory negligence from negligent entrustment:

Negligent entrustment is a form of direct liability - a bailor is negligent in entrusting property to bailee.

Sheehan case: P was a trespasser; at the time of his injury, wasn't a passenger nor in a public crossing or space in which the public were licensed to travel.

Nonliability to trespassers

regarding wrongful death statutes: Each state statute will give a list of beneficiaries, will prioritize who's in the first, second ranks. If somebody is living in first rank, don't go to second rank, second rank can't bring claim. But if nobody lives in first rank, keep going till there's somebody in existence. If nobody exists in the ranks, nobody can bring a wrongful death claim.

Normally the spouse of the decedent, or spouse and kids, are beneficiaries. Spouses, alternative, may be in first rank and kids second. Aka, if spouse living kids can't file claim. ("Spouse" doesn't include civil unions, but some states have provisions that allow them to collect in a wrongful death claim. This is purely statutory) Children, p639: includes the biological kids. Unborn kids: do have a right to ds as beneficiaries as if they were born if the unborn kid is seeking to be a beneficiary.

What about public officials on your land, like firemen, police, sanitation workers, meter readers?

Normally, police and firepeople aren't treated like invitees. If anything, they're treated like licensees. Why: they get paid to do their job, assume the risks to do that job. However, there are some cases where they're treated like invitees. The meter reader, garbage man does not assume that risk - they are invitees, bc they come to benefit you. • So the law is all over the place.

In most js, no contribution is permitted among intentional wrongdoers. ALSO: non-immune tortfeasors may not seek contribution or indemnity from those who are immune. Workers comp statutes prevent employee from suing employer in a tort. In most js, a party sued for negligence may file a cross-claim for contribution, may implead another person as a third-party D from whom he is seeking contribution, or may bring a separate action for contribution.

Not all js protect a settling D from contribution: some permit the nonsettling D to seek contribution from the settling D. not super popular bc discourages settlement.

Trespass to chattel (TTC): chattel is real property, personal property. if you interference with one's chattel, you have to either damage the chattel or you have to dispossess the owner of the chattel for a sufficient length of time that it will be warranted that you pay some money to the owner or possessor of the chattel. If you keep one's car for three days, damages are the rental cost of the car for three days. If you miss a job interview bc didn't have car, those consequential damages can be added to the rental value to give you full compensation.

Note 5, p88: there must be an actual injury to the chattel in trespass to Chattel. In girl case, no TTC because didn't hurt dog, didn't dispossess owner of the dog. CompuServe case: deals with the internet and emails. The gen rule is that an electronic signal, including emails, that interfere with search engines or computer servers can be a TTC. The property would be the internet system. The electronic signals one sends to that system will be deemed to be an interference. What was the injury: it slowed down the processing system, made it less valuable to the owners and the subscribers, injured the goodwill of the owners to their customers - the value of the co was being damaged. Didn't destroy it, but slowed it down, made it less valuable and efficient.

Collateral sources ALSO include: if your spouse takes time off of work to help you, care for you. D has to pay the fair cost of those services; doesn't matter that it was free; we can put a value on that, and the D has to pay that

Note: if D negotiates with a hospital, insurance carrier, etc to lower P's costs, then they DON'T have to pay P the amount. If a discount of med bills is done before an injury, then D only has to give the percentage of the bill that was left after the reduction.

employer is vicariously liable for employee acting in scope of his or her employee. But the exception is that when you get in your car and come to work, you're not acting in the scope of your employment, and same when you go home. "going and coming rule" say that when going to and from work, you''re not acting in the scope of your employment, so employer is not vicariously liable.

Now if your employer expects you to meet with a colleague, do some deals on your way home, you ARE acting in the scope of employment. If you're driving a co car that needs to be serviced, get it serviced on the way home, can say you were acting in scope of employment. So there are exceptions even to this exception. But even this lady, in this case, was due to another exception: she got sick on the job bc pesticides were sprayed at her work, she got nauseous, and on the way home she rear-ended someone. P sued her employer. What'd court say: employer made her sick, extrapolated from that that bc pesticides at work she was exposed to caused her illness, employer shouldn't be shocked, should've foreseen she could've been involved in an accident on the way home. Bc this injury was foreseeable, from something that happened at work, the going and coming rule doesn't apply. The gist of this: if, as an employer, you do something at work that causes some condition in your employee, that in turn causes employee to hurt someone on the way home, employer will be vicariously liable for employee's negligence, even if weren't technically operating in the scope of their employment

Regarding trespass: Has to be some sort of physical invasion, either by D personally, or by throwing an object, to have trespass. Smoke, shining a spotlight are more in the line of a nuisance.

Nuisance: interference with one's use and enjoyment of the land. Trespass: infringement of one's ownership of the land. If it's a nuisance, have to show actual injury to your body or your right to use the land. Trespass in interference with your right of ownership and control. A nuisance is an interference with one's use and enjoyment of a land. If it's a nuisance, have to show actual injury to your body or land. Trespass has to be some sort of physical manifestation. Trespass has to be some kind of physical manifestation, in contrast to nuisance.

McGuire v. Almy

Nurse sues the patient who hit her. D's defense: didn't have the capacity to understand what they were doing. Court says that doesn't matter, that the D, when they intentinally caused injury, they're liable in the same circumstances as a normal person. If she knew she was going to hit the nurse, doesn't matter what caused her to act that way. Read policy reasons why, p28: tends to make insane people's watchers more careful; insane person shouldn't enjoy the comfort of not being liable; also courts don't want to intro a big body of litigation on determining mental capacity Some js won't allow a healthcare provider taking care of an insane person to sue the patient for an intentional tort bc they assumed the risk.

a. Facts: D was trying to pass P, as he passed him, his tire blew out, causing it to hit P. b. Issue: was D negligent for driving with defective tires? Would an ordinary person have known the tire was in such bad shape? BREACH case: did D fail in his duty by driving on worn tires. c. Analysis: tires are in "universal use." Any ordinary person, regardless of mechanical experience, knows that when a tire is worn badly as this one was it's dangerous to drive on it. All drivers must be held to a knowledge of these facts. Owners MUST know d. Uses objective standard of a reasonable third party to evaluate whether there was a breach

Objective Standard of Care - Declair v. McAdoo

a. Facts: D built a hay rick close to P's cottages. D's warned it was likely to catch fire and endanger the cottages. Despite these warnings, D said he'd chance it; well, it did catch fire, and the cottages were then destroyed. b. Issue: should D be evaluated by the standard of a "prudent man" or the standard of his own personal intelligence. c. Rule: D's bound to act as a reasonable person would in a similar circumstance. d. Analysis: judge says the man shouldn't be evaluated on if he was acting to his best judgment; liability not commiserate with an individual's judgment, but the standard of ordinary prudence. 1. This intros objective standard of care - not what that individual would do. a. So: doesn't matter if, in the tugboat case, Connors used their best judgment in not having replacement bargees. Doesn't matter that P here used his best judgment either.

Objective Standard of Care - Vaughan v. Menlove (1837, England)

There are two defs of a battery. Both require intent. That is, the act must be done for the purpose of, or with intentional certainty, that touching will occur, and then touching must actually occur.

Old def: touching must be done in anger, insolent, and rude. P35. New def under restatement: the touching can be either harmful or offensive (one or the other). But under both you need intent to do the touching. Under the restatement section 18, p38: implicit in this is that you can just attempt an assault but commit a battery.

Baseball case: "the inherent nature of the game is such as to require the landowner to take reasonable precautions for the protection of the traveling public.

Once a landowner alters a condition of his land, it becomes an 'artificial' one for the purposes of trots and the owner must exercise reasonable care for the protection of those outside the premises.

In most states, the defense of assumption of risk has been carried beyond the concept of consent and applied to any factual situation where the P consciously and voluntarily places himself in a position where he is subject to a known risk. The rel tween this type of assumption to contributory negligence has caused issues.

One explanation: assumption of risk involves the encountering of subjectively known risk; contributory negligence may involve a P exposing himself to a danger of which eh was subjectively unaware but which would have been apparent had he used due care. It is essential to show that the P had actual knowledge of the risk. The best way to show this is to secure a direct admission from the P or someone who overheard him

Summers v. Tice is a 1 Cause in fact case. What was alleged negligent act: negligently shooting P while hunting. Negligently shot in his directly, didn't use reasonable care when you did that

One of you shot him in the eye. Now look at cause in fact - in this situation, where P is helpless and not negligent, but one of you are, we're going to switch the burden of proof to one of you. one of you must prove by preponderance of the evidence gonna have to prove it's not you. if you can't prove, both are going to be the cause in fact.

Some js have directed that a portion of the punitive damages award be paid to a state fund Some courts say a party can be liable for punitive damages only for their own conduct and not vicariously liable for the acts of others, like agents or employees:

Others say when the agent or employee commits an act that makes them liable for punitive damages, the principle or employer should be liable as well. Most court take the middle position, hold the principle liable only if they authorized or ratified the act, was reckless in employing the agent, or agent was employed in a managerial capacity and acting in the scope of employment.

Three factors you must est to use implied assumption of the risk:

P Must know: there's a risk; appreciate the magnitude of the risk; voluntarily encounter the risk.

'Interest' and damages:

P could argue that D should've been paying me interest on an injury I suffered x years later. Should pay interest from day of injury till they pay settlement. There are diff rules regarding when interest is supposed to start accruing. Also gotta decide how much interest to pay - diff j have diff rules again. But MOST js don't go back to the date of interest - give you interest from other days, like the day the cause of action was filed, date you demand payment from D, date expenses were actually incurred by P (ex, med expenses, lost wages). If a court DOES award you interest, the interest amount is taxable for income taxes.

Brekdown of Coney v. J.L.G. Industries, Inc.

P died while operating a machine, his estate sued the manufacturer of the machine. Two Ds: manufacturer and employer. • Employer, employee, and manufacturer can all be negligent. Although distinct parties, if their negligence combines to produce an indivisible injury, the Ds can be j and s liable. So the manufacturer and employer could both be negligent at the same time in creating the injury. • There's something called worker's comp: lose your right to sue your employer, but get to get your money immediately, don't have to prove breach, just that you got hurt on the job. • Workers comp isn't an issue inn this case. What the manufacturer is that the employee should have also sued the employer. Aka, I manufacturer cannot be j and s liable with the employer - should get some of the money from the employer, and if you can't bc of workers comp, then that's too bad, from me all you can get is my percentage fault.

Breakdown of the Summers v. Tice situation

P is innocent. Don't know which one of you it was, but you both were negligent. When P is innocent in this situation, we'll switch the burden of proof to Ds. This IS AN: Alternative way to prove cause in fact

Whittaker v. Sandford

P stuck on boat by cult leader. Not allowed go on the land unaccompanied.

Knell v. Feltman provides that:

P sues D1, D1 can sue D2. P doesn't have to sue D2 at all. if j and s liable, D1 can file a third party suit and bring D2 into the lawsuit. Basically measn that if P wins money against me, I can get you to pay your fair share of whatever I have to pay. OR D1 can file a totally separate lawsuit to get the money from D2.

Vicarious liability: is used to find that a D can pay. Ex: Bussard v. Minimed, Inc.

P sues employer of person who rear-ended her. D argues that the "coming and going rule" meant that their employee wasn't in the scope of her employment during her commute home, thus they shouldn't be held vicariously liable. Vicarious liability requires that the employee be acting in the course of her employment, which case law defines expansively. Thus, acts necessary to the comfort, convenience, health, and welfare of the employee while at work, though strictly personal and not acts of service, don't take the employee outside the scope of employment. Despite this wide reach, courts have not defined it so broadly to include an employee's commute. Going and coming rule. There are exceptions to this rule, though: one is when an employee endangers others with a risk arising from or related to work. Here, bc employee's job contributed to the accident, ARE vicariously liable

Rowland v. Christian breakdown:

P was a guest, asked to use bathroom. Hurt when handle on water faucet cracked. D had known that handle was cracked, had complained to manager of his building. Fundamental principle: person is responsible for an injury occasioned to another by his want of ordinary care or skill in the management of his property or person, provided that the person has not willful or wantonly brought the injury on themselves. Majority opinion: critique of categories of trespasser, leasee, invitee. Some js have eliminated or altered these categories, esp in regards to licensee and invitee There's considerable agreement that the general negligence standard should be applied to all persons invited or permitted on the premises, there is less accord as to how the trespasser should be handled.

An attorney has to get paid for handling these cases. But economic damages like past med expenses, lost wages is money they actually need. But pain and suffering - just to compensate you for the mental anguish you're going through. One of the way the attorney gets paid is out of the pain and suffering amount.

Pain and suffering is 'gravy' - don't need money to pay any bills. So frequently attorneys like to take their amount out of the pain and suffering. There enlies a policy reason to not cap the pain and suffering. Aka, when courts cap pain and suffering, they are de-incentivizing lawyers to take cases. • Same thing with punitive damages. Since are designed to punish, not to compensate, out of that money the attorney will frequently get their attorneys' fees.

Parvi v. City of Kingston: elements of false imprisonment in this j are listed on p49. What, in this case, is the most relevant: conscious knowledge. Here, they say that he was conscious, so he could prove his claim.

Proof of this: got in the car when they told him to, asked to be let out somewhere else - did NOT want to go to the gold course. Court says you only have to be conscious at the time of the confinement. Restatement takes this a little bit farther, p 50: he has to KNOW that he's being confined. Element 4: there are certain laws that allow one to be confined against their will. will discuss later.

Two basic forms of comparative fault are utilized by 45 states: 1. Pure comparative fault 2. Modified comparative fault

Pure comparative fault: P's damages are reduced in proportion to the percentage negligence attributed to him; for ex, P responsible for 90% of the negligence that caused his injuries may recover 10% of his damages. In modified comparative fault: Ps recover as in pure js, but only if the P's negligence either does not exceed (fifty percent rule) or is less than (49 percent rule) the D's negligence. Modified comparative negligence: P's recovery is reduced by the percentage of fault attributable to P as long as the P's fault is not "as great as" D's. or not greater than D's.

Comparative negligence: P is in three-car pileup, all are negligent. B is 45%, would pay 45% of damages; C responsible for the other 45%; P is ten percent. two categories of comparative negligence:

Pure comparative negligence: you substract P's percentage and can collect the rest. Modified comparative negligence: fifty percent rule and forty-nine percent rule. When you compare P's negligence with the Ds, you combine the Ds negligence (if more than one), and P can collect up until the point where her negligence equals the Ds' negligence. If goes above fifty percent, can't collect anything from the D. Forty nine percent rule: P's neg has to be less than Ds, not equal to. Forty-six states use some variety of pure fifty percent or forty-nine percent rule.

Slocum v Donahue: again dealing with contribution. Ford settled with decedent's mom and dad. After they settled, Donahue tries to get contribution and/or indemnity from Ford. Why'd the court hold that Ford does not have to give contribution or indemnity: in most js, if you are a settling D before the verdict comes against the other co-D, that other D cannot come after you for contribution even though you may be j and s, have been a joint tortfeasor. Why have this rule: policy reason - if you allow D1 to get contribution from someone who's already settled, D2 won't settle. Denying contribution against a settling D promotes settlement.

Qualification to this rule: if you can show settlement is a sham, that they weren't really trying to resolve Ford's responsibility - were just trying to stick it to Donahue, colluded to go after him. another way settlement can be set aside: inadequate consideration. Normally court doesn't get involved in settlement amounts; but this can change if it's a wrongful death case. Ex, a dollar isn't enough for a life. but generally, court doesn't care - if kids are involved, class action, court has a duty though to adequately represent absent kids or Ps. But if just A sues B, court doesn't get involved.

Pagelsdorf v. Safeco Ins. Co. of America

Regarding defective premises: holding is that landlord must exercise ordinary care to his tenant and others on the premises with permission Here, balcony collapsed on second story - dryrot. The "implied warranty of habitability" creates a duty to leasor to deliver the premises in a habitable condition

CompuServe Inc. v. Cyber Promotions, Inc.: Concerns the right of online service to send unsolicited email. P trying to protect its servers from Ds' bulk email.

Restatement section 14: trespass to chattels has evolved from its og common law application, concerning primarily the asportation of another tangible property, to include the unauthorized use of personal property: its chief importance now, is that there may be recovery for interferences with the possession of chattels which are not sufficiently important to be classed as conversion, and so to compel the D to pay the full value of the thing with which he has interfered." In section 217: trespass to chattel may be committed by intentionally using or intermeddling with the chattel in possession of another. Restatement section 218 says that recovery may be had for a trespass that causes harm to something in which the possessor has a legally protected interest. Here, there was in fact harm - people terminated services bc of the spam emails.

Trespass to chattels: Glidden v. Szybiak

Restatement section 218: "One who without consensual or other privilege to do so uses or otherwise intentionally intermeddles with a chattel which is in possession of another is liable for a trespass to such person if, (a) the chattel is impaired as to its condition, quality or value, or (b) the possessor is deprived of the use of the chattel for a substantial time, or (c) bodily harm is thereby caused to the possessor or harm is caused to some person or thing in which the possessor has a legally protected interest. Unlike trespass to land, trespass to chattels will not lie unless there is actual dispossession or actual damage to the chattel itself or to its owner or his property. can't get nominal damages.

Hardy v. LaBelle's Distributing Co.: Employee shut up in showroom. Here, P did not ask to leave, was not told she couldn't leave, nothing done to compel her to stay, said she would've stayed voluntarily.

Restatement suggests that, in addition to physical barriers, a false imprisonment can be accomplished by force, threat of force, duress, or asserted legal authority. Seizing of P's property sometimes may provide the "restraint" necessary to constitute false imprisonment. Is generally agreed that false imprisonment resembles assault in that threats of future action aren't enough. Thus the action does not lie where the D merely threatens to call the police and have P arrested unless he stays.

§ If adult standard is applied, bc it's held that activity was inherently dangerous, did he breach his duty? He tried to apply brakes but applied gas - adult probably would not have done the same thing. i. Could argue a reasonable adult wouldn't have sped. ii. Also, he was distracted by the loud noise - adult'd normally put on brakes, do something other than keep speeding. Whatever a distracted, reasonable person would have done is what he should have done (jury will have to decide) § Note that he could be held to a different standard of care when he was in the grocery store. Since he's 14, and not doing an inherently dangerous activity or an adult activity. Look at what a reasonable prudent child of the same age would do.

Robinson Application - online hypo. On exam, make argument as to a kid doing/not doing an adult activity; for it being/not being inherently dangerous. Analyze for it existing/not existing. Tell what rules are, argue how that under each rule one is liable, then turn around and show how he isn't. Do we think the boy would be held to an adult standard? (ALSO: just because a child might be held to an adult standard, you can possibly argue they acted as a reasonable adult would).

Ranson v. Kitner

Rule in this case: if they intend the act, doesn't matter if they are mistaken about what precisely they were doing. Trespass of chattel here. they had the intent, the purpose of shooting the dog. The fact that they made a mistake is totally irrelevant. They shot, intended to hit what they shot. Another ex: if you grab the wrong person in a store. The only time a mistaken battery would not get you to be liable is if you are defending yourself.

i. If kid's under age of 7, is presumed incapable of negligence. Even if he gets in a car, drives down the road, is presumed to be inclusively incapable of negligence. ii. Kids 7-14 are going to be presumed to be incapable of negligence BUT may be proved capable of negligence. § How do you prove 7-14 may be proved capable: they've been instructed to not do x, have done x before. If they have experience, know the risk involved, can appreciate that risk, then you can prove they're capable of negligence. iii. If you're over 14, you're going to be presumed capable of negligence BUT may be proven incapable of negligence. § How: have to prove there's something about their upbringing that shows they don't understand the situation, don't know they're presenting harm to another. § Ex: kid in the Amazon is raised by natives; goes to US, doesn't know the rules, which side of the road to walk on. SO: his upbringing makes him unable to appreciate what he's doing exposes another to a risk of harm. iv. If you're in a Rule of 7 jurisdiction, note it only talks about CAPACITY. If you're under 7, you're not capable of negligence. If you're older, you may be proved capable. THEN we ask if you should be held to adult or child standard of care. Rule of 7: Says if you have even the capacity to be sued. Then and only then do you look at the standard of care

Rule of 7

Murphy v. Martin Oil Co. - ex of Survival statute Survival statutes provide that causes of action for injury to all tangible property survive the death of either party. The maj of statutes also allow personal injury actions to survive. Only a few states permit claims for intangible interests of personality to survive.

Since the survival statute merely conts the decedent's own cause of action, any defenses that might have been set up against him if he had lived are still available to the D, like comparative or contributory negligence

Facts of Elbaor v. Smith:

Smith hurt in an accident. Went to hospital, had many docs, including Elbaor. Sues all docs, makes "Mary Carter" agreement with all docs except Elbaor.

Ex of a non-delegable duty: dutyof a general contractor to construct a building safely, duty of landowner to maintain their property in a reasonably safe condition The brake repair co will ALSo be negligent for not fixing the brakes - P will be suing two people In short: there's no hard and fast rule as to what's a non-delegable duty.

Some areas: cleaning grocery stores, emergency room cases - docs aren't employees of hospitals but indep contractors (exposes public to serious risk of harm if not done with due care - despite being indep contractors, hospital still liable, though they can get indemnification from the docs if not negligent in hiring them), state liable for harm to foster kids despite the fact that foster parents are indep contractors (can also be directly liable for negligently engaging foster parents)

Contribution only exists between joint tortfeasors who are j and s liable. If severally liable, you will never be entitled to contribution bc you're already only paying your share.

Some js by statute have changed the common law rule that two people who cause an indivisible injury are j and s liable, say that the only ones who are j and s are those who were acting in concert. The only who act independently to cause an individual injury will just be severally liable, only responsible for his percentage fault share.

The family car or family purpose doctrine is a court created legal fiction by which the owner of a car is held vicariously liable when the car is negligently driven by a member of the immediate household.

Some older cases held that the mere presence of the owner in the car est his right of control over the driving. A number of js have departed from this idea. Doesn't relieve the owner of a duty to object to negligent driving, though. Many states have statutes that make the owner of a car vicariously liable for injury caused by the negligent operation of a car as long as it's being used with the owner's consent. What if owner is a rental co: many js have held that such cos are liable as the car owners.

There's some intentional torts, like defamation and fraud, that require the person to know they have made a false statement.

Someone insane might not be able to know it was false. If they can't form the requisite ability to know it was false, can't be liable for that type of intentional tort.

a. Facts: P fell in a post office after bumped by Burson, a blind operator of a concession stand in the building. Burson was going to the bathroom, bumped into P, who was older, slighter than him. P argues Burson was negligent bc he didn't carry a cane with him b. Issue: Was Burson negligent for going to bathroom without a cane c. Rule: handicapped people's "conduct...must be reasonable in light of his knowledge of his infirmity....must take precautions ...which an ordinary reasonable man'd take if he were blind." Analysis: Burson wasn't negligent. Expert testimony showed it wasn't uncommon for blind people to rely on techniques other than canes when moving in familiar settings. Burson was familiar with his surroundings, had good mobility skills. Engaged in no other action that could be seen as possibly negligent.

Standard of Care for the handicapped - Roberts v. State of Louisiana

vicarious liability ex: O'Shea v. Welch

Store manager, D, hit P at a service station. Driving to the district office to deliver football tickets to pass out to managers. Approximately half of he states have applied some form of the "slight deviation" rule: here, it must be decided if the employee was on a frolic or a detour; the latter is a deviation that is sufficiently related to the employment to fall in its scope, while the former is the pursuit of the employee's personal business as a substantial deviation from or an abandonment of the employment. If an employee wholly abandons, even temporarily, the employer's business for personal reasons, the act isn't within the scope of employment, and employer isn't liable under respondeat superior for the employee's conduct during that lapse. Act that are necessary to the comfort, convenience, health and welfare of the employee while at work aren't outside the scope of employment, if the conduct isn't a substantial deviation from the duties of employment.

In a personal injury case, a P is awarded a lump sum to compensate for all future pecuniary losses. Most js require that the lump sum award for future losses be reduced to its "present value."

That is, the jury's instructed to award the amount of money in a lump that will produce for the P the amount the P would have earned or will need for a future operation.

Where the statute of limitations on the personal injury action had run before the decedent's death, no survival action can be brought. But regarding the wrongful death action, some js have held that the statute runs against the death action only from the date of death, even though at that time the decedent's own act would've been barred while he was living. Others hold that that it runs from the time of the og injury and consequently that the death action may be lost before it ever has accrued.

The accrual of the cause of action/when the statute begins to run is dependent on the wording of the wrongful death statute. Some specify that the cause of action accrues at death. Of those statutes that don't specify the accrual of the cause of action, some have interpreted their statutes as providing that the cause of action accrues at death while others have held that the action accrues at the time of the injury that then causes death

Historically, many js divided implied assumption of the risk into categories of primary and secondary assumption of the risk:

The category of primary assumption of the risk collects cases that might more accurately be described as cases where the D owed no duty to the P or where the D didn't breach the limited duty owed to the P rather than that P assumed the risks inherent in the activity. Ex, those who attend baseball games outside the area behind home assume the risk of balls being hit into the stands. The category of secondary assumption of the risk collects cases where the P acts voluntarily but unreasonably to encounter a known risk. Many courts have ruled that implied secondary assumption of risk doesn't remain an affirmative defense separate and apart from contributory negligence.

Physical harm to property: you can have both person injury damages to your body (burn case, five or six categories) and physical harm to property. Ex of physical harm to property: if somebody runs into your car, harms it, you can sue them for negligence in harming your car/property.

The damages for property damages: if property is completely destroyed, they will be the fair market value of that property on the date and location where the negligence caused the damages. If the damaged property is only damaged, but not ddestroyed, then one way of assessing damages would be the diff of the fair market value before and after the damages. Sometimes when property is damages, another way to calculate damages is the repair costs. Another way of getting damages: if property is not damaged physically but only dispossessed. Ex, if somebody steals your car and rides around it in for a week - you didn't have loss of your car for a week. Your damages might be the rental price for having to ride around town for a week. You lost the use, the rental value of whatever your car is if there is no physical damage or dimination of value in the car Stocks and bonds: they stole your stocks, you couldn't sell them when they doubled in value when you weren't recorded as the owner of stocks. Some js would give you the highest intermediate value of the stocks from the day you lost them til the day you got them back. highest intermediate value Things like mom's dress, granny's china: have lotta sentimental value, maybe not a lot of market value. Courts struggle with that. sometimes the replacement value, plus a little bit more to compensate you for your attachment. Animals: when negligently killed, you can't get pain and suffering for their loss in most js if you, youre relative was negligently killed. Can sue for the vet bill, the price of the animals, but in most js not for emotional distress. NOTE: TN DOES allow about 5,000 in emotional distress damages for a lost pet if it's negligently caused by someone other than a vet.

Murrell v. Goertz: D assaulted P. D was making monthly collections for delivery of newspaper. Employer, D denies D was newspaper's employee. P argues D was servant of newspaper either by agreement, or by creating the belief that he worked for them. Question of independent contractor:

The demarcation tween an independent contractor and a servant isn't clearly drawn. A contractor is one who's engaged to do a certain service for another according to his own methods and manner, free from control of his employer in all matters connected with the performance of the service except as to the result thereof. The parties agree that the decisive test fro determining if a person is an employee or an independent contractor is the right to control the physical details of the work. Co had no input into the decision to hire D, had no knowledge of his employment; D had no direct contact with co. while co est certain policies to which all distributors were to adhere, such do nor rise to that level of supervision as to make D a servant. One who arranges for work to be done by an independent contractor isn't vicariously liable for the contractor's torts. The distinction between a servant and an indep contractor has been said to lie in the fact that the latter does the work in his own time, in his own way, and under no one's directions but his own, so that the one who selected the contractor has no control or right of control over the manner in which it's done. Ex: foster parents are deemed independent contractors of the state

In order for a person to be classified as an invitee, it's sufficient that he go on the land in furtherance of the owner's business. It's not necessary that the invited person gain an advantage by his entry on the land. A social guest is considered a licensee and has been defined as one who enters the premises of the owner by permission, but for the licensee's own purposes. Therefore, a social guest is a person who goes on another's property for companionship, diversion, or entertainment

The duty owed by the owner of premises towards an invitee is greater than that owed towards a licensee. A social guest as a licensee, generally must take the premises of his host as he finds them. However, the owner of the premises has a duty to warn the licensee of any hidden dangers which are unknown to his guest, of which he, the owner, has knowledge, and to refrain from injuring his guest willfully or wantonly. Towards an invitee, owner of the premises has a duty to exercise reasonable care in keeping the premises reasonably safe for use by the invitee. It was first held that there was no duty to a licensee except to refrain from inflicting willful or wanton injury; and there are still courts which hold this. Most courts have now overruled older decisions and hold that the D, in conducting active operations, is under a duty of reasonable care toward licensees. This duty extends not only to licensees who are discovered, but to those whose presence might be reasonably anticipated. If D landholder knows about a dangerous, latent condition on his premises that a visitor is likely to encounter, he is under a duty to warn licensee about it

There's some level of tension between first amend and intentional infliction of emotional distress.

The issue clash between right to protest, speak your mind, versus somebody having to hear what you're saying and be injured should be reconciled. Issue is what rights does the listener have. Where do you draw the line? P68: these are the four elements of intentional infliction of emotional distress we need to know for exam.

Tolling: statutes of limitation contain in them provisions that stop the running of the time within which to file for various reasons.

The most common of these provisions that toll the statute for minors, legally insane or incompetent, people that are out of state, and those Ds who have concealed their identity. Ordinarily, the tolling stops/clock starts running when the minor is of age or when D returns to the state or when the incompetent becomes competent again - like when somebody wakes after a coma. What if the incompetence is permanent? In a few js, the appointment of a guardian starts the clock; others have an outside time limit for filing actions on behalf of incompetent Ps, whether or not they have been restored to competency or had a guardian appointed. In other, appointment of a guardian doesn't affect the tolling bc it doesn't remove the disability. Alt: D's misrepresentation of his identity tolls the statute. In addition to tolling specified in the statute itself, many js recognize equitable rolling where the D fraudulently concealed injury from the P or concealed his identity. Ex, doc conceals med malpractice from patient. Such equitable tolling isn't available if someone other than the D has concealed D's identity.

Taylor v. Vallelunga: Doctrine of transferred intent. Daughter has ED from father's beating. Even if daughter's distress was severe, she has to prove she has had physical manifestations of emotional distress. Many courts require some kinds of physical manifestation. But even assuming she had physical manifestation, has to show extreme and outrageous. Beating somebody to a bloody pulp is extreme and outrageous. The main problem here though is that the D didn't know she was present. So the rule has developed that in order for a bystander to collect, P has to show that D knew they were present at the time they were doing the extreme and outrageous conduct. If don't know, then she can't prove that he had the intent to cause intentional infliction of emotional distress.

The rule has developed that in order for a bystander to collect damages for IIED, P has to show that the D knew she was present at the time he was doing the extreme and outrageous conduct. And if the D didn't know she wasn't present, can't prove he had intent to cause her IIED. Many states also tend to limit it to family members who are nearby when they see D commit an act that is extreme and outrageous on a family member. That's a way to limit the ds - if you didn't limit it, anybody walking by who randomly walking by could sue for IIED. Restatement is going to limit it to family members, so not just every random bystander can sue (unless they're pregnant, small kids, elderly).

regarding discovery rule, some js require that injury, malpractice occurs: in one of these js, you know normally P isn't going to know a sophisiticated malpractice claim occurred unless another doc tells her. frequently she's not going to know until someone tells her.

The standard is when a reasonable prudent person knew or should have known about the injury. This will be a determination for the jury whether or not a reasonable person would have known about the injury or not. Another thing: what is the injury - is it when her tubes weren't tied or when she got pregnant? Some js would say that the injury is when she gets pregnant.

make a distinction between willful and wanton negligence. If you discover someone on your land, doing active thing, must not act willfully and wantonly to hurt him. Trend is once you discover him, take reasonable care to guard from harm. Once discover him, have to drive the not recklessly, drive in a way to warn him - flashing lights, whistles, whatever a reasonable person would do to keep from injuring him. There are exceptions:

The tolerated trespasser is an exception: you know he's there. If you know, have to use reasonable care to guard against injuring him. must warn him of latent conditions; if warning doesn't do the job, have to take precautions to remove the condition if a reasonable prudent person would do that. Anticipated trespassers are another exception: ex you know your neighbor will walk across your back yard. You can anticipate that. have to use reasonable caution to guard against harm to him during the time you now he'll probably be there. That could be warning him, but if warning doesn't work, have to use reasonable care to make the premises safe for him.

Kline v. 1500 Massachusetts Ave. Apartment Corp. Issue: if landlord owed a duty to use reasonable care to guard against criminal behavior by a third party. Tenant robbed in a common hallway. Ordinarily, landlord could say the criminal act is superseding - supersedes any negligence he had in not having security available. So dealing with two issues: 1. If in first instance duty is owed, 2. Whether that duty is cut off by a third party. Duty here would be owed because it falls under one of six common law exceptions. But does the criminal intentional act cut off the liability/superseding intervening cause (Watson case): here court held that the negligence wasn't cut off by the criminal acts bc they were foreseeable.

The trend regarding superseding criminal acts is to see if it was foreseeable. If foreseeable, you're still liable. Whether the duty is breached depends on whether you're in a high or low crime area. What type of security must be provided: at minimum, the security that was provided the day that tenant entered into the lease. May need more, depending on whether criminal activity increases. In some areas, like hotels, crime is foreseeable, esp in high crime areas. Note four: any time anywhere someone has a chance of being robbed, when you go out into public you expose yourself to the risk of criminal activity - because of that, before a landlord is held to have a duty to guard against criminal behavior, there must be a HEIGHTENED foreseeability of criminal activity. What that means: ex there have been other mugging in this hallway, building, parking lot, etc - is historically known to have occurred. KNOW: sometimes a j, in respect to crime, might require heightened foreseeability and not just foreseeability. Note 6, p578: takes it farther than just landlords - extends rule of condo associations, board of directors of those associations. Ex when you have a condo, there's some stuff you can't do. If you're hurt bc you can't do something, then the board may be liable. This could also extend to homeowners' associations, bc they have control

Imputed contributory negligence: not a way to vicarious negligence. Way to say the owner is contributorily or comparative negligent bc of negligence of driver. Many js have gotten rid of this.

There are situations where negligence of one will be imputed to another: employer and employee, derivative claims. P781. Wrongful death claims are derivative claims: are imputed to the survivors who try to get money based upon the loss of benefits to the dead person. Can use the negligence of the deceased and impute it to the survivors, survivors recovery will be reduced by the dead's negligence. Mostly, a mom or dad's negligence in watching a kid won't be imputed to the kid when the kid sues a tortfeasors. But some js will. If there's contributory negligence, can impute negligence of contributorily neg party to the spouse/employer/car owner

1) that there is no genuine issue of material fact 2) they're entitled to a motion for judgment as a matter of law. i. Judgment as a matter of law: if there's a general issue of material act, D's not entitled to judgment as a matter of law bc jury could believe P's witnesses.

There are two standards for a summary judgment motion; filer must show:

Fisher v. Carrousel Motor Hotel: this is a battery, not an assault. They took the plate, they invaded the sphere of his personhood. There was no physical contact, but they snatched the plate out of his hand. court says that's a battery - how's they reach that conclusion? Anything someone is holding in their hand is to offend their dignity.

There is a sphere around a person, anything in that sphere is almost considered to be part of his or her body. Anything that's intimately connected with a person's body, if you touch it, constitutes a battery. Touching it invades his dignity This case is meant to emphasize someone to commit battery; you just have to touch something near or around him. If you yank somebody's hat off, that's battery. What we're trying to protect is the inviolability, dignity of one's body; when you invade the sphere around them, you've committed battery if you have the requisite intent or certainty. Reason why there's no assault: they used racial slurs, but there's no evidence that the P was placed in a reasonable apprehension of an imminent battery. Point where assault occurs: when P is placed in a reasonable apprehension of an imminent battery. In old timey case: at the point when a woman jerks her head back bc she's afraid of getting hit with a hatchet, assault occurred

In Bierczynski, if the two racers were racing in concert, why should one have to suffer paying all the money to the P, and one not?

They gave the one that pays a benefit - he can sue the other one and get a fair allocation of the amount of money to be paid, get a reallocation. If one who pays was 75% negligent, can get 25% reimbursement from the other D if the allocation of the fault during contribution is by percentage of fault. The remedy for one D who pays the total amount is to seek contribution against the other tortfeasor. One way to do that is to get percentage fault allocation. Another way to get allocation between Ds is through paratashare - D1 pays 50%, D2 pays 50%, if D1 has paid it all, can get a 50% reimbursement from the absent D. if absent D has no money, contribution isn't worth very much

Punitive damages: to punish or deter egregious behavior. Normally get them if the behavior of D was intentional or reckless. Some js may allow punitive damages for gross negligence (extreme departure from what reasonable prudent person would do) but usually need intentional or reckless behavior. Ex: Cheatham v. Pohle:

This is intentional behavior - there's nothing negligent/accidental about this. Main issue in the case: fifth amendment due process - can't take somebody's property without notice. This court ads another aspect: even when you give them notice, you can't capriciously take their property. If you take someone's property arbitrarily, that's a capricious taking of the property. If the property is not vested, then the Fifth Amendment is not applicable - aka, if the property right is not vested, you have no right to it. you have to have a vested property interest - have no such interest in punitive damages, so not taking can occur except if the jury has already awarded it to you. then they can't take your money away without giving you due process. You have no right to punitive damages unless the jury has awarded it to you. if they haven't done that, court can take away even the claim of punitive damages. You have no vested right to any tort claim or any tort damages unless the jury has awarded you a specific sum of money. Before that, can take away or restrict the claim any way they want to. A P that is capped at 500k for pain and suffering can't say you took away the other 100k I'm entitled to and thus violated my due process. Negligent torts: don't get punitive damages if it's simple negligence. Simple negligence is speeding, allowing a barrel to fall on somebody's head. Have to show that they were intentional or reckless in those acts to get punitive damages

The possessor of land is subject to liability to another as an invitee only for harm sustained while he is on the land within the scope of his invitation. Thus an invitee ceases to be an invitee after the expiration of that time he becomes a trespasser or a licensee will dependon whether the possessor does or doesn't consent to his remaining on the land. If the invitee goes outside the area of his invitation, he becomes a trespasser or a licensee, depending on whether he goes there without the consent of the possessor, or with such consent.

Thus, one who goes into a shop which occupies part of the building is a trespasser if he goes into the residential part of the premises without the shopkeeper's consent; but he is a licensee if the shopkeep permits him to go to the toilet or invites him to pay a social call. The scope of duty owed by an occupant to an invitee is one of reasonable care in all the circumstances.

Indemnity versus contribution:

Traditionally, while contribution allowed a tortfeasor to be partially reimbursed for money paid in judgment or settlement, indemnity was available to shift the entire cost of the judgment or settlement from a tortfeasor whose liability to the P wasn't based on its own wrongful conduct, but imposed on it by law bc of its relationship with the tortfeasor whose wrongful conduct caused the injury. For ex, the employer's liability for employee's conduct, car owner's liability for a driver's conduct, retailer's liability for the manufacturer's product would give rise to a cause of action for indemnity. NOTE: indemnity not available to a D jury found to be negligent.

If it's neg trespass, have to show actual damages; if didn't do any actual harm to the land, you'd lose (but if it's intentional, don't have to show that, can collect nominative and maybe punitive ds).

Trespass to land, don't have to show real damages - just entry to your land; can get nominal damages. Trespass to chattel: have to show real damages, either disposition or actual harm to the chattel. If you can show a trespass of land was intentionally done, can also get punitive damages. In trespass of chattel, usually get actual damages, and if can't show if intentional and reckless, can get punitive damages as well. For trespass of land, can get nominal damages. Trespass to chattel, you need real damages.

application of professional malpractice to Be Careful hypo: Before evaluating breach, look at duty - duty's dependent upon if jurisdiction follows nat standard of care, local standard, or similar locality standard a. If, on the exam, doesn't give a jurisdiction, you'll have to discuss it, apply rule in all 3 jurisdictions (say that there's three ways to look at the duty). b. Once you decide which applies, look at knowledge, best judgment, use of due care - all these are applicable to locality standards. Determine standard, then evaluate 3 elements that determine breach.

Two different causes of action can be brought here: medical malpractice; informed consent

IIED in terms of dead bodies:

Two situations: intentional infliction of emotional distress AND conversion. Most people will acknowledge an intentional infliction of emotional distress claim in this situation. We know people will suffer emotional distress if you mishandle their relatives. Second question: are those corpses property. if they are, then who owns them. The court has struggled with dead bodies in this context. For some purposes, dead bodies are property that's owned by the relatives or heirs. However, property rights are normally determined by state law, so it depends on what the state says. Note: if coroner takes a brain, tissue, probably don't have a claim, but you would with intentional infliction of emotional distress If state statute says a body is property, a coroner can't just take body parts from a dead body bc you own it.

Seaborne-Worsley v. Mintiens - imputed contributory negligence

Under the "controversial doctrine of imputed negligence," the negligence of one individual may be imputed to another who was otherwise without fault. In the realm of car torts, the doctrine has been applied to ascribe the negligence of a permissive driver of a car to the owner of the car if the driver operates the car negligently while the owner is a passenger. Contrast between negligent entrustment and imputed negligence: negligence entrustment is based on the negligent action of the car owner - ie, entrusting the car to an unreliable driver - and doesn't depend on if the owner is present at the time of the accident. It's a form of direct negligence and isn't a theory of vicarious liability. By contrast, the doctrine of imputed negligence isn't premised on any negligence of the car owner, but only on the owner's ownership and presence at the time of the accident. Court here doesn't hold up that wife is liable.

Should P be entitled to pain and suffering damages in a legal malpractice action? Pain and suffering over the fact that your lawyer screwed up dad's will?

Usually not. Can get money you lost, but not any money for your frustration. However, emotional distress IS available if the distress is foreseeable due to the attorney's egregious conduct. Ex, if somebody mishandles a dead body so that the body is hurt, destroyed, etc that's a very sensitive matter that may allow for emotional harm damages against the attorney. Immigration and kids being separated from parents: emotional stress may be a component of damages in that case. Litigation-induced stress: most courts don't allow damages for that. If in criminal case, P can sue for legal malpractice, but usually can't get damages just bc it was emotional or frustrating to be in jail.

respondeat superior - concept applied to hold the employer vicariously liable for their employees' negligence.

Vicarious liability implies that employer did not personally do anything wrong, but is being held liable bc the law says that if an employee is operating during the scope of her employment, employer is vicarious liable, under respondeat superior, for any injuries that employee causes. Hold employer liable for negligence of employee. The operative thing for this concept of vicarious liability: employee was operating in the scope of his employment. Ex, driving delivery truck from town to town.

Wallace v. Rosen breakdown:

W was student's parent, alarm sounded, R teacher told her to move it, touched her back, W fell. "for battery to be an appropriate instruction, the evidence had to support an inference nt only that R touched W, but that she did so in a rude, insolvent, or angry manner" "Absent expression to the contrary, consent is assumed to all those ordinary contacts which are customary and reasonably necessary to the common intercourse of life, such as a tap on the shoulder, casual jostling"

In Chang case, court says it's a question of fact as to whether it was foreseeable to Suzuki, when it made the car; it was foreseeable if that this accident could occur on the highway, a rescuer could show up, did what this rescuer did.

Was it foreseeable to them that a rescuer would show up and act as this rescuer did? Possibly - it's foreseeable that when you have people at a crash scene, people will show up, cops will tell them what to do. Some injury is foreseeable under direct cause - aka, in this theory of proximate cause, any other injury will also give liability to D, even if it wasn't foreseeable. Another possible intervening cause: truck that hit him. if it was negligently driven into him, have to ask if a reasoanable prudent person would think it's possible that a truck will come and negligently hit somebody with a flare standing x amount of distance away. i. Pittman says he thinks all of that is foreseeable. The only thing that could bring this chain under foreseeable test or direct cause test would be if truck driver intentionally drove into him. that'd be so extraordinary that nobody could foresee it.

• There's a companion to IIED: NeGLIGENT IED.

Way this works: D violates a duty to a family member who's near the accident scene, can sue for neg IED. Aka, the violence directed to my rel caused me IED. There are rules that govern neg IED - must est duty, breach, caution, damages.

Bartlett v. New Mexico Welding finds the exact opposite of Coney:

We go from contributory to comparative, get rid of j and s. each D can only be liable for his percentage fault. Both cases use comparative negligence. If in a comparative negligence case, can either keep j and s liability or get rid of it. the reason why first case keeps j and s is bc they say there's still only one injury, P still needs contribution, so why put her in a bad position - why put the risk of insolvency of one D on the P when both Ds were negligent - let Ds figure out percentage fault among themselves. In Coney, P suffers a risk of loss. Better hope that all Ds have money so can collect percentage fault from each. If one can't provide, can only get from the other one their percentage fault. If in both situations there's comparative negligence, and Ds are joint tortfeasors, may be liable to j and s liability. Question is who should bear the risk - it's a policy determination that courts decide differently.

Effects of a statute: once a court has adopted the statute/regulation/ordinance as the duty, the question is how does jury treat a violation of the statute?

What's the effect of violating a statute that's been adopted as the duty? Depends on jurisdiction.

a. Standard: same as a motion for sj - could a reasonable jury find for the P? i. If judge thinks so, thinks they could believe P's witnesses, will deny motion of judgment as a matter of law 1. Judge will likely deny motion for judgment as a matter of law; then D'll do their Examination in Chief. a. Calls witnesses that challenge P's witnesses, challenge there were damages. Doesn't ask leading questions; afterwards, P can cross-question and CAN ask leading questions then.

When P finishes calling witnesses, they rest their case. D usually then rises, asks for a motion for judgment as a matter of law/directed verdict

Some states have said that possession by a bona fide purchaser or other innocent converter isn't a sufficiently serious defiance of the owner's rights and hold that the possessor is liable only if refusing to return the goods on demand. Note: in most states, however, a conversion occurs as soon as the D takes dominion and control over the goods in a manner inconsistent with P's ownership.

When a converter offers to return the converted goods and the owner accepts, the return doesn't bar the action for conversion, but it must be taken into account to reduce the damages recovered. If the chattel is in the same condition as when it was taken, hasn't changed in value, and P hasn't suffered special damage through being deprived of it, the effect may be to reduce his recovery to nominal damages. Trad rule was that D couldn't force goods back upon owner in reduction of the ds. Some js though have held that when the conversion is an innocent one, the court may require the P to take back the goods and credit the D with tehri value. If someone buys your stolen watch, they're not a converter unless you ask them to give it back. historically, though, the rule is that the moment they buy it they're a converter. Depends on j. some js say not a converter til you ask him to return it and he doesn't. but the common law rule is that he's a converter when he buys it You don't have to accept the goods back, but if you do your diminution will be based on the time you didn't have it.

State farm case: in order to have violation of Due Process, one has to take property. What kind of property was taken here: money, 145mill.

When jury awards punitive damages against you, they're taking away your property; have to give you notice (there's going to be a trial, you better be ready), whatever rules that they have to take away your money must be fair and not done in an arbitrary, capricious way, can't be excessive. If the punitive damage award is excessive, then that's a violation of your due process by taking your money excessively. There must be some standard by which we can judge whether an award of punitive damages is excessive or not. If excessive, it's a violation of your Due Process right bc jury is taking away your property right without due process of law. •Excessive punitive damages award: taking away your property without due process of law. Substantively, it's not fair and capricious.

Assumption of Risk - Express. Ex: Seigneur v. national Fitness Institute, Inc.

When signed up for membership, Seigneur disclosed she had a history fo lower back problems, including a herniated disc, and that her general physical condition was poor. K had an Exculpatory Clause. This clause was clear, court upheld. K of adhesion, but doesn't mean they had disparate bargaining power - plenty of other gyms P could've gone to.

Campbell v. Weathers: customer allowed to go to the back of store. When he went there, status went from invitee to licensee.

When went to get a box, is serving his own purpose, not that of the store. Store only has to warn him of dangers that are not open and obvious. We didn't know the light was blown - we thought you could see the stairs. Bc didn't know, didn't have an opportunity to warn you about it.

differentiate between wrongful death and survival statutes:

Wrongful death is a statute you'd use if you're a beneficiary of the dead person. Give the beneficiaries a claim against a tortfeasor for any damages they've suffered bc tortfeasor negligently killed the decedent. In contrast, a survival action is a claim that the decedent himself had for damage they personally suffered, either property or personal injury damage, that they suffered form the tortfeasor. There are diff rules that apply to the diff claims. The v first statute that's discussed is the Wrongful Death statute. It's strictly construed, which means if something isn't provided for in the statute, normally the court won't enforce it. for ex, if statute says the 'spouse' may bring a claim, a live in girlfriend can't. they're strictly construed.

Sometimes, P will develop a second, more serious injury after an initial, less serious symptoms were noted. Generally:

Wtatute begins to run when P finds some form of actionable harm, not necessarily its fullest manifestation. Ex: cause of action against car manufacturer based on alleged design defect accrued on day of accident, not a year later when P was diagnosed with a resultant disease. What if injury happens in one state, but suit is filed in another: choice of law rules in most states provide that the law of the forum state determines the statute of limitations

if you expose employee to something on the job that makes them sick or drunk and hurt someone on the way home, you can be vicariously liable for harms caused by their negligence.

You can either sue employer for being directly liable for serving alcohol, or can sue them under vicarious liability concept, saying that technically wasn't operating in scope, but you gave her some kind of substance that you should've foreseen would've caused injury. Why here they sue under vicarious liability: no evidence employer did something negligent.

Gentry didn't go to jury. What can you change about the facts so that it should go to jury as per Reynolds: alleged negligent act was not maintaining the stairs.

a. Agent's alleged negligent act: negligently handled the weapon - didn't unload it, caried in on shoulder instead of pointing to the ground. Must pinpoint negligence of every D on a test. (he's definitely negligent - look at likelihood of harm, burden of precaution, etc). b. Lady who owned house's negligent act was in the way she maintained her steps. So why don't we go to the jury on whether she was negligent in the way she was maintaining her steps? They couldn't prove by a preponderance of the evidence that the disrepaired condition of the steps was a but for cause, or a substantial factor cause, in causing the fall, which in turn caused the gun to discharge. You have to show that the negligence in maintaining the steps was a but for or a substantial factor that caused the fall.

notes after Bierczynski:

a. Although the D can't affect the cause of action between the P and other potential Ds, the D can enforce whatever right to contribution or indemnity the D may have against another potential D by impleading, bringing a third-party complaint against, that other party or by later bringing a separate suit for contribution or indemnity against the other party b. "Joint and several liability" means that each of fseveral tortfeasors can be sued jointly with the others for the amount of the P's loss, and that each is also individually liable for the full amount of P's damages. The P can collect from any one of them or any group of them up to the full amount of P's damages as determined by the trier of fact c. The mere presence at the commission of a tort doesn't render an observer liable as if he were a participant. Note that those who encouraged or incited assailant or tortfeasor, however, would likely be liable.

1. "Joint and several liability" means that each of several tortfeasors can be sued jointly with the others for the amount of the P's loss, and that each is also individually liable for the full amount of P's damages. P can collect from any one of them or any group of them up to the full amount of P's damages as determined by trier of fact. There are three types of factual situation in which joint and several liability usually is imposed.

a. Bierczynski illustrates the first - that in which the tortfeasors acted in concert. b. When Ds fail to perform a common duty to the P. i. Included in this category are cases involving the liability of two parties based on their rel to each other, eg the liability of a master for servant's acts. c. When Ds act independently to cause an indivisible harm.

discuss proof of negligence under "the three ways in which a jury can be instructed on how to treat a violation of an adopted statute:

a. Breaking the statute is just ONE FACTOR to be considered in determining whether D acted as a reasonable prudent person would. Consider the statute in light of all the other evidence, circumstances; then decide if D acted as a reasonable person.

1. If the thing that makes the D negligent occurs, then it will also probably est proximate cause under the foreseeability test. So there's a connection between breach and proximate cause.

a. Burden and likelihood will inform the analysis of proximate cause under the foreseeability test. Aka, the foreseeability of a particular risk makes the D liable as a breach if the likelihood is high and burden is low. Aka, if you find a breach, you're probably going to find proximate cause. If the risk that materializes is the risk that made the person have a breach, the likelihood of it is also going to probably est foreseeability for proximate cause. Connection between breach and proximate cause dealing with foreseeability. Both have some element of likelihood of harm.

informed consent evaluation when applying professional malpractice to Be Careful hypo:

a. Considering her age, this might immediately be an issue. Perhaps he didn't tell her all the risks involved with the treatment. Also had a duty to inform her about the steroid - since that was brought up, he should've told her about both. i. Informed consent duty attaches at beginning of the treatment, and any time he recommends a treatment doc has a duty to give informed consent. Say risks, benefits, risks and benefits of alt treatments; risks, benefits of doing nothing. ii. If she mentions something else, then he has to say if it's an accepted treatment, say risks and benefits of it if it is. iii. Two standards with respect to informed consent: § reasonably prudent professional standard (duty to disclose risks/benefits that a reasonable prudent doc would disclose/risks of doing nothing/alt treatments; breaches when he doesn't disclose what a reasonable doc would - is an objective standard set by med profession. o Some people don't like this bc docs can get together, decide what they will/will not disclose), states differ on which standard applies. § reasonable prudent patient standard iv. If patient is unconscious, in an emergency situation, will ask family member. If no family member, they'll treat without informed consent.

dealing with two possibilities in Smith v. Providence Health Services:

a. Dealing with the loss of chance of better condition. That better condition could be a stroke victim that doesn't have paralysis in both legs, only one leg. b. Person could live instead of die. Misdiagnosis of cancer can lead to death. 1. These change the types of damages - instead of giving you damages for death, will give you damages for losing your chance of survival. One type of injury is the value of your life. in these types of cases, the damages is the loss of chance, and not your life. a. So what does the loss of chance look like: here, lost the chance of 33% improvement over his current condition. How much is that worth? i. What some courts do is that they say 'if you can show you lost some chance,' we'll give you the full value of your life. usually happens in cancer cases. Some js say we won't give you the full value of your life, just the percentage value of your life, aka the value of the reduction that you lost.

Notes after Watson:

a. Depending on j, criminal conduct doesn't automatically interrupt the causal ink. b. Probably impossible to state any comprehensive rule as to when a D will continue to be eliable for negligence after an intervening criminal act of a third person. Ex of where liability has still been found: i. Student left at park by school, was raped; criminal act doesn't cut off negligence of leaving her in a crimeful area. Question of fact for the jury. ii. D leaves keys in P's door iii. Co hired a sex offender as security guard liable when he accosted someone iv. RR co liable for putting off a passenger at the wrong stop, where she got raped, if it had reason to know of risk to passenger's safety

know that market share is a way to est cause in fact

a. Diff states have diff rules about what the exact market is - ie national market, market of the pharmacy from which a drug was bought, market of the state/city/etc. b. Note 4, p 333: most courts don't extend market share liability beyond drug cases/DES context. Though this note gives some other products that may be invoked

physician oriented standard:

a. Duty is: for a doc to give the risk and benefit disclosures that a reasonable prudent member of the profession/doc would give. Breach occurs when he doesn't give risk and benefit disclosures that a reasonable prudent doc would give. b. This's decided by docs getting together, deciding what risk disclosures they're going to give to somebody in x position. Breach occurs when a doc doesn't give benefit or risk disclosures that a reasonable doc would give. c. Lotta people don't like this bc docs set their own standards. d. Physician standard helps docs, patient standard helps patients more. i. Then why have doc standard of care: docs aren't boogeyman - care about taking care of patients. ALSO: if a state has probs keeping docs, then can't make it too hard for docs to make a living.

patient-oriented standard/reasonable prudent patient standard

a. Duty: to disclose those risks, benefits that're material to a patient's medical treatment. b. Definition of material risk - any risk that'd have an impact on the patient deciding to have the treatment. i. Duty isn't what this particular patient would find as material; it's an objective standard, what the objective reasonable patient would want to know: risks that are pertinent to the reasonable patient. c. Breach occurs when doc doesn't disclose a material risk or benefit that a reasonable prudent patient would want to know. How do you know this? Jury'll decide what risks are material, want to be known by a reasonable prudent patient. i. Patient may say "I want to know if I'll have a scar, if I knew I wouldn't have had surgery" but jury may say "reasonable person wouldn't find the scar to be material, wouldn't stop patient from getting a life-saving procedure." If doc says there's a 3% risk you die, a reasonable prudent person'd want to know that. The more serious the risk, the more likely it will be deemed to be material.

reorganization/things to know about proximate cuase:

a. Eggshal resul b. Rescuer doctrine c. Social host liability d. Suicides e. Third generation DES Ps. Ex Chan case: people can disagree on what is, is not foreseeable. In practice, the tests can be blended together, you might have to talk about direct cause and foreseeability test for one jury instruction. If you don't know, make all the tests work for you, explain them all to the jury.

Daubert causes us to ask whether expert testimony will be admissible. There are two levels to this:

a. Expert must have knowledge or expertise in the relevant field. i. Experts in this case did have that. aka, need to be docs and not quacks or pilots. b. Whether or not the expert's opinion that is given is credible to be accepted. i. Here, the opinion was not credible enough to be accepted. ii. Always going to get experts fighting with each other. That's not going to prevent one of them from testifying. But the opinion they give must be based on scientific knowledge, done by scientific methods. Court isn't concerned with their conclusion; concerned with the method that was used to arrive at the opinion. The court says that in order to have this opinion admitted into evidence, there's a certain process that must occur. There's a certain way the expert must go about showing that their opinion was based on good science and scientific methods. iii. Court cares that good scientific method caused them to reach this opinion. Say that the two factors we look at: was your research done pre-lawsuit (there's was not - only produced research to support this suit. Strike against it, bc normally research is done to improve knowledge). 2. Was research peer-reviewed. Those are the two ways we decide if something is good science. Just care about the method, the process. We'll let the other expert challenge the conclusion.

Kramer Service, Inc. v. Wilkins

a. FACTS: Appellant hotel owner challenged a judgment of the circuit court which entered a verdict for appellee visitor in the visitor's personal injury action against the hotel owner arising from an accident in which broken glass fell on the visitor's head. The visitor was a business associate of a hotel guest. The guest notified the hotel clerk that there was a break in the glass of the transom of his room. Later, the guest and visitor had a business meeting in the guest's room. When the visitor opened the door to leave, the broken glass fell onto his head, causing a wound on the temple, as well as two other wounds. The temple wound never healed and a cancer developed. The hotel owner's requested instruction that the cancer should not have been considered by the jury was refused and the jury awarded a large sum. a. ISSUE: May a hotel owner be was liable for a visitor's injury? Yes CONCLUSION: Court affirmed as to liability because there was sufficient competent evidence that the condition had existed for a sufficient length of time to charge the hotel owner with responsible notice thereof, and a reasonably prudent and careful operator should have foreseen the likelihood of injury. Court reversed, remanded as to the damages bc the expert testimony on whether the wound was the cause of the cancer entitled the hotel owner to the requested instruction. Court affirmed the judgment finding the hotel owner liable for P'sinjuries. Court reversed, remanded as to the amount of damages.

Yun v. Ford Motor Co.: Brief

a. FACTS: Chang Hak Yun (Chang) was struck by an automobile on the Garden State Parkway while retrieving a spare tire that had fallen off of a Ford van in which he was a passenger. Approximately seven months later, he died of the injuries sustained. Plaintiffs brought suit against the defendants, claiming that the apparatus connecting the spare tire to the rear of the van was defective. Plaintiffs claimed that the accident was a result of the "negligent manufacture, distribution, service and/or warranty" of the van and its parts by Ford, the manufacturer, and Castle, the dealership. Plaintiffs amended their complaint to add defendants Universal and Kim. Plaintiffs alleged that Universal was "responsible for the [negligent] installation, assembly, manufacture and/or distribution of a conversion kit to the defectively manufactured 1987 Ford [v]an." Plaintiffs contended that Kim had "improperly serviced the 1987 van and caused a hazardous condition to occur." In their third amended complaint plaintiffs alleged that Miller was "responsible for the [defective] manufacture of the spare tire carrier." b. ISSUE: Was the alleged defect in the attachment of the spare tire the proximate cause of Chang's death? No c. CONCLUSION: Under the most liberal interpretation, conduct constituting proximate cause "need only be a cause which sets off a foreseeable sequence of consequences, unbroken by any superseding cause, and which is a substantial factor in producing the particular injury." An alleged defect in the spare tire assembly caused the spare tire and other parts to fall off the van and roll across the Parkway. Because the van in which Chang was travelling came safely to rest at the side of the Parkway, his actions were "highly extraordinary." Chang's attempt to retrieve the parts involved crossing the Parkway in both directions--an activity which cannot be described as anything short of extraordinarily dangerous, if not suicidal, as the action proved. In the process of returning from the middle of the Parkway, Chang was struck by Mrs. Linderman and fatally injured. Logic and fairness dictate that liability should not extend to injuries received as a result of Chang's senseless decision to cross the Parkway under such dangerous conditions. Common sense should have persuaded Chang, who was only a passenger, to wait for assistance or abandon the bald tire and damaged assembly. The van could have been driven safely home.

joint tortfeasors ex - Bierczynski v. Rogers. Brief:

a. FACTS: Ds were involved in a speed competition in cars on the public highway when one of the D's cars crashed into Ps' car. Plaintiffs brought an action against defendants for concurrent negligence. After a trial, the jury found defendants were each negligent and the negligence of each defendant was the proximate cause of the accident. The defendant that did not strike plaintiffs' automobile appealed. Defendant contended it was error for the trial court to submit the issue of proximate cause to the jury, insofar as he was concerned. The court affirmed the lower court's decision. b. ISSUE: Could all defendant's be held accountable for damages to plaintiff? Yes c. CONCLUSION: Court held that participation in a motor vehicle race on a public highway was an act of concurrent negligence which imposed liability on each participant for any injury to a non-participant that resulted from the race. Court affirmed and held the issue of whether D's conduct was a proximate cause of the accident was a proper issue for the jury.

A. H.E. Butt Groc. Co. v. Resendez - actual notice case (evidence of negligence ex)

a. FACTS: P c fell near a grape display at D grocery store. b. ISSUE: Did P present any evidence that the grape display created an unreasonable risk for customers? No c. CONCLUSION: Court found the customer sampling bowl was recessed below the table's surface and had a railing around its edges, floor was a non-skid surface, mats were around the display tables, and warning cones were nearby. Court stated the mere fact a store had a sampling display could not be evidence of a condition on the premises that posed an unreasonable risk of harm. P had the burden to prove D had actual or constructive knowledge of a condition, the condition posed an unreasonable risk of harm, D didn't exercise reasonable care to reduce or to eliminate the risk, and D's failure to use such care proximately caused her injuries. P claimed the sampling display resulted in an unreasonable risk of harm. However, P presented no evidence the display created an unreasonable risk of customers falling on grapes.

Bartlett v. New Mexico Welding Supply, Inc. Brief:

a. FACTS: P hurt in a car accident involving an unknown driver, who quickly turned in front of plaintiff causing her to stop suddenly, and defendant, who struck plaintiff's car from behind. The jury was instructed to determine the proportions of fault among the parties and found defendant 30 percent at fault and the unknown driver 70 percent at fault. The trial court ordered a new trial because the jury was not instructed on joint and several liability. On appeal, the court reversed the order granting a new trial. b. ISSUE: Is a tortfeasor liable for all of the damages caused by concurrent tortfeasors under a theory of comparative negligence? No c. CONCLUSION: Court held that the trial court properly instructed jury to ascertain the percentage of negligence of each participant and that a concurrent tortfeasor could not be held liable for damages caused by an unknown concurrent tortfeasor. The court held that in a comparative negligence case, a concurrent tortfeasor was liable only for the percentage of damages for which he was held accountable.

Brief for Derdiarian:

a. FACTS: Plaintiff employee was injured on an excavation job when he was struck by a car driven by a man suffering an epileptic seizure. Plaintiff and plaintiff's wife sued defendant employer, defendant driver, and defendant contractor for negligence, claiming that the employer failed to maintain a safe work site by erecting a traffic barrier. The trial court found for plaintiff, apportioning D employer's liability at 55 percent. On appeal, D employer argued that there was no causal link to P's injuries. b. ISSUE: Was there sufficient evidence that inadequate safety precautions on the work site by employer were the proximate cause of the accident? Yes c. CONCLUSION: The court affirmed the decision holding defendant employer liable for negligence because plaintiff employee's injuries were a foreseeable result of defendant employer's failure to maintain safe work site. From the evidence in the record, the jury could have found that the defendant negligently failed to safeguard the excavation site. A prime hazard associated with such dereliction is the possibility that a driver will negligently enter the work site and cause injury to a worker. That the driver was negligent, or even reckless, does not insulate defendant from liability.

Brief for Watson v. Kentucky & Indiana Bridge & R.R. Co.

a. FACTS: The victim brought a negligence action against the railroads and a tank car company to recover damages for the personal injuries he sustained from a gas explosion that occurred after a train derailment. It was undisputed that the explosion was caused by a lighted match thrown onto the street by an individual, who claimed to have used it to ignite a cigar. The trial court gave the jury peremptory instruction that although the railroad company may have been negligent in permitting the tank car to be derailed, such negligence was not the proximate. Instead, it was the individual's act of throwing the match purposely that was the proximate cause thereof. Following such instruction, the jury returned a verdict in favor of the railroads and tank car company. b. ISSUE: May the trial court determine proximate cause for the jury? No c. CONCLUSION: The court reversed and remanded the judgment in part because in holding that the individual in lighting or throwing the match acted maliciously or with the intent to cause the explosion, the trial court invaded the province of the jury. It was for the jury and not the trial court to determine from all the evidence whether the lighting of the match was done by the individual inadvertently or negligently, or whether it was a wanton and malicious act.

Boyce v. Brown:

a. Facts: D doc, P patient. Helped her broken ankle, years later P consulted D again on her ankle pain. D tried to help, but she didn't improve. Two years later P went to him again, then went to another doc who found necrosis around the screw D'd originally put in. i. Alleged act of malpractice: D didn't take an X-ray of her ankle when he did follow-up on. P wants to sue for not taking an X-ray. ii. Expert says he would have taken an xray, but we're not asking a subjective question of what that expert would have done - would a reasonable prudent doc have taken an x-ray? iii. Expert never testifies that X-ray SHOULD have been taken - probably bc he didn't know what condition it was in during that time. They SHOULD've asked him: 'assuming it was in the same condition then as it was when she came to you, would a reasonable doc have taken an xray.' b. Rule: standard of medical practice in the community must be shown by affirmative evidence. No presumption of negligence should arise from the fact that a treatment was unsuccessful. Negligence by a doc must be established by expert testimony unless the negligence is so grossly apparent that a layman would have no hard time recognizing it. c. Analysis: testimony of other docs that they'd have given diff treatment than D isn't enough to est malpractice unless the treatment deviated from one of the methods of treatment approved by the community's standard. Court thinks it's too far to say it's negligence not to take X-ray; too expensive, don't always give a satisfactory diagnosis.

Violation of a statute ex - Osborne v. McMaster

a. Facts: D's clerk sold to P's intestate a deadly poison without labeling it poised, as required by statute. P took it, died. 1. What was the statute: poison has to be labeled. 2. P was in the class - meant to protect people that buy poison. a. Is ANYONE that comes in contact with this poison in the realm of people meant for this statute to protect? Yes - they could potentially use the substance and get injured.

Hodges v. Carter (professional malpractice case)

a. Facts: Ds were P's lawyers; P claims Ds were negligent in prosecuting for him bc they failed to have process properly served and sue out alias summonses/additional summons. Attorneys served by following local custom, put it in the mail; mail should not have been used, should have been served in-person. No jurisdiction bc Ds were inappropriately served. Trial court though said it was ok bc it was local custom to send by mail, continued to trial. Appellate court held that mail service was not sufficient; needed physical service. Bc appellate court held that in-person service was needed, statute of limitations continued to run, eventually it ran out bc you served the summons incorrectly. P then decides to sue lawyers bc couldn't sue og D. Court said: lawyer not guilty of malpractice - can't be held answerable for a mere error of judgment. b. Rule: Rule 11 - attorneys must act in good faith and in an honest belief that his advice, acts are well founded and in client's best interest. Not answerable for a mere error of judgment of a mistake in a point of law which hasn't BEEN SETTLED by the court of last resort and, thus, reasonable doubt may be entertained by well-informed attorneys. Analysis: P has given no evidence showing that Ds breached their duty. They were acting as lawyers had in the state for years.

Violation of a statute ex: Martin v. Herzog

a. Facts: P and husband were hit by D's car. D charged with negligence for not driving farther away from the center of the highway. P charged for not driving with his lights on b. Rule: a statute designed for the protection of life is not to be brushed aside as a form of words, its commands reduced to the level of cautions. c. Conclusion: law says there must be lights. To ignore that law is to fall short of diligence. 1. Statute says you should have lights on your buggy. Question is if the court should adopt that ordinance. Court did adopt it, but court tells jury that they could give it whatever weight they wanted. Could just use it as evidence of negligence. This court in this jurisdiction says that once you adopt a statute, a violation of the statute must be negligence per se - jury has no ability to determine negligence. That means driver of the buggy must be negligence from the sole fact that they didn't have lights.

Standard of care for mental illness - Breunig v. American Family Ins. Co.

a. Facts: P was hit by a car driven on the wrong side of the road by Erma Veith. Veith had a mental episode. D is Veith's insurance co. b. Issue: does a person with a mental illness have liability for their torts? Here, did this woman any forewarning of her mental episode? (if so, then she's liable; if not, then no). c. Rule: generally speaking, people with mental illnesses are held liable on a case by case basis - do they have forewarning of the illness, is the illness such that it affects their ability to understand their duty to act as a reasonable person. d. Analysis: court holds that the idea "insanity is no defense" is too broad when a driver suddenly has a mental disorder. Holds that sudden mental eps are equivalent to heart attacks, fainting and shouldn't be treated under the general insanity rule. There's evidence she had forewarning; this question of forewarning should be left to jury.

A. Morrison v. MacNamara - deals with board-certified lab doing a smear test.

a. Facts: P went in for a UTI test. Test administered while standing; P fainted, hit his head b. Procedure: in trial court, P presented an expert who testified that the nat standard of care requires patient sit or lie down during the test. Trial court refused to instruct jury that a national standard applied, instead instructing them that local standard applied. P appeals. c. Issue: is a national standard or a local standard sufficient? d. Rule: locality rule states that the conduct of members of the medical profession is measured by standard of conduct expected of other members in the same locality. Locality rule was premised on notion that the disparity in edu, access to advances in science between rural, urban docs required they be held to a different standard of care e. Analysis: locality rule's unique to med field. Other courts have adopted a nat standard of care - situations of 19th c no longer apply. Docs train according to national standards, National standard should be followed. f. Conclusion: local standard is sufficient, court agrees with P

Lack of Informed Consent ex - Moore v. The Regents of the U of CA (here, using the reasonable patient standard, not reasonable physician, for informed consent).

a. Facts: P went to Golde for leukemia treatment. Golde advised P get spleen removed, P consented. Golde removed it to facilitate his research. Golde patented a cell line from Moore's cells, licensed it. P filed a complaint containing thirteen causes of action. b. Issue: Golde failed to disclose his economic interest in P's cells before obtaining consent to the medical procedures by which the cells were extracted. c. Rule: Doc must disclose personal interests unrelated to patient's health, whether research or economic, that may affect his professional judgment; doc's failure to disclose such interests may give rise to a cause of action for lack of informed consent d. Analysis: doc potentially has conflicting loyalties if has an eco interest in their care. A doc who's seeking patient's consent for a med procedure must, to satisfy his fiduciary duty and to obtain the patient's informed consent, disclose personal interests unrelated to the patient's health, whether research or eco, that may affect his medical judgment.

violation of a statute ex - Stachniewicz v. Mar-Cam Corp

a. Facts: fight broke out in bar; when it ended, P had head injuries, amnesia. b. Issue: whether violations of OR laws constitute negligence as a matter of law. c. Rule: a violation of a statute constitutes negligence when the violation results in injury to a member of the class of persons intended to be protected by the statute and when the harm is the kind which the statute was enacted to prevent. d. Analysis: Bc P was within class of people intended to be protected by the statute, harm was the kind it was intended to prevent, hold that the trial court erred in not treated the alleged violations of the regulation as negligence as a matter of law.

Violation of a statute ex: Zeni v. Anderson

a. Facts: instead of using snow covered sidewalk, which would have made her walk across the street twice to get to her job, P walked on a pedestrian snowpath with her back to oncoming traffic. D hit P b. Issue: was there contributory negligence bc P broke state statute which required people to walk on sidewalks when available c. Rule: statute can be rebutted by a showing an adequate excuse under the facts, circumstances of the case. "if there is sufficient excuse or justification, there is ordinarily no violation of a statute and the statutory standard is inapplicable." d. This case classifies 3 approaches to the question of what effect to give to the violation of a statute into three groups: 1. Negligence per se; 2. Prima facie (rebuttable presumption) negligence; 3. Some evidence of negligence. An unexcused violation is negligence per se. Statute has a lot of wiggle room, just says has to be 'practicable' to walk on sidewalk. Couldn't follow statute, so did what a reasonable person would do, next best thing. Her excuse was enough to rebut presumption that she was negligence for not walking on the sidewalk. MI is a rebuttal state, so statute doesn't apply, look to reasonable person standard.

Ex of professional malpractice (evaluated on education, best judgment, due care): Heath v. Swift Wings, Inc.

a. Facts: pilot was flying a plane in which his wife, son, friend were riding. Crashed, all died. Pilot's daughter brought a case on behalf of the estates of his son and wife. Expert witness said that pilot didn't act as a reasonable pilot should have. b. Issue: (breach) was pilot negligent in operation of plane? c. Rule: in trial court, was said that "negligence should be more specifically defined as the failure to exercise that degree of ordinary care which an ordinary prudent pilot, having the same training as the pilot here, would have used in the same circumstances." Analysis: trial court erred in introing subjective standard of care, specifically in "having the same training, experience as the pilot here." Professional standard remains objective. For ex, recognized standard for a doc is "the standard of professional competence customary in similar communities among physicians engaged in his field of practice." Should look at standard of all pilots, not just pilots like this one.

Consider the issue of whether it's fair/appropriate to apply the statute in Perry: court says other facts in this are whether there already exists a common law duty to do what the statute says you shouldn't do. If there is no common law duty, then that's a strike against it being appropriate, fair, workable, and wise.

a. Here, court says there's no common law duty to come to the aid of another (swimming pool don't have to say drowning man). i. So says Ds may have suspected abuse, but there's no common law duty to do anything. Statute imposes a duty that didn't exist in common law, and thus it's a strike against adopting it. Ideally just adopt statutes that affirm common law duty. ii. The jury, in applying the reasonable person standard, wouldn't find a duty here bc there's no common law duty to come to the aid of another b. Second factor: whether statute gave notice as to how to satisfy the statute. Here, they did have notice what to do. But we necessarily wouldn't, bc if we just hear a crying kid, we don't know what's going on. i. Under this statute, unless we want to get sued, have to call cops every time we see a bruise on a kid. So the court says this statute does not give all people to whom it'd be applicable notice on how to fulfill it, what circumstances to fulfill it. c. Another factor: if you look at the sanction, whether they could go to jail or get a fine, see how disproportionate is the fine/sentence tween the abuser and the person that reports it. when you see a big disparity, it's not appropriate to adopt one of these statutes. i. Court takes it a step further - not only do you have to compare the criminal and civil sanctions, but also compare the amount of liability Ds'd suffer criminally against what they'd suffer civilly. D may get prosecuted, common law, get a fine. When you see a disparity between the criminal sanction under the statute and tort liability, that's a factor against incorporating the statute.

notes following Sindell deal with products liability, takes us farther than negligence - have generic manufacturer of drug

a. Here, the case says that if the manufacturer makes a fraudulent in its advertisement or its package insert, that that misrepresentation can't be used against the og manufacturer if the P took the drug made by another manufacturer. But prof says that doesn't wholly agree - the generic drug manufacturer must use the same labelling as the og drug manufacturer. b. Things you should be able to sue them - generic seller has to use the same language as the og manufacturer. So in the case, though, you have to show that you took a co's drug before you can sue them.

what does Palsgraf add to our knowledge: zone of danger

a. How do you know if something is in the zone of danger? The person injured has to be in the zone of injury. If the one who's injured is not in the zone of danger? There's no duty. b. The majority opinion dismisses the case based on duty. We don't even get to proximate cause analysis. This is because no duty was owed to P because she wasn't in the zone of danger. She wasn't in the zone of danger, factually, bc the package appeared to the guard to be harmless. What would have been in the zone of danger of the guard's negligent act: anyone nearby - foreseeable that they might have been hurt if the guy fell on him, etc. c. P was far down the platform. Not in the radius of the fall of the package, falling man. So Cordoza says she wasn't in zone of danger of the things that could be foreseeably seen. d. If the package had had dynamite on it, the zone of danger would be greatly expanded, would have included everybody in the blast radius. Guard would have to be put on notice to change his behavior, to not do a negligent act because the gravity of harm has increased. A reasonable person, knowing that, would change his behavior i. So it depends on what a reasonable prudent person would have known at the time of his negligent act. If the package was marked, he would have known a lot more risks.

Richardson v. Chapman Brief:

a. ISSUE: Were the damages awarded excessive? Yes b. CONCLUSION: Court held the car driver's economist was not required to use "neutral" figures-amounts having no relation to the damages alleged by the parties-in explaining the concept of present cash value to the jury. The economist's methodology was reasonable because he did not make a prediction of future growth and inflation rates, he was consistent in his treatment of inflation, and he did not adopt a method that would under or over compensate the car driver. The court reduced the differential between the jury's award for the car driver's future medical expenses and the higher figure presented in the testimony because only a portion of the difference was allowed for expenses that the car driver was likely to incur, but that were not included in the calculations. Court found that the car passenger's award for pain and suffering was excessive and reduced it.

1. Res ipsa is supposed to be usable when you don't know what caused the instrumentality that injured you. in chair case, didn't know what cause the chair. When you know what caused your injury, don't need to use res ipsa.

a. If you know the employee pushed the barrel out the window, can just use ordinary duty. When you can explain in your complaint exactly what happened, won't be able to use res ipsa.

If court does adopt the statute as the duty, duty becomes not 'what a reasonable person...' but to do what statute says should be done. Breach occurs when somebody doesn't do what statute says to do. Will be up to jury to decide if person did, didn't do what the statute says. Normally, a licensing statute won't be adopted as the duty:

a. If you practice law without a license, there's a criminal statute; you can be prosecuted for practicing without it. If you drive a car without a license, can be criminally prosecuted. b. Court usually won't adopt these statutes as a duty bc the failure to have a license isn't what would make you directly liable for an injury to someone. i. If you're a lawyer, you're directly liable if you commit malpractice - don't have requisite knowledge, skills; use best judgement; use due diligence. Having a license is no guarantee you have those 3 qualities. Failure to have one is no definitive evidence you're negligent. ii. If you're sued for malpractice, going to be sued for not having requisite knowledge; due care; best judgment, not for not having a license. Might be put in jail for not having a license, but won't be successfully sued if you have 3 factors.

1. What's the test we can use to determine when cause in fact exists: something is a cause in fact of an accident if it a substantial factor in causing an accident - if accident would not have occurred but for that negligence.

a. In Perkins, since this accident would've happened even if they hadn't been speeding, speeding wasn't a cause in fact of the accident. 1. There's only one cause of action, so 'but for' can be used to determine if the negligence of train in speeding was a cause of fact in causing injury of P. If injury would have occurred even without the speeding, was NOT a 'but for' cause.

Watson v. Kentucky & Indiana Bridge & R.R. Co.: Only difference between this case and Felix case is that in Felix, car negligently drove into the hole - in the risk of harm created by not having barricades, was foreseeable, not cut off D's negligence.

a. In Watson, there's a question if the person intentionally threw the match. If it was intentional, then his behavior is so extraordinary that it will cut off the liability of the D for spilling gas. b. In both cases, the injury that occurs was within the risk that made the negligent behavior in both of those cases negligent in the first place. What is NOT in the risk is that somebody intentionally drives car in the hole, lights the match. Intentional behavior isn't foreseeable as identified by these cases c. In Watson, D's negligence exists in having no barricades, in spilling gas. You have an intervening act - the car goes negligently goes into the hold, a man throws a match into the gasoline possibly intentionally. If not intentional, won't cut off D's negligent. If intentional, WILL cut off D's negligence - is superceding. Won't be proximate cause.

Notes following Bartlett:

a. In both Coney and Bartlett, the negligent conduct of more than one actor combined to produce a particular injury to P. the negligence of each is found to be a proximate cause of that injury. The injury has therefore been treated as indivisible. b. Ds in states that adopted comparative negligence argue that the concept of comparative negligence should be applied to compare the negligence of the Ds to each other as well as to compare the negligence of the Ds to the P and that each D be responsible for only that part of the injury that the percentage of fault apportioned to him bears to the total negligence of all tortfeasors. c. Compare Several liability and joint and several liability: the principal difference between the two systems arises when the share of one or more tortfeasors can't be collected from them. Under j and s, P can collect from one or more of the tortfeasors and leave it up to that D either to obtain contribution from the others or to bear it alone. Under several liability, each tortfeasors pays no more than its apportioned share, and the injured party bears the loss of any uncollectible share. d. Some states have attempted to ameliorate the effect of an insolvent D by providing for a reallocation of the share of the insolvent D among the other parties according to their respective percentages of fault

Chang test introduces the intervening cause situation.

a. Intervening cause: can apply whether you're using direct cause or foreseeability. In either of these tests there's something called an intervening cause. SO they're actually using the foreseeability test here.

Takeaways from Boyce v. Brown:

a. It's not enough the expert said he wouldn't personally follow D's practice; he must also testify that the practice didn't conform with the standard of care of an ordinary member of the profession b. Proof that a professional violated standard of care usually must be est by expert testimony unless negligence is so obvious that it's within jurors' common knowledge c. Random notes i. If the matter's in the jury's common knowledge, don't need an expert - don't need an expert to tell you a doc is wrong for cutting off the wrong leg of a client. ii. you want an expert in the correct field - if you're dealing with an obgyn, you need a gyno witness, not a cardiologist. You also want to get an expert that is board-certified. SO for your experts: same specialty, board-certified.

Normally on foreseeability, normally the judge will say if you find the D was negligent in breaching his or her duty. In order to analyze proximate cause, you will use the foreseeability test. If you find it was foreseeable, then the foreseeability test has been met." ALT: if you find the act was possible, then you shall find it was foreseeable, and you shall find it was the proximate cause of the P's injury. Even if something is highly unlikely, judge will probably let it go to the jury - jury decides if it's reasonable if its possible.

a. Jury might find that a super remote risk is still possible. In analyzing duty, breach, and proximate cause, a jury could come to the conclusion that a 1% risk was possible, therefore it's foreseeable, therefore proximate cause is est. IF THE GRAVITY OF HARM IS OF SUCH A SUBSTANTIAL CALAMITY or destruction or death or serious injury, the court is more likely to let the case go to the jury on the proximate cause issue, just like it will with the duty, breach issue. If risk is low but potential harm is super high, judge will probably let case go to the jury on proximate cause. Ex, Chan case, appellate court said hold on we have a death, so let it go to the jury - doesn't matter if he might have been foolish. b. On test, make that argument for foreseeability. Remember, you're just trying to get to the jury. Make the best argument you can - say judge, in this previous case like mine, they let it go to the jury even though there was a 1% foreseeability, therefore you should let me go to the jury. Argue the alternative, too

Ortega v. Kmart gives the rule that for this j, if you can prove that there was a substance on the floor you fell on, can prove that no reasonable inspection had been done by the store for a reasonable period of time, that in itself is enough to take the case to the jury.

a. Jury will decide, looking at that evidence, whether more likely than not it was or was not there and a reasonable person would have removed it. Aka, don't know how long something was there, but you gotta use whatever you can to get the case to the jury. b. Here, no idea about the appearance of the milk. Can't be an indication as to how long it was there but what can be was that no inspection was done in a certain amount of time. c. Just the fact that they didn't do an inspection in a reasonable period of time is enough to get the case to jury. Jury may/may not find it was there long enough, but if it never gets to jury, you're going to automatically loose.

Alleged negligent act in Reynolds: no hand rail, not lit. the probable thing that's going to occur if you don't have these things is that someone's going to fall. That's what happened here. What this case basically says is that when the probable thing occurs, that's going to be enough to get to the jury; P just has to show that the negligence was more likely than not the cause of the injury.

a. Just the occurrence of the accident can be enough to get to the jury - the probable thing occurred. P doesn't have to show absolutely that the negligence in not having a light, not having a handrail, was beyond a reasonable doubt; just has to show that those negligent things were more likely than not the cause of her injury. Your negligence was probably the reason that she was injured, so we're going to the injury. Jury will decide if that negligence was more likely than not the cause of the injury. The fact that there are other factors that could have caused her injury isn't going to make us dismiss the case at sj stage, j as a matter of law stage. We're going to look at the probably thing, let it go to the jury.

in Daubert, court used Frye test to show that the experts were not able to est cause in fact:

a. Must determine nothing less than whether the experts' testimony reflects 'scientific knowledge,' whether their finidngs are 'derived by the scientific method,' and whether their work product amounts to 'good science.' We must ensure that the proposed expert testimony is 'relevant to the task at hand,' ie that is logically advances a material aspect of the proposing party's case.

'exercise of best judgment' under professional duty:

a. Must exercise his best judgment in carrying out a particular action (ie in lawyers case, best judgment in filing the complaint). b. What does it mean for a prof to exercise their best judgment? i. If reasonable prudent doc would only use a cat scan, and a doc decides to use an x-ray, you can argue that a reasonable prudent doc would not do that. ii. Doc's exercise of his best judgment is only going to give him a defense if the decision he makes is within acceptable practice. iii. Ex, if only a reasonable doc would order a cat or MRI, and doc uses xray, they didn't use best judgment, can't use this defense - can't go outside of acceptable practice that an ordinary member of the profession would do. iv. Have to act within confines of the profession - is an objective, not subjective standard. There may be many diff treatments - as long as treatment's acceptable, doc can choose that; but if outside of acceptable treatments, won't fall in the reasonable judgment rule.

res ipsa and experts:

a. Normally, jury has layperson's knowledge when res ipsa is applicable - know that barrels usually don't' fall from windows without negligence. Res ipsa normally just applicable to things that don't need an expert. b. However, there are time you do need an expert - even when the matter may be outside the knowledge of a layperson, depending on the j, res ipsa may be used even if outside confines of laypeople's knowledge. Ex, if paralysis occurred bc of surgery. c. If incident is outside of laypeople's knowledge as to whether the item usually happens with, without negligence, usually, then need an expert. Note 1A-G on p274.

Notes after Fuller:

a. Note 1: generally, courts view suicide as a deliberate, intentional act that breaks the chain of causation. b. Notes 2: early on, courts resolutely denied claims based on causing suicide if the decedent had even a remote awareness of what he was doing. This was broadened somewhat when some courts permitted recovery if the decedent's injury caused an irresistible impulse to commit suicide. Note 3 says if somebody encourages somebody to commit suicide, if they intentionally tried to cause it, and somebody intentionally commits suicide then, that won't be enough to cut off the promoter's liability in promoting it. those who promote suicide can't say that the suicide's intentional killing cuts off their liability.

1. If judge decides to not adopt a statute as duty, duty then's to do what a reasonable prudent person would do.

a. Note: even if you don't adopt the statute, what a reasonable person may be to do what the statute says. Have to look at likelihood of harm, gravity of harm, burden of precaution.

zone of danger discussion: There's this concept that what makes a breach exist is that the D's act must expose somebody to a risk of harm.

a. Notes, p368 - if you were driving your milk truck in the middle of the desert, the closest care was miles and miles away, you can drive as fast as you can go, and you're not liable for any injury bc there's nobody there who's in the risk of your driving. Speeding is not relational to anybody who could get hurt. Not exposing anyone to unreasonable risk of harm. What makes your act negligent is that you're exposing somebody to a risk of harm. Cordoza said somebody must be in your zone of danger for you to have a duty.

'use of due care' in professional duty:

a. Once you embark upon an acceptable practice, must use reasonable care in its execution i. Ex, a pilot took the choice of landing a plane in an emergency - a reasonable pilot would've done certain acts when landing. ii. If doc chooses surgery for cancer, but doesn't identify a nerve, paralyzes you, didn't use due care in performing an acceptable treatment - a reasonable doc would locate the nerve, not cut it.

1. Res ipsa basically involves breach. How do you prove breach.

a. One way to prove breach is though circumstantial evidence (banana peel), in res ipsa there's also circumstantial evidence, just to the extent that we don't know for sure caused something to happen, but bc D had possession or control or the right to control the item, and the accidents usually don't occur without negligence, that's enough circumstantial evidence to get the case to the jury.

1. Byrne v. Boadle - Res ipsa loquitor

a. P walks past D's shop, barrel of flour fell on him from a window above the shop. P was bound to give proof of negligence, but there wasn't any evidence unless the occurrence is itself evidence of negligence. Aka, the mere fact of the accident having occurred is evidence of negligence. a. 1st req of res ipsa loquitor is a basis of past experience which reasonably permits the concision that such events don't ordinarily occur unless someone has been negligent. b. P doesn't know how barrel fell, but inference is that D or emps were handling barrel when it fell. We don't know this for sure; only know what probably happened. Only need probability, though. The mere fact that it fell, that D had control and possession of it, is enough for us to believe this D probably was negligent

three elements (possession of knowledge of skill; exercise of best knowledge; use of due care) are the DUTY OF ALL PROFESSIONALS. If one's not met, that's breach.

a. P'll always have to say what the prof should have done. Jury will then have to evaluate that claim, evaluate the 3 elements of the duty, when the prof did what they did. Have to use expert witnesses to do this, have expert testimony from P. There'll be dueling experts - each side will have one. Jury will have to decide who is telling the truth. b. Note that malpractice is intertwined with contract law as well.

foreseeability test (for proximate cause):

a. P345. Note 4. Alleged negligent act in note 4 was that they didn't do appropriate screening. The question becomes assuming that they were negligent in doing the screening, assuming that the proper screening would've noticed the terrorists, what about proximate cause? i. Depends on what test you use. Here, seem to use foreseeability test (diff test than what we've had). Held that when terrorists killed people, that destruction was proximately caused by the negligent screening. ii. Negligence in screening people when they got on the plane was proximate cause of the destruction of the building. It's foreseeable if you don't do proper screening, bad people might hijack the plane. b. Note 5 deals with intentional torts - the concept here is that if you commit an intentional tort, then the lines of foreseeability are going to be drawn further out. Almost as if when you do a negligent act, if anything can be traced to that act, we're not going to ask if it's foreseeable. If you commit that act, anything that can be related to directly flowing from it is going to be approximately caused. Whether it's foreseeable or not, if it can be traced back to the negligent act, it will be.

Scott v. Bradford takeaways

a. Patient suing under theory of informed consent must allege and prove: i. doc failed to inform him adequately of a material risk before securing his consent to the proposed treatment ii. if he'd been informed of risks = no consent to treatment iii. the adverse consequences that were not made known did in fact occur and he was injured by submitting to treatment. § As a defense, a doc may prove P knew risks, full disclosure would be detrimental to patient's best interests, that an emergency existed requiring prompt treatment; and patient was in no condition to decide for himself.

Gentry v. Douglas Hereford Ranch, Inc.

a. RULE: A negligence action requires proof of 4 elements: (1) existence of a duty; (2) breach of the duty; (3) causation; and (4) damages. If P fails to offer proof of one of these elements, the action in negligence fails and summary judgment in favor of D is proper. The causation element requires proof of both cause in fact and proximate cause. b. FACTS: The decedent was shot when a person carrying a rifle stumbled and accidentally pulled the trigger. Accident occurred on property owned by the ranch and leased to the cattle co. The personal representative filed suit against the ranch, the cattle company, and the shooter, who subsequently filed bankruptcy. The personal representative alleged that the ranch and the cattle co. negligently failed to properly maintain the property and they were vicariously liable for shooter's negligence under the doctrine of respondeat superior. Court affirmed the summary judgment in favor of the ranch and the cattle co. c. ISSUE: Is speculation sufficient to defeat a motion for summary judgment, where cause in fact of an accident cannot be proven? No d. CONCLUSION: The personal representative offered no substantial evidence that any condition on the property caused the shooter to stumble and fall. Therefore, the issue of causation was subject to speculation. The representative also failed to establish that the shooter was an employee of the ranch or the cattle company. Moreover, it was undisputed the shooter was not acting on behalf of the ranch or the cattle company at the time of the shooting. Therefore, ranch and the cattle co. were not vicariously liable.

Brief for Fuller:

a. RULE: An act of suicide, as a matter of law, is not a superseding cause in negligence law precluding liability. b. FACTS: Decedent, Dr. Lewis, committed suicide some 7 mths after a car accident from which he had walked away believing he was uninjured. P executor brought wrongful death action. At jury trial, defendants were found at fault. The theory of the case was that defendants, owner and operator of the vehicle which struck decedent's automobile, were responsible in tort for the suicide as a matter of proximate cause and effect. c. ISSUE: Was the evidence of cause of decedent's suicide sufficient to withstand dismissal of the wrongful death complaint? Yes CONCLUSION: P executor's complaint in a wrongful death action should not have been dismissed, since there was evidence sufficient to find decedent's suicide was proximately caused by brain damage incurred through D's negligent operation of a motor vehicle.Although decedent changed his will and wrote suicide notes claiming sanity and directing destruction of one note so that the "outcome of the case" would not be jeopardized, indicating that he intended and was aware of his acts, the evidence supports a finding that the suicide was an irresistible impulse. It is a fair issue of fact whether the suicide was a rational act or an irresistible impulse, where preceded by trauma, brain damage, epileptic seizures, aberrational conduct, depression and despair.

Bartolone v. Jeckovich - proximate cause ex

a. RULE: D must take P as he finds him and hence may be held liable in damages for aggravation of a preexisting illness. Nor may Ds argue that P should be denied recovery because his condition might have occurred even without the accident. b. FACTS: On October 4, 1976, plaintiff was involved in a four-car chain reaction collision in Niagara Falls for which defendants were found liable. Plaintiff sustained relatively minor injuries consisting of whiplash and cervical and lower back strain for which he was treated with muscle relaxants and physical therapy but was not hospitalized. Subsequently, however, he suffered an acute psychotic breakdown from which he has not recovered. The theory on which plaintiff's case was tried was that the accident aggravated a preexisting paranoid schizophrenic condition which has totally and permanently disabled him. The trial court granted the liable parties' motion to set aside the jury verdict and to order a new trial unless the injured party would stipulate to a reduced verdict. The injured party refused to so stipulate and appealed. c. ISSUE: Are Ds liable for the aggravation of the pre-existing illness of P? Yes d. CONCLUSION: The court reversed the order and reinstated the verdict. The court found that the trial court record presented ample evidence that the plaintiff suffered from a psychotic illness but that he had been able to function in a relatively normal manner until the time of the accident. The court held that the liable parties were obligated to take the injured party as they found him and that they were, thus, liable for damages for the aggravation of a preexisting illness.

Sindell v. Abbott Laboratories

a. RULE: For purposes of market share liability in cases involving diethylstilbestrol, it is probably impossible, with the passage of time, to determine market share with mathematical exactitude. But just as a jury cannot be expected to determine the precise relationship between fault and liability in applying the doctrine of comparative fault or partial indemnity, the difficulty of apportioning damages among the defendant producers in exact relation to their market share does not seriously militate against the rule of market share liability. With regard to the liability of independent tortfeasors, where a correct division of liability cannot be made the trier of fact may make it the best it can. b. FACTS: Two class actions for personal injuries were filed against a company manufacturing diethylstilbestrol (DES). The complaint alleges that the cos were jointly and individually negligent in marketing and promoting DES as a safe and efficacious drug to prevent miscarriage, without adequate testing or warning, and without monitoring or reporting its effect. The complainants, daughters who were allegedly injured by their mothers' ingestion of DES, were unable to identify the specific manufacturer of the drugs taken by their moms. Ds moved to dismiss the case on this basis, which was approved by trial court. The case was appealed to CA's SC. c. ISSUE: Was the motion to dismiss properly granted? No d. CONCLUSION: In reversing the dismissals the court adopted the theory of market share liability based on the principle that tween innocent Ps and negligent Ds the latter should bear the cost of injury. Therefore, once Ps joined the manufacturers of a substantial percentage of DES, defendants were required to prove they could not have manufactured the injury-causing product. Absent such proof, liability for damages could be apportioned based on each defendant's share of the appropriate market.

Palsgraf v. Long Island R.R. Co.: Brief

a. RULE: If no hazard is apparent to the eye of ordinary vigilance, an act innocent and harmless, at least to outward seeming, with reference to a plaintiff, does not take to itself the quality of a tort because it happened to be a wrong, though apparently not one involving the risk of bodily insecurity, with reference to someone else. In every instance, before negligence can be predicated of a given act, back of the act must be sought and found a duty to the individual complaining, the observance of which would have averted or avoided the injury. The ideas of negligence and duty are strictly correlative. b. FACTS: Plaintiff ticket-holding passenger Helen Palsgraf was standing on a platform of defendant Long Island Railroad Company. A man carrying a package jumped aboard the car of a moving train at a nearby platform. A guard in the car reached to help him in, and a guard on the platform pushed the man from behind. The package was dislodged, fell onto the rails, and exploded. The shock of the explosion caused a scales to fall onto plaintiff passenger, who filed this lawsuit for injuries due to defendant's alleged negligence. The trial term court (the Supreme Court) entered a verdict for plaintiff. In 1927, the Supreme Court, Appellate Division affirmed the judgment in favor of plaintiff. These two lower courts held that the negligent acts of the railroad's employees caused the package that contained explosives to be thrown under the train where they exploded. Defendant railroad sought appeal to the Court of Appeals of New York. c. ISSUE: In an action for injuries sustained during an explosion when a package was dropped on a nearby rail, was defendant railroad liable for negligence due to its guards' conduct in pushing the man carrying the package? No d. CONCLUSION: In 1928, the Court of Appeals of New York reversed the judgments of the appellate division and the trial term of the Supreme Court. Ruling in favor of defendant railroad, the Court dismissed plaintiff passenger's complaint. The Court found that the conduct of the railroad's guards was not a wrong or negligence in relation to plaintiff, standing far away. There was nothing in the situation to suggest to the most cautious mind that the parcel wrapped in newspaper would spread wreckage through the station. The Court explained that negligence is not actionable unless it involves the invasion of a legally protected interest--the violation of a right, which in this case was claimed to be the right to be protected against interference with one's bodily security. Bodily security is not protected against all forms of interference or aggression, but only against some. Negligence is the absence of care, according to the circumstances. The Court concluded that there was no negligence because defendant railroad could not have reasonably foreseen that its employees' conduct would have resulted in injury to plaintiff Palsgraff. The Court noted that the plaintiff had sued in her own right for a wrong personal to her, and not as the vicarious beneficiary of a breach of duty to another.

Summers v. Tice - cause in fact ex

a. RULE: In Oliver v. Miles, 144 Miss. 852, 110 So. 666, 50 A.L.R. 357, two hunters fired at partridges across a public highway in reckless disregard of the rights of persons using this road, and a traveler on the highway was shot. In the absence of proof as to which hunter fired the shot causing the injury to the third person who was not of their party, the court held that the defendants were jointly engaged in the unlawful enterprise of shooting across a highway and were jointly liable. b. FACTS: Plaintiff brought a negligence suit against Respondents for the loss of his right eye and lip injuries. Plaintiff Summers and Respondents went on a hunting expedition together on the open range and plaintiff alleged that at a time when plaintiff was at a point approximately 200 feet from the defendants, each of them negligently discharged his gun, with the result that shot was lodged in the respondent's right eye and in his upper lip. The court then found each defendant guilty of negligence and that the negligence of each defendant was a proximate cause of the injury to plaintiff. Defendants appealed. c. ISSUE: Are the defendants jointly liable for the injuries sustained by plaintiff? No d. CONCLUSION: The court reversed and held that under the reasoning of these rules from "Restatement of the Law of Torts", defendants could not be held liable. To uphold plaintiff's contention the court would be forced to hold that if A, B and C go hunting together in a place where they have a right to go and at all times on the expedition the conduct of A is careful, skillful and prudent, but that by the unforeseen negligent conduct of B, C is injured, nevertheless, in spite of his own exemplary conduct, A is liable for the injury to C. There is no reason in logic or law for such a holding and no decision to any such effect has been called to the court's attention.

Kelly v. Gwinnell Brief:

a. RULE: In most cases the justice of imposing a duty is so clear that the cause of action in negligence is assumed to exist simply on the basis of the actor's creation of an unreasonable risk of foreseeable harm resulting in injury. However, more is needed, more being the value judgment, based on an analysis of public policy, that the actor owed the injured party a duty of reasonable care. Whether a duty exists is ultimately a question of fairness. The inquiry involves a weighing of the relationship of the parties, the nature of the risk, and the public interest in the proposed solution. b. FACTS: P, the victim of a drunken driving accident, sued D drunken driver and D host, who had served alcoholic beverages to the drunken driver. Summary judgment was granted in favor of defendant host. On appeal, the court reversed. c. ISSUE: Can defendant host be held liable for plaintiff's accident? Yes d. CONCLUSION: Where a host continues to serve alcohol directly to a guest even after the guest's visibly intoxicated, knowing the guest will soon be driving, host may be liable for consequences of the drunken driving. Court reasoned that the policy considerations of just compensation for victims and deterrence of drunken driving supported imposing a duty on the social host. Court declared its expansion of liability to be prospective.

Enright v. Eli Lilly & Co. is another public policy case. Says that public policy is used in other places besides just alcohol. Brief:

a. RULE: Limiting liability to those who ingested the drug or were exposed to it in utero serves the purpose of confining liability within manageable limits, and does not unduly impair the deterrent purposes of tort liability. Thus, there is no basis for a departure from the rule that an injury to a mother which results in injuries to a later-conceived child does not establish a cause of action in favor of the child against the original tortfeasor. b. FACTS: A mother took the drug diethylstilbestrol (DES) while pregnant with P, and P's reproductive system was adversely affected. As an adult, P had troubled pregnancies before giving birth to her daughter prematurely, causing her daughter's numerous health problems. As a result, P brought several actions against several manufacturers of DES on behalf of her daughter, claiming negligence, breach of warranty, strict liability, and fraud. Trial court dismissed claims brought on behalf of Ps daughter but on appeal, the appellate court reinstated the daughter's claim against the manufacturers of DES. The case was appealed by the manufacturer's to the Court of Appeals of New York. c. ISSUE: Should the liability of manufacturers of the drug diethylstilbestrol (DES) extend to the so-called "third generation"? No d. CONCLUSION: The court ruled that, even though courts and the legislature had given litigants great latitude in DES cases, they could not make manufacturers liable to victim's children. Court ruled that their job was to make litigation and liability manageable, and extending liability to the extent sought by plaintiff could have negative consequences.

cause in fact ex - Perkins v. Texas and New Orleans R. Co.

a. RULE: Negligence is a cause in fact of the harm to another if it was a substantial factor in bringing about that harm. b. FACTS: D's train traveling E. P's husband was a passenger in a car traveling southbound. A warehouse obstructed the view of the car's driver and the train's engineer and brakeman. Train approached intersection with its headlight burning, bell ringing, its whistle blowing. Train struck the right side of the car, killing its occupants. The car's speed was undetermined. Train was speeding. Parties conceded that driver was negligent in driving upon the track in front of the train and that his negligence was a proximate cause of the husband's death. The district court awarded damages to P. Appellate court affirmed trial court's judgment, finding that D was negligent in operating the train in excess of the speed limit. State SC reversed appellate court's judgment. c. ISSUE: Was the excessive speed of D's train a cause in fact of the fatal collision? No. d. CONCLUSION: P failed to show that D's negligence in travelling at an excessive speed was a substantial factor in bringing about the accident because the record reflected that the fatal accident would have occurred irrespective of the excessive speed of the train.

Anderson v. Minneapolis, St. P. & S. St. M. Ry. Co.

a. RULE: One who negligently sets a fire is not liable if another's property is damaged, unless it is made to appear that the fire was a material element in the destruction of the property. If a fire set by the engine of one railroad company unites with a fire set by the engine of another company, there is joint and several liability, even though either fire would have destroyed plaintiff's property. But if one of the fires is of unknown origin, there is no liability. b. FACTS: P Anderson owned property near defendant railway company's tracks. Anderson sued for property damages caused by a fire alleged to have been caused by sparks from a' locomotive engine. The fire started in a bog near Anderson's land and smoldered there for several months, when it flared up and burned his property shortly before it was reached by one of the great fires sweeping through the area that day. The jury returned a verdict for Anderson. The railway company appealed the judgment. c. ISSUE: Was defendant railway company liable for the damages on P's property caused by a fire alleged to have been caused by sparks from a locomotive engine? Yes d. CONCLUSION: The state supreme court affirmed the judgment because the trial court did not abuse its discretion in allowing Anderson to amend the complaint to conform to proof at trial. Moreover, the trial judge's instructions to the jury in the absence of counsel were correct statements of law, and the trial court was not obliged to notify counsel before responding to the jury's question. The court also held that the railway company could not escape liability for a fire started by one of its engines by showing that the fire united with another of no responsible origin and that Anderson's property was destroyed by the combined fires, if it appears that the railroad fire was a material element entering into the destruction of the property.

Ryan v. New York Central R.R. Co.

a. RULE: That a building upon which sparks and cinders fall should be destroyed or seriously injured must be expected, but that the fire should spread and other buildings be consumed, is not a necessary or an usual result. That it's possible, and that it's not unfrequent, cannot be denied. The result, however, depends, not upon any necessity of a further communication of the fire, but upon a concurrence of accidental circumstances, such as the degree of the heat, the state of the atmosphere, the condition and materials of the adjoining structures and wind' direction. These are accidental, varying circumstances. The party has no control over them, isn't responsible for their effects. b. FACTS: The railroad negligently caused a fire that destroyed its woodshed. The fire spread to the landowner's property 130 ft away from the shed and destroyed a building. The landowner sued the railroad to recover for the damage to the building. The court affirmed the decision to nonsuit the landowner. c. ISSUE: A building in a populous city takes fire, through the negligence of the owner or his servant; the flames extend to and destroy an adjacent building: Is the owner of the first building liable to the second owner for the damage sustained by such burning? No CONCLUSION: A man may insure his own house or his own furniture, but he cannot insure his neighbor's building or furniture, for the reason that he has no interest in them. To hold that the owner must not only meet his own loss by fire, but that he must guarantee the security of his neighbors on both sides, and to an unlimited extent, would be to create a liability which would be the destruction of all civilized society. No community could long exist, under the operation of such a principle. In a commercial country, each man, to some extent, runs the hazard of his neighbor's conduct, and each, by insurance against such hazards, is enabled to obtain a reasonable security against loss. To neglect such precaution, and to call upon his neighbor, on whose premises a fire originated, to indemnify him instead, would be to award a punishment quite beyond the offense committed. It is to be considered, also, that if the negligent party is liable to the owner of a remote building thus consumed, he would also be liable to the insurance companies who should pay losses to such remote owners. The principle of subrogation would entitle the companies to the benefit of every claim held by the party to whom a loss should be paid.

Coney v. J.L.G. Industries, Inc. Brief:

a. RULE: The key conceptual distinction between strict products liability and negligence is that in strict liability the plaintiff need not prove faulty conduct on the part of the defendant in order to recover. The focus is on the condition of the product. In applying comparative negligence to strict products liability, the court stated a direct comparison of the defective product with P's negligence is both conceptually and pragmatically inappropriate. The only conceptual basis for comparison is the causative contribution of each to the particular loss or injury. In apportioning damages courts are really asking how much of the injury was caused by the defect in the product versus how much was caused by the plaintiff's own actions. b. FACTS: In a wrongful death suit against a manufacturer based on strict liability, D requested that D's fault, if any, be compared to the total fault of the all parties and any judgment against defendant reflect only defendant's percentage of the overall liability. It appears that the victim died from injuries sustained while operating a defective hydraulic aerial work platform. The trial court struck the defenses, but certified three questions for appeal. The appellate court declined defendant's application for leave to appeal. c. ISSUE: Is the defense of comparative negligence applicable to strict products liability actions, such that the D would be relieved of any liability? Yes, but the defense would only mitigate liability. d. CONCLUSION: Court held (1) comparative negligence defense is applicable in strict products liability actions since the manufacturer's liability would remain strict, but the responsibility for damages would be lessened by the extent the consumer's conduct contributed to the injuries; (2) comparative fault would not eliminate joint and several liability since liability would still be apportioned to the degree each D's conduct proximately caused the injuries; and (3) joint and several liability would not deny Ds equal protection of the law.

Atlantic Coast Line R. Co. v. Daniels - proximate cause ex

a. RULE: Though negligence is discovered in relation to one of the causes which have preceded the injurious effect, it does not follow that the author of the negligence is to be held legally responsible for the injury. In the first place, to judge the transaction according to the natural probabilities which men's minds take as the basis for passing judgment upon the course of human affairs, it may appear that causes other than the negligent one referred to so preponderated in bringing about the result as to lead us to say, from a human point of view, that the injury was just as likely to have ensued (with only its details somewhat varied, perhaps) if the negligent thing had not occurred. In such cases we exempt the author of the negligence from liability. b. FACTS: The injured party was operating his car when he came to a railroad track. As the injured party approached, the crossing the bars were up, the car's engine was given maximum power, and then as he came upon the tracks the bar was suddenly lowered. The injured party was penned upon the tracks; he threw on the brakes and stopped the automobile. As the injured party moved to the third track, the train dashed by on the second track. The injured party was so unnerved he forgot the condition in which he had left his levers and the engine "kicked back" throwing the injured party. In affirming the court held the injured party's fright was a condition rather than a cause of his injury. c. ISSUE: Was railroad's negligence considered the proximate cause of the injuries? Yes CONCLUSION: Nothing appears in the case made by the facts stated in the petition to require a belief that, according to the ordinary course of human probability, the fright and the consequent injury would have occurred if the negligence had not operated in a direct causal way upon the conditions surrounding the transaction. We find P coming lawfully and prudently within the range of D's activities; as he comes he is uninjured; while he is within the range of those activities the defendant violates the normal standard--acts as a reasonably prudent person would not have acted under the circumstances (for we are assuming the defendant's negligence); and before P gets from within the range of these activities he is hurt as the result of a condition to the creation of which these activities contributed in a causal way; wherever else we look among the sum total of the general causes which joined to create the condition and to characterize it, we find no other variationfrom the normal course of prudence; we except all these other causes, class them as merely a part of the condition or innocuous medium through which the defendant's negligent activities became effectual, and say that the defendant's negligence was, therefore, the sole, direct, and proximate cause of the plaintiff's injury.

Reynolds v. Texas and Pac. Ry. Co.

a. RULE: Where D's negligence greatly multiplies the chances of an accident to a plaintiff and is of a character naturally leading to its occurrence, the mere possibility that it might have happened without the negligence is not sufficient to break the chain of cause and effect between the negligence and the injury. Courts, in such matters, consider the natural and ordinary course of events and do not indulge in fanciful suppositions. b. FACTS: Ps purchased tickets to board the carrier's eastbound passenger train, at a depot at a certain station, at about midnight. When the train arrived at the station, it was behind time, and passengers were allegedly warned to "hurry up." As the passenger hastened down the unlighted steps, which ran paralleled to the track, from the depot to the cars, she made a misstep and was precipitated beyond the narrow platform in front and down the slope beyond. They sued for damages against the defendant. c. ISSUE: if the mere possibility that the injury would have resulted even without the negligence relieves the tortfeasor from liability: No. CONCLUSION: On appeal, court found: (1) there was no sufficient light on the steps, and such absence rendered the passage from the depot to the train insecure and constituted negligence on carrier's part and a failure to perform its duty of providing safe modes of ingress and egress between its depots and its trains; (2) bc carrier's negligence greatly multiplied the chances of passenger's accident, the mere possibility that the passenger might've made a misstep and fallen even in broad daylight, was insufficient to break the chain of cause and effect between the carrier's negligence and the passenger's injury; and (3) hence, the evidence connected the accident with the negligence.

Hill v. Edmonds - concurrent cause ex.

a. RULE: Where separate acts of negligence combine to produce directly a single injury, each tort-feasor's responsible for the entire result though his act alone might not've caused it. b. FACTS: At the close of the injured party's case, trial court dismissed the complaint against the tortfeasor who, on a stormy night, left a tractor truck parked without lights in the middle of a road. The car in which the injured party was a passenger collided with the truck from the rear. From the testimony of the driver of the car, trial court concluded that the driver was guilty of negligence and was solely responsible for the collision. c. ISSUE: When multiple acts caused a single harm, but each single act cannot commit the harm, may each tortfeasor be sanctioned? Yes d. CONCLUSION: When separate acts of negligence combined all together cause one direct harm, the liability is directed to each tortfeasor even if each act individually may not have caused the harm. Still, the driver left his truck without the lights contributing to harm. The court reversed the trial court's dismissal of the injured party's negligence claim and granted the injured party a new trial.

Morrison v. MacNamara takeaways:

a. Real issue is whether locality standard of care should be used - here, court said that the rule should be that DC isn't under the locality rule; is a center for national standards. Court says we're not going to follow locality rule; instead, they followed nat standard. i. Nat standard required person to be sitting or lying; local rule said he could stand b. They chose the nat standard bc the lab was board-certified - have to have met a testing standard that is national. c. Normally, nat standard is going to be a higher standard of care, docs're going to have to know more. Locality standard still exists in some places. i. In Oxford, standard is looking at towns of the same size. Docs had better be doing what docs are doing in similar localities. d. Locality rule/similar locality rule/nat standard of care deals with what standard are we going to judge the prof's three duties by. Are we going to judge them by what's done locality, in a similar locality, or what's doing on a nat standard. Narrows pool of experts. i. Locality rule exists bc long ago docs in rural areas didn't know what docs in big cities knew - didn't go to same schools. Isn't true anymore. ii. Only benefit to locality rule is that it induces docs to work in smaller comms. Make it more advantageous for them. Nat standard of care's slowly moving into the forefront, becoming maj in most js. If you're board certified, going to be held to a nat standard of care no matter where you ar

discussing evidence of negligence ex: Joye v. Great Atlantic and Pacific Tea Co.

a. Rule: storekeeper isn't an insurer of his customers' safety but owes them a duty of ordinary care in keeping aisles in a reasonably safe condition. b. Facts: P fell on a banana peel in D supermarket. Litigation ensued, P won, D challenged, asserting he fulfilled his duty to keep the aisles in a reasonably safe condition. c. Issue: Did P provide sufficient evidence to show that D was liable? No. d. Conclusion: P failed to show D had constructive notice of the peel. Burden was on him to show it'd been there long enough to give D constructive notice. Here, P offered no direct evidence as to how long it'd been on floor, thus, judgment was reversed in favor of D Says that the jury wouldn't be able to decide if banana had been there a long time. a. Would you have dismissed bc somebody couldn't prove more likely than not it had been there a long time? Some judges would let this go to the jury. § Jury can still find there's not enough evidence for it being there long enough for them to have seen it.

discussing evidence of negligence ex: Jasko v. F.W. Woolworth Co.

a. Rule: when the operating methods of a proprietor are such that dangerous conditions are continuous or easily foreseeable, then actual or constructive notice of the specific condition need not be proved. b. Facts: P injured when leaving D's store - fell on a slice of pizza on the terrazzo floor. c. Issue: Is the store liable for P's injuries despite being unaware that a pizza was dropped on the floor or that the store knew of its presence? Yes. d. Conclusion: court held that a dangerous condition was created by D's method of sale. The practice of selling pizza on waxed paper to customers who consumed it while standing created the reasonable probability that food would drop to the floor. Food on a terrazzo floor will create a dangerous condition. Therefore, notice to D of the specific item on the floor didn't have to be shown. Court also found that factual issues regarding D's negligence had been raised.

Even more notes after Watson:

a. Second injury caused by weakened condition resulting from first injury: there are some recurrent patterns falling into this classification, in which the cases have indicated a more or less crystallized response, holding the first D liable for the second injury. b. SOMETIMES intentional acts are foreseeable. Note 1 tells us that some criminal acts that are intentional will not automatically cut off D's negligence if it was foreseeable.

majority opinion in Elbaor:

a. She entered into "Mary Carter" settlement agreements with other docs. Those agreements provided that the Ds would pay her a certain amount of money and remain as Ds, participate in the trial, and be paid back all or part of the settlement money paid to Smith out of the recovery against the remaining D, Elbaor. Elbaor requested that the Mary Carter agreements be voided as against public policy or, alternatively, that the settling Ds be dismissed from the trial b. A Mary Carter agreement exists, under our definition, when the P enteres into a settlement agreement with one D and goes to trial against the remaining Ds. The settling D, who remains a party, guarantees the P a minimum payment, which may be offset in whole or in part by an excess judgment recovered at trial. c. Note that this creates a great incentive for the settling D to ensure that the P succeeds in obtaining a sizable recovery, and thus motivates the D to assist greatly in the P's presentation of the case (as occurred here). Marcy Carter agreements generally, but not always, contain a clause requires the settling D to participate in the trial on the P's behalf d. Many js have decided to tolerate the ill effects of Mary Carter agreements, presumably bc they believe that the agreements promote settlement. Holds that public policy favoring far trials outweighs our public policy favoring partial settlements. So this j declares them void as violative of sound public policy

Daubert tells us there are two types of considerations when you're dealing with statistics:

a. Show that this drug is capable of causing defects b. Show that it more likely than not caused THIS patient's defects. i. Do this by showing that the woman took the drug, and that the population who took the drug was more than doubled of the population that didn't take the drug. But first have to just show it's in the class of drugs that do cause defects.

In Daubert, who was the D? the pharma co. the alleged act of negligence: negligently produced this drug/products liability. Act of negligence: they did not do what a reasonable prudent co would do in developing and selling their drug. a. Is duty or breach an issue in this case? No, we're mainly looking at causation. Court only discusses causation, cause in fact. How does P attempt to show that manufacturing the drug was a cause in fact of the birth defects? Got experts to try to prove this.

a. Sometimes you need an expert to show cause in fact when the issue is beyond the knowledge of a layperson. b. Court says that without even taking meds, 1 in 1,000 women end up with kids that have birth defects. This woman may have had this problem even if she didn't take drugs. Has to show that BUT FOR taking drug, kids wouldn't have defects. Standard's by the preponderance of the evidence; show more likely than not that if she hadn't taken the drug, kids wouldn't have had defects. Expert testimony has to est cause in fact.

takeaways from Heath v. Swift Wings:

a. Standard is expressed in objective form - the training and skill of an ordinary member of the profession. b. In practice: All pilots are held to the same standard of care, regardless of how long they've been flying. Can't say "I didn't know how to land bc I haven't had enough exp." c. Standard of care is that of a reasonable prudent pilot is a same or similar circumstances. There is some pool of knowledge, skills that all pilots must have if they're going to fly planes. You MUST know how to fly, land, have those skills.

In Stachniewicz, have two different things - a statute and an ordinance.

a. Statute said: don't give alcohol to people already drunk. Designed to protect people in the bar. Also possibly anybody driving on the road, if drunk guy drives home drunk. i. If you're a P attorney, going to have to work hard to expand the class! Lookit legis. History for this. Wider class: anybody that could be hurt by a drunken person. What kind of injury: all kinds of injury. b. Court didn't adopt statute, instead adopted regulation. Didn't adopt statute bc meant to prevent drunken contact that results in injury to customers. Didn't adopt, bc it'd be very hard to enforce the 'already visible intoxicated' part. Aka, must be fair, workable, and wise to adopt the statute. c. Not fair, workable, and wise: we don't know if a drink sold after the person was drunk started the fight. Might've been a drink sold before they were visibly drunk. d. Regulation just says you shouldn't allow rowdy people to be in bars threatening people, acting crazy because it's going to lead to a fight. 1. Adds 3rd element (within class; injury type): is it fair/appropriate to adopt this statute.

a. Cause in fact: there's a direct connection between the D's breach and P's damages. Ex, him hitting me across the head with a bat is a cause in fact of my head injury. The hit is a substantial factor causing the hurt head.

a. Substantial factor defined using 'but for' test: but for this factor, injury would not have occurred.

Takeaways from United States v. Carroll Towing Co.

a. The alleged negligent act: owner of the boat, (case of contributory negligence) should've had a bargee onboard. b. Carroll's affirmative offense: bargee should've been onboard. c. Likelihood comes into play bc it's a busy waterway. Boats constantly coming in and out, likely one could hit another. ALSO: burden is low (just have a bargee) and gravity is high d. You don't even have to have the bargee there 24/7, just where the probability is high. Busy part of the day, he needs to be there. e. As a lawyer, need to be able to argue these three factors are high or low for both P and D.

Rescue doctrine is something to consider when evaluating proximate cuase.

a. The duty that is owed to the victim who's trying to be rescued by rescuer; whatever the duty was owed to the victim is also the duty that was owed to the rescuer. b. This duty does away with the Palsgraf zone of danger question. Ex: car couldn't run off the road five hours before the rescuer got onto the scene. Recuer doesn't have to be in the zone of danger - contrast to Palsgraf. But the duty owed to the victim is what's owed to the rescuer. c. A breach to the victim is also a breach to the rescuer. The cause in fact of the injury to the victim is also a cause in fact of the injury to the rescuer. The only difference is proximate cause - the victim is probably going to be able to est proximate cause.

elements of res ipsa:

a. The instrumentality was under the D's control i. "The instrumentality must be under the management of the D or her employees:" may be actual control or the RIGHT to control it. ii. Aka, handling the barrel vs. the barrel being in the D's warehouse. Have right to use it, have right to prevent negligence. b. That these types of accidents usually don't occur without negligence i. Doesn't have to be D's negligence - can be anyone's negligence that'd normally cause this type of action to occur. c. Those are the two major elements of res ipsa. Some js add other elements - ex, event not have been brought about by P's actions. i. Can't have contributory negligence in these js if want to use res ipsa. Most js don't use this third element. d. Another possible element some js do/don't use: D must have superior knowledge. i. Aka, an accident has occurred, but sometimes they just freakishly happen without anybody's knowledge or negligence. So it's not fair to hold D responsible for something they didn't cause. ii. In falling barrel case, by having access to the property D is in the better position to know how the barrel fell than the P. So you ask the D with superior knowledge to come forward and explain what happened. iii. If D doesn't have superior knowledge, then you need to reshuffle the equities, D and P basically in same position; P should have to prove who had possession of barrel and they acted negligently.

Immediate or remote test (for proximate cause): an ex is Ryan v. New York Central RR Co.

a. Was the injury to the P's property immediate or remote? Immediate or remote is defined by the concept of time and distance. So what would've been immediate is that the fire left the engine, landed first on D's own woodshed. He can't sue himself. Court says that the immediate consequences of negligence were the damages to your own property. The adjoining property owner would be remote. So the adjoining party wasn't able to collect. b. Letting the fire escape was the negligent act, was a cause in fact of the property's destruction, but wasn't proximate cause because the harm was remote because of the physical distance and time. c. Under this test, must show that the destruction of your property was the immediate result of D's negligence. § Notes provde that NY has changed this rule. Other js, immediacy is extended for four miles (note 2). § What's the difference? NY wants to generate commerce via railroads, so they value corps more. § If you value something, you limit its liability. § Another factor - insurance. In NY, you're a homeowner, better buy insurance to protect yourself from fire. NY wants to protect industry by restricting liability through proximate cause, put burden on homeowners to buy insurance to protect their property.

more notes after Watson:

a. What if attempt at rescue is utterly foolish: no liability bc D not bound to foresee, guard against such extraordinary conduct. b. Suppose the injured rescuer is a professional, like a cop or firefighter: most states don't permit recovery by professionals, at least against the property owner or driver whose negligence occasioned the response, under the so-called "firefighter's rule" or the "professional rescuer's rule." Rule precludes recovery only if the risk creaed was the type of risk reasonably anticipated by the job. This rule has been limited or abolished in some js. c. Cases generally agree that the D who caused the danger will be liable to someone who's injured trying to escape from it. d. Attempt to alleviate harm caused by D: the primary ex of this is the case in which a doc treats the P's injuries and negligently aggravates them. Is the og D liable for the aggravation? Answer is usually yes.

In Chang case, the wheel assembly was negligently placed on the truck. That's the allegedly negligent act.

a. What the reasonable risks from this: that it will fall off the car, roll into the street, on the side of the road. What is foreseeable that can happen? The majority opinion says it was not reasonable to foreseeable that Chang would run across the road. He has to prove that putting the assembly on was a proximate cause of his injury. If they did put it on, it is a cause in fact. But for it being put on improperly, falling off, would not have crossed road. But it's not a proximate cause bc his actions were extraordinary/not foreseeable. Manufacturer could not have sene someone who acted as oddly as Chang did. Here, the initial judge said it was not foreseeable, said it was extraordinary. But ultimately, in the last court, the SC held that this was a question for the jury to decide. It's possible someone would try to go aacross traffic to get his tire if it falls off.

1. When you commit a negligent act, there are certain risks that stem from that act. Ex, if you speeding, there's risk you'll hit somebody, run into a ditch, or lose control 2. The risk of leaving oil on the water - the risk of leaving a flammable debris on the water with things floating on it, the risk of all of that is that a fire will occur.

a. What they're saying here is that the risk of somebody dropping molten iron, and causing a fire, is foreseeable. Independent causes, if not committed negligently, will not cut off the og tortfeaser's negligence, break the chain. Dropping of the cotton was a foreseeable force, but apparently the court thought that that was a foreseeable thing. 1. Most things that are third-party induced are independently induced, having nothing to do with the oil spill. But what they did is they started a force that was within the risk that was created by the original negligent act. 2. The risk has to be negligently foreseeable to a reasonable prudent person at the time it occurred. 3. Wagon mound tests - are the foreseeability tests (foreseeable = something that is possible that falls in the realm of the D's negligent conduct) applied different ways

in first Wagon Mount case, it applies the foreseeable test. Says it wasn't reasonably foreseeable that there would be fire. In the second case, the P were the two ships that were docked at the dock. But we're talking about the same spillage of oil, same negligent act. How then can we get two different results?

a. What's the difference in the probable cause analyses: first trial judge says that it was not foreseeable in the mind of the engineer that oil would catch on fire. But the second one said that the engineers should've known that it was possible. b. So if it's possible, it's foreseeable. Now the question is: HOW foreseeable. Court seems to be saying that even a little likelihood makes it possible, makes it foreseeable. If it's possible, then a reasonable person would remove the oil given the likelihood of harm and burden of precaution. Gravity is high, burden is low - no utility having it there. c. Foreseeability doesn't have to be 50% likely; just has to be possible. Look at the risk as opposed to a probability. If its very risky, dangerous, remove it.

notes after Derdiarian:

a. When courts have decided that the intervening act should cut off liability, it's labelled a "superseding" cause. b. The test for determining whether an intervening force constitutes a superseding cause is often couched in terms of foreseeability; a person is deemed to have foreseen the normal consequences of his conduct, but is not responsible for extraordinarily negligent intervening acts of third persons. If, for example, vagrants break into a condemned house before it's knocked down, start a fire, and burn down neighboring houses, Ds are liable if the neighborhood is known to be visited by vagrants. Another ex: farmers built corn up to an intersection, obscured view. Drivers failing to stop at that obstructed view intersection constituted an efficient intervening cause. c. One standard type of intervening force is called an Act of God. aka, an act of nature.

Three exceptions have been identified where the public interest will render an exculpatory clause unenforceable:

a. When the party protected by the clause intentionally causes harm mor engages in acts of reckless, wanton, or gross negligence b. When the bargaining power of one party to the k is so grossly unequal so as to put that party at the mercy of the other's negligence c. When the transaction involves the public interest.

If the conduct of a third party was negligent, and if it was foreseeable negligent behavior, won't cut off D's negligence. If conduct was intentional, will cut off D's negligence, keeping D from being proximate cause

a. When you have an intervening force between the negligence and the injury, have to look at whether its intentionally done to decide if D is still the proximate cause of the injury. i. What does intervening cause mean? The jury will be told the law on when an act is superceding, when it's not. ii. If reasonable minds can differ as to whether it was or wasn't intentionally done, it'll have to go to the jury. iii. If D says that P was a contributing cause, you must look at either of their negligence, see if that negligence was cause in fact, if, separately, were a proximate cause of P's injury. If so, then you'll determine the percentage fault of each party. Concurrent cause question.

H.E. Butt Groc. Co. v. Resendez is just like Woolworth case; the offending thing is the method of display or sale. Here, court decided that the display was not dangerous.

a. Whereas the pizza method of selling was more likely than not negligent, the display bowl was not more likely than not negligent. b. No evidence a reasonable person wouldn't make the display that way. Actual notice case, method of sale case. Had duty to use reasonable care in the method that they use to sell things.

Smith tell us that by losing a substantial percentage of one's ability to recover, patient should be compensated for that percentage. One of the reqs of loss of chance theory is that you have to specify the loss of recovery percentage that you lost.

a. Why's' the percentage important? You have to prove cause in fact by a preponderance OF THE EVIDENCE. You have to show that whatever the injury is, that the negligence was the probable cause. Aka, have to show that it was a more likely than not cause. Have to show that negligence in not treating a stroke was a cause of disability. But for that negligence, he wouldn't have ended up with an injury. b. Here, can't show that but for the negligence he wouldn't have had his injury - can't show that more likely than not he would not've had the disability if the stroke had been treated appropriately. Can only say that had he been treated appropriately, he would've had a 33 percent change of a better outcome. That it'd be 33 percent greater isn't a preponderance. Court says we need to change this rule, though, bc otherwise negligent doc are going to be able to go free. To get rid of the harshness of a doc being able to go scott free bc P can't show more likely than not he wouldn't have had these injuries, we're going to change burden of proof and types of damages allowed

alleged negligent act in Kramer Services Inc. v. Wilkins: having a door with a glass frame that needed to be changed out. That's why the glass fell and cut guest. Alleged negligent act is having a door that was in disrepair. Was the negligent act a cause in fact of some of the P's injuries?

a. Yes, it caused the cut on his head. Even if you couldn't get money for the cancer, you should be able to est that the negligence in maintaining the glass was the cause of the cut. But for glass falling on his head, he would not have developed the cut. The negligence was a cause in fact of the cut using either the but for test or the substantial factor test. b. The cancer, the court says, there's no expert testimony to show that the cut caused the cancer. Need experts bc outside the knowledge of a layperson. None of the experts said that the cut, which was negligently caused, was a probable cause of the cancer. They said it was possible, but we need more than possible, we need more likely than not. i. Court says that when you have an area that's in med expertise, if you don't have a med expert, you're going to lose.

In the dissenting opinion for Palsgraf, the dissenting opinion, Andrews, says that if the guard owed a duty to the man he pushed because the man was in the zone of danger, if you owe a duty to anyone then you owe a duty also to the person that you injured even though the person you injured wasn't in the zone of danger. Aka, you owe a duty to one, you owe a duty to society at large.

a. You injure one person, you're liable for all. Andrews would cut this case off through proximate cause. He'd say there was duty to man pushed, breach happened when he pushed him. b. He says let's cut train co's liability off by using Direct cause Test for proximate cause. There's no superceding or intervening cause, there was both foreseeable and unforeseeable harm. Therefore, co is guilty under the Direct Cause test.

Kelly v. Gwinnell Notes:

a. You serve alcohol to minors, you'll be held liable for any accidents they cause b. Commercial dispensers of liquor: dram shop acts. These acts generally provide a civil cause of action against commercial furnishers of alcohol for damages resulting from a consumer's intoxicated state. c. Employers: some js have applied liability to employers who serve alcohol at co functions when somebody later went and hurt somebody afterwards d. Designated driver ex: designated driver can change there mind, in some js, no liability on them, just on bar or bartender. e. If you loan your car to someone intoxicated, or know they have a history of drinking, liable.

discuss presumption of negligence under "the three ways in which a jury can be instructed on how to treat a violation of an adopted statute:

a. You're negligent unless you can rebut it. b. The statute may offer an excuse, or D can offer an excuse c. If you're in a presumption state, that means D violates a statute can offer an excuse if one is allowed by the statute that's violated. If no excuse is allowed via the statute they break, then in most states, can give one of these excuses: § The violation is reasonable bc of the actor's incapacity § Violator neither knows nor should know of the occasion for compliance v Ex, car lights are working when you get in your car, but halfway to town they blew. Your excuse is you didn't know, couldn't and shouldn't know. § Violator is unable after reasonable diligence of care to comply v Ex, a car hits black ice § Violator is confronted by an emergency not due to his own misconduct v Ex: suddenly drive up on a truck in the road you couldn't see til just then § Violator's compliance would involve a greater risk of harm to him or others. v Ex: Zeni case. Compliance with walking on sidewalk would've exposed her to more ice and more risk than walking with her back to traffic. d. If don't have one of those excuses, some js allow you to still say you acted as reasonable person under the circumstances. Once presumption is rebutted, totality of circumstances is looked at. Then ask if D acted as a reasonable prudent person would. e. A mere violation of the statute isn't going to be enough to find negligence. Jury'll decide if a reasonable prudent person in same or similar circumstance would've done the same

In Sindell, CA required that the no. of manufacturers that are joined in the lawsuit must comprise a substantial share of the market. Here, the six that were joined comprised 90% of the market. Some states, like WA and FL, see note C on p332, say that the P doesn't have to jjoin a substantial number of manufacturers; can join as few as one. That manufacturer must bring in any other manufacturer into the lawsuit.

a. diff states have different rules as to how many Ds have to be sued. b. Also have different rules on if a D, once sued, can exonerate itself by showing by a preponderance of the evidence that it didn't produce the pill taken. In some states, a D, once sued, is not allowed to exonerate itself. But most states do. c. Once once one D exonerates itself by a preponderance of the evidence it didn't produce x, then sometimes the remaining share of the market is reallocated among the other Ds. Lets say D A proves that it didn't produce it; the market share is allocated to the remaining Ds, fifty/thirty/twenty percent split evenly. § In some Js, there is no reallocation. So if the one who has 28% shows that he did not manufacture the drug, than whatever the percentage the others have, if can'ts how they didn't do it, market share will be reallocated among them A. CONCLUSION: So states have diff rules as to one D, D can exonerate itself, whether not once a D has left the lawsuit if the percentage of the market is reallocated.

medical malpractice evaluation when applying professional malpractice to Be Careful hypo:

a. doc must have knowledge, training of an ordinary member of the profession and use due care, best judgment. Breach'll be judged by a locality rule, similar locality rule, or nat standard of care. i. This knowledge's outside knowledge of an ordinary person, so you'll need an expert to tell you if a reasonable doc in same circumstances would've prescribed steroid or the other drug. ii. There'll be experts on both sides, say whether it was/wasn't ok to prescribe the steroid (might've been too new, not have enough info at the time for it to be standard of care). iii. Look at what the specific CDC requirement was, if patient was sick enough to meet tit. Let's assume that under the CDC's rule in regards to using the steroid she was sick enough to it - CDC rule still doesn't est the medical standard of care. § That's est by the medical profession, who might reject the CDC and use their own practice, which might be to prescribe the other drug. If they say the custom is to prescribe the other drug, going to have to establish why. § CDC can inform what the standard of care is, but that's it - doesn't est what custom of the medical profession is. iv. This is all under the question of did the doc violate the standard of care when he did not prescribe the steroid and prescribe the other drug - expert will be needed to say what a reasonable member of the prof in a similar circumstance with the knowledge, due care, and best judgment would have done.

Thought experiment for James v. Wormuth: Instead of using res ipsa, what do you use to sue the doc: medical malpractice and talk about standard of care, whether the doc should've left that wire. Talk about if doc should've left it, did he breach his duty of care in breaking the wire in the patient in the first place - use training, best judgment, due care. i. Aka, if something was intentionally done, need to sue doc under medical malpractice. If it WASn"T intentionally done, then res ipsa can be used. Yabarra case, p283:

a. n a situation where patient is unconscious, and she suffers an injury that normally doesn't happen without negligence, burden is going to shift to everybody that had access to her body. When there's this shift, all will be jointly liable unless one can prove it wasn't his fault - aka, going to have to rat on each other. Some courts, when patient is vulnerable, accident is the type that usually doesn't occur without negligence, burden is going to shift to all of the Ds, who had access to patient's body and control over her during the time when the accident could occur.

Sindell establishes market share liability. Were able to prove the drug in question did cause the injury, but couldn't tell exactly who manufactured it. Court is concerned about who can be liable for this. There are six manufacturers that produce the majority of the drug in this market. They look alike, don't know if she took a pill from just one or all of them. Market share liability:

a. when you only have some of the people that COULD have produced the pill (there are others, besides these, that sell), unfair to use Sumers v. Tice in this situation. In Summers, each was liable for the whole. But here, each is not liable for the whole. Each is liable only for his percentage share of the market, the amount of drugs he sold. 10% of market, responsible for 10% of the damages. Cause in fact case bc P can't prove that either one of the six main ones was in fact the maker of the pill that caused their injury

What is an assault:

act for the purpose of placing someone in an imminent apprehension in contact. An assault is to place someone in an imminent apprehension of contact. THAT"S THE DEF

a. another exception to the application of child's standard of care. Applied when kid is doing an activity normally conducted by an adult. i. Both inherently dangerous activities and adult activities are deemed according to location. Ex, in MS hunting may be deemed a child activity. Not so for NY. ii. In real life, you'll look at real cases to decide if there's been a determination on if something is an adult or dangerous activity. If no case, have to look at likelihood and gravity of harm. Use case law to apply to new fact patterns.

adult activity rule under child's standard of care:

Standard: (after the D has rested her case) same for sj motion - whether a reasonable jury could find for P on either breach, causation, or damages

after D does their examination in chief, P and D then ask AGAIN for a motion of directed verdict (under some states)/judgment as a matter of law.

trial stage - assuming sj motion is denied, case goes to trial

after discovery stage, we have:

1. Judge then reads the law to the jury. Will give them the jury instructions which both parties have agreed to - takes a long time; jury then deliberates. a. Jury considers testimonies, determines credibility issues, issues of fact. Jury comes back with a verdict.

after each side makes a motion for a directed verdict in the trial, if motion is denied, each side makes their closing statement - aka, this is what we said the witnesses would say, and they did. P goes first.

1. P then presents the Plaintiff's Case in Chief a. Calls witnesses to show that D had duty, breach, causation, damages. b. Witnesses are sworn in, P lawyer does examination, asks direct and not leading questions. D lawyer then cross-examines, CAN ask leading ?s i. May call Fact or Damages (ex, a doctor) Witnesses

after jury's chosen, sides make opening statements. P is first, D is second. each says what they'll prove, list major laws.

discovery stage. Once complaint and answer are filed, judge calls parties for a Pre-Discovery Conference. Judge gives dates here, including discovery cutoff date. Formal discovery involves filing interrogatories, doc requests, depositions

after pleading/pre-litigation, we have:

a. Names randomly chosen; these people are set in audience, judge chooses 6 (fed) or 12 (state) to sit in jury box b. Judge, lawyers ask questions to see if qualified to be jurors. c. Two reasons jurors will be dismissed: 1) Challenges for cause - if they hate the system, don't believe in trials, are otherwise compromised 2) Preemptory challenges - each side has like five. Can dismiss for no reason (just not for race or gender)

after pretrial conference, jury is then picked. In fed system, they're chosen from voter registration rolls.

1. answer the complaint 2. file a 12(b)(6) motion

after receiving the complaint, D's lawyer has two choices:

1) Motion for JNOV (judgement notwithstanding the verdict) - "judge, reverse what jury did, render a verdict in our favor. Say as a matter of law we win." a. Standard: about the same for sj motion - has to show no reasonable jury could find that the party that won should indeed have won. 2) Motion for a new trial a. Both post-trial motions can be made at the same time. b. Standard for a new trial motion: whether something improper happened at the trial, like judge allowed a witness he shouldn't have

after trial stage, we get post-trial stage. The losing side has an opportunity to ask the court to overturn the jury's decision via two post-verdict motions:

some states have amended their wrongful death statutes to provide specifically for recovery for loss of companionship in:

all cases, not just those involving a minor child

Restatement second section 46(2)

allows recovery if D knows of bystander's presence AND 1. the conduct was directed at a member of bystander's immediate family or 2. bystander suffers bodily harm as a result of her distress

Courts have long agreed that an employer can't insulate himself from liability by imposing safety rules or by instructing his employees to proceed carefully. Respondeat superior isn't limited to negligence cases;

an employer may be held liable for the intentional torts of his employee when the intentional acts are reasonabley connected with the employment and so within its "scope." Counter ex: not liable when employee tries to get revenge on his wife. Generally, whether an intentional tort is in the scope of employment is a question for the jury Most courts say that the principle is liable for punitive damages only if they authorized or ratified the act, was reckless in employing or retaining the agent, or the agent was employed in a managerial capacity and was acting in the scope of employment. To be distinguished from the vicarious sliability cases are cases in which the employer is liable for its own negligence in hiring an employee without adequately checking their background or failing to train them properly

a. Boy's going to argue he was in an emergency situation due to the sounding of a car accident; was an emergency bc it was an imminent thing that distracted him. b. Strongest argument that an emergency argument doesn't apply: wasn't a direct impact to him, his harm wasn't imminent; the distance of the collision is relevant here. ALSO: could have been expected. Wrecks often happen in parking lots; noise of a wreck isn't too uncommon to hear. Perhaps he should have anticipated that there'd be wrecks; should have also anticipated loud noises because he was on a scooter, more exposed to noises. c. Contributory negligence: he's distracted AND speeding. If hadn't been speeding, would've had more time before he was up on her. Speeding was of his own making, was a part of the emergency. You can't use emergency doctrine if you caused the emergency.

application of emergency standard of care to the Be Careful hypo:

a. Is just like mediation. But instead of having a mediator to settle disputes, is kind of a trial-like process; but instead of judge there's an arbitrator. i. Multiple arbitrators - one for P, D, and neutral (panel's chairman). Mostly there's 3 people that'll hear facts, witnesses like a jury b. If arbitrator decides against you, there's not much you can appeal c. Can be contractually obligated to do arbitration - contractual arbitration. Most arbitrations that occur happen because of agreements via contract. d. In scheduling conference, judge can order parties to go to arbitration

arbitration:

more about punitive damages:

are not designed to compensate; are designed to punish the D. tends to be an amount awarded above and beyond compensatory damages. Ex med expenses to pout at 100k, but bc D's activity was so egregious the jury awardes 100k and an additional amount to punish him.

future physical and mental pain:

attorneys have to talk about how far in the future P is anticipated to have these pains. Need to make jury feel how painful this is going to be in the future and for how many years in the future. Can do "a day in the life" vid to show what they go through, assume what they will go through. (how much is a day of pain worth? Attorneys suggest amount, jury imposes amount. Amount is just something jury pulls from thin air based upon the evidence that's presented to them. That's why you need that day in the life video. Ask for x number of dollars per hour. Judge will look at certain standards to decide if an amount is too much; here, uses "maximum recovery" rule. Also can look at other case in the j, see how it compares to other amounts. Same goes for past and future pain. Look at past standards on what has been awarded. In some js, tort reform has put a limit on the amount that can be recovered total for all kinds of pain and suffering).

If you look at negligence per se with an excuse and compare it to a presumption, they are:

basically the same thing. Once the excuse is offered, the violation of the statue is no longer outcome determinate that negligence occurred. What's important is whether the jury finds whether, acting in the totality of circumstances, if they acted as a reasonable prudent person. On exam: give analysis of what violation of statute means, explain the three effects, the ways of negligence per se. If there're facts regarding those, make an argument for and against them

statutes of repose - says that you bought the toaster in 2010, statute says you have 10 years to bring suit about the toaster being negligently made. If toaster burns you in 11th year, statute of repose has expired before injury occurred. Even though toaster is defective, can't bring suit. Why have statute of repose:

bc at some point, manufacturer needs to know there's no risk they'll be sued by you for the toaster. It's a policy reason - to limit liability of manufacturers. Starts ticking day one after you buy the product. Normally applies to airplanes, buildings. If there's design effect, you have x years to bring case.

Fourth way to find j and s liability:

by statute. Some state laws say that if you have the P and two Ds, j and s only applies for those two Ds for the one who's above fifty percent negligent. Aka, if D2 is only 25% liable, will only be responsible for his percente. But the other D could be held liable for everything.

in joint and several liability, how can the D who pays more of the money get contribution:

by two methods: 1. can bring in the absent D if the absent D can be found through what's called a third party practice; everything resolved in one trial. 2. If the D who pays all of the money, who's sued first, doesn't bring the absent D into the suit, will have to bring a later suit against D2. In that case, the absent D gets to re-litigate the percentage of fault, would get to re-litigate his liability, bc you cannot bind an absent D bc of due process.

Summarize notes after Watson:

c. depends on j as to whether intentional behavior will or will not cut off D's negligenece. The trend is that if the intentional act is foreseeable, it won't cut off D's original negligent behavior, which will then be proximate cause. d. Gives exs of where intentional intervention did not cut off D's negligence. Has to be extraordinary, not foreseeable. e. Most courts will not allow someone who's left their keys in their car negligent for what a criminal, who steals their car, does. BUT if it's a high crime area, one could argue that it would actually be foreseeable. f. Intentional behavior doesn't have to be criminal. Ex, teacher knows kids will fight at recess. If you leave students unsupervised, they will do bad things, hurt each other. Just because they may not have committed a criminal beatdown, it was still intentional behavior, still harmful, should've foreseen that. can't argue the students' intentional behavior cut off the school's liability. If the intervening conduct is not necessarily criminal, but still intentional, usually cuts off D's liability

i. Facts: P was pulled behind a snowmobile owned by D; child was driving. P's thumb was severed. P sued both child driver and owner. ii. Issue: whether the minor driver is to be held to an adult standard of care. iii. Rule: "it's a child's duty to exercise the same care that a reasonably careful child of the same age, intelligence, maturity, training and experience would exercise under the same circumstances." iv. Analysis: this rule discourages kids from engaging in inherently dangerous activities. Trad childhood activities don't apply under this rule, since they usually aren't capable of grave harm. Since the child was operating a vehicle inherently dangerous and capable of causing harm, must be held to adult standard of care.

child's standard of care ex - Robinson v. Lindsay

a. Some courts permit D to show he's has been criminally punished for the same wrong or punished in an earlier civil suit to lower the amount of punitive damages imposed. The applicable burden of proof for establishing entitlement to punitive damages is:

clear and convincing evidence in most js. This standard is between preponderance of the evidence and the criminal standard 'beyond a reasonable doubt.'

Montgomery Ward & Co., Inc. v. Anderson - explains Collateral sources

collateral source: implies P is getting money from another party than D or Ds reps (aka, insurance co, parents, spouses, etc.) The things that are NOT paid on behalf of the D, that came from sources unconnected to D, are deemed to be collateral sources. Many js won't deduct those amounts from the amount that D owes the P.

Diff between contribution and indemnification:

contribution is something that exists between joint tortfeasors. Ex in Biercz case - if one pays more than his share, can seek contribution from the other. Is avilabe to a joint tortfeasors when he pays more than his share. Indemnity: exists to people who are vicariously liable. The employer and employee have a common duty. Employee negligently drives a car while employer is away; employer is liable for the employer if employee is acting during the scope of employment. Employer may have told employee to drive v safely, given good instructions, but employee doesn't follow them. But the mere fact that the employee was acting negligently in employment will make employer liable, even though they're not negligent. A person who is held vicariously liable for someone else, and wasn't negligent, can get indemnification from the negligent party/employee. Employee needs to repay employer for what they had to pay. Often, employer is the only one that's sued. Employer can sue the employee to be reimbursed IF they are not negligent. Breached their duty if didn't tell employee not to speed. If negligent, then doesn't have a right to indemnity, just a right to contribution (joint tortfeasors - negligent at the same time, has common duty with employee). • Indemnification usually gives you more money than contribution - contribution just divies

So in Gentry, the only issue is whether the stair maintenance was a cause of the man falling:

court says we don't know that, we don't know what happened? What would you want to change to create that chain of 'but for?' i. If there was proof that he fell because of the steps. If you can do that, it's just like the Reynolds case. Put him on the stairs, badly dsrepaired, it's probably that somebody will fall. Then we go to the jury for them to decide if the steps were indeed the cause in fact of the fall ii. BUT FOR the steps disrepair, there'd be no fall, no discharge, no death. Connect the chain!

economic damages:

damages that can be reduced to a sum certainty of money. Aka, those are the damages that you can add up and get a total number. Ex: med expenses, lost wages past and future. Can be determined by physical evidence - bills, pay stubs. The economic damages are those that must be inflated from year to year and reduced to the present value.

conversion and intellectual property:

ds may be whatever the monetary value is, or the value of the hours' worth of work he spent creating it- average rate of an accountant, architect, etc may be the value of what you have to pay when convert intellectual property docs. "where info is gathered and arranged at some cost and sold as a commodity on the market, it's properly protected as property. where ideas are formulated with labor and inventive genius, as in the case of literary works or scientific researches, they are protected. And where they constitute instruments of fair and effective commercial competition, those who develop them may gather their fruits under the protection of the law."

The survival statute, p648: generally, if a claim exists on behalf of decedent, his estate will bring suit. Ds normally include any loss wages, med expenses that the decedent incurred from date of injury to date of death. Also includes any paid and suffering. Lost wages, destruction of property, pain and suffering are the types of ds you get in a survival claim - aka, same types of ds he'd get if he were living. Frequently, both survival and wrongful death claims can be brought at same time;

estate brings survival, beneficiary brings the other, but they'll be apart of the same complaint. Sometimes for ex a wife can be both the beneficiary and the head person in the estate. The problem is that you want to make sure there's no double recovery. Area in which there's likely to be double recovery is lost wages. (calculate by inflating, deflating). Way you avoid double recovery is that you give a credit to the D for any amount he's already paid for the survival claim. So judge might say ok how much of the lost wages would go to beneficiaries, then deduct that amount from the future lost wages that'd originally go the estate and just give them the amount that's overage - rest goes to beneficiaries. There's no double recovery.

(situation where retaining an independent contractor won't insulate employer from vicarious liability) Inherently dangerous activities or peculiar risk of harm:

ex: when a contractor was engaged to transport giant logs over a road, where an obvious special danger arises unless they're properly anchored and secured; to transport prisoners from state to state. Under such circumstances, those injured by contractor's negligence would be able to recover through vicarious liability form the person who hired the contractor. NOTE: the exception for inherently dangerous activities doesn't apply when the contractor's negligence is deemed 'collateral' to the inherent risk of the activity; aka, not recognizable in advance as particularly likely to occur or as calling for special precaution.

things that DO normally get an instruction of res ipsa:

falling elevator, escape of gas or water from a main, escape of electricity from wires, train derailment, explosion of a boiler

Diff tween false imprisonment and false arrest:

false arrest is when you're held in custom, in jail - in false imprisonment, you're just held in a physical space. False imprisonment can be committed by negligence, as well as trespass, but in those situation you need an actual injury.

what's the proximate cause test used in the Chang case:

foreseeability test.

Court must decide if an employee was on a frolic or a detour - this is almost always done by jury

frolic: abandonment of employer's business while in pursuit of employee's personal business detour: slight deviation from employer's own business for employee's own resaons

Wagner v. State: state would not be liable if guy's act was intentional. Court found that it was indeed intentional. This case is about mental deficiency, what impact it has upon intent:

he has to be so incompetent that he doesn't understand he's touching her for it not to be battery. As long as he understood that, he understands he's touching her, it doesn't matter that some delusion might b causing the touch. Only thing that's relevant is that he's touching her. That's the first element: whether he had a deliberate contact. Second element: the contact must be harmful or offensive. He doesn't have to know that it's offensive - it just has to be offensive to a reasonable prudent person

in Bierczynski, even though D didn't come into contact with P's car:

he's liable bc he acted in concert with the other D to cause injury

permanent disability and disfigurement:

her scars, inability to walk, no hair, all will be considered disabilities; jury can put a dollar amount on that. attorneys can suggest an amount, jury decides figure. Here, there's a laundry list of all of her disabilities.

(via Moore) Other things doc has to disclose:

his physical or mental disabilities (material to your decision if you want treatment from him; might be an alcoholic, drug addict) (when material, must be disclosed - HIV is one courts really disagree on - usually don't have to disclose if doc doesn't do invasive procedures - look at jurisdiction's policy for HIV).

In the notes following Smith v. Providence Health services, we get a discussion of legal malpractice cases. The issue presented there is slightly different. When you have a legal malpractice case, there are two cases under consideration - one case is the 'P's lawsuit' or 'other legal matter' for which lawyer committed malpractice.

i. Aka, P brings a suit against Kroger/etc, her attorney commits malpractice, lets case be dismissed bc didn't file some required legal pleading. P sues for malpractice. Ie, two cases - case against Kroger, case against lawyer for malpractice. In order to prevail in a malpractice case against the lawyer, must prove 'the case within a case.' Must prove that they'd have prevailed in the og case against Kroger AND must prove that the judgment against Kroger would've been collectible. Then of course has to prove the lawyer committed malpractice, and therefore bc of that she's not able to previail in og case. Ie, prove in og case would've won, the amount of damages that would've been given, that the damages would've been collectible. Otherwise, case will be dismissed. ii. There's a different rule for criminal case. In criminal case, must show that the underlying case was reversed bc of the mistake the lawyer made. If can't show that the case would've been reversed, can't be successful in suit. Must prevail in the case within the case. iii. Third situation - what if the case is settled? P says you settled my case for x amount, but it was worth y. if you hadn't been negligent, I would've got y. sues for lawyer's activities in the settlement. If can prove that a reasonable prudent attorney would've gotten a better settlement. This is hard to do. § Who can est this? Maybe the person on the other side of the settlement. In either three of these situations, lawyer can be held liable if P can prove they would have won the case within the case

Davison v. Snohomish County takeaways:

i. Alleged negligent act: county had duty to build stronger railway. ii. Court held that county didn't have a duty - burden of precaution too high; would be too expensive to create railways that'd stop cars from crashing iii. To consider: Lawyer must pinpoint the negligent act. In this case, act was not building railing strong enough. Could have pinpointed a diff fact, then case might have been different iv. There's another case like 40 years later - engineering' progressed, what's considered reasonable precaution had changed. Burden of precaution changed. 1) Another factor to consider - if it's technologically feasible to create a detour, then bc foreseeability is so high you'd have a different story. Then question would be whether a prudent county, given foreseeability of harm, would simply create a detour around the bridge (burden not too great). v. If this was just a decorative bridge, with no functionality: then you can argue that this is a dangerous instrumentality, and the mere having it is such that the risk outweighs the bridge's utility, it's negligent to have it. ex with turntable case. If a dangerous thing has no use, then its instrumentality means it is negligent to have it. That'd be like taking a yeti cup of nuclear waste to class.

In Cruz, P trying to sue the seller of the van via res ipsa - have to say there's some defect in these airbags that made them explode. Have to show the defect was in van when it was sold.

i. Can't prove defect was in the car at the time when it left the dealership three years ago. In three years, a lot can happen to a car. Can't use res ipsa, bc can't show D had control over the vehicle when the defect presented itself. ii. If you wanna use res ipsa against a manufacturer, it's best if the accident occurs right after sale. The longer after sale it occurs, the more likely some other act caused the instrumentality to fail. § If you buy a TV, it catches on fire, can you sue the manufacturer? You have to show that you didn't do something that caused the defect. Show that you maintained tv the way it was supposed to be maintained. Those will help you prove more likely than not that it was in the same condition it was when it exploded that it was when it left the seller.

medical malpractice:

i. Duty: doc must have the same knowledge, skills, and abilities as an ordinary member of the profession; exercise their best judgement; use due care in carrying out treatment ii. Breach occurs when doc doesn't have these three qualities - expert testimony needed to determine this. Expert will testify what an ordinary member of the profession would do

Burden of Precaution ex - Davison v. Snohomish County

i. Facts: 1926, Ps were driving car over bridge; lost control, car hit bridge railing, broke through, fell to ground. Ps sued county for negligence in construction and maintenance of bridge's "elevated approach" ii. Rule: "Roads must be built and traveled, and to hold that the public can't open their roads until they're prepared to fence the roads with barriers strong enough to hold a team and wagon when coming in violent contact with them...would put a burden on the public that it couldn't bear. It'd prohibit the building of new roads, bring financial ruin to the counties undertaking to maintain the old ones." v Issue: was county negligent for having "insufficient" railing to prevent cars from skidding off bridge's approach (breach); (duty) did county have a duty to make railing strong enough to stand a car hitting it. Analysis: county can't be expected to have super-secure railings (wouldn't fly today).

Burden of Precaution ex - United States v. Carroll Towing Co.

i. Facts: there was a boat on which was some flour owned by P. Employees of D negligently shifted boat's mooring lines, boat broke free of pier, drifted up against a tanker, propeller broke a hole in boat, boat sank, lost the flour ii. Procedure: trial court found that Conner Co. (boat's owner) partially responsible for not having a bargee on board when it was damaged by D's negligence iii. Issue: (duty) Was Conners negligent in not having bargee on board iv. Analysis: Co. has reasonable forewarning something'd happen to boat while at pier. This requires a bargee. Bargee doesn't have to be onboard always, but this bargee gone almost 24 hours. Therefore, Co. didn't meet its responsibility to guard against likely damage. Conclusion: Conners Co. found partly responsible for the losses due to the absence of the bargee.

In Larson v. St. Francis Hotel, P assumes chair came from hotel - doesn't know for sure. Has to show more likely than not it came from the hotel.

i. Has to show that the hotel had control over the chair before it fell (this court requires proof of EXCLUSIVE control); court says that the people in the hotel rooms have some control over the chairs in the room, hotel has the right to control the chairs; ie, two different entities have the right to control this chair, we don't know whose possession it was in when it fell. ii. No evidence that despite the fact that hotel had the right to control the chair, no evidence they WERE controlling it, just as likely that one of the guests was handling the chair when it fell. P can't est exclusive control. iii. But even in a j dealing with the right to control, don't have to show exclusive, she probably would've lost - court says in order to prevent a chair from falling when tenant has possession of a room, hotel would have to keep a guard in every room (this is an old case); note that now, just have a window that can't open. Today, you'd focus on the hotel not having windows that were strong enough; negligent to not have such windows. § Probably don't even need res ipsa for that - can just use ordinary duty and breach. Can just say they had a duty to use reasonable care to construct windows that can't be opened, get an expert to say that's possible pursuant to the custom, say that they breached custom as well as duty, and bc of that breach they were negligent. In reality, this case could get dismissed at summary judgment stage. P couldn't show evidence on who had control of chair

MORE on medical malpractice evaluation when applying professional malpractice to Be Careful hypo:

i. Her age, condition will be part of asking what a reasonable prudent doc would've done. Maybe the standard would be to give the drug to a 15 but not a 75 year old. ii. Also there could possibly be an argument about whether the doc was acting from overt or implicit racial bias) (all patients, regardless of race, who are the same level of sick should receive the same level of care - gotta consider that, talk about racism in medi profession; going to need to question if her race motivated any of the decisions made by docs; lookit research regarding that. § Docs might not recommend same treatments for black people as for white. § May have to ask who doc is treating, what their trends are. Then might have other theories come up, like discrimination § Another factor is that she might not have had insurance - must treat her as they would a patient that does have insurance. Can't treat her with substandard meds when there's a more efficacious med available. § Even though doc may have been black, doesn't mean he couldn't discriminate. Can be inter-race racism. iii. Jury's going to have to decide on a breach, if you get docs to get up and say the doc did breach their duty in some respect, and it was based on race, the jury may be influenced by implicit bias. § Also, when you get to jury, have to look at how juries relate to docs - aka, 60% doc wins bc they're a respected member of comm; P may not have that influence. Gotta convince the jury to go along with YOUR expert. iv. So the question becomes how far do you drill down on the malpractice issue. They have to have knowledge, best judgment, due care according to the locality/similar locality/nat standard of care. Have to have expert witnesses going both ways. And then you have the race issue coming into play, and whom jury will identify with. that's all just on medical malpractice issue

In Smith v. Providence Health, we got a divide in opinion as to whether P can bring a loss of chance as a case:

i. In some js, the answer is yes, they'll follow Smith - aka, at the time the misdiagnosis or mistreatment occurred, P did not have more than a fifty percent chance of living or getting a better outcome IF they had the correct treatment. ii. In some js, like MS, the state DOES require that the P have more than a fifty percent chance of surviving or a better outcome. § Note 3, p309: mostly, if a state does recognize the loss of chance suit, mostly limited to med cases, courts won't extend to other cases like products liability.

Factors to consider when deciding if the adoption of a statute is far, workable, and wise:

i. Is there a preexisting common law duty to do what statute says § Normally what this means is 'previously, via case law or statute, has legislature or judge said that a reasonable prudent person would do the thing that the statute says should, should not be done. § Aka, if this thing the statute proscribes is something that previously a reasonable prudent person would already be required to do, that's one strike in favor of adopting the statute. § Case in point: no duty of a swimmer to jump in a pool and save someone - strike against adopting a statute that might say he should. ii. Is statute's mandate clear enough for D to know exactly what's expected of him. § If statute says to not park on street, don't have to debate what that means. But in Perry, statute's more nebulous - "anybody having cause to believe kid's being abused has a duty to report it to state authorities." If you see the abuse, you know what's expected of you; but if you just hear kid crying, don't really know. This is why judge says statute isn't clear enough on what you should have to have notice about the abuse. iii. Look at the disparity between the criminal penalty imposed on the perpetrator and the penalty imposed on the one that does not report § In Perry, penalty for molester's much higher than the abuse's reporter. When you have that disparity in the penalty of the statute, the non-reporter's sanctioned criminally at a much smaller rather than the perpetrator, that's a strike against adopting it. § You also look at the criminal and the tort liability. Perry, legislature imposed a small penalty if criminal prosecution follows. Tort civil damage remedy is much larger - court says when you have such a large disparity between civil and criminal liability, big strike against it. iv. Is the statute's adoption designed to impose liability for indirect or direct action. § In Perry, sued for indirect action, not actual molestation. § When it's direct, that's a factor in favor of adopting the statute. When indirect, that's a factor against adopting the statute as the duty. v. Would the adoption impose liability without fault/strict liability. § If it does, that points against adoption - normally in tort law, tort damages are imposed for things that one DOES to cause injury and not strict liability (strict liability - you own a Kroger. Anyone injured on the property can sue you and get damages for not having to show negligence). In Perry, you have to know it's abuse and through your own volition and fault decide to not report. So strike against adoption.

in applying Robinson to online hypo, does McAdoo case have any relevance? (says you must have an understanding of the dangerous capabilities of a vehicle)

i. Same standard applies to scooter; need to know the condition of movable parts. In knowing the condition of your movable parts, question is: did you act as a reasonable prudent person in driving a car, or scooter, with bald tires? ii. Brakes on a scooter is the same thing - on a scooter, are a visible part. You can see it, you can evaluate it. iii. McAdoo rule: You're going to be deemed to know the condition of movable parts on your instrumentality. § THEN: deeming that you knew, DID YOU ACT as a reasonable prudent person would - can't say you didn't know, you're going to be deemed to know, jury'll be told to assume you knew tires were bald. § Are you then negligent? Depends. May have acted reasonably if you had to go to the hospital. § Anytime there's an ambiguity in a hypo, point that out. Look at diff possibilities. Esp note type of scooter, where accident occurred. That'll determine whether he's held to an adult standard of care

If fires remain three feet, create six feet fire that burns down the fire, but when this six foot fire is travelling to the house, F jumps out and throws a cigar into the fire, is that cigar a substantial factor in causing the house to burn down?

i. You'd want to know about the cigar: was it highly flammable, had some kind of explosive in it. if so, you could say it was a substantial factor. 1. You have to know what the rule is in your j. some js use just one test, some use both. Whether they use a substantial factor and one cause is involved, can use but for to define it. if there's more than one, can't use 'but for.'

lack of informed consent

i. physician oriented standard § duty: to disclose the risks that a reasonable prudent doc in the same or similar circumstances would § breach: when risks, benefits, option of doing nothing are not disclosed § need an expert to testify as to what risks an ordinary member of the profession would disclose. ii. patient oriented standard § Duty: to disclose all material risks that a reasonable prudent patient would want to know about. § Breach occurs when a material risk isn't disclosed. § Don't need an expert to tell you when a risk is material, but do need an expert to tell you what the risks are, alternative treatments are, what risks are of doing nothing. Jury decides if it was material from that info.

Yellow Cab Co. case:

if P is in D2's car, and D2 is husband. Spouses are protected like employers, parents. Immunity. If a D is immune cuz of workers comp, parent hild rule, husband wife rule, or any other basis for immunity, a D cannot seek contribution from someone who is not subject to suit by the P. governmental immunity: sometimes can't sue gov unless they let you • D1 can't sue D2 if 2 is immune to suit by the P, can't get contribution from them.

Another group of people who are able to get indemnity:

if a Ford dealer is liable, acted negligently, Ford may be vicariously liable even though they're the one sued. Can get the dealer to pay them back. manufacturers and distributors, contractors and subcontractors. • Knell says that D1, Feltman, can get contribution from someone like Knell who was not even sued. Knell can't complain that he has no judgment against him. can bring into the suit, and all that will be resolved. • What are the methods by which contribution is assigned: if D1 has successfully brought D2 into the lawsuit, how will contribution be determined? Proportional/parater; or percentage. Depends on j - some have percentage fault, some have parater.

"On the day he died, decedent must have a claim against tortfeasor:" either a personal injury or property damage claim arising out of tortfeasor's negligent act. Ex: car accident, D hits P and kills him instantaneously. On that day, if he had lived, would've had a personal injury claim against D. if he dies ten later, on the day that he dies he'd still have acclaim - only reason he'd not have a claim is if statute of limitations had run. If statute of limitations has run, claim is barred. In what other ways would decedent not have a claim:

if he settled before he died, if filed the lawsuit and lost. Both situations, wouldn't have had a claim on the day he died. In other words, the beneficiaries claim is derivative of the decedent's claim. That's the precondition that exists - day of his death must have a claim.

thought exercise for Blyth v. Birmingham Waterworks Co.

if instrumentality had deteriorated through age, when you installed it you didn't make it durable enough, then issue would be how long would a reasonable person make it strong enough to last. Expert testimony may come in and answer this question. Takeaways: Compare to note that talks about building houses in Chicago and LA to withstand earthquakes - in LA there's duty, bc they're foreseeable, but not in Chicago

Assumption of the risk v. comparative negligence:

if they assume you took on the risk, you can't recover from D at all; totally cuts off D's liability. Comparative neg is just recognition that people have diff percentage fault to the injury.

So the tree is considered to be something diff from the ordinary natural or artificial rule. Are an exception to that rule. They're treated differently. What's the duty in respect to trees:

if you are an adjoining landowner to a public road or public place, tree injures someone on public road or sidewalk or space, the duty is to use reasonable care to prevent your trees from falling from your property to somewhere else.

Continuing Trespass

if you give someone permission to be/bring something onto your land, and they overstay their welcome or go outside the bounds of their invitation

Original tortfeasor doctrine:

in a situation where one party's negligence leads to other parties causing harm, the original tortfeasor ais liable, along wit the later acting tortfeasor, for the P's injuries. Ex: a driver hits a pedestrian, breaking his leg. While pedestrian is lying helplessi n the street, bicyclist runs over his arm. At the hospital, pedestrian is treated negligently by doc, aggravating his leg injury. Driver would be responsible for all injuries. Bicyclist would be responsible for only the arm injury. Doc would be responsible for aggravation of leg injury.

wrongful death and statute of limitations: some wrongful death statutes have their own statue of limitations, that takes precedent over more general statute of limitations.

in contrast, some js have held that the gen statute of limitations runs against he death action ONLy from the date of death, even though perhaps at the time the decedent's own action for personal injury or property loss would've been barred while he was living. aka, might no longer have claim for med malpractice, but claim for wrongful death starts at his death.

Note following Daubert:

in some cases, the court will relax the 'more likely than not' rule. Sometimes, court will relax doubling rule. Workers comp cases will do this. Show just that there's a reasonable probability, ie the increase in the risk in the population that took the drug is more than the people that didn't take the drug; doesn't have to be exactly double or more. a. But still must show cause in fact, that there was some substance or environment on the job that caused the harm, reasonable cause of the harm. Has to show that the injury on the job was in fact the cause of the ultimate injury.

if you release one joint tortfeasor, general rule is that you release all tortfeasors. But some js say that we get away from common law; it depends on the intent of the parties; look at their intent in the written doc; some js don't care if you call it a release or a covenant not to sue.

in some js that follow the rule if you release all, they apply that to not only people who were acting in concert, if they acted negligently at same time and injured you, people who also injure you subsequently. Ex, if car driver hit you, then hospital does malpractice, if you release doc you also release hospital - release one, release all, even if they were negligent at different times. Called the Original Tortfeasor Rule - driver I s responsible for everything that happens during, after crash; he's the reason why you're in the hospital. So if you release the driver, in some js you also release the doc.

Note 16, p675: once TN did away with contributory neg, went to comparative neg, did away with last clear chance. Also changed rules with res ipsa;

in some js, in res ipsa, a third element is that P didn't contribute to her own negligence. If in a comparative neg state, should do away with that factor bc she can still collect. Aka, in res ipsa some js provide that the P's own neg can't be the cause of the accident - a comparative neg state, P CAN be a part of the injury, so should still be able to use res ipsa even though her negligence played a part in the accident and her injury.

Negligent v. intentional trespass

intentional is when you know you're trespassing and you do it anyway. If it's negligent trespass, you have to show damages, like any other negligence case. If it's intentional, don't have to show harm, but you do if it's negligent trespass.

Willful, wanton, and reckless conduct:

intermediate class of conduct between negligence and intentional torts. This type of conduct is usually defined as consisting of a deliberate and conscious disregard for a known high degree of probability of harm to another

under comparative negligence, most courts have found that the last clear chance doctrine _____ necessary

is not

under comparative neg systems, there's the possibility of having the jury consider both the negligence of beneficiary and decedent since the neg of either one may not automatically bar a claim. How do this:

jury allocates percentage of neg to beneficiary and decedent and then reduces the award to the beneficiary by that total amount. In modified comparative neg systems, recovery by that beneficiary is barred if the threshold is reached.

Exs of battery that involve less direct contact:

knowingly serving wrong meatballs to Muslims, unauthorized surgery conducted on P, expose of D to virus like herpes (consent to sex doesn't bar action)

Judicial proceedings and litigation immunity: type of immunity:

litigation immunity protects attorneys from tort liability for questions asked during deposition that Ps alleged constituted intentional infliction of emotional distress. The "absolute privilege" to publish defamation in the course of judicial, legislative, or executive proceedings is really an immunity of those engaged in the proceedings, conferred bc of the public interest in protecting them from suit

Is there a duty to wear a helmet/face mask/seat belts? In common law, such failure not deemed to be contributory or comparative negligence. But as it relates to seat belts, many js have seat belt statutes that allow the failure to wear a seat belt to be used against a P who sues the driver of another car or who sues the manufacturer of her car. Suit against the manufacturer: the car was not crash-worthy.

maj of js interpret their statues to permit evidence of seatbeltuse or nonuse in cases alleging crashworthiness against the manufacturer, but ont those alleging negligence against the driver of the other car. Aka: if there's a seatbelt statute, and your failure to wear it can be used to allocate a percentage of fault to the person who didn't wear the belt, and a suit against the manufacturer of a car based upon crashworthiness of the car happens, the other D in the other car cannot use failure to wear a belt to allege that some fault should be assigned to P for not wearing a seat belt. Some js say that if a P fails to wear a seatbelt, her damages will be reduced by 5%. if says that, then can't be reduced beyond that. five percent cap on reduction applicable against another driver, but not against manufacturer of car. NEDD TO KNOW: failure to wear a seatbelt can be used to reduce some D's responsibility for the damages and a certain percentage of fault can be assigned to the P, sometimes that percentage of fault is stated in the statute. Same thing with failure to wear a helmet. No duty in common law. But if duty is imposed by statute, failure can be used to reduce P's recovery; can be sued to assign percentage fault to P that will reduce her recovery - same thing can be said about face masks - no duty, but if there is a duty by certain ordinances, state law, then the failure to wear them can be used to assign percentage fault to P thereby reducing her recovery by that percentage fault.

categories/considerations regarding economic damages:

medical expenses; lost wages; loss or impairment of future earning capactiy; damages calculation/future inflation; fed income tax.

If doc fails to inform a patient that they are contagious/carry a disease, then a third party person, affected/caught disease from the og patient can sue the doc. Same if they don't disclose that a medication can cause them to have seizures, have seizure while driving, hit somebody What if doc is examining a patient bc of an employer, insurer, or bc of litigation:

most courts have found that the physician owes a limited duty, sounding in med malpractice, to examine the patient without harming her but not to provide diagnosis or treament.

more about compensatory damages:

most personal injury cases ask for these damages. They are damgaes by the way of money that's supposed to put you back in the same place you would've been if D hadn't hurt you. put you in a pre-injury state. Can do that by giving lost wages, med expenses, etc. there are different types of compensatory damages - look at case with young burn victim

Loss of consortium:

most states recognize a claim by the spouse of an injured person for loss of conjugal relations, society, companionship, household services, etc. during the time period that the injured person is recovering or for a permanent loss. Etc, the husband of an injured woman can sue the D for loss of consortium. Usually has to be brought in the same case as the wife's claim, though.

In STATE law, you go to the MS or TN statute and pull up their wrongful death and survival statutes. Will tell you when the claim exists, who are the beneficiaries of the claim, the statute of limitations for the claim. Defined by state statute. State SCs tells when claim can be brought, who beneficiaries are. But there are some commonalities across the diff states:

most will lay a precondition to the bringing of the claim. One, almost universal, is that a wrongful death claim will exist in the beneficiaries on the date of the death of the decedent. This is the precondition: provided that on the day of the death of the decedent, the decedent would've had a claim against the tortfeasor. THEn the beneficiaries can bring a claim for wrongful death.

regarding next of kin in wrongful death claims:

normally if nobody else is alive we go to them. These are the cousins, nephews, anybody who is a blood relative. Most states have intestate laws, that say who your property goes to if you die without a will; in these, will list next of kin, tell you which types of next of kin take priority over the others. But if there's no beneficiaries in the classes listed by the statute, there is no right to bring a wrongful death claim by the tortfeasor.

(situation where retaining an independent contractor won't insulate employer from vicarious liability) Apparent authority:

one who expressly or impliedly represents that another party is his servant or agent may be held vicariously liable for the latter's negligent acts to the extent of that representation. This result may occur even though the negligent party is an independent contractor or even when there's no employment or contractual rel whatsoever between the negligent actor and the party making the representation. This theory allows the injured party who reasonably relies on the representation to hold the party who made the misrepresentation liable. Ex: franchisors held vicariously liable for the negligent conduct of their independent contractor franchises. Has also been applied incases holding hospital vicariously liable for the med malpractice of independent docs practicing on their premises.

Independent contractors: a person normally who commits a tort, ask if the tortfeasor is an employee or an independent contractor. If an employee, vicarious liability is usually est. but if an independent contractor, a whole separate set of rules kick in. employer usually not liable for contractor Indep contractor def:

one who has control over the manner and means of his work. Ex: need to cut down a tree - you call out a tree removal co, that co can cause harm by felling the tree wrongly. You'd like to say they're indep contractors. How'd that work: depends on whether you dictated to them how to cut it down, what tools they use, control the manner of their work. Just tell them the result you want, but didn't tell them how = indep contractor In Murrell case, the paper hired someone to deliver papers. Told him certain things, but didn't tell him the manner and means of how to deliver it, whether to use a car, bike, etc. those other things they told would be like the Results that you want. You can tell them result, but not manner and means. Results would include the time frame you want delivery. Manner and means would be where to put it, can't use a bike. But exactly what they do, the nuts and bolts, are left to them. But you may still be directly liable in hiring them - have a duty to act as a reasonable prudent person when engaging an independent contractor.

Future (and past) medical expenses:

past - after the injury but before the trial. Future - after the trial. Past is easily calculated; just look at medical bills that you've had, add up your expenses. Future will be based upon two people testifying: some doc that will give an opinion on what treatment she'll need in the future, what equipment, special meals, etc; and an economist who's going to price out each item, tell you how much x surgery will cost y years down the road - projects into the future how much treatment will cost. Up to the attorneys to gather all of the evidence that is needed on each and every element of med expenses.

Anderson v. Sears, Roebuck & Co.: is a products liability case. Asks what categories of damages would a P like this P be entitled to (assuming that the D is negligent and their negligence caused injury) (note distinction between injury and damages: damages ask how much P can get if can est duty, breach, causation). This case lists different categories of compensatory damages:

past physical and mental pain future physical and mental pain future medical expenses loss of earning capacity permanent disability and disfigurement

b4 filing a client's suit, you have to do some pre-litigation investigation. Rule 11 says you must investigate the facts AND the law; must do a pre-litigation investigation a. If you don't appropriately investigate facts and law: Rule 11 sanctions. b. Must make sure the facts and the law are well grounded for your lawsuit.

pre-litigation stage = what you do before the suit is filed. Rule 11 says:

There's a particular def for invitee, licensee. There's a trespasser def: normally someone on your land without your knowledge or consent. So what's the duty owed to a trespasser: it depends on the discovery of it, when you learn they're on the land

prior to discovery, what's the duty landowner owes: no duty. That means for our big hypo: all the train's bad acts, negligence could be irrelevant if ATV is a trespasser. The only thing that matters is once he discovers, reasonable care is taken from that point onwards to prevent harm to trespasser. Once he's discovered, what's the duty: 1. With respect to passive conditions (a sinkhole, trap door, pit bull), the duty is to warn them of the passive condition if it's not open and obvious. If it is open and obvious, common law rule is that you don't have to warn them. Another ex: just something on your land, like a pool. 2. Active condition: when you're actively doing something, like if mowing your yard you can run them over with lawnmower, if shooting you shoot them.);

Mary Carter agreements (Elbaor v. Smith):

provides that if you say A sues B, C, and D; C may settle with A, but other Ds may decide to go to trial. C can enter into a mary carter agreement - pay x amount, will stay in the lawsuit as a defendant and will help A win their lawsuit against the other Ds. If A gest some money from other Ds, A is to reimburse C for the money that they paid. • Main opinion in Elbaor: says MC agreements are void and not enforceable, bc it's collusion, and jury thinks that C, as a defendant, shouldn't be coming after other Ds. But many js allow these agreements bc they promote settlement. C's more likely to settle with A if there's a possibility they'll get that money back. • But in Elbaor, j says that as public policy they value justice more than settlement. • Depends on j whether MC are enforceable. Some js view them as collusion. • Dissenting opinion: frequently Ds turn on each other anyway. Normally the one that's at fault the others turn on, say that it wasn't us, it was them. As long as the jury knows for C to stay in the case, and C is trying to turn on other Ds, that's ok. •Also, court may give D and E more preemptory challenges (right to strike a jury for no reason) since they've essentially been ganged up on. change the procedures to make it fair.

In McIntyre case, TN case, court analyzed three types of comparative neg:

pure (subtract P's percentage, can collect the rest), fifty percent rule (P's negligence can be fifty percent or less but can't be above fifty; if above, won't be able to collect anything from D; if below, then can collect the amount that attributable to the Ds, but not her percentage).

judicial control of amounts recovered:

remittitur, additur. Before either of these is granted, has to be some standard applied to see if the damages jury awarded were correct and reasonable. And so the rule that was applied in the burn case was the Maximum Recovery Rule. The second standard: if the verdict is too low or high and shocks the conscience of the court (that's the standard). Another rule: whether the rule is grossly excessive or inadequate. Use these three standards - if one's been violated, they will order a remittitur or an additur (in state; in fed, can only give one of these). There's a preference that once the jury comes up with an amount, usually judge isn't going to overturn that. but sometimes jury's are fired up by P's attorneys, just give away money, and vv. Sometimes they make mistakes, and that's when the court will apply those three standards, grant remitter or additur. But the presumption is going to be that the jury got it right.

Some js will apply the discovery rule only when a foreign object is left in patient's body. But in many js, discovery rule applies to everything, not just foreign objects. We need to know:

some js have cause of action accrue on date of injury, on some it occurs on date P knew or should have known of injury; some js accrues when she knew of injury AND knew it happened bc of med malpractice

regarding wrongful death: Statutes provide that the action is to be brought by personal representatives of the deceased for the benefit of the named beneficiaries. Some statutes provide that the action should be brought by the beneficiaries themselves. Possible beneficiaries:

spouse, domestic partner/civil union party/reciprocal beneficiary, child, stepchild, child born out of wedlock, unborn child, parent, next of kin Most js say stepkids don't count. Unborn kid: every j has a cause of action for prenatal injuries to a viable, unborn kid who's then born alive and later dies. Some, however, won't allow recovery if the kid is viable but stillborn. Others won't allow recovery if the kid isn't viable at the time of injury, even if born alive. And others recognize a claim even if the baby was neither viable nor born alive. In most js, a parent is a beneficiary only if a kid dies without a spouse or children. Some of those statutes require the parents of adult kids to show financial dependence to qualify as a beneficiary. Under some statutes, if no designated beneficiaries are living at the time of wrongful death, the action files. Other statues have a catchall category, like 'next of kin' if there are no other named statutory beneficiaries

non-economic damages:

tend to be pain and suffering. It also includes loss of function or appearance. Can't be reduced to a sum certainty bc jury basically just pulls down an argument out of the air. Bc can't b reduced to a sum of money, most juries won't discount them down to the present day. Whatever the jury thinks you're entitled to is what you get. Not based by a computation of an expert. Don't need an expert for non-economic damages bc no way to calculate it other than to tell jury to give P a reasonable amount

a. P must show that D's breach caused her injuries. After proves that, she can get damages

the fourth element of negligence, damages:

Selders v. Armentrout Holding:

the measure of ds for the wrongful death of a minor child should be extended to include the loss of the society, comfort, and companionship of the kid.

under rescue doctrine, WHO can rescuer sue:

the og tortfeasor, the victim (if the victim was half negligent for accident, should be able to get half of his damages from the victim)

past physical and mental pain:

the pain P suffered before the trial, things that have already happened

a. Breach occurs when D doesn't comply, either through omission or commission, to comply with their duty i. Omission: failure to act, build a fence around your pool ii. Commission: your action wasn't done prudently or reasonably b. Note that Breach is an objective standard, not subjective. i. Jury decides if a reasonable person would/wouldn't do something, but they don't use a subjective standard of 'what would I do, what would a 105-year-old do.' ii. It's what this imaginary prudent person, not what an individual juror or a particular person would do. § Jury's supposed to know what that person would do based upon their life experiences. Ex, they know from life exp. that a reasonable person doesn't usually look under their car to see if a baby is under it when driving away.

the second party of negligence, breach:

The measure of ds for conversion is:

the value of the property converted. Usually this is the mkt value. Ds, though, may be available both for the fmv and for loss of use of the chattel until it's replaced.

Acting in concert means:

there's a plan between A and B to do a specific thing. Why Summers v. Tice doesn't apply - both negligent, but won't in concert; just acting negligent at the same time. Why, in Summers, burden shifted for them to prove they weren't negligent: both Ds were in court, P didn't know who shot her; if they couldn't prove which one did it, would be jointly and severally liable. • In this Bierczynski case, the drivers were acting in concert - why they are jointly and severally liable.

Statute of limitations: every civil cause of action will have an applicable statute of limitations - after the injury occurs, P has a certain number of years to file a suit. If doesn't file suit in that period, her claim will be barred. If she should subsequently try to bring a barred claim, D will raise an affirmative complaint. Under statute of limitations:

there's frequently ambiguity when an injury occurs but P doesn't know injury has occurred until later date. When does statute run - date of injury or date when injury was discovered? Here, p692, doc did a sterilization, but he botched it up, patient got pregnant, she wants to sue for malpractice. P693, statute of limitations says that for med malpractice suits, the action must be commenced in one year after the cause of action accrued. When does cause of action accrue? Date of sterilization, the injury, or when she got pregnant. D wants the date of injury to be when he did sterilization. P wants, and GETS, the date when she discovers. HERE< court says the statute starts to run when she knows or should have known that something is wrong. This also applies to leaving foreign objects in a patient.

(situation where retaining an independent contractor won't insulate employer from vicarious liability) nondelegable duties:

there's no obvious criterion by which it can be determined whether a duty is 'delegable' or not. Exs given in Maloney of a nondelegable duty: duty of a condemning agent to protect a severed parcel from damage, the duty of a general contractor to construct a building safely, the duty to exercise due care when an independent contractor is employed to do work which the employer should recognize as necessarily creating a condition involving an unreasonable risk of harm to others unless special precautions are taken, duty of landowners to maintain their property in a reasonably safe condition

O'Shea v. Welch: manager was delivering football tickets to a branch office; on the way, he starts to pull into service station to get an estimate for his, which he uses for work, while pulling off he hits another car; car sues his employer, says he was operating in scope of his employment. Employer says he wasn't operating in scope of employment - was his own personal car. Court says:

there's two principles that apply: he was your employee, going to deliver tickets in his own private car - if that's in the scope of his employment, then the first issue is if while going to branch office he was operating in his job. Let's assume he was (though really we need more discovery); second: when he turned off the highway, is he still operating in the scope of his employment. Did his turning to get an estimate take him out of the scope of employment. Court says that's a question of fact. His car has a duel private-business use. Getting service on it can satisfy a dual prupose. If you use a car for work and private stuff, stopping off at a service station may be still in the scope of employment. Pittman argues that if he was going home after work, and he uses his personal car for travelling around from office to office, stopping to get it service may still be in the scope of employment. Could be part of the incidentals of his job. Another thing in this case: diff tween a frolic and a detour/slight deviation. If on a frolic, outside the scope of employment. If just a slight deviation, he's still in the scope. Jury's going to have to decide if his turning off the road was frolic or detour IF decides he was acting in scope of employment. Are you getting to station to get car serviced so you can go to work, or are you going to meet your bf. If the latter, not serving your master's business, are on a frolic.

More about equitable relief:

things like an injunction; ex if somebody has stolen money from you, invested, made millions off of it. disgorgement - taking that profit and giving it back to you. normally not money - usually a declaratory judgment saying you have certain rights. Could be specific performance that makes them give you some property. In these damages, there is an injunction. Ex, could be sell the business and divide the profits.

Common law collateral source rule:

this is applied when P receives compensation from any source collateral to the tortfeasors; these may include health or med insurance, employee benefits, gov benefits. When benefits are conferred upon P gratuitously, as when they are nursed without charge by a spouse, the prevailing rule is that P is entitled to recover the reasonable value of the services from the D, since they are a gift to the P and not the D and thus should not reduce D's liability

the foreseeability test for proximate cause only applies to:

those harms that result from the risks that made the actor's conduct tortious (but, I mean, this is subjective, kinda) ex: D bus driver was speeding - he risked a crash with another car, but not created a snow-swirl that would blind another driver that followed in his wake

Some exs of things that historically res ipsa hasn't been applied to

tire blowout, man falls down stairs. you're just as likely to be the case any any D.

A release implies you've settled with a D, signed a settlement or a covenant not to sue. What if, after the accident with A, you're hurt by B and decide to sue B bc you think your only injury is from B, but after you settle with A you realize you have an injury from A? Can you go back and ask for more?

usually can't go back and ask for more. Usually when you release the D, release is worded so that you release them for all injuries and injuries that could happen. Only caveat: if D committed fraud in some way, or you can prove they broke k (settlement is a k).

Sullivan v. Crabtree

v FACTS: Ps brought suit to recover damages arising from the death of their son, who was killed while a guest in a motor truck which swerved off the highway and overturned down a steep embankment. Suit was brought against driver. Jury returned a verdict in favor of D. On review the trial court found that the doctrine of res ipsa loquitur did apply to cases involving motor vehicle wrecks. v ISSUE: Did the jury have sufficient evidence to rule in favor of defendant? Yes v CONCLUSION: judgment for D was affirmed bc there was sufficient evidence to support jury's verdict in favor of D, and Ps didn't present any other assignments of error that could be considered by the court. The cause of the death sued for was D's loss of control of the truck. This may have been due to his own negligence, or it may have been due to no fault of his -- an unavoidable accident resulting from the brakes giving way or the breaking of some part of the control mechanism of the truck. Since such conflicting inferences might be reasonably drawn from the evidence, it was for jury to choose the inference they thought most probable; and court cannot say that there was no evidence to support their verdict for D. a. When an auto goes off the road in a one car accident, the general impression is that it's the type of accident that ordinarily doesn't occur without the driver's negligence. i. When P est the res ipsa elements, the case goes to jury for them to look at the totality of the circumstances, apply the burden of persuasion. Has to find if, more likely than not, D was negligent when the truck went off the road. Here, jury decided no, we can't tell - P wasn't able to tip the scales in her favor. Could've been gravel on the road, brakes could've given out. We don't know what it was that caused the accident. P couldn't prove what it was, so she loses

James v. Wormuth

v FACTS: doc informed patient he'd not been able to find a guide wire after taking a biopsy, that he'd determined that it was better to leave it. patient then returned to doc complaining of pain she attributed to the wire, and that it was so significant that it disrupted her ability to work. Approximately 2 mths after the 1st procedure, doc did 2nd operation, wherein he removed the wire. Thereafter, patient sued for malpractice based on res ipsa loquitur. The trial court granted a directed verdict in favor of defendants, doctor and practice, in the patient's medical malpractice action. On appeal, the appellate court affirmed the trial court's decision. v ISSUE: Should the doctor be found liable for medical malpractice? No. v CONCLUSION: Court found that whether doc was in control of the operation did not address the question of whether he was in exclusive control of the wire, since several others participated in the procedure. Doc exercised his professional judgment when he chose to leave the wire in the patient, and patient did not present any expert evidence that by so doing, the doctor departed from accepted standards of medical care. Therefore, the directed verdict was properly entered.

forewarning ex (subelement for likelihood of harm, an element of breach): Blyth v. Birmingham Waterworks Co.

v Facts: 25 years b4, D'd installed water mains in the streets. The main next to P's house sprung a leak during a "severe frost;" water escaped into P's house, creating damages. v Rule: D has the duty to act as a reasonable company would when installing a water main to avoid harm to others v Issue: (breach analysis) was D negligent in not making the water main strong enough to withstand an extreme frost (alternatively, duty analysis: did co have a duty to make main strong enough to stand extreme frost) v Analysis: D wasn't negligent bc didn't neglect to do what a person "taking reasonable precautions" would've done. Reasonable man'd act according to temp of ordinary years; this year much colder than previous years. D'd only acted in provision of temps they knew of. No reasonable man could provide foresight against these temps. § Co had no foreseeable reason to expect freeze to be that cold. Therefore, didn't breach duty bc likelihood of harm was so low. v If there's forewarning bc of climate change, ie every year gets colder and colder, there might be a case for breach

Forewarning ex - Pipher v. Parsell

v Facts: P is suing Ds because one of Ds was driving, the other grabbed steering wheel and took car off the road. Did this 2x - 2nd time, P was hurt. P says D had duty to guard P against 2nd occurrence. v Issue: did driver breach by not doing something to stop passenger v Rule: driver has a duty to use reasonable care to protect passenger against unreasonable risk of harm. § Would there have been a breach if accident happened on first pull of the arm? No - was an unforeseen event. But if he knew this other D had pulled his arm b4 on a different occasion, then on 1st pull he would've been liable - it was foreseeable. v After 1st pull, driver needs to do something to guard against harm. 30 sec is long enough to cuss someone out, pull over. In this case, driver had forewarning there was a likelihood of harm. It's this likelihood that makes a breach occur if a precaution isn't taken that a reasonable person would do in a similar circumstance.

more details about nominal damages:

v small amount to vindicate your rights. Normally just one dollar. It's awarded not to compensate for injury, but to vindicate your rights. Ex if you have a track of land, neighbor might claim adverse possession. you go to court, ask for an injunction and nominal damages just so you can show that you won.

Two ways to get employer:

vicarious liability for employee's negligence when acting in scope of employment (applies even when person is going home, if harm occurs on job), direct liability (employer themselves did something negligent, like service alcohol or negligently engage a contractor/security guard/etc. and proximately cause someone else's injury).

"what's reasonable to expect of kids of like age, intelligence, experience." Aka, child must act as a reasonable child of the same age would act IF DOING A CHILDHOOD ACTIVITY. This is called the Child's Standard of Care.

what is the child's standard of care:

i. Then have to decide if that's an adult or a child activity; do only adults drive those types of scooters, or do kids? ii. If it was like a car, his age'd be irrelevant - adult standard of care. If like a toy, still ask if the location would warrant it being an adult activity

when applying Robinson to the online hypo: Note that we don't know how large his scooter is, if a license is required or not; was it like a motorcycle, or like a kid's scooter?

discuss comparative and contributory neg in wrongful death and survival claims:

when the action is brought under a survival act, it's in theory still on behalf of the decedent, and the negligence of even a sole heir has been held not to prevent recovery through contributory neg doctrine since the heir isn't the P. Under the wrongful death act, the recovery is for the beneficiaries, and their contributory or comparative neg of a sole or all beneficiaries generally is held to preclude the action on the same principle that would bar any other P in interest. When one of several beneficiaries is contributorily neg, the gen view is that the action isn't barred for those who weren't neg, but that recovery is diminished to the extent of the share of ds of the neg beneficiary, who is denied all share in the proceeds. Generally, the neg of one parent isn't imputed to the other in a claim arising out of the death of their kid (ex, if they're involved in the wreck that killed the kid)

slight negligence:

when utmost care is used and you don't do something that causes an accident. Ex: (only a few js use slight negligence) driving a truck at night at 60mph down the center of the highway - aka, you just don't give a damn. 1. Ex: drive 70mph around a curve, try to pass another car. Basically means to not do anything. When utmost care standard is used and you don't use it. Defined on p231. Slight - failure to use great care. Means that you don't do anything.

Some js have separate statute of limitations dealing with DES and asbestos bc disease caused by them usually don't show up til years later. Say you were exposed to malpractice in state 1, state 2 bring the lawsuit - which statute of limitations apply?

where malpractice occurs, or where you filed suit? Some js apply statute the state in which you file the suit.

What distinguishes cases where cause is lack of informed consent vs. medical battery (Scott v. Bradford)

whether an expert witness is needed. i. Primary consideration in med battery case is whether patient authorized a procedure. Ex, no expert's needed to est standard for informed consent if surgeon operated on right eye after obtaining consent to operate on left eye

Some js that have gone from contributory to comparative neg have done away with implied secondary assumption of the risk, instead look at "going into the building" as negligence

will become part of comparative negligence, your act will reduce your recovery.

regarding wrongful death: most courts have held that a judgment for or against the decedent in an action for his injuries commenced during his life, or the compromise and release of such an action:

will operate as a bar to any later suit founded upon his death. aka, if he wins a suit for injuries, then can't later bring wrongful death or survival claim. In efect: decedent "sells out" the beneficiaries' claim because their action is "derivative," arising out of the wrong og done to him. This helps prevent double recovery.

Bundt v. Embro Brief:

§ FACTS: 2 cars hit at an intersection, which was controlled by a stop sign, and the passengers in both cars were injured. Both groups of passengers (Bundt and the Mondini family) brought a negligence action in the Supreme Court, suing the owners and operators of both automobiles as well as Peckham, a construction corporation, which had been doing some repair work on the road. More specifically, the complaint alleged that both automobiles were operated in a negligent manner and that defendant Peckham, while repairing the highway near the intersection, had negligently obstructed the view of the stop sign, resulting in the automobile driven by defendant Embro colliding into the car driven by defendant Di Belardino. Plaintiffs also allege that Peckham's negligence, combined with the negligence of the other defendants, caused the accident. Defendants advised the court that plaintiffs had instituted a separate action against the State in another court proceeding and that a decision rendered by that court had granted a judgment in favor of the plaintiffs, of which the judgment had already been satisfied. Ds argued that since they were joint tortfeasors with the State, the satisfaction of the judgment against the State discharged them. Ds sought leave to amend their answers. § ISSUE: Was the judgment already satisfied? Yes § CONCLUSION: The court held that when one was injured by the joint wrong of several parties, the injured party could recover his damages against either or all of the parties. However, although there may be several suits and recoveries, there could only be one satisfaction. Accordingly, the court awarded defendants leave to amend their answers

Larson v. St. Francis Hotel

§ FACTS: P stepped out under D hotel's marquee, hi by a chair that fell from sky. § ISSUE: Does res ipsa loquitur apply under the circumstances of this case, where the cause of the accident was unexplained and might be due to one of several causes for some of which the defendant was not responsible? No § CONCLUSION: P failed to prove D had exclusive control of the falling chair, or that injury wouldn't have occurred had D used ordinary care. The accident ordinarily might have happened despite D's reasonable care under circumstances. Thus, res ipsa loquitur couldn't apply where cause of accident was unexplained, might be due to several causes of which D was not responsible.

Cox v. Pearl Investment Co. Brief:

§ FACTS: Ps sought compensation for injuries plaintiff sustained when she fell on property rented by tenant but owned by defendant. Plaintiff's complaint acknowledged that the tenant rented the property from defendant, but the complaint only charged defendant with negligence. Apparently, the tenant had compensated plaintiffs, who executed a release document with the tenant. Defendant moved for summary judgment and alleged that the release of the tenant operated as a release of all tortfeasors. Defendant was granted summary judgment. On appeal, the court reversed. § ISSUE: Does the execution of a "Covenant Not to Proceed with Suit" release all the tort-feasors from their liabilities regardless of the presence of an expressed reservation in the instrument regarding others who may be liable to the injury? No CONCLUSION: Where a contract had the effect of releasing one joint tortfeasor while reserving the right to sue the others, who could be liable, it should not be treated otherwise. The expressed reservation in the instrument evinced a clear manifestation that plaintiffs were not receiving full compensation, and their right to bring an action against others who caused the damages could not be foreclosed.

Instrumentality ex - Lubitz v. Wells

§ Facts: 11 year old was playing in yard with Lubitz, swung a golf club, hit her. D is the dad who left club in the yard. § Issue: (breach analysis) was D negligent leaving golf club in the yard (note that child was negligent for swinging it) § Rules: under negligence, D has a duty to act as a reasonable person would in a similar circumstance. § Analysis: didn't breach duty bc had no reason to think a golf club would be a dangerous weapon to leave around a. Instrumentality of the object concerns whether it has the propensity and opportunity (are you in a desert or a backyard?) to hurt someone. i. ALSO depends on whether it HAS in the past been used to hurt someone. If this was the 2nd time the kid had hit someone with a club, outcome probably'd be different Instrumentality of some objects, like a hoe, is more nebulous. Would there have been forewarning from a sharp hoe as opposed to a dull hoe? Let the jury decide

Crucial factor to consider under burden of precaution (element of breach): Utility of instrumentality. Ex: Chicago v. Krayenbuhl

§ Facts: child played on a railroad turntable close to a public path. Railroad co. had a policy that turntable be locked when it wasn't being used, but it wasn't usually followed. Playmates set turntable in motion, child's foot was cut off. § Rule: Loomis v. Terry, 17 Wend. 496, 31 Am.Dec. 306 - "the business of life must go forward;" Should have use up to when benefits no longer outweigh danger. § Issue: was there an error in the instructions that were given to the jury? BETTER: (duty) did rr co have a duty to lock the turntable. § Analysis: "in all cases of this kind in the determination of negligence, regard must be to the character and location of the premises, the purpose for which they are used, the probability of injury." a. Offending act: did not lock the turntable b. Likelihood of harm: high. Kids walk close by. Gravity of harm: high. Kid's foot was chopped off. Burden of precaution: VERY low. All they had to do was lock it. c. The co isn't negligent for just having a turntable - it has practical uses. We want it to exist, but you need to take precautions. v Burden of precaution is v low - not asking for a cement wall, just a lock, and employees to follow your own rule.

eggshell rule test (for proximate cause): ex is Bartolone v. Jeckovick

§ If person has a pre-existing condition, D "takes the P as he finds him." That means that if there's a pre-existing condition. Had a latent condition in his brain, but the accident morphed, made the latent condition active. Could have been latent cancer, pre-diabetes - if the accident spurs a latent condition to be active and destructive, can fall in this case § Even if you don't know P had a condition that is latent, if it morphs into a destructive condition as a result of the accident, you're liable. § Notes say that some states make a difference between an underlying mental condition and underlying physical condition. Argument NOT for this: people with mental illness have hospital bills too. § Eggshell rules extends proximate cause (may be limited if is limited to physical conditions, applies only to physical, not mental conditions).

Via Daubert, we know that the two ways we decide if something is good science is that 1. an expert must have the knowledge or expertise in the relevant field; and 2. whether the expert's opinion that's given is credible to be accepted. So the question/priority is: did the test they used scientific methods/formulas, was research based upon the way that scientists would do the research. Look at whether there was pre lawsuit research done, whether that research was peer-reviewed

§ If you don't have research done before lawsuit, that's not peer reviewed, then the court says the expert can still give his opinion, but he'd better say it's clearly, better say what methods he used, step by step, and must be able to point to some authoritative source, show that the steps he used are at least supported by a minority of scientists in the field. § If a minority would look at your steps and say that's good science, then your opinion will come in and be accepted on the issue of cause in fact. i. Here, experts laid out what they did step by step, but they had no definitive source saying that the way that they did it was the appropriate way to test if the drug caused birth defects

in applying Robinson to online hypo, is Klein case relevant? Yes - boy wasn't wearing a mask

§ In Klein, there's a growing custom of using fake glass. There's a custom in many stores that you should wear a mask. This particular state didn't have a law that you have to wear a mask inside. Raises question of whether he has to wear a mask. Lack of criminal penalty doesn't necessarily cover tort liability. § Possible tort liability to wear mask around store. Did he neglect his duty when he didn't have a mask on? Note likelihood of harm, gravity, burden to determine this. § Frank had a lung condition. Burden is higher. In this case, there'll probably need to be an expert to testify what steps someone with a lung condition could do to still protect others from harm. v IN CONTRAST: we know he intended to bring a mask. Fact that he has a lung condition may not come into play since it's obvious he intended to comply with custom § If there's a custom of wearing masks in public spaces, regardless of whether there is a law or not, what does this custom do? Custom's only something that jury considers when deciding whether he acted as a reasonable prudent person. The fact that there's a custom to wear a mask is only RELEVANT, not determinative.

In applying Robinson to online hypo, is Breunig relevant? Doesn't say you have to be diagnosed with anything; just have to have forewarning to be liable. Do boy's prior hallucinations matter if they happened just at night?

§ Maybe - could argue that he had no forewarning they could happen in daytime. In the jurisdiction of Breunig, is going to say that he had no forewarning. § Will become a jury question - should he have foreseen a daytime hallucination. § If you're going to argue he did have forewarning, you'd go through his abuse, get an expert to say an abused person is more skittish and would know that.

constructive notice:

§ They didn't know it was there. Issue then will be was it there long enough to discover it before you fell. If it was, will be deemed to have had constructive notice. Aka, you should assume that they knew it was there bc it was long enough for them to know it; will be assumed to have known. If a reasonable person would've removed it, will then be liable. § If item wasn't there long enough, can't be proved to have been there long enough for them to have discovered it, judge won't send case to jury. § Unless P can show it was there long enough to be discovered, case dismissed. If P can prove it, goes to jury to decide if, given the totality of the circumstances, store did/didn't do something a reasonable store'd do to alleviate a dangerous condition a. Actual notice, they KNEW it was there. Constructive is more objective, has to be proven. If judge says it wasn't definitively there long enough, case will be dismissed

Direct Cause Test (for proximate cause): ex is a. In re Arbitration Between Polemis and Furness, Withy and Co., Ltd. How do you define direct cause:

§ in order to have direct cause, at the time that the D did the negligent act, it was foreseeable to them that some damage or injury would occur. If it's foreseeable that some damage or injury would occur, will be liable for what is both foreseeable and unforeseeable in an unbroken chain of events without any intervening causes. Aka, if there's some foreseeable injury from negligence act, D will be liable for both liable for both foreseeable and unforeseeable injury in an unbroken chain of event without any intervening acts (ex, if somebody kicked the plank in between the negligent unloading and the plank actually causing the spark - an intervention). If you can show a superceding, intervening cause, that will cut off proximate cause. § Direct cause is est if the D did foresee or should have foreseen some damage - will be liable for foreseeable as well as unforeseeable damages in an uninterrupted chain of events without any intervening cause. § Foreseeable even there: the plank falls on somebody's head. Unforeseeable: plank caused a spark that burned the boat. § To argue this test, there must be a foreseeable damage as well as unforeseeable. Aka, D may argue that it wasn't foreseeable that the damage that occurred didn't occur (of course, won't be able to argue this successfully). At the time of the alleged negligent act, a reasonable D, if they should have foreseen an injury, is going to be liable for the foreseeable injury as well as the unforeseeable injury as well as those injuries occur in an unbreakable change of events without any supervening or intervening causes.

actual notice

§ means that they observed it was there. If they observed it was there, didn't remove it, should indeed be negligent. Failed to do what a reasonable person would. § When do stores usually have actual notice: they cause the dangerous condition; OR saw it there b4 you fell on it. Can occur when employee caused it, didn't remove it, or when they saw it b4hand and didn't remove it.

distinctions regarding comparative negligence, j and s, and several liability:

· Comparative negligence: if you have different players that combine together, all be negligence, then in these js, jury should allocate a percentage fault to each party. · Some js, even that adopt comparative negligence, say it's a diff issue than j and s. P might be 20% negligent, but can still get 80% from either one of the Ds - can both still be j and s. · Two cases in book: in one, keeps j and s despite comparative negligence, in the other we get rid of it. · If you do away with j and s, the scheme is several liability. · Comparative negligence: if you have more than one D, percentage fault will be allocated amongst them. If in a comparative negligence state, can keep j and s or get rid of it. · Reason to keep j and s: why place the risk of insolvency on the P · Bottom line: who should bear the risk. · Can be in a j and s state or a severally liable state. But even in severally liable states, usually keep j and s when people are acting in concert. · In severally liable states, jury will decide percentage fault of each D; Ds responsible only for their percentage.

Should punitive damages be available against the estate of a dead tortfeasor? What about if, in a legal malpractice case, P wants to collect the money that they would have gotten in punitive damages?

· Should punitive damages be available against the estate of a dead tortfeasor? Majority of js don't allow punitive damages against an estate. · Depends on j whether a P can, in a legal malpractice suit, get from their lawyer the amount of money they would have received in punitive damages.

Under j and s liability, the D that pays the damages has a benefit:

· can sue the other D and get a fair allocation of the fault, money to be paid, reallocated between them. · Sometimes that reallocation is done under percentage fault. What percentage at fault was each party. · One remedy for j and s (for party that has to pay the whole): Seek contribution against the other tortfeasor (one way to do that - percentage fault allocation) · Another way to do allocation: a pro rata share - equally among the Ds, fifty/fifty.

Seigneur v. National Fitness Institute, Inc.

• Assumption of risk - two types: express and implied • Express: you either have a written k that has an assumption of the risk provision in it or you expressly state orally that you assume the risk. Either one is sufficient. • As in this case, will normally have you sign an Exculpatory clause. Here, it released club from any negligence perpetrated by its own employees and it released them from her negligence and any other person in that health club. • One issue: can a health spa get you to sign a release that says you release it from its own negligence; it's ok to release them from your negligence or somebody else's, but their own is another deal. This court says that in the context of a health club or gym, those releases are ok as long as the agreement clearly provides for them. The court referenced certain entities where the release of the entity from the consequences of its own negligence wouldn't be enforced. P682: schools, hosppitals, public utilities, doc offices, lawyer's offices, hospitals, common carriers. Normally if these entities get you to sign a release from their own negligence, it's going to be void as against public policy: bc usually those releases come in a k of adhesion tha you can't negotiation about. Those services are deemed to be essential services, you have to have it, are at a disadvantage. Once you agree to such a release, you disincenivize them to be careful. If you know that none of your guests can sue you for your own negligence, you might do horrible job, not keep things sanitary. • But since a gym is not an essential thing, you can protect yourself. Can't protect yourself in a hospital - you HAVE to go. You don't have to go to a gym. It's not essential. Therefore the clause is enforceable if you agree to release a gym or other non-essential business bc it's not against public policy for people to negotiate such k terms, have these Exculpatory k and releases. • Benefit to these exculpatory ks: if two people that are negotiating over non-essential services agree to a release, the court will enforce it. • Second thing: the court in this j says that despite the fact that it is not essential, and we normally enforce these type of ks in non-essential services, is there something else about this k that would make it appropriate to not enforce this clause as being against the public interest? Pointed out Tunkl factors as to when the public interest is in effect and sufficient enough for one of these ks to not be enforced. • KNOW the Tunkl factors. • This particular type of exculpatory k where the P released a co or a gym from liability for any injury they suffered from their negligence or from the co's negligence, are gonna be enforced unless it involves an essential business and you can est that the Tunkl factors weight in your favor • If it's a situation where there's a hospital gym, it depends on who the P is - are they a patient, prescribed specific med care, or just a general member of the public. If a pateitn, won't enforce Exculpatory ks.

(Brief) Owners and occupiers of land - outside the premises - Taylor v. Olsen

• FACTS: P sued for damages for injuries she sustained when her car, on a dark and windy January evening, struck a tree, which shortly before had splintered and fallen across a Clackamas County road. The trial court directed verdicts for defendant Clackamas County, the owner of the right-of-way on which the tree was located, and Marion Olsen, the adjoining landowner who was alleged to be in possession of the same location. Plaintiff appeals from the judgment entered on the directed verdict for Olsen • ISSUE: Will the possessor be held liable if he should have known of the danger? No •CONCLUSION: There was no evidence to suggest that chopping or drilling into the trunk would have been a normal or expected way to examine a standing tree in the absence of external indications that it might not be structurally sound. The only reference to such a procedure came from defendant Olsen himself, when called by P as an adverse witness. D had testified that the tree, located on county property immediately adjacent to D's land, was about 60 years old and approximately 85 feet tall. It was divided above ground into two separate trunks, "clubby" or short, and leaned a few degrees toward the road. Defendant also testified that a double-trunk tree might rot from water seeping into the tree where the trunks joined, but that he did not observe signs of such rot.

(Brief) Licensees - Barmore v. Elmore

• FACTS: P was attacked by D's son while in D's home to discuss business. P sued Ds after he was stabbed by the D's son, alleging that Ds, as landowners, were negligent in failing to protect P, as an invitee, from a dangerous condition on the premises. The dangerous condition referred to the son's mental illness. • ISSUE: Is the plaintiff considered an invitee, and was there a special duty to warn him about the son's mental health issues? No and no •CONCLUSION: The court found plaintiff's status as the time of the incident was that of a licensee-social guest and thus, the only duty owed to plaintiff by defendant's was to warn plaintiff of hidden dangers unknown to plaintiff of which the defendants had knowledge. Defendant's son's past violent behavior did not qualify as a hidden danger as ten years since the son had last exhibited violent behavior and because the son and plaintiff had previous physical contract. The judgment of the trial court was affirmed

(Brief) Salevan v. Wilmington Park, Inc.

• FACTS: P was walking on the public street adjacent to property owned by defendant Wilmington Park, Inc. Wilmington Park owned, maintained, and rented the property for baseball games. Although a fence separated the property from the street, a baseball struck Salevan in the head. Salevan filed a lawsuit against Wilmington Park in Delaware superior court seeking to recover damages for personal injuries she sustained. • ISSUE: Was Salevan entitled to recover damages for personal injuries she suffered when she was struck in the head by a baseball that came from Wilmington Park's baseball facility? Yes • CONCLUSION: After a bench trial, the trial judge found that, while Wilmington Park took precautions to protect people passing along the street adjacent to its property, those precautions were insufficient. The judge further found that Wilmington Park knew, or should have known, that the precautions taken initially were insufficient to protect the public engaged in its lawful right, that is, using the highways. The judge rendered judgment for Salevan and awarded her $ 250,000 in damages.

(Brief) On the premises - trespassers: Sheehan v. St. Paul & Duluth Ry. Co.

• FACTS: Plaintiff Sheehan was injured when he became entangled in railroad track owned by defendant St. Paul & D. R. Co. ("St. Paul") and was thereafter struck by train. Sheehan filed a lawsuit against St. Paul in federal district court seeking to recover damages for his injuries. The undisputed evidence established that, at the time of his injury, Sheehan was a trespasser on St. Paul's tracks. The trial court directed a verdict for St. Paul, and Sheehan appealed. • ISSUE: Was St. Paul liable to Sheehan for the injuries that he sustained? No. •CONCLUSION: On appeal, the court affirmed the trial court's judgment. The court ruled that Sheehan was a trespasser who ventured upon a track for a purpose of his own and assumed all risks of conditions which were found there, including operation of engines and cars. The obligation of St. Paul was to make all reasonable effort to avert injuries that could be controlled. There was no evidence to weaken the force of statements furnished by engineer, fireman, and brakeman, declaring that in spite of every means used to stop the train by exerting their utmost effort immediately upon hearing the Sheehan's cry, the train could not be stopped in time because of the conditions at the time, such as slippery state of the rails and the inability of using sand to slow the train.

Implied assumption of the risk ex: Rush v. Commercial Realty Co. Implied assumption is similarly an assumption of the risks by the P. it's not expressed as such in writing or by mouth, but her conduct indicates an implied assumption of the risk.

• In order for there to be implied assumption of the risk: P must have actual knowledge of the risk, she must appreciate the magnitude of the risk. Then she must voluntarily encounter the risk. Direct evidence: "yes, I knew there was a risk, yes I voluntarily used it." circumstantial evidence can also be used - would a reasonable prudent person have know that x was a risk. Ex, go to a baseball game, get hit with a ball. Note: you assume the risk you'll be hit by a ball, but not that you'll be trampled by unruly fans. • You must voluntarily encounter a known risk, and you must understand the magnitude of the risk. You might walk across the floor, know it's a little slippery, but if there's sign that says 15 people have fallen today, you assume the risk now that you may fall, you now understand and appreciate that people fall here. otherwise, you might not have assumed the risk - at best, might say you were just negligent, didn't assume the risk bc didn't understand the magnitude of the risk.

Slocum v. Food Fair Stores of Florida:

• P sought damages for mental suffering and an ensuing heart attack allegedly caused by the insulting language of D's employee while she was a customer in its store • What if slurs focus on racial or sexual characteristics - most courts have found them not so outrageous as to be intolerable in a civilized society.

Brief: McIntyre v. Balentine (shift from contributory negligence to comparative negligence)

• RULE: TN SC rejected the pure form of comparative fault and adopted a system of modified comparative fault, the "49 percent rule." So long as a P's negligence remains less than D's negligence the P may recover; in such a case, P's damages are to be reduced in proportion to the percentage of the total negligence attributable to P. • FACTS: P and D were involved in a car accident resulting in severe injuries to P. Based upon evidence introduced at trial establishing that P had been intoxicated at the time, the jury returned a verdict in favor of D, holding that P's contributory negligence completely barred P from any recovery. P appealed. Ct of appeals rejected P's arguments that jury should've been instructed on comparative negligence, as it was not the law at the time, and held that the presumption of intoxication provided by Tenn. Code Ann. § 55-10-408(b) (1988) was admissible evidence in a civil case. After replacing the contributory negligence system with a comparative negligence system, TN SC remanded the case for retrial in accordance with this new system and affirmed the introduction of the evidence regarding P's intoxication as a violation of a penal statute was negligence per se in TN. • ISSUE: Is P barred from recovering in a negligence action if he is also at fault? No • CONCLUSION: In all trials where the issue of comparative fault is before a jury, the trial court shall instruct the jury on the effect of the jury's finding as to the percentage of negligence as between the plaintiff or plaintiffs and the defendant or defendants. The attorneys for each party shall be allowed to argue how this instruction affects a plaintiff's ability to recover.


Kaugnay na mga set ng pag-aaral

Cultural Anthropology - Applied Perspective: Chapter 2: The Concept of Culture

View Set

Lab Safety Review Quiz - Lab Flow

View Set

✨THE DEFINITIVE Darby's Simulated NBDHE Board Exam 4

View Set